You are on page 1of 236

SOLUTIONS TO CALCULUS VOLUME 1 BY TOM APOSTOL.

ERNEST YEUNG
Fund Science! & Help Ernest nish his Physics Research! : quantum super-A-polynomials - a thesis by Ernest Yeung
http://igg.me/at/ernestyalumni2014
Facebook : ernestyalumni
gmail : ernestyalumni
google : ernestyalumni
linkedin : ernestyalumni
tumblr : ernestyalumni
twitter : ernestyalumni
weibo : ernestyalumni
youtube : ernestyalumni
indiegogo : ernestyalumni
Ernest Yeung is supported by Mr. and Mrs. C.W. Yeung, Prof. Robert A. Rosenstone, Michael Drown, Arvid Kingl, Mr .
and Mrs. Valerie Cheng, and the Foundation for Polish Sciences, Warsaw University.
SOLUTIONS TO VOLUME 1 One-Variable Calculus, with an Introduction to Linear Algebra
I 2.5 Exercises - Introduction to set theory, Notations for designating sets, Subsets, Unions, intersections, complements.
Exercise 10. Distributive laws
Let X = A (B C), Y = (A B) (A C)
Suppose x X
x A and x (B C) =x A and x is in at least B or in C
then x is in at least either (A B) or (A C)
x Y, X Y
Suppose y Y
y is at least in either (A B) or A C
then y A and either in B or C
y X, Y X
X = Y
Let X = A (B C), Y = (A B) (A C)
Suppose x X
then x is at least either in A or in (B C)
if x A, x Y
if x (B C), x Y x Y, X Y
Suppose y Y
then y is at least in A or in B and y is at least in A or in C
if y A, then y X
if y A B or y A C, y X (various carvings out of A, simply )
if y (B C), y X y X, Y X
X = Y
1
Exercise 11. If x A A, then x is at least in A or in A. Then x A. So A A A. Of course A A A.
If x A A, then x is in A and in A. Then x A. So A A A. Of course A A A.
Exercise 12. Let x A. y A B if y is at least in A or in B. x is in A so x A B. =A A B.
Suppose b B and b / A. b A B but b / A. so A A B.
Exercise 13. Let x A , then x is at least in A or in . If x , then x is a null element (not an element at all). Then
actual elements must be in A. =A A.
Let x A. Then x A . A A . =A = A .
Exercise 14. From distributivity, A (A B) = (A A) (A B) = A (A B).
If x A (A B), x A and x A B, i.e. x A and x is at least in A or in B.
= x is in A and is in B or is not in B. Then x A. = A (A B) A. Of course, A A (A B).
=A (A B) = A (A B) = A.
Exercise 15. a A, a C and b B, b C. Consider x A B. x is at least in A or in B. In either case, x C.
=A B C.
Exercise 16.
if C A and C B, then C A B
c C, c A and c B
x A B, x A and x B. Then c C, c A B. C A B
Exercise 17.
(1)
if A B and B C then
a A, a B.b B, b C.
then since a B, a C, c C such that c / B.
a A, a B so a ,= ca. =A C
(2) If A B, B C, A C since, a A, a B, b B, b C. Then since a B, a C. A C
(3) A B and B C. B C or B = C. A B only. Then A C.
(4) Yes, since a A, a B.
(5) No, since x ,= A (sets as elements are different from elements)
Exercise 18. A(B C) = (AB) (AC)
Suppose x A(B C)
then x A and x / B C =x / B C
then x is not in even at least one B or C
=x (AB) (AC)
Suppose x (AB) (AC)
then x is at least in (AB) or in (AC) =x is at least in A and not in B or in A and not in C
then consider when one of the cases is true and when both cases are true =x A(B C)
Exercise 19.
Suppose x B
_
AF
A
then x B, x /
_
AF
A
x /
_
AF
A =x / A, A T
since A T, x B, x / A, then x

AF
(B A)
2
Suppose x

AF
(B A)
then x B A
1
and x B A
2
and . . .
then A T, x B, x / A
then x / even at least one A T
=x B
_
AF
A
Suppose x B

AF
A
then x /

AF
A
then at most x A for A T but one
then x is at least in one B A
=x
_
AF
(B A)
Suppose x
_
AF
(B A)
then x is at least in one B A
then for A T, x B and x / A
Consider A T
= then x B

AF
A
Exercise 20.
(1) (ii) is correct.
Suppose x (AB) C
then x AB, x / C
then x A and x / B and x / C
x / B and x / C =x / even at least B or C
x A(B C)
Suppose x A(B C)
then x A, x / (B C)
then x A and x / B and x / C
=x (AB) C
To show that (i) is sometimes wrong,
Suppose y A(B C)
y A and y / B C
y / B C
then y / B or y C or y / C
(where does this lead to?)
Consider directly,
Suppose x (AB) C
then x is at least in AB or in C
then x is at least in A and / B or in C
Suppose x = c C and c / A
3
(2)
If C A,
A(B C) = (AB) C
I 3.3 Exercises - The eld axioms. The goal seems to be to abstract these so-called real numbers into just xs and ys that
are purely built upon these axioms.
Exercise 1. Thm. I.5. a(b c) = ab ac.
Let y = ab ac; x = a(b c)
Want: x = y
ac +y = ab (by Thm. I.2, possibility of subtraction)
Note that by Thm. I.3, a(b c) = a(b + (c)) = ab +a(c) (by distributivity axiom)
ac +x = ac +ab +a(c) = a(c + (c)) +ab = a(0 +b) = ab
But there exists exactly one y or x by Thm. I.2. x = y.
Thm. I.6. 0 a = a 0 = 0.
0(a) = a(0) (by commutativity axiom)
Given b !and 0 !, exactly one b s.t. b a = 0
0(a) = (b + (b))a = ab ab = 0 (by Thm. I.5. and Thm. I.2)
Thm. I.7.
ab = ac
By Axiom 4, y !s.t. ay = 1
since products are uniquely determined, yab = yac =(ya)b = (ya)c =1(b) = 1(c)
=b = c
Thm. I.8. Possibility of Division.
Given a, b, a ,= 0, choose y such that ay = 1.
Let x = yb.
ax = ayb = 1(b) = b
Therefore, there exists at least one x such that ax = b. But by Thm. I.7, there exists only one x (since if az b, and so
x = z).
Thm. I.9. If a ,= 0, then b/a = b(a
1
).
Let x =
b
a
for ax = b
y = a
1
for ay = 1
Want: x = by
Now b(1) = b, so ax = b = b(ay) = a(by)
=x = by (by Thm. I.7)
Thm. I.10. If a ,= 0, then (a
1
)
1
= a.
Now ab = 1 for b = a
1
. But since b ! and b ,= 0 (otherwise 1 = 0, contradiction), then using Thm. I.8 on b,
ab = b(a) = 1; a = b
1
.
Thm.I.11. If ab = 0, a = 0 or b = 0.
ab = 0 = a(0) =b = 0 or ab = ba = b(0) =a = 0. (we used Thm. I.7, cancellation law for multiplication)
Thm. I.12. Want: x = y if x = (a)b and y = (ab).
ab +y = 0
ab +x = ab + (a)b = b(a + (a)) = b(a a) = b(0) = 0
0 is unique, so ab +y = ab +x implies x = y( by Thm. I.1 )
Thm. I.13. Want: x +y = z, if a = bx, c = dy, (ad +bc) = (bd)z.
(bd)(x +y) = bdx +bdy = ad +bc = (bd)z
So using b, d ,= 0, which is given, and Thm. I.7, then x +y = z.
4
Thm. I.14. Want: xy = z for bx = a, dy = c, ac = (bd)z.
(bd)(xy) = (bx)(dy) = ac = (bd)z
b, d ,= 0, so by Thm. I.7, xy = z.
Thm.I.15. Want: x = yz, if bx = a, dy = c, (bc)z = ad
(bc)z = b(dy)z = d(byz) = da
d ,= 0 so by Thm. I.7, by z = a, byz = abx
b ,= 0 so by Thm. I.7, yz = x
Exercise 2. Consider 0 +z = 0. By Thm. I.2, there exists exactly one z, z = 0. By Axiom 4, z = 0. 0 = 0.
Exercise 3. Consider 1(z)z(1) = 1. Then z = 1
1
. But by Axiom 4, there exists distinct 1 such that z(1) = 1, so z = 1.
Exercise 4. Suppose there exists x such that 0x = 1, but 0x = 0 and 0 and 1 are distinct, so zero has no reciprocal .
Exercise 5. a + (a) = 0, 0 + 0 = 0. Then
a + (a) +b + (b) = (a +b) + (a) + (b) = 0
(a +b) = a + (b) = a b
Exercise 6. a + (a) = 0, b + (b) = 0, so
a + (a) +b + (b) = a + (b) + (a) +b = (a b) + (a) +b = 0 + 0 = 0
(a b) = a +b.
Exercise 7.
(a b) + (b c) = a + (b) +b + (c) = a + (b + (b)) + (c) = a c
Exercise 8.
(ab)x = 1 (ab)
1
= x
a(bx) = 1 a
1
= bx
b(ax) = 1 b
1
ax
a
1
b
1
= (abx)x = 1(x) = (ab)
1
Exercise 9. Want: x = y = z, if
z =
a
b
a = zt b +t = 0
y =
(a)
b
by = u a +u = 0
x =
_
a
b
_ _
a
b
_
+x = v +x = 0 vb = a
a + (a) = vb +by = b(v +y) = 0
if b ,= 0, v +y = 0, but v +x = 0
by Thm. I.1 , x = y
b +t = 0, then z(b +t) = zb +zt = zb +a = z(0) = 0
a +zb = 0 =a = zb = by
since b ,= 0, z = y so x = y = z
Exercise 10. Since b, d ,= 0, Let
z =
ad bc
bd
(bd)z = ad bc by previous exercise or Thm. I.8, the possibility of division
x =
a
b
bx = a
t =
c
d
dt = c (By Thm. I.3, we know that b a = b + (a) )
5
dbx +bdt = (bd)(x +y) = ad bc = (bd)z
b, d ,= 0, so x +y = z
I 3.5 Exercises - The order axioms.
Theorem 1 (I.18). If a < b and c > 0 then ac < bc
Theorem 2 (I.19). If a < b and c > 0, then ac < bc
Theorem 3 (I.20). If a ,= 0, then a
2
> 0
Theorem 4 (I.21). 1 > 0
Theorem 5 (I.22). If a < b and c < 0, then ac > bc.
Theorem 6 (I.23). If a < b and a > b. In particular, if a < 0, then a > 0.
Theorem 7 (I.24). If ab > 0, then both a and b are positive or both are negative.
Theorem 8 (I.25). If a < c and b < d, then a +b < c +d.
Exercise 1.
(1) By Thm. I.19, c > 0
a(c) < b(c) ac < bc
bc (ac) = ac bc > 0. Then ac > bc (by denition of > )
(2)
a < b a + 0 < b + 0 a +b + (b) < b +a + (a) (a +b) b < (a +b) + (a)
By Thm.I.18 (a +b) +(a +b) + (b) < (a +b) (a +b) + (a)
b < a
(3)
If a = 0 or b = 0, ab = 0, but 0 0
If a > 0, then if b > 0, ab > 0(b) = 0. If b < 0, ab < 0(b) = 0. So if a > 0, then b > 0.
If a < 0, then if b > 0, ab < 0(b) = 0. If b < 0, ab > 0(b) = 0. So if a < 0, then b < 0.
(4)
a < c so a +b < c +b = b +c
b < d so b +c < d +c
By Transitive Law , a +b < d +c
Exercise 2. If x = 0, x
2
= 0. 0 + 1 = 1 ,= 0. So x ,= 0.
If x ,= 0, x
2
> 0, and by Thm. I.21 , 1 > 0
x
2
+ 1 > 0 + 0 = 0 x
2
+ 1 ,= 0
=x R such that x
2
+ 1 = 0
Exercise 3.
a < 0, b < 0, a +b < 0 + 0 = 0 ( By Thm. I.25)
Exercise 4. Consider ax = 1.
ax = 1 > 0. By Thm. I.24 , a, x are both positive or a, x are both negative
Exercise 5. Dene x, y such that ax = 1, by = 1. We want x > y when b > a.
xb ax = xb 1 > 0 =bx > 1 = by
b > 0 so x > y
Exercise 6.
6
If a = b and b = c, then a = c
If a = b and b < c, then a < c
If a < b and b = c, then a < c
If a < b and b < c, then a < c (by transitivity of the inequality)
=a c
Exercise 7. If a b and b c, then a c. If a = c, then by previous proof, a = b.
Exercise 8. If a b and b c, then a c. If a = c, then by previous proof, a = b.
Exercise 8. If a or b is zero, a
2
or b
2
= 0. By Thm. I.20, b
2
0 or a
2
0, respectively.
Otherwise, if neither are zero, by transitivity, a
2
+b
2
> 0.
Exercise 9. Suppose a x. Then a x 0.
If a R so y R, such that a y = 0.
Consider y + 1 R (by closure under addition).
a (y + 1) = a y 1 = 0 1 < 0 Contradiction that a y + 1
Exercise 10.
If x = 0, done.
If x > 0, x is a positive real number. Let h =
x
2
.
=
x
2
> x Contradiction.
I 3.12 Exercises - Integers and rational numbers, Geometric interpretation of real numbers as points on a line,
Upper bound of a set, maximum element, least upper bound (supremum), The least-upper-bound axiom (completeness
axiom), The Archimedean property of the real-number system, Fundamental properties of the suprenumand inmum.
We use Thm I.30, the Archimedean property of real numbers, alot.
Theorem 9 (I.30). If x > 0 and if y is an arbitrary real number, there exists a positive integer n such that nx > y.
We will use the least upper-bound axiom (completeness axiom) alot for continuity and differentiation theorems later.
Apostol states it as an axiom; in real analysis, the existence of a sup for nonempty, bounded sets can be shown with an
algorithmto zoominto a sup with monotonically increasing and monotonically decreasing sequence of guesses and showing
its difference is a Cauchy sequence.
Axiom 1 (Least upper-bound axiom). Every nonempty set S of real numbers which is bounded above has a suprenum; that
is, theres a real number B s.t. B = supS.
Exercise 1. 0 < y x.
=n(y x) > h > 0, n Z
+
, h arbitrary
y x > h/n =y > x +h/n > x
so let z = x +h/n Done.
Exercise 2. x R so n Z
+
such that n > x (Thm. I.29).
Set of negative integers is unbounded below because
If m Z

, x > m, then x is an upper bound on Z


+
. Contradiction of Thm. I.29. =m Z such that m < x < n
Exercise 3. Use Archimedian property.
x > 0 so for 1, n Z
+
such that nx > 1, x >
1
n
.
Exercise 4. x is an arbitrary real number. By Thm. I.29 and well-ordering principle, there exists a smallest n + 1 positive
integer such that x < n + 1 (consider the set of all m + 1 > x and so by well-ordering principle, there must be a smallest
element of this specic set of positive integers).
If x = n for some positive integer n, done.
Otherwise, note that if x < n, then n + 1 couldnt have been the smallest element such that m > x. x > n.
Exercise 5. If x = n, done. Otherwise, consider all m > x. By well-ordering principle, there exists a smallest element n such
7
that n > x.
If x + 1 < n, then x < n 1, contradicting the fact that n is the smallest element such that x < n. Thus x + 1 > n.
Exercise 6. y x > 0.
n(y x) > h, h arbitrary , n Z
+
y > x +h/n = z > x
Since h was arbitrary, there are innitely many numbers in between x, y.
Exercise 7. x =
a
b
Q, y / Q.
x y =
a by
b
If a by was an integer, say m, then y =
_
a mb
b
_
which is rational. Contradiction.
xy =
a
b
y
1
=
ay
b
If ay was an integer, ay = n, y =
n
a
, but y is irrational. =xy is irrational.
x
y
y is not an integer
Exercise 8. Proof by counterexamples. We want that the sum or product of 2 irrational numbers is not always irrational. If y
is irrational, y + 1 is irrational, otherwise, if y + 1 Q, y Q by closure under addition.
=y + 1 y = 1
Likewise, y
1
y
= 1.
Exercise 9.
y x > 0 =n(y x) > k, n Z
+
, k arbitrary. Choose k to be irrational. Then k/n irrational.
y >
k
n
+x > x. Let z = x +
k
n
, z irrational .
Exercise 10.
(1) Suppose n = 2m
1
and n + 1 = 2m
2
.
2m
1
+ 1 = 2m
2
2(m
1
m
2
) = 1 m
1
m
2
=
1
2
. But m
1
m
2
can only be an integer.
(2) By the well-ordering principle, if x Z
+
is neither even and odd, consider the set of all x. There must exist a
smallest element x
0
of this set. But since x
0
Z
+
, then there must exist a n < x such that n + 1 = x
0
. n is even or
odd since it doesnt belong in the above set. So x
0
must be odd or even. Contradiction.
(3)
(2m
1
)(2m
2
) = 2(2m
1
m
2
) even
2m
1
+ 2m
2
= 2(m
1
+m
2
) even
(2m
1
+ 1) + (2m
2
+ 1) = 2(m
1
+m
2
+ 1) = sum of two odd numbers is even
(n
1
+ 1)(n
2
+ 1) = n
1
n
2
+n
1
+n + 2 + 1 = 2(2m
1
m
2
)
2(2m
1
m
2
) (n
1
+n
2
) 1 odd, the product of two odd numbers n
1
, n
2
is odd
(4) If n
2
even, n is even, since for n = 2m, (2m)
2
= 4m
2
= 2(2m
2
) is even.
a
2
= 2b
2
. 2(b
2
) even. a
2
even, so a even.
If a even a = 2n.a
2
= 4n
2
If b odd , b
2
odd. b has no factors of 2 b
2
,= 4n
2
Thus b is even.
8
(5) For
p
q
, If p or q or both are odd, then were done.
Else, when p, q are both even, p = 2
l
m, q = 2
n
p, m, p odd.
p
q
=
2
l
m
2
n
p
=
2
ln
m
p
and at least m or p odd
Exercise 11.
a
b
can be put into a form such that a or b at least is odd by the previous exercise.
However, a
2
= 2b
2
, so a even, b even, by the previous exercise, part (d) or 4th part. Thus
a
b
cannot be rational.
Exercise 12. The set of rational numbers satises the Archimedean property but not the least-upper-bound property.
Since
p
q
Q R, n
p1
q1
>
p2
q2
since if q
1
, q
2
> 0,
np
1
q
2
q
1
q
2
>
q
1
p
2
q
1
q
2
np
1
q
2
> q
1
p
2
n exists since (p
1
q
2
), (q
1
p
2
) R.
The set of rational numbers does not satisfy the least-upper-bound property.
Consider a nonempty set of rational numbers S bounded above so that x =
r
s
S, x < b.
Suppose x < b
1
, x < b
2
x S.
r
s
< b
2
< nb
1
but likewise
r
s
< b
1
< mb
2
, n, m Z
+
So its possible that b
1
> b
2
, but also b
2
> b
1
.
I 4.4 Exercises - An example of a proof by mathematical induction, The principle of mathematical induction, The
well-ordering principle. Consider these 2 proofs.
N +N + +N = N
2
(N 1) + (N 2) + + (N (N 1)) + (N N) = N
2

j=1
j =
N1

j=1
j
N
2
+N = 2
N

j=1
j =
N

j=1
j =
N(N + 1)
2
An interesting property is that
S =
n

j=m
j =
n

j=m
(n +mj)
So that
N

j=1
j =
N

j=m
j +
m

j=1
j =
N

j=m
j +
m(m+ 1)
2
=
N(N + 1)
2
N

j=m
j =
N(N + 1) m(m+ 1)
2
=
(N m)(N +m+ 1)
2
Another way to show this is the following.
S = 1+ 2+ + (N 2)+ (N 1)+ N
but S = N+ N 1+ + 3+ 2+ 1
2S = (N + 1)N S =
N(N + 1)
2
9
Telescoping series will let you get

N
j=1
j
2
and other powers of j.
N

j=1
(2j 1) = 2
N(N + 1)
2
N = N
2
N

j=1
(j
2
(j 1)
2
) =
N

j=1
(j
2
(j
2
2j + 1)) =
N

j=1
(2j 1) = 2
_
N(N + 1)
2
_
N = N
2
N

j=1
(j
3
(j 1)
3
) = N
3
=
N

j=1
(j
3
(j
3
3j
2
+ 3j 1)) =
N

j=1
(3j
2
3j + 1)
=3
N

j=1
j
2
= 3
N(N + 1)
2
+N = N
3
=
2N
3
+ 2N 3N
2
3N
2
=
N(N + 1)(2N + 1)
6
=
N

j=1
j
2
N

j=1
j
4
(j 1)
4
= N
4
=
N

j=1
j
4
(j
4
4j
3
+ 6j
2
4j + 1) =
N

j=1
4j
3
6j
2
+ 4j 1 =
= 4
N

j=1
j
3
6
N(N + 1)(2N + 1)
6
+ 4
N(N + 1)
2
N = N
4
=
N

j=1
j
3
=
1
4
(N
4
+N(N + 1)(2N + 1) 2N(N + 1) +N) =
1
4
(N
4
+ (2N)N(N + 1) N(N + 1) +N)
=
1
4
(N
4
+ 2N
3
+ 2N
2
N
2
N +N) =
1
4
N
2
(N
2
+ 2N + 1) =
1
4
(N(N + 1))
2
2
Exercise 1. Induction proof.
1(1 + 1)
2
N+1

j=1
j =
n

j=1
j +n + 1 =
n(n + 1)
2
+n + 1 =
n(n + 1) + 2(n + 1)
2
=
(n + 2)(n + 1)
2
Exercise 6.
(1)
A(k + 1) = A(k) +k + 1 =
1
8
(2k + 1)
2
+k + 1 =
1
8
(4k
2
+ 4k + 1) +
8k + 8
8
=
(2k + 3)
2
8
(2) The n = 1 case isnt true.
(3)
1 + 2 + +n =
(n + 1)n
2
=
n
2
+n
2
<
n
2
+n +
1
4
2
and
_
2n + 1
2
_
2
1
2
=
(n + 1/2)
2
2
=
n
2
+n + 1/4
2
Exercise 7.
(1 +x)
2
> 1 + 2x + 2x
2
1 + 2x +x
2
> 1 + 2x + 2x
2
0 > x
2
= Impossible
(1 +x)
3
= 1 + 3x + 3x
2
+x
3
> 1 + 3x + 3x
2
=x
3
> 0
By well-ordering principle, we could argue that n = 3 must be the smallest number such that (1 +x)
n
> 1 + 2x + 2x
2
. Or
we could nd, explicitly
(1 +x)
n
=
n

j=0
_
n
j
_
x
j
= 1 +nx +
n(n 1)
2
x
2
+
n

j=3
_
n
j
_
x
j
10
and
n(n 1)
2
> n
n
2
n > 2n
n
2
> 3n
n > 3
Exercise 8.
a
2
ca
1
, a
3
ca
2
c
2
a
1
a
n+1
ca
n
ca
1
c
n1
= a
1
c
n
Exercise 9.
n = 1,

1 = 1
_
1
2
+ 1
2
=

2
_
(

2)
2
+ 1
2
=

3
_
(

n)
2
+ 1
2
=

n + 1
Exercise 10.
1 = qb +r
q = 0, b = 1, r = 1
2 = qb +r, q = 0, r = 2, b = 1, 2 or r = 0, q = 2; q = 1, r = 0
Assume n = qb +r; 0 r < b; b Z
+
, b xed
n + 1 = qb +r + 1 = qb + 1 +r = qb + 1 +b 1 = (q + 1)b + 0
Exercise 11. For n > 1, n = 2, 3 are prime. n = 4 = 2(2), a product of primes.
Assume the k 1th case. Consider
k
j
, 1 j k.
If
k
j
Z
+
, only for j = 1, j = k, then k prime.
If
k
j
Z
+
, for some 1 < j < k,
k
j
= c Z
+
. c, j < k.
Thus k = cj. c, j are products of primes or are primes, by induction hypothesis. Thus k is a product of primes.
Exercise 12. n = 2. G
1
, G
2
are blonde. G
1
has blue eyes. Consider G
2
. G
2
may not have blue eyes. Then G
1
, G
2
are not all
blue-eyed.
I 4.7 Exercises - Proof of the well-ordering principle, The summation notation. Exercise 1.
(1)
n(n+1)
2
=

4
k=1
k = 10
(2)

5
n=2
2
n2
=

3
n=0
2
n
= 1 + 14 = 15
(3) 2

3
r=0
2
2r
= 2

3
r=0
4
r
= 170
(4)

4
j=1
j
j
= 1 + 4 + 27 + 4
4
= 288
(5)

5
j=0
(2j + 1) = 2
5(6)
2
+ 6(1) = 36
(6)

1
k(k+1)
=

n
k=1
_
1
k

1
k+1
_
= 1
1
n+1
=
n
n+1
Exercise 9.
n = 1(1)(3) + 5 = 2 = 2n
n = 2(1)(3) + 5 + (1)7 + 9 = 4 = 2n
n
2n

k=1
(1)
k
(2k + 1) = 2n
n + 1
2(n+1)

k=1
(1)
k
(2k + 1) =
2n

k=1
(1)
k
(2k + 1) + (1)
2n+1
(4n + 3) + (1)
2n+2
(4n + 5) =
= 2n + 2 = 2(n + 1)
Exercise 10.
11
(1) a
m
+a
m+1
+ +a
m+n
(2)
n = 1
1
2
=
1
1

1
2
=
1
2
n + 1
2(n+1)

k=n+2
1
k
=
2n

m=1
(1)
m+1
m

1
n + 1
+
1
2n + 1
+
1
2n + 2
=
2n

m=1
(1)
m+1
m
+
1
2(n + 1)
+
(1)
2n+1+1
(2n + 1)
=
2(n+1)

m=1
(1)
m+1
m
Exercise 13.
n = 12(

2 1) < 1 < 2 since


1
2
>

2 1
n case (

n + 1

n)(

n + 1 +

n) = n + 1 n = 1 <

n + 1 +

n
2

n
=
1
2
(
_
1 +
1
n
+ 1)
n + 1 case (

n + 2

n + 1)(

n + 2 +

n + 1) = n + 2 (n + 1) = 1

n + 2 +

n + 1
2

n + 1
=
1 +
_
1 +
1
n+1
2
> 1
So then, using the telescoping property,
n1

n=1
2(

n + 1

n) = 2(

m1) <
m

n=1
1

n
<
m

n=1
2(

n 1) = 2(

m1) < 2

m1
I 4.9 Exercises - Absolute values and the triangle inequality. Exercise 1.
(1) [x[ = 0iffx = 0
If x = 0, x = 0, x = 0 = 0. If [x[ = 0, x = 0, x = 0.
(2)
[ x[ =
_
x if x 0
x if x 0
=
_
x if x 0
x if x 0
(3) [x y[ = [y x[ by previous exercise and (1)(x y) = y x (by distributivity)
(4) [x[
2
=
_
(x)
2
if x 0
(x)
2
if x 0
= x
2
(5)

x
2
=
_
x if x 0
x if x 0
= [x[
(6) We want to show that [xy[ = [x[[y[
[xy[ =
_
xy if xy 0
xy if xy 0
=
_
xy if x, y 0 or x, y 0
xy if x, y 0 or x, y 0
[x[[y[ =
_
x[y[ if x 0
x[y[ if x 0
=
_

_
xy if x, y 0
xy if x, y 0
xy if x, y 0
xy if x, y 0
(7) By previous exercise, since

x
y

= [xy
1
[ = [x[[y
1
[

1
y

=
_
1
y
if
1
y
0
1
y
if
1
y
0
1
[y[
=
_
1
y
if
1
y
0
1
y
if
1
y
0
12
(8) We know that [a b[ [a c[ +[b c[.
Let c = 0 =[x y[ [x[ +[y[
(9) x = a b, b c = y.
[x[ [x y[ +[ y[ [x[ [y[ [x y[
(10)
[[x[ [y[[ =
_
[x[ [y[ if [x[ [y[ 0
[y[ [x[ if [x[ [y[ 0
[x[ [x y[ +[ y[ =[x[ [y[ [x y[
[y[ [y x[ +[ x[ =[y[ [x[ [y x[ = [x y[
Exercise 4.

If k = 1 . . . n; a
k
x +b
k
= 0
_
n

k=1
a
k
(xa
k
)
_
2
=
_
x
n

k=1
a
2
k
_
2
=
_
n

k=1
a
2
k
__
n

k=1
(xa
k
)
2
_
=
_
n

k=1
a
2
k
__
n

k=1
b
2
k
_
Proving a
k
x +b
k
= 0 means x =
b
k
a
k
, a
k
,= 0
(a
1
b
1
+a
2
b
2
+ +a
n
b
n
)
2
=
n

j=1
a
2
j
b
2
j
+
n

j=q
a
j
a
k
b
j
b
k
==
n

j=1
a
2
j
b
2
j
+
n

j=k
a
2
j
b
2
k
=a
2
j
b
2
k
a
j
a
k
b
j
b
k
= a
j
b
k
(a
j
b
k
a
k
b
j
) = 0
if a
j
, b
k
,= 0, a
j
b
k
a
k
b
j
= 0 =a
k
_
b
j
a
j
_
+b
k
= 0
Exercise 8. The trick of this exercise is the following algebraic trick (multiplication by conjugate) and using telescoping
property of products:
(1 x
2
j
)(1 +x
2
j
) = 1 x
2
j
+2
j
= 1 x
2
j+1
1

j=1
1 +x
2
j1
=
1

j=1
1 x
2
j
1 x
2
j1
=
1 x
2
n
1 x
if x = 1, 2
n
Exercise 10.
x > 1
x
2
> x
x
3
> x
2
> x
x
n+1
= x
n
x > x
2
> x
0 < x < 1
x
2
< x
X
3
< x
2
< x
x
n+1
= x
n
x < x
2
< x =x
n+1
< x
Exercise 11. Let S = n Z
+
[2
n
< n!.
By well-ordering principle, smallest n
0
S. Now
2
4
= 16, 4! = 24. So S starts at n = 4.
Exercise 12.
13
(1)
_
1 +
1
n
_
n
=
n

j=0
_
n
k
__
1
n
_
j
=
n

k=0
n!
(n k)!k!
_
1
n
_
k
k1

r=0
_
1
r
n
_
=
k1

r=0
_
n r
n
_
=
_
1
n
k
_
n!
(n k)!
n

k=1
1
k!
k1

r=0
_
1
r
n
_
=
_
1
n
k
_
n!
(n k)!
(2)
(1 +
1
n
)
n
= 1 +
n

k=1
_
1
k!
k1

r=0
(1
r
n
)
_
< 1 +
n

k=1
1
k!
< 1 +
n

k=1
1
2
k
= 1 +
1
2

_
1
2
_
n+1
1
2
= 1 + (1
_
1
2
_
n
)
< 3
The rst inequality obtained from the fact that if 0 < x < 1, x
n
< x < 1. The second inequality came from the
previous exercise, that
1
k!
<
1
2
k
.
(1 +
1
n
)
n
=
n

k=0
_
n
k
__
1
n
_
k
= 1 +
1
n
+
n1

k=1
_
n
k
__
1
n
_
k
= 1 +
1
n
+
n1

k=2
_
n
k
__
1
n
_
k
+
n
1
_
1
n
_
> > 2
Exercise 13.
(1)
S =
p1

k=0
_
b
a
_
k
=
1
_
b
a
_
p
1
b
a
p1

k=0
b
k
a
p1k
= a
p1
1
_
b
a
_
p
1
b
a
=
b
p
a
p
b a
(2)
(3) Given
n
p
<
(n + 1)
p+1
n
p+1
p + 1
< (n + 1)
p
We want
n1

k=1
k
p
<
n
p+1
p + 1
<
n

k=1
k
p
n = 21
p
<
2
p+1
p + 1
< 1
p
+ 2
p
p = 1
1 < 2
2
/2 = 2.2 < 1 + 2 = 3
p 2
1 < 8/3 < 1 + 4 = 5
I 4.10 Miscellaneous exercises involving induction. Exercise 13.
(1)
(2)
(3) Let n = 2.
21

k=1
k
p
= 1
p
= 1,
n
p+1
p + 1
=
2
p+1
p + 1
2

k=1
k
p
= 1 + 2
p
What makes this exercise hard is that we have to use induction on p itself. Let p = 1.
1 <
2
1+1
1 + 2
= 2 < 1 + 2
1
= 3
14
Now assume pth case. Test the p + 1 case.
2
p+2
p + 2
=
2(p + 1)
p + 2
_
2
p+1
p + 1
_
> 1
since p + 2 < 2p + 2 = 2(p + 1) for p Z
+
For the right-hand inequality, we will use the fact just proven, that 2
p
(p) > 0 and pth case rewritten in this manner
(1 + 2
p
) >
2
p+1
p + 1
=(1 + 2
p
)(p + 1) > 2
p+1
So
(p + 2)(1 + 2
p+1
) = (p + 2) + ((p + 1) + 1)2
p
(2) = (p + 2) + 2(p + 1)2
p
+ 2
p
(2) >
> (p + 2) + 2(2
p+1
(p + 1)) + 2
p
(2) = p + 2
p+2
+ 2
p+1
> 2
p+2
So the n = 2 case is true for all p Z
+
.
Assume nth case is true. We now prove the n + 1 case.
n

k=1
k
p
=
n1

k=1
k
p
+n
p
<
n
p+1
p + 1
+n
p
<
n
p+1
p + 1
+
(n + 1)
p+1
n
p+1
p + 1
=
(n + 1)
p+1
p + 1
n+1

k=1
k
p
=
n

k=1
k
p
+ (n + 1)
p
>
n
p+1
p + 1
+
(n + 1)
p+1
n
p+1
p + 1
=
(n + 1)
p+1
p + 1
We had used the inequality proven in part b, n
p
<
(n+1)
p+1
n
p+1
p+1
< (n + 1)
p
.
Exercise 14. Use induction to prove a general form of Bernoullis inequality.
1 +a
1
= 1 +a
1
(1 +a
1
)(1 +a
2
) = 1 +a
2
+a
1
+a
1
a
2
1 +a
1
+a + 2
Test the n + 1 case
(1 +a
1
)(1 +a
2
) . . . (1 +a
n+1
) (1 +a
1
+a
2
+ +a
n
)(1 +a
n+1
) =
= 1 +a
1
+a
2
+ +a
n
+a
n+1
+a
n+1
(a
1
+a
2
+. . . a
n
)
1 +a
1
+a
2
+ +a
n
+a
n+1
Note that the last step depended upon the given fact that all the numbers were of the same sign.
For a
1
= a
2
= = a
n
= x, then we have (1 +x)
n
1 +nx.
(1 +x)
n
=
n

j=0
_
n
j
_
x
j
= 1 +nx
Since x and n are arbitrary, we can compare terms of x
j
s. Then x = 0.
Exercise 15.
2!
2
2
=
1
2
3!
3
3
=
2
9
< 1.
So weve shown the n = 2, n = 3 cases. Assume the nth case, that
n!
n
n

_
1
2
_
k
, where k is the greatest integer
n
2
.
(n + 1)!
(n + 1)
n+1

(n + 1)n
n
_
1
2
_
k
(n + 1)
n+1
=
_
n
n + 1
_
n
_
1
2
_
k
=
_
1
1
n + 1
_
n
_
1
2
_
k
<
1
2
_
1
2
_
k
=
_
1
2
_
k+1
where in the second to the last step, we had made this important observation:
k
n
2
=k +
1
2

n + 1
2
=
1
n + 1

1
2k + 1
<
1
2
Exercise 16.
15
a
1
= 1 <
1 +

5
2
a
2
= 2 <
_
1 +

5
2
_
2
=
1 + 2

5 + 5
4
=
6 + 2

5
4
a
n+1
= a
n
+a
n1
<
_
1 +

5
2
_
n
+
_
1 +

5
2
_
n1
=
_
1 +

5
2
_
n _
1 +
2
1 +

5
_
=
=
_
1 +

5
2
_
n
_
2(1

5)
1 5
+
4
4
_
=
_
1 +

5
2
_
n+1
Exercise 17. Use Cauchy-Schwarz, which says
_

a
k
b
k
_
2

a
2
k
__

b
2
k
_
Let a
k
= x
p
k
and b
k
= 1. Then Cauchy-Schwarz says
_

x
p
k
_
2

x
2p
k
_
n =

(x
2p
k
)
(

x
p
k
)
2
n
We dene M
p
as follows:
M
p
=
_
n
k=1
x
p
k
n
_
1/p
So then
nM
p
p
=
n

k=1
x
p
k
M
2p
=
_

n
k=1
x
2p
k
n
_
1/2p
nM
2p
2p
=
n

k=1
x
2p
k

x
2p
k
= nM
2p
2p

(nM
p
p
)
2
n
= nM
2p
p
M
2p
2p
M
2p
p
=M
2p
M
p
Exercise 18.
_
a
4
+b
4
+c
4
3
_
1/4

_
a
2
+b
2
+c
2
3
_
1/2
=
2
3/2
3
1/2
since
a
4
+b
4
+c
4

64
3
Exercise 19. a
k
= 1,

n
k=1
1 = n
Now consider the case of when not all a
k
= 1.
a
1
= 1
a
1
a
2
= 1 and suppose, without loss of generality a
1
> 1. Then 1 > a
2
.
(a
1
1)(a
2
1) < 0
a
1
a
2
a
1
a
2
+ 1 < 0 =a
1
+a
2
> 2
(consider n + 1 case ) If a
1
a
2
. . . a
n+1
= 1, then suppose a
1
> 1, a
n+1
< 1 without loss of generality
b
1
= a
1
a
n+1
b
1
a
2
. . . a
n
= 1 =b
1
+a
2
+ +a
n
n (by the induction hypothesis)
(a
1
1)(a
n+1
1) = a
1
a
n+1
a
1
a
n+1
+ 1 < 0, b
1
< a
1
+a
n+1
1
=a
1
+a
n+1
1 +a
2
+ +a
n
> b
1
+a
2
+ +a
n
n
=a
1
+a
2
+ +a
n+1
n + 1
16
1.7 Exercises - The concept of area as a set function. We will use the following axioms:
Assume a class /of measurable sets (i.e. sets that can be assigned an area), set function a, a : /R.

Axiom 2 (Nonnegative property).


(1) S /, a(S) 0

Axiom 3 (Additive property). If S, T /, then S T, S T /and


(2) a(S T) = a(S) +a(T) a(S T)

Axiom 4 (Difference property). If S, T /, S T then T S /and


(3) a(T S) = a(T) a(S)

Axiom 5 (Invariance under congruence). If S /, T = S, then T /, a(T) = a(S)

Axiom 6 (Choice of scale). rectangle R /, if R has edge lengths h, k then a(R) = hk

Axiom 7 (Exhaustion property). Let Q such that


(4) S Q T
If only one c such that a(S) c a(T), S, T such that they satisfy Eqn. (??)
then Q measurable and a(Q) = c
Exercise 1.
(1) We need to say that we consider a line segment or a point to be a special case of a rectangle allowing h or k (or both)
to be zero.
Let T
l
= line segment containing x
0
, Q = x
0
.
For Q, only Q
By Axiom 3, let T = S.
a(T S) = a() = a(T) a(T) = 0
Q T
l
=a() a(Q) a(T
l
) =0 a(Q) 0
=a(Q) = 0
(2)
a
_
_
N
_
j=1
Q
j
_
_
=
N

j=1
a(Q
j
)
if Q
j
s disjoint. Let Q
j
= x
j
.
Since a(Q
j
) = 0. By previous part, a
_

N
j=1
Q
j
_
= 0
Exercise 2. Let A, B be rectangles. By Axiom 5, A, B are measurable. By Axiom 2, A B measurable.
a(A B) =
_
a
2
+b
2
d +ab (
1
2
ab +
_
a
2
+b
2
d) =
1
2
ab
Exercise 3. Prove that every trapezoid and every parallelogram is measurable and derive the usual formulas for their areas.
A trapezoid is simply a rectangle with a right triangle attached to each end of it. T
r
= R + T
1
+ T
2
. T
1
, T
2
are right
triangles and so by the previous problem, T
1
, T
2
are measurable. Then T
r
is measurable by the Additive property axiom (note
that the triangles and the rectangle dont overlap).
17
We can compute the area of a trapezoid:
T
r
= R +T
1
+T
2
=a(T
r
) = a(R) +a(T
1
) +a(T
2
)
a(T
r
) = hb
1
+
1
2
h(b
2
b
1
)/2 +
1
2
h(b
2
b
1
)/2 =
1
2
h(b
1
+h
2
)
P = R (a parallelogram consists of a right triangle rotated by and attached to the other side of the same right triangle;
the two triangles do not overlap). Since two right triangles are measurable, the parallelogram, P is measurable.
Using the Additive Axiom, a(P) = 2a(T) = 2
1
2
bh = bh
Exercise 4. A point (x, y) in the plane is called a lattice point if both coordinates x and y are integers. Let P be a polygon
whose vertices are lattice points. The area of P is I +
1
2
B1, where I denotes the number of lattice points inside the polygon
and B denotes the number on the boundary.
(1) Consider one side of the rectangle lying on a coordinate axis with one end on the origin. If the rectangle side has
length l, then l + 1 lattice points lie on this side (you have to count one more point at the 0 point. Then consider the
same number of lattice points on the opposite side. We have 2(l + 1) lattice points so far, for the boundary.
The other pair of sides will contribute 2(h1) lattice points, the 1 to avoid double counting. Thus 2(l +h) = B.
I = (h 1)(l 1) by simply considering multiplication of (h 1) rows and (l 1) columns of lattice points
inside the rectangle.
I +
1
2
B 1 = hl h l + 1 + (l +h) 1 = hl = a(R)
(2)
(3)
Exercise 5. Prove that a triangle whose vertices are lattice points cannot be equilateral.
My way: I will take, for granted, that we know an equilateral triangle has angles of /3 for all its angles.
Even if we place two of the vertices on lattice points, so that its length is 2L, and put the midpoint and an intersecting
perpendicular bisector on a coordinate axis (a picture would help), but the ratio of the perpendicular bisector to the third
vertex to half the length of the triangle is cot /3 =
1

3
. Even if we go down by an integer number L, L steps down, we go
out to the third vertex by an irrational number

3L. Thus, the third vertex cannot lie on a lattice point.


Exercise 6. Let A = 1, 2, 3, 4, 5 and let /denote the class of all subsets of A. (There are 32 altogether counting A itself
and the empty set ). (My Note: the set of all subsets, in this case, /, is called a power set and is denoted 2
A
. This is
because the way to get the total number of elements of this power set, [2
A
[, or the size, think of assigning to each element a
yes, if its in some subset, or no, if its not. This is a great way of accounting for all possible subsets and we correctly
get all possible subsets.) For each set S in /, let n(S) denote the number of distinct elements in S. If S = 1, 2, 3, 4 and
T = 3, 4, 5,
n(S
_
T) = 5
n(S

T) = 2
n(S T) = n(1, 2) = 2
n(T S) = n(5) = 1
n satises nonnegative property because by denition, theres no such thing as a negative number of elements. If S, T are
subsets of A, so are S

T, S

T since every element in S

T, S

T is in S. Thus n could be assigned to it, so that its


measurable. Since n counts only distinct elements, then n(S

T) = n(S) + a(T) a(S

T), where a(S

T) ensures
there is no double counting of distinct elements. Thus, the Additive Property Axiom is satised.
For S T, then x T S, x T, x / S Now S T, so x S, x T. Thus T S is complementary to S with
respect to T. n(S) +n(T S) = n(T), since n counts up distinct elements.
18
1.11 Exercises - Intervals and ordinate sets, Partitions and step functions, Sum and product of step function. Exercise
4.
(1)
[x +n] = y x +n, y Z; y n x
[x] +n = z +n x +n
If y n < z, then y < z +n x +n. then y wouldnt be the greatest integer less than x +n
=y = z +n
(2)
= y
2
x [x] = y
2
x y
2
1 x
x y
1
= [x] = y
2
1 = [x] 1; ( and y
1
= y
2
1 since y
2
> x )
If x is an integer [x] = [x]
(3) Let x = q
1
+r
1
, y = q
2
+r
2
; 0 r
1
, r
2
< 1.
= [q
1
+q
2
+r
1
+r
2
] =
_
q
1
+q
2
q
1
+q
2
+ 1 if r
1
+r
2
1
[x] + [y] = q
1
+q
2
[x] + [y] + 1 = q
1
+q
2
+ 1
(4)
If x is an integer , [2x] = 2x = [x] + [x +
1
2
] = [x] + [x] = 2x
[x] + [x +
1
2
] = q +
_
q if r <
1
2
2q + 1 if r >
1
2
[2x] = [2(q +r)] = [2q + 2r] =
_
2q if r <
1
2
2q + 1 if r >
1
2
(5)
[x] + [x +
1
3
] + [x +
2
3
] = q +
_
q if r <
2
3
q + 1 if r >
2
3
+
_
q if r <
1
3
q + 1 if r >
1
3
=
_

_
3q if r <
1
3
3q + 1 if
1
3
< r <
2
3
3q + 2 if r >
2
3
[3x] = [3(q +r)] = [3q + 3r] =
_

_
3q if r <
1
3
3q + 1 if
1
3
< r <
2
3
3q + 2 if r >
2
3
Exercise 5. Direct proof.
[nx] = [n(q +r)] =
_

_
nq if r <
1
n
nq + 1 if
1
n
< r <
2
n
nq +n 1 if r >
n1
n
Exercise 6.
a(R) = hk = I
R
+
1
2
B
R
1
b

n=a
[f(n)] = [f(a)] + [f(a + 1)] + + [f(b)]
[f(n)] = g f(n), g Z, so that if f(n) is an integer,g = f(n), and if f(n) is not an integer, g is the largest integer such
that g < f(n), so that all lattice points included and less than g are included.
Exercise 7.
19
(1) Consider a right triangle with lattice points as vertices. Consider b + 1 lattice points as the base with b length.
Start from the vertex and move across the base by increments of 1.
The main insight is that the slope of the hypotenuse of the right triangle is
a
b
so as we move 1 along the base, the
hypotenuse (or the y-value, if you will) goes up by
a
b
. Now
(5)
_
na
b
_
= number of interior points at x = n and below the hypotenuse line of the right triangle of sides a, b,
including points on the hypotenuse
b1

n=1
_
na
b
_
+
1
2
((a + 1) +b) 1 =
ab
2
Now
(a 1)(b 1)
2
=
ab
2

a
2

b
2
+
1
2
=
b1

n=1
_
na
b
_
=
(a 1)(b 1)
2
(2) a, b Z
+
b1

n=1
_
na
b
_
=
b1

n=1
_
a(b n)
b
_
(reverses order of summation)
b1

n=1
_
a
an
b
_
=
_

b1
n=1
_
an
b
a

if
an
b
a4 is an integer (but a
_
n
b
1
_
cant be!)

b1
n=1
__
an
b
a

1
_
otherwise
=
b1

n=1
__
an
b
a
_
1
_
=
b1

n=1
__
an
b
_
a
_
(b 1) =
=
b1

n=1
_
an
b
_
+a(b 1) (b 1)
b1

n=1
_
na
b
_
=
(a 1)(b 1)
2
Exercise 8. Recall that for the step function f = f(x), theres a partition P = x
0
, x
1
, . . . , x
n
of [a, b] such that f(x) = c
k
if x I
k
.
Given that
s
(x) =
_
1 x S
0 x / S
.
If x [a, b], then x must only lie in one open subinterval I
j
, since real numbers obey transitivity.
n

k=1
c
k

I
k
(x) = c
j
for x I
j
=
n

k=1
c
k

I
k
(x) = f(x)x [a, b]
1.15 Exercises - The denition of the integral for step functions, Properties of the integral of a step function, Other
notations for integrals. Exercise 1.
(1)
_
3
1
[x]dx = (1) + 1 + (2) = 2
(2)
_
3
1
[x +
1
2
]dx =
_
7/2
1/2
[x]dx = (1)
1
2
+ (1)(1) + (2)(1) +
1
2
3 = 4
(3)
_
3
1
([x] + [x +
1
2
])dx = 6
(4)
_
3
1
2[x]dx = 4
(5)
_
3
1
[2x]dx =
1
2
_
6
2
[x]dx =
1
2
((2)1 + (1) + (1) + 2 + 3 + 4 + 5) = 6
(6)
_
3
1
[x]dx =
_
3
1
[x]dx =
_
1
3
[x]dx = 3 +2 +1 = 6
Exercise 2.
s =
_
5/2 if 0 < x < 2
1 if 2 < x < 5
20
Exercise 3. [x] = y x so y x.
y 1 x, otherwise if y 1 x, y + 1 x and so y wouldnt be the largest integer x.
=[x] + [x] = y y 1 = 1
Or use Exercise 4(c), pp. 64.
_
b
a
([x] + [x])dx =
_
b
a
[x x]dx =
_
b
a
(1)dx = a b
Exercise 4.
(1) n Z
+
,
_
n
0
[t]dt =

n1
t=0
t =
(n1)(n1+1)
2
=
(n1)n
2
(2)
Exercise 5.
(1)
_
2
0
[t
2
]dt =
_
2
1
[t
2
]dt = 1(

2 1) + 2(

2) + 3(2

3) = 5

3
(2)
_
3
3
[t
2
]dt =
_
3
0
[t
2
]dt +
_
0
3
[t
2
]dt =
_
3
0
[t
2
]dt +
_
0
3
[t
2
]dt = 2
_
3
0
[t
2
]dt
_
3
2
[t
2
]dt = 4(

5 2) + 5(

5) + 6(

6) + 7(

7) + 8(3

8)
16

8
_
2
0
[t
2
]dt +
_
3
2
[t
2
]dt = 21 3

8
=
_
3
3
[t
2
]dt = 42 2(3

2 +

3 +

5 +

6 +

7)
Exercise 6.
(1)
_
n
0
[t]
2
dt =
_
n
1
[t]
2
dt =

n1
j=1
j
2
=
(n1)n(2n1)
6
(2)
_
x
0
[t]
2
dt =

[x1]
j=1
j
2
+q
2
r where x = q +r, q Z
+
, 0 r < 1.
_
x
0
[t]
2
dt =
q(q 1)(2q 1)
6
+q
2
r = 2(x 1) = 2(q +r 1)
=q(q 1)(2q 1) + 6q
2
r = 12q + 12r 12
=x = 1, x = 5/2
Exercise 7.
(1)
_
9
0
[

t]dt =
_
9
1
[

t]dt = 3(1) + 5(2) = 13


_
1
0
6[

t]dt = 3(1) + 5(2) + 7(3) = 34 =


(4)(3)(17)
6
Assume
_
n
2
0
[

t]dt =
n(n 1)(4n + 1)
6
_
(n+1)
2
0
[

t]dt =
_
n
2
0
[

t]dt +
_
(n+1)
2
n
2
[

t]dt =
n(n 1)(4n + 1)
6
+n((n + 1)
2
n
2
) =
=
(n
2
n)(4n + 1) + 6n(2n + 1)
6
=
4n
3
+n
2
4n
2
n + 12n
2
+ 6n
6
=
4n
3
+ 9n
2
+ 5n
6
indeed ,
(n + 1)(n)(4(n + 1) + 1)
6
=
(n
2
+n)(4n + 5)
6
=
4n
3
+ 5n
2
+ 4n
2
+ 5n
6
21
Exercise 8.
_
b+c
a+c
f(x)dx =
_
b+cc
a+cc
f(x (c))dx =
_
b
a
f(x +c)dx
Exercise 9.
_
kb
ka
f(x)dx =
1
1
k
_
(kb)/k
(ka)/k
f
_
x
1/k
_
dx = k
_
b
a
f(kx)dx
Exercise 10. Given s(x) = (1)
n
n if n x < n + 1; n = 0, 1, 2, . . . p 1; s(p) = 0, p Z
+
. f(p) =
_
p
0
s(x)dx.
So for f(3) =
_
3
0
s(x)dx, we need to consider n = 0, 1, 2.
s(0 x < 1) = 0
s(1 x < 2) = (1)(1)
s(2 x < 3) = 2;
s(3 x < 4) = 3
So then
f(3) = (1)(1) + 2(1) = 1
f(4) = 1 + (3)(1) = 2
f(f(3)) = f(1) = 0
We obtain this formula
f(p) =
_
p
2
(1)
p+1
p even
p1
2
(1)
p+1
p even
since
f(p + 1) = f(p) +
_
p+1
p
s(x)dx =
_
p1
2
(1)
p+1
p even + (1)
p
p
=
_
p
2
p even
p1
2
p odd
+
_
p
p
=
_
p
2
p1
2
=
=
_

(p+1)
2
if p + 1 even
p
2
if p + 1 odd
Thus, p = 14, p = 15.
Exercise 11.
(1)
_
b
a
s(x)dx =
n

k=1
s
3
k
(x
k
x
k1
)
_
b
a
s +
_
c
b
s =
n1

k=1
s
2
k
(x
k
x
k1
) +
n2

k=n1
s
3
k
(x
k
x
k1
) =
n2

k=1
s
3
k
(x
k
x
k1
) =
_
c
a
s(x)dx
(2)
_
b
a
(s +t) =

n3
k=1
(s +t)
3
k
(x
k
x
k1
) ,=
_
b
a
s +
_
b
a
t
(3)
_
b
a
cs =

n
k=1
(cs)
3
(x
k
x
k1
) ,= c
_
b
a
s
(4) Consider these facts that are true, that x
k1
< x < x
k
, s(x) = s
k
; x
0
= a +c, x
n
= b +c,
x
k1
c < x c < x
l
c =y
k1
< y < y
k
so then s(y +c) = s
k
.
n

k=1
s
3
k
(x
k
x
k1
) =
k

k=1
s
3
k
(x
k
c (x
k1
c)) =
=
n

k=1
s
3
k
(y
k
y
k1
) =
_
b
a
s(y +c)dy
(5) s < t,
_
b
a
s =

n
k=1
s
3
k
(x
k
x
k1
).
if 0 < s, s
3
< s
2
t < st
2
< t
3
if s < 0t, s
3
< and t
3
> 0
if s < t < 0, s
3
< s
2
t, s(st) < t(ts) = t
2
s
ts > t
2
t
2
s < t
3
s
3
< s
2
t < t
2
s < t
3
22
Then
_
b
a
s <
_
b
a
t.
Exercise 12.
(1)
_
b
a
s +
_
c
b
s =

n1
k=1
s
k
(x
2
k
x
2
k1
) +

n2
k=n1
s
k
(x
2
k
x
2
k1
) =

n3
k=1
s
k
(x
2
k
x
2
k1
) =
_
c
a
s
(2)
_
b
a
(s+t) =

n3
k=1
(s+t)
k
(x
2
k
x
2
k1
) =

n3
k=1
(s
k
+t
k
)(x
2
k
x
2
k1
) =

n3
k=1
s
k
(x
2
k
x
2
k1
)+

n3
k=1
t
k
(x
2
k
x
2
k1
)
since P
3
= x
k
is a ner partition than the partition for s, P
1
, t, P
2
, then consider
s
k
(y
2
j
y
2
j1
) = s
k
((x
2
k+1
x
2
k
) + (x
2
k
x
2
k1
)), so
n3

k=1
s
k
(x
2
k
x k 1
2
) +
n3

k=1
t
k
(x
2
k
x k 1
2
) =
n1

j=1
s
j
(x
2
j
x j 1
2
) +
n2

j=1
t
j
(x
2
j
x j 1
2
) =
=
_
b
a
s +
_
b
a
t
(3)
_
b
a
cs =

n
k=1
cs
k
(x
2
k
x
2
k1
) = c

n
k=1
s
k
(x
2
k
x
2
k1
) = c
_
b
a
s
(4)
_
b+c
a+c
s(x)dx =

n
k=1
s
k
(x
2
k
x
2
k1
) where
s(x) = s
k
if x
k1
< x < x
k
x(y +c) = s
k
if x
k1
< y +c < x
k
=x
k1
c < y < x
k
c =y
k1
< y < y
k
where P

= y
k
is a partition on [a, b]
_
b
a
s(y +c)dy =
n

k=1
s
k
(y
2
k
y
2
k1
) =
=
n

k=1
s
k
((x
k
c)
2
(x
k1
c)
2
) =
n

k=1
s
k
(x
2
k
2x
k
c +c
2
(x
2
k1
2x
k1
c +c
2
)) =
=
n

k=1
s
k
(x
2
k
x
2
k1
2c(x
k
x
k1
)) ,=
n

k=1
s
k
(x
2
k
x
2
k1
)
(5) Since x
2
k
x
2
k1
> 0,
_
b
a
sdx =

n
k=1
s
k
(x
2
k
x
2
k1
) <

n
k=1
t
k
(x
2
k
x
2
k1
) =
_
b
a
tdx
Note that we had shown previously that the integral doesnt change under ner partition.
Exercise 13.
_
b
a
s(x)dx
n

k=1
s
k
(x
k
x
k1
);
_
b
a
t(x)dx =
n2

k=1
t
k
(y
k
y
k1
)
P = x
0
, x
1
, . . . , x
n
, Q = y
0
, y
1
, . . . , y
n

Note that x
0
= y
0
= a; x
n
= y
n2
= b.
Consider P

Q = R. R consists of n
3
elements, (since n
3
n + n
2
some elements of P and Q may be the same. R is
another partition on [a, b] (by partition denition) since x
k
, y
k
R and since real numbers obey transitivity, x
k
, y
k
can be
arranged such that a < z
1
< z
2
< < z
n32
< b where z
k
= x
k
or y
k
.
(s +t)(x) = s(x) +t(x) = s
j
+t
k
if x
j1
< x < x
j
; y
j1
< x < y
j
If x
j1
y
j1
, let z
l1
= y
j1
, x
j1
and
If x
j
y
j
, let z
l
= x
j
, y
j
Let s
j
= s
l
; t
k
= t
l
(s +t)(x) = s(x) +t(x) = s
l
+t
l
, if z
l1
< x < z
l
_
b
a
(s(x) +t(x))dx =
_
b
a
((s +t)(x))dx =
n3

l=1
(s
l
+t)l)(z
l
z
l1
) =
n3

l=1
s
l
(z
l
z
l1
) +
n3

l=1
t
l
(z
l
z
l1
)
In general, it was shown (Apostol I, pp. 66) that any ner partition doesnt change the integral R is a ner partition. So
n3

l=1
s
l
(z
l
z
l1
) +
n

l=1
t
l
(z
l
z
l1
) =
n

k=1
s
k
(x
k
x
k1
) +
n2

k=1
t
k
(y
k
y
k1
) =
_
b
a
s(x)dx +
_
b
a
t(x)dx
23
Exercise 14. Prove Theorem 1.4 (the linearity property).
c
1
_
b
a
s(x)dx +c
2
_
b
a
t(x)dx = c
1
n

k=1
s
k
(x
k
x
k1
) +c
2
n2

k=1
t
k
(x
k
x
k1
) =
=
n3

l=1
c
1
s
l
(z
l
z
l1
) +
n3

l=1
c
2
t
l
(z
l
z
l1
) =
n3

l=1
(c
1
s
l
+c
2
t
l
)(z
l
z
l1
) =
=
_
b
a
(c
1
s +c
2
t)(x)dx
We relied on the fact that we could dene a ner partition from two partitions of the same interval.
Exercise 15. Prove Theorem 1.5 (the comparison theorem).
s(x) < t(x) x [a, b]; s(x)(z
l
z
l1
) < t(x)(z
l
z
l1
) (z
l
z
l1
> 0)
_
b
a
s(x)dx =
n

k=1
s
k
(x
k
x
k1
) =
n3

l=1
s
l
(z
l
z
l1
) <
n3

l=1
t
l
(z
l
z
l1
) =
n2

k=1
t
k
(y
l
y
k1
) =
=
_
b
a
t(x)dx
=
_
b
a
s(x)dx <
_
b
a
t(x)dx
Exercise 16. Prove Theorem 1.6 (additivity with respect to the interval).
Use the hint: P
1
is a partition of [a, c], P
2
is a partition of [c, b], then the points of P
1
along with those of P
2
form a
partition of [a, b].
_
c
a
s(x)dx +
_
b
a
s(x)dx =
n1

k=1
s
l
(x
k
x
k1
) +
n2

k=1
s
k
(x
k
x
k1
) =
n3

k=1
s
k
(x
k
x
k1
) =
_
b
a
s(x)dx
Exercise 17. Prove Theorem 1.7 (invariance under translation).
P

= y
0
, y
1
, . . . , y
n
; y
k
= x
k
+c;
=x
k1
+c < y < x
k
+c
x
k1
< y c < x
k
y
k
y
k1
= x
k
+c (x
k1
+c) = x
k
x
k1
s(y c) = s
k
if x
k1
< y c < x
k
, k = 1, 2, . . . n
_
b
a
s(x)dx =
n

k=1
s
k
(x
k
x
k1
) =
n

k=1
s
k
(y
k
= y
k1
) =
_
yn
y0
s(y c)dy =
_
b+c
a+c
s(x c)dx
1.26 Exercises - The integral of more general functions, Upper and lower integrals, The area of an ordinate set ex-
pressed as an integral, Informal remarks on the theory and technique of integration, Monotonic and piecewise mono-
tonic functions. Denitions and examples, Integrability of bounded monotonic functions, Calculation of the integral
of a bounded monotonic function, Calculation of the integral
_
b
0
x
p
dx when p is a positive integer, The basic properties
of the integral, Integration of polynomials.
Exercise 16.
_
2
0
[(x 1)(3x 1)[dx =
24
_
2
1
(x 1)(3x 1)dx =
_
2
1
(3x
2
4x + 1)dx = (x
3
2x
2
+x)

2
1
= 2
_
1
1/3
(1 x)(3x 1)dx = (x
3
2x
2
+x)

1
1/3
=
4
27
_
1/3
0
(x 1)(3x 1)dx =
4
27
So the nal answer for the integral is 62/27.
Exercise 17.
_
3
0
(2x 5)
3
dx = 8
_
3
0
(x
5
2
)
3
dx = 8
_
35/2
5/2
x
3
dx = 8
1
4
x
4

1/2
5/2
=
39
2
Exercise 18.
_
3
3
(x
2
3)
3
dx =
_
3
0
(x
2
3)
3
+
_
x
3
(x
2
3)
3
=
_
3
0
(x
2
3)
2
+
_
3
0
(x
2
3)
3
= 0
2.4 Exercises - Introduction, The area of a region between two graphs expressed as an integral, Worked examples.
Exercise 15. f = x
2
, g = cx
3
, c > 0
For 0 < x <
1
c
, cx < 1 (since c > 0). So cx
3
< x
2
(since x
2
> 0).
_
f g =
_
x
2
cx
3
=
_
1
3
x
3

c
4
x
4
_

1/c
0
=
1
12c
3
_
f g =
2
3
=
1
12c
3
; c =
1
2

2
Exercise 16. f = x(1 x), g = ax.
_
f g =
_
1a
0
x x
2
ax =
_
(1 a)
1
2
x
2

1
3
x
3
_

1a
0
= (1 a)
3
1
6
= 9/2 = a = 2
Exercise 17. = 2
_
1
1

1 x
2
dx
(1)
_
3
3
_
9 x
2
dx = 3
_
3
3
_
1
_
x
3
_
2
= 3(3)
_
1
1
_
1 x
2
=
9
2
Now
_
kb
ka
f
_
x
k
_
dx = k
_
b
a
fdx
(2)
_
2
0
_
1
1
4
x
2
dx = 2
_
1
0
_
1 x
2
dx =
2
4
=

2
(3)
_
2
2
(x 3)

4 x
2
dx
_
2
2
x
_
4 x
2
dx = (1)
_
2
2
x
_
4 x
2
=2
_
2
2
x
_
4 x
2
= 0
3
_
2
2
2
_
1
_
x
2
_
2
dx = (6)(2)
_
1
1
_
1 x
2
= 6
Exercise 18. Consider a circle of radius 1 and a twelve-sided dodecagon inscribed in it. Divide the dodecagon by isosceles
triangle pie slices. The interior angle that is the vertex angle of these triangles is 360/12 = 30 degrees.
Then the length of the bottom side of each triangle is given by the law of cosines:
c
2
= 1 + 1 2(1)(1) cos 30

= 2
_
1

3
2
_
=c =

3
2
25
The height is given also by the law of cosines
h = 1 cos 15

=
_
1 + cos 30

2
=

1 +

3
2
2
The area of the dodecagon is given by adding up twelve of those isosceles triangles
(12)
1
2
_
_

1 +

3
2
_
1

2
_
_
_
_
_

3
2
_
_
= 3
So 3 < .
Now consider a dodecagon thats circumscribing the circle of radius 1.
(12)
1
2
_
_
2

_
1

3
2
1 +

3
2
_
_
(1) = 12
_
2

3
2
_
>
Exercise 19.
(1) (x, y) E if x = ax
1
, y = by
1
such that x
2
1
+y
2
1
1
=
_
x
a
_
2
+
_
y
b
_
2
= 1
(2)
y = b
_
1
_
x
a
_
2
2
_
a
a
b
_
1
_
x
a
_
2
= 2ba
_
1
1
_
1 x
2
= ba

2
(2) = ba
Exercise 20. Let f be nonnegative and integrable on [a, b] and let S be its ordinate set.
Suppose x and y coordinates of S were expanded in different ways x = k
1
x
1
, y = k
2
y
1
.
If f(x
1
) = y
1
, g(x) = k
2
f
_
x
k1
_
= k
2
y
1
= y.
integrating g on [k
1
a, k
1
b],
_
k1b
k1a
g(x)dx =
_
k1b
k1a
k
2
f
_
x
k
1
_
dx = k
2
k
1
_
b
a
f(x)dx = k
2
k
1
A
2.8 Exercises - The trigonometric functions, Integration formulas for the sine and cosine, A geometric description of
the sine and cosine functions. Exercise 1.
(1) sin = sin 0 = 0. sine is periodic by 2, so by induction, sin n = 0.
sin 2(n + 1) = sin 2n + 2 = sin 2n = 0
sin (2(n + 1) + 1) = sin (2n + 3) = sin ((2n + 1) + 2) = sin (2n + 1) = 0
(2) cos /2 = cos /2 = 0
by induction, cos /2 + 2j = cos /2(1 + 4j)
cos /2 + 2j = cos (4j 1)/2, j Z
+
Exercise 2.
(1) sin /2 = 1, sin /2(1 + 4j) = 1, j Z
+
.
(2) cos x = 1, cos 0 = 1, cos 2j = 1
Exercise 3.
sin x + = sin x +/2 +/2 = cos x +/2 = sin x
cos x + = cos x +/2 +/2 = sin x +/2 = cos x
Exercise 4.
26
sin 3x = sin 2xcos x + sin xcos 2x = 2 sin xcos
2
x + sin x(cos
2
x sin
2
x) = 3 cos
2
xsin x sin
3
x =
= 3(1 sin
2
x) sin x sin
3
x = 3 sin x 4 sin
3
x
cos 3x = cos 2xcos x sin 2xsin x = (cos
2
x sin
2
x) cos x (2 sin xcos x) sin x = cos x 4 sin
2
xcos x
cos 3x = 3 cos x + 4 cos
3
x
Exercise 5.
(1) This is the most direct solution. Using results from Exercise 4 (and it really helps to choose the cosine relationship,
not the sine relationship),
cos 3x = 4 cos
3
x 3 cos x
x = /6
cos 3/6 = 0 = 4 cos
3
/6 3 cos /6 = cos /6(4 cos
2
/6 3) = 0
=cos /6 =

3/2, sin /6 = 1/2( by Pythagorean theorem )


(2) sin 2/6 = 2 cos /6 sin /6 =

32, cos /3 = 1/2 (by Pythagorean theorem)


(3) cos2/4 = 0 = 2 cos /4 1, cos /4 = 1/

2 = sin /4
Note that the most general way to solve a cubic is to use this formula. For x
3
+bx
2
+cx +d = 0,
R =
9bc 27d 2b
3
54
Q =
3c b
2
9
S = (R +
_
Q
3
+R
2
)
1/3
T = (R
_
Q
3
+R
2
)
1/3
x
1
= S +T b/3
x
2
= 1/2(S +T) b/3 + 1/2

3(S T)
x
3
= 1/2(S +T) b/3 1/2

3(S T)
Exercise 6.
tan x y =
sin x y
cos x y
=
sin xcos y sin y cos x
cos xcos y + sin xsin y
_
1
cos x cos y
1
cos x cos y
_
=
tan x tan y
1 + tan xtan y
if tan xtan y ,= 1
Similarly,
tan x +y =
sin x +y
cos x +y
=
sin xcos y + sin y cos x
cos xcos y sin xsin y
=
tan x + tan y
1 tan xtan y
, tan xtan y ,= 1
cot x +y =
cos x +y
sin x +y
=
cos xcos y sin xsin y
sin xcos y + sin y cos x
=
cot xcot y 1
cot y + cot x
Exercise 7. 3 sin x +/3 = Asin x +Bcos x = 3(sin x
1
2
+

3
2
cos x) =
3
2
sin x +
3

3
2
cos x
Exercise 8.
C sin x + = C(sin xcos + cos xsin ) = C cos sin x +C sin cos x
A = C cos , B = C sin
Exercise 9. If A = 0, Bcos x = Bsin /2 +x = C sin x + so C = B, = /2 if A = 0.
If A ,= 0,
Asin x +Bcos x = A(sin x +
B
A
cos x) == A(sin x + tan cos x)
=
A
cos
(cos sin x + sin cos x) =
A
cos
(sin x +)
where /2 < < /4, B/A = tan , C =
A
cos
Exercise 10. C sin x + = C sin xcos +C cos xsin .
C cos = 2, C sin = 2, C = 2

2, = /4
Exercise 11. If A = 0, C = B, = 0. If B = 0, A = C, = /2. Otherwise,
27
Asin x +Bcos x = B(cos x +
A
B
sin x) =
B
cos
(cos xcos + sin sin x) = C cos x +
where
A
B
= tan , = , C =
B
cos
.
Exercise 12.
sin x = cos x =
_
1 cos
2
x =cos x = 1/

2 =x =

4
Try 5/4. sin 5/4 = cos 3/4 = sin /4 = 1/

2.
cos 5/4 = sin 3/4 = cos /4 = 1/

2. So sin 5/4 = cos 5/4. x = 5/4 must be the other root.


So = /4 +n (by periodicity of sine and cosine).
Exercise 13.
sin x cos x = 1 =
_
1 cos
2
x = 1 + cos x
=1 cos
2
x = 1 + 2 cos x + cos
2
x =0 = 2 cos x(1 + cos x)
cos x = 1, x = /2 + 2n
Exercise 14.
cos x y + cos x +y = cos xcos y + sin xsin y + cos xcos y sin xsin y = 2 cos xcos y
cos x y cos x +y = sin xcos y sin y cos x + sin xcos y + sin y cos x = 2 sin xcos y
sin x y + sin x +y = sin xcos y sin y cos x + sin xcos y + sin y cos x = 2 sin xcos y
Exercise 15.
sin x +h sin x
h
=
sin (x +h/2) cos h/2 + cos (x +h/2) sin h/2 sin (x +h) cos h/2 cos x +h/2 sin h/2
h
=
sin h/2
h/2
cos (x +h/2)
cos x +h cos x
h
=
cos (x +h/2) cos h/2 sin (x +h/2) sin h/2 (cos (x +h/2) cos h/2 + sin (x +h/2) sin h/2)
h
=
sin h/2
h/2
sin (x +h/2)
Exercise 16.
(1)
sin 2x = 2 sin xcos x
if sin 2x = 2 sin x and x ,= 0, x ,= n, cos x = 1 but x ,= n = x = 2n
(2) cos x +y = cos xcos y sin xsin y = cos x + cos y.
cos xcos y cos x cos y = sin y
_
1 cos
2
x
Letting A = cos x, B = cos y,
A
2
B
2
+A
2
+B
2
2A
2
B 2AB
2
+ 2AB = 1 A
2
B
2
+A
2
B
2
A
2
+B
2
A
2
B AB
2
+AB = 1/2
B
2
(1 A) +B(AA
2
) +A
2
1/2 = 0
B =
A(1 A)
_
A
2
(1 A)
2
4(1 A)(A
2
1/2)
1 A
= A
1

1 A
(A
2
(1 A) 4(A
2
1/2))
1/2
=
= A
1

1 A
(3A
2
A
3
+ 2)
1/2
Note that 1 B 1, but for [A[ 1.
Solve for the roots of 3A
2
A
3
+ 2, A
0
= 1, 1 +

3, 1

3. So suppose cos x = 9/10. Then there is


no real number for y such that cos y would be real and satisfy the above equation.
(3) sin x +y = sin xcos y + sin y cos x = sin x + sin y
=sin y(1 cos x) + sin y +cos xsin y = 0, =y = 2n
Checking our result, we nd that sin (2n +y) = sin 2n + sin y(1)
28
(4)
_
y
0
sin xdx = cos x[
y
0
= (cos y 1) = 1 cos y = sin y
=1 cos y =
_
1 cos
2
y
1 2 cos y + cos
2
y = 1 cos
2
y =cos y(cos y 1) = 0; y =
2(j + 1)
2
, 2n
Exercise 17.
_
b
a
sin xdx = cos x[
b
a
= cos b + cos a
(1)

3
2
+ 1
(2)

2
2
+ 1
(3)
1
2
(4) 1
(5) 2
(6) 0 We were integrating over one period, over one positive semicircle and over one negative semicircle.
(7) 0 We had integrated over two equal parts, though it only shaded in up to x = 1.
(8)

2
2
+

3
2
Exercise 18.
_

0
(x + sin x)dx = (
1
2
x
2
cos x)

0
=

2
2
(1 1) =

2
2
+ 2
Exercise 19.
_
/2
0
(x
2
+ cos x)dx = (
1
3
x
3
+ sin x)

/2
0
=
1
3
(/2)
3
+ 1
Exercise 20.
_
/2
0
(sin x cos x)dx = (cos x sin x)[
/2
0
= 1 (1) = 0
Exercise 21.
_
/2
0
[ sin x cos x[dx = ( by symmetry )2
_
/4
0
(cos x sin x)dx = 2(sin x + cos x)[
/4
0
= 2(

2 1)
Exercise 22.
_

0
(
1
2
+ cos t)dt = (
1
2
t + sin t)

0
=

2
Exercise 23.
_
2/3
0
(
1
2
+ cos t)dt +
_

2/3
(
1
2
+ cos t)dt = (
t
2
+ sin t)

2/3
0
+ (
t
2
+ sin t)

2/3

= 2(

3
+

3
2
)

2
=

6
+

3
Exercise 24. If < x
2
3
,
_
x

(
1
2
+ cos t)dt =
_

x
(
1
2
+ cos t)dt =
_
t
2
+ sin t
_

x
=

2

x
2
sin x
If 2/3 x 2/3,
_
2/3

(
1
2
+ cos t)dt +
_
x
2/3
(
1
2
+ cos t)dt =
/6
+

3/2 + (t/2 + sin t)[


x
2/3
= x/2 + sin x /3

3/2 +

3/2 /6 =
x
2
+ sin x /3
If 2/3 x ,

3/2 +
_
x
2/3
(1/2 + cos t)dt =

3/2 + (t/2 + sin t)[


2/3
x
= /3 +

3 x/2 sin x
Exercise 25.
_
x
2
x
(t
2
+ sin t)dt = (
1
3
t
3
+cos t) =
x
6
x
3
3
+ cos x cos x
2
Exercise 26.
_
/2
0
sin 2xdx =
_
cos (2x)
2
_

/2
0
= (1/2)(1 1) = 1
Exercise 27.
_
/3
0
cos x/2dx = 2 sin x/2[
/3
0
= 2
1
2
= 1
Exercise 28.
29
_
x
0
cos (a +bt)dt =
_
x
0
(cos a cos bt sin a sin bt)dt =
_
cos a
b
sin bt sin a(cos bt/b)
_

x
0
=
=
cos a
b
sin bx +
sin a
b
(cos bx 1) =
1
b
sin a +bx sin a/b
_
x
0
sin (a +bt)dt =
_
x
0
(sin a cos bt + sin bt cos a)dt =
_
sin a
b
sin bt
cos a
b
cos bt
_

x
0
=
=
1
b
(cos bx +a + cos a)
Exercise 29.
(1)
_
x
0
sin
3
tdt =
_
x
0
3 sin t sin 3t
4
dt =
_

3
4
cos t + cos 3t/12
_

x
0
= 3/4(cos x 1) +
cos 3x 1
12
=
=
1
3

3
4
cos x +
1
12
(cos 2xcos x sin 2xsin x) = 2/3 1/3 cos x(2 + sin
2
x)
(2)
_
x
0
cos
3
tdt =
_
x
0
1
4
(cos 3t + 3 cos t)dt =
_
1
4
sin 3t
3
+
3
4
sin t
_

x
0
=
=
1
12
(sin 2xcos x + sin xcos 2x) +
3
4
sin x =
1
12
(2 sin xcos x + sin x(2 cos
2
x 1)) =
=
sin xcos
2
x + 2 sin x
3
Exercise 30. Now using the denition of a periodic function,
f(x) = f(x +p); f(x + (n + 1)p) = f(x +np +p) = f(x +np) = f(x)
and knowing that we could write any real number in the following form,
a = np +r; 0 < p, r R; n Z
then
_
a+p
a
f(x)dx =
_
r+p
r
f(x +np)dx =
_
r+p
r
f(x)dx =
_
p
r
f +
_
r+p
p
f(x)dx =
=
_
p
r
f +
_
r
0
f(x p)dx =
_
p
r
f +
_
r
0
f =
_
p
0
f
Exercise 31.
(1)
_
2
0
sin nxdx =
1
n
_
2n
0
sin xdx =
1
n
(cos x)

2n
0
=
1
n
(1 1) = 0
_
2
0
cos nxdx =
1
n
_
2n
0
cos xdx =
1
n
sin x

2n
0
= 0
(2)
_
2
0
sin nxcos mxdx =
_
2
0
1
2
(sin (n +m)x + sin (n m)x)dx = 0 + 0 = 0
_
2
0
sin nxsin mxdx =
_
2
0
1
2
(cos (n m)x + cos (n +m)x)dx = 0 + 0 = 0
_
2
0
cos nxcos mxdx =
_
2
0
1
2
(cos (n m)x + cos (n +m)x)dx = 0 + 0 = 0
While
_
2
0
sin
2
nxdx =
_
2
0
1 cos 2nx
2
dx =
_
2
0
cos
2
nxdx =
_
2
0
1 + cos 2nx
2
dx =
30
Exercise 32. Given that x ,= 2n; sin x/2 ,= 0,
n

k=1
2 sin x/2 cos kx = 2 sin x/2
n

k=1
cos kx =
n

k=1
sin (2k + 1)
x
2
sin (2k 1)
x
2
= sin (2n + 1)
x
2
sin x/2
= sin nxcos x/2 + sin x/2 cos nx sin x/2 =
= 2 sin nx/2 cos nx/2 cos x/2 + sin x/2(1 2 sin
2
nx/2) sin x/2 =
= 2(sin nx/2)(cos (n + 1)x/2)
Exercise 33. Recall that
cos (2k + 1)x/2 cos (2k 1)x/2 = cos kx +x/2 cos kx x/2 =
= cos kxcos x/2 sin kxsin x/2 (cos kxcos x/2 + sin kxsin x/2) =
= 2 sin kxsin x/2
2 sin x/2
n

k=1
sin kx =
n

k=1
(cos (2k + 1)x/2 cos (2k 1)x/2) = cos (2n + 1)x/2 cos x/2 =
= cos nx +x/2 cos x/2
Now
sin nx/2 sin nx/2 +x/2 = sin nx/2(sin nx/2 cos x/2 + sin x/2 cos nx/2) =
= sin
2
nx/2 cos x/2 + sin x/2 cos nx/2 sin nx/2 =
=
_
1 cos nx
2
_
cos x/2 +
sin nx
2
sin x/2 =
=
1
2
(cos x/2 cos x/2 cos nx + sin nxsin x/2) =
1
2
(cos x/2 cos (nx
x
/2)
Then
2 sin x/2
n

k=1
sin kx = 2 sin nx/2 sin
1
2
(n + 1)x
n

k=1
sin kx =
sin nx/2 sin
1
2
(n + 1)x
sin x/2
Exercise 34. Using triangle OAP, not the right triangle, if 0 < x < /2
1
2
cos xsin x <
1
2
sin x <
x
2
=sin x < x
Now if 0 > x > /2, sin x < 0,
[ sin x[ = sin x = sin x = sin [x[ < [x[
2.17 Exercises - Average value of a function. Exercise 1.
1
ba
_
x
2
dx =
1
3
(b
2
+ab +a
2
)
Exercise 2.
1
10
_
x
2
+x
3
=
7
12
Exercise 3.
1
40
_
x
1/2
=
4
3
Exercise 4.
1
81
_
x
1/3
=
45
28
Exercise 5.
1
/20
_
/2
0
sin x =
2

Exercise 6.
1
/2/2
_
cos x = 2/
Exercise 7.
1
/20
_
sin 2x = 1/(1 1) = 2/
Exercise 8.
1
/40
_
sin xcos x =
1

31
Exercise 9.
1
/20
_
sin
2
x =
1

(x sin 2x/2)

0
=
1
2
Exercise 10.
1
0
_
cos
2
x =
1
2
Exercise 11.
(1)
1
a0
_
x
2
= a
2
/3 = c
2
=c = a/

3
(2)
1
a0
_
x
n
=
1
a
1
n+1
x
n+1

a
0
=
a
n
n+1
= c
n
=c =
a
(n+1)
1/n
Exercise 12.
A =
_
wf/
_
w
_
wx
2
= k
_
x
_
x
3
=
1
4
x
4
= k
1
2
x
2
; k =
1
2
, w = x
_
x
4
=
1
5
x
5
= k
1
3
x
3
; k =
3
5
, w = x
2
_
x
5
=
1
6
x
6
= k
1
4
x
4
; k =
2
3
, w = x
3
Exercise 13.
A(f +g) =
1
b a
_
f +g =
1
b a
_
f +
1
b a
_
g = A(f) +A(g)
A(cf) =
1
b a
_
cf = c
_
1
b a
__
f
A(f) =
1
b a
_
f
1
b a
_
g = A(g)
Exercise 14.
A(c
1
f +c
2
g) =
_
w(c
1
f +c
2
g)
_
w
=
c
1
_
wf
_
w
+
c
2
_
wg
_
w
= c
1
A(f) +c
2
A(g)
f g w > 0( nonnegative ), =wf wg
Exercise 15.
A
b
a
(f) =
1
b a
_
b
a
f =
1
b a
_
_
c
a
f +
_
b
c
f
_
=
_
c a
b a
_
_
_
c
a
f
c a
_
+
b a (c a)
b a
_
b
a
f
b c
a < c < b
0 <
c a
b a
< 1
Let t =
c a
b a
=A
b
a
(f) = tA
c
a
(f) + (1 t)A
b
c
(f)
A
b
a
(f) =
_
b
a
wf
_
b
a
w
=
_
c
a
w
_
b
a
w
_
c
a
wf
_
c
a
w
+
_
_
b
a
w
_
c
a
w
_
b
a
w
_
_
b
c
wf
_
b
c
w
0 <
_
c
a
w
_
b
a
w
< 1 since w is a nonnegative function. Let t =
_
c
a
w
_
b
a
w
=A
b
a
(f) = tA
c
a
(f) + (1 t)A
b
c
(f)
Exercise 16. Recall that x
cm
=


or r
cm
=

rdm
M
.
32
x
cm
=
_
L
0
x
_
L
0
1
=
L
2
I
cm
=
_
r
2
dm =
_
x
2
(1) = L
3
/3
r
2
=
I
cm
_
L
0
1
= L
2
/3 =r =
L

3
Exercise 17.
x
cm
=
_
l/2
0
x +
_
L
L/2
2xdx
L
2
+ 2(L L/2)
=
yL
2
12
I
cm
=
_
L/2
0
x
2
+
_
L
L/2
2x
2
= 5L
3
/8 r
2
=
5L
3
/8
3L/2
=
5L
2
12
=r =

5L
2

3
Exercise 18. (x) = x for 0 x L
x
cm
=
_
xxdx
_
xdx
=
1
3
x
3

L
0
1
2
x
2

L
0
=
2
3
L
I
cm
=
_
x
2
xdx = L
4
/4
r
2
=
L
4
/4
L
2
/2
= L
2
/2 r =
L

2
Exercise 19.
x
cm
=
_
xxdx +
_
x
L
2
dx
_
xdx +
_
L/2
=
1
3
x
3

L/2
0
+
L
2
(x
2
/2)

L
L/2
1
2
x
2

L/2
0
+
L
2
(L L/2)
= 11L/18
I
cm
=
_
x
2
xdx +
_
x
2
L/2dx = L
4
31/192
r
2
= I
cm
/(L
2
3/8) = L
2
31/72 r =

31L
6

2
Exercise 20. (x) = x
2
for 0 x L
x
cm
=
_
xx
2
dx
_
x
2
= 3L/4
I
cm
=
_
x
2
x
2
dx = L
5
/5
r
2
=
I
cm
1
3
L
3
=
3
5
L
2
r =
_
3
5
L
Exercise 21.
x
cm
=
_
L/2
0
xx
2
dx +
_
L
L/2
x
L
2
4
dx
_
L/2
0
x
2
dx +
_
L
L/2
L
2
4
dx
= 21L/32
I
cm
= int
L/2
0
x
2
x
2
dx +
_
L
L/2
x
2
L
2
4
dx = 19L
5
/240
r
2
=
I
cm
L
3
/6
= 19L
2
/40 =r =

19L/2

10
Exercise 22. Be exible about how you can choose a convenient origin to evaluate the center-of-mass from
33
Let = cx
n
c
_
L
0
x
n
dx =
1
n + 1
L
n+1
c = M
=c =
(n + 1)M
L
n+1
c
_
L
0
xx
n
dx = c
1
n + 2
L
n+2
=
n + 1
n + 2
ML =
3ML
4
x
cm
=
_
x
M
=
3L
4
=
_
x =
n + 1
n + 2
=
3
4
=n = 2
=
3M
L
3
x
2
Exercise 23.
(1)
1
/2 0
_
3 sin 2t =
6

(2)
1
/2 0
_
9 sin
2
2t = 9/2 =v
rms
= 3

2/2
Exercise 24. T = 2 (just look at the functions themselves)
1
2
_
2
0
160 sin t2 sin (t /6) = 80

3
2.19 Exercises - The integral as a function of the upper limit. Indenite integrals. Exercise 1.
_
x
0
(1 + t + t
2
)dt =
x +
1
2
x
2
+
1
3
x
3
Exercise 2. 2y + 2y
2
+ 8y
3
/3
Exercise 3. 2x + 2x
2
+ 8x
3
/3 (1 + 1/2 +1/3) = 2(x +x
2
+ 4x
3
/3) + 5/6
Exercise 4.
_
1x
1
(1 2t + 3t
2
)dt = (t t
2
+t
3
)

1x
1
= 2x + 2x
2
x
3
Exercise 5.
_
x
2
t
4
+t
2
=
1
5
t
5
+
1
3
t
3

x
2
=
x
5
5
+
x
3
3
+
40
3
Exercise 6.
_
x
2
x
t
4
+ 2t
2
+ 1 =
_
t
5
5
+
2
3
t
3
+t
_

x
2
x
=
1
5
(x
10
x
5
) +
2
3
(x
6
x
3
) +x
2
x
Exercise 7.
_
2
3
t
3/2
+t
_

x
1
=
2
3
(x
3/2
1) + (x 1)
Exercise 8.
_
2
3
t
3/2
+
4
5
t
5/4
_

x
2
x
=
2
3
(x
3
x
3/2
) +
4
5
(x
5/2
x
5/4
)
Exercise 9. sin t[
x
i
= sin x
Exercise 10.
_
t
2
+ sin t
_

x
2
0
=
x
2
2
+ sin x
2
Exercise 11.
_
1
2
t + cos t
_

x
2
x
=
x
2
x
2
+ cos x
2
cos x
Exercise 12.
_
1
3
u
3
+
1
3
cos 3u
_

x
0
=
x
3
3
+
1
3
(cos 3x 1)
Exercise 13.
_
1
3
v
3
+
cos 3v
3
_

x
2
x
=
x
6
x
3
3
+
1
3
(cos 3x
2
cos 3x)
Exercise 14.
_
1cos 2x
2
+x =
_
1
2
x
sin 2x
4
+
1
2
x
2
_

y
0
=
y
2

sin 2y
4
+
y
2
2
34
Exercise 15.
_
cos 2w
2
+ 2 sin
w
2
_

x
0
=
(cos 2x 1)
2
+ 2 sin
x
2
Exercise 16.
_
x

(
1
2
+cos t)
2
dt =
_
x

1
4
+cos t+cos
2
t =
1
4
(x+)+sin x+
1
2
_
t +
sin 2t
2
_

=
3
4
(x+)+sin x+
1
4
sin 2x
Exercise 17.
_
x
0
(t
3
t)dt =
1
3
_
x

2
(t t
3
)dt
Note that t
3
t < 0 for 0 < t 1 and t
3
t > 0 for t > 1. t t
3
< 0 for t >

2.
1
4
x
4

1
2
x
2
=
1
3
_
1
2
t
2

1
4
t
4
_

2
=
1
6
x
2

1
12
x
4
=
1
3
x
4

2
3
x
2
= 0 =x = 0, x =

2
_
1
0
(t
3
t)dt +
_

2
1
(t
3
t)dt cancel each other out.
Exercise 18. f(x) = x [x]
1
2
if x is not an integer; f(x) = 0 if x Z.
For any real number, x = q +r, 0 r < 1, q Z. So then
x [x] = r
f(x) = r
1
2
(1) To show the periodicity, consider
f(x + 1) = x + 1 [x + 1]
1
2
= r
1
2
= f(x) sincex + 1 = q + 1 +r, [x + 1] = q + 1
x + 1 [x + 1] = r
1
2
(2) P(x) =
_
x
0
f(t)dt =
_
x
0
(t
1
2
) =
1
2
x
2

1
2
x because given 0 < x 1, then q = 0 for x, so we can use r = t.
To show periodicity,
P(x + 1) =
_
x+1
0
f(t)dt =
_
1
0
f(t)dt +
_
x+1
1
f(t)dt = 0 +
_
x
0
f(t + 1)dt =
_
x
0
f(t)dt = P(x)
since
_
1
0
f(t)dt =
1
2
(x
2
x)

1
0
= 0
(3) Since P itself is periodic by 1, then we can consider 0 x < 1 only. Now x [x] = r and P(x) =
1
2
(r
2
r). So
P(x) =
1
2
((x [x])
2
(x [x])).
(4)
_
1
0
(P(t) +c)dt = 0 =
_
1
0
P(t)dt = c
0 t 1 so P(t) =
1
2
(t
2
t)
=
_
1
0
P(t)dt =
1
2
_
1
3
t
3

1
2
t
2
_

1
0
=
1
2
1
6
= c =
1
12
(5) Q(x) =
_
x
0
(P(t) +c)dt
Q(x + 1) =
_
x+1
0
(P(t) +c)dt =
_
1
0
(P(t) +c)dt +
_
x+1
1
(P(t) +c)dt =
= 0 +
_
x
0
(P(t + 1) +c)dt =
_
x
0
(P(t) +c)dt = Q(x)
so without loss of generality, consider 0 x < 1
=Q(x)
_
x
0
1
2
(t
2
t) +
1
12
=
1
6
x
3

1
4
x
2
+
x
12
35
Exercise 19. g(2n) =
_
2
0
f(t)dt
Consider
_
1
1
f(t)dt =
_
1
0
f(t)dt +
_
0
1
f(t)dt =
_
1
0
f(t)dt +
1
1
_
0
1
f(1t)dt =
=
_
1
0
f +
_
0
1
f(t)dt = 0
Consider that
_
3
1
f(t)dt =
_
1
1
f(t + 2)dt =
_
1
1
f(t)dt = 0. Then, by induction,
_
2n+1
1
f =
_
2n1
1
f +
_
2n+1
2n1
f(t)dt = 0 +
_
1
1
f(t + 2n)dt =
_
1
1
f(t)dt = 0
(1)
g(2n) =
_
1
0
f +
_
2n1
1
f +
_
2n
2n1
f =
_
1
0
f +
_
0
1
f(t)dt =
_
1
0
f +
_
0
1
f(t)dt
=
_
1
0
f +
_
0
1
f = 0
(2)
g(x) =
_
x
0
f =
_
x
0
f(t)dt =
_
x
0
f(t)dt = g(x)
g(x + 2) =
_
x+2
0
f(t)dt =
_
2
0
f +
_
x+2
2
f =
_
x
0
f(t + 2)dt =
_
x
0
f(t)dt = g(x)
Exercise 20.
(1) g is odd since
g(x) =
_
x
0
f(t)dt =
_
x
0
f(t)dt =
_
x
0
f(t)dt = g(x)
Now
g(x + 2) =
_
x+2
0
f =
_
2
0
f +
_
x+2
2
f = g(2) +
_
x
0
f(t + 2)dt = g(2) +
_
x
0
f(t)dt = g(2) +g(x)
=g(x + 2) g(x) = g(2)
(2)
g(2) =
_
2
0
f =
_
2
1
f +
_
1
0
f =
_
2
1
f +A =
_
0
1
f(t + 2)dt +A =
_
0
1
f(t)dt +A =

_
0
1
f(t)dt +A = 2A
g(5) g(3) = g(2)
g(3) = g(2) +
_
3
2
f(t)dt = 2A+
_
1
0
f(t + 2)dt = 2A+A = 3A
=g(5) = 3A+ 2A = 5A
(3) The key observation is to see that g must repeat itself by a change of 2 in the argument. To make g(1) = g(3) = g(5),
theyre different, unless A = 0!
Exercise 21. From the given, we can derive
g(x) = f(x + 5), f(x) =
_
x
0
g(t)dt
=f(5) =
_
5
0
g(t)dt = g(0) = 7
(1) The key insight I uncovered was, when stuck, one of the things you can do, is to think geometrically and draw
a picture.
g(x) = f(x + 5) = g(x) = f(x 5)
=g(x) = f(x 5)
36
(2)
_
5
0
f(t)dt =
_
0
5
f(t + 5)dt =
_
0
5
g(t)dt =
_
5
0
g(t)dt =
_
5
0
g(t)dt =
_
5
0
g(t)dt = f(5) = 7
(3)
_
x
0
f(t)dt =
_
x5
5
f(t + 5)dt =
_
x5
5
g(t)dt =
_
x5
0
g +
_
0
5
g = f(x 5) +
_
5
0
g(t)dt =
f(x 5) +
_
5
0
g(t)dt = f(x 5) +f(5) = g(x) +g(0)
where weve used f(x 5) = g(x) in the second and third to the last step.
3.6 Exercises - Informal description of continuity, The denition of the limit of a function, The denition of continuity
of a function, The basic limit theorems. More examples of continuous functions, Proofs of the basic limit theorems.
Polynomials are continuous.
Exercise 1. lim
x2
1
x
2
=
1
limx2 x
2
=
1
4
Exercise 2.
limx0(25x
3
+2)
limx0(75x
7
2)
= 1
Exercise 3. lim
x2
(x2)(x+2)
(x2)
= 4
Exercise 4. lim
x1
(2x1)(x1)
x1
= 1
Exercise 5. lim
h0
t
2
+2th+h
2
t
2
h
= 2t
Exercise 6. lim
x0
(xa)(x+a)
(x+a)
2
= 1
Exercise 7. lim
a0
(xa)(x+a)
(x+a)
2
= 1
Exercise 8. lim
xa
(xa)(x+a)
(x+a)
2
= 0
Exercise 9. lim
t0
tan t =
limt0 sin t
limx0 cos t
=
0
1
= 0
Exercise 10. lim
t0
(sin 2t +t
2
cos 5t) = lim
t0
sin 2t + lim
t0
t
2
lim
t0
cos 5t = 0 + 0 = 0
Exercise 11. lim
x0
+
|x|
x
= 1
Exercise 12. lim
x0

|x|
x
= 1
Exercise 13. lim
x0
+

x
2
x
= +1
Exercise 14. lim
x0

x
2
x
= 1
Exercise 15. lim
x0
2 sin x cos x
x
= 2
Exercise 16. lim
x0
2 sin x cos x
cos 2x sin x
= 2
Exercise 17. lim
x0
sin x cos 4x+sin 4x cos x
sin x
= 1 + lim
x0
2 sin 2x cos 2x
sin x
= 1 + 2
_
lim
x0
2 sin x cos x cos 2x
sin x
_
= 5 Exercise 18.
lim
x0
5 sin 5x
5x
lim
x0
3 sin 3x
3x
= 5 3 = 2 Exercise 19.
37
lim
x0
sin
_
x+a
2
+
xa
2
_
sin
_
x+a
2

_
xa
2
__
x a
=
= lim
x0
_
sin
x+a
2
cos
xa
2
+ sin
xa
2
cos
x+a
2

_
sin
x+a
2
cos
xa
2
sin
xa
2
cos
x+a
2
_
x a
_
=
= lim
xa
2 sin
xa
2
cos
x+a
2
x a
= cos a
Exercise 20. lim
x0
2 sin
2
x/2
4(x/2)
2
=
1
2
_
lim
x0
sin x/2
x/2
_
=
1
2
Exercise 21. lim
x0
1

1x
2
x
2
_
1+

1x
2
1+

1x
2
_
= lim
x0
1(1x
2
)
x
2
(1+

1x
2
)
=
1
2
Exercise 22. b, c are given.
sin c = ac +b, a =
sin cb
c
, c ,= 0.
if c = 0, then b = 0, a R.
Exercise 23. b, c are given.
2 cos c = ac
2
+b, a =
2 cos cb
c
2
, c ,= 0.
If c = 0, then b = 2, a R.
Exercise 24.
tangent is continuous for x / (2n + 1)/2
cotangent is continuous for x / 2n
Exercise 25. lim
x0
f(x) = . No f(0) cannot be dened.
Exercise 26.
(1) [ sin x 0[ = [ sin x[ < [x[. Choose = for a given .
Then > 0, > 0 such that [ sin x 0[ < when [x[ < .
(2)
[ cos x 1[ = [ 2 sin
2
x/2[ = 2[ sin x/2[
2
< 2[
x
2
[
2
=
[x[
2
2
< 2/2 =
If we had chosen
0
=

2 for a given . [x 0[ < =

2.
(3)
[ sin x(cos h 1) + cos xsin h[ [ sin x[[ cos h 1[ +[ cos h[[ sin h[ <

2
+

2
=
[ cos x +h cos x[ = [ cos xcos h sin xsin h cos x[ = [ cos x(cos h 1) sin xsin h[
[ cos x[[ cos h 1[ +[ sin x[[ sin h[ <

2
+

2
=
since > 0
1
,
2
> 0 such that [ cos h 1[ <
0
; [ sin h[ < whenever [h[ < min (
1
,
2
)
Choose
3
such that if [h[ <
3
; [ cos h 1[ <

2
; [ sin h[ <

2
Exercise 27. f(x) A = sin
1
x
A.
Let x =
1
n
.
[f(x) A[ = [ sin n A[ > [[ sin n[ [A[[ > [1 [A[[
Consider [x 0[ = [x[ =
1
n
(n). Consider
0
=
|1|A||
2
. Then suppose a (n) [x 0[ but [f(x) A[ >
0
. Thus,
contradiction.
Exercise 28. Consider x
1
n
, n Z
+
, n > M(n) (n is a given constant)
f(x) =
_
1
x
_
= [n] = n, for m > M(n), x =
1
m
f(x) > M(n)
so > 0, we cannot nd =
1
n
such that [f(x) A[ < for x < .
So f(x) as x 0
+
.
38
Consider
1
n
x > 0, n Z

; n > M(n).
f(x) =
_
1
x
_
= [n] = n < M(n)
Since integers are unbounded, we can consider n < A, so that
[f(x) A[ > [[f[ [A[[ = n [A[ > M(n) [A[. Choose n such that M(n) [A[ > 0
Exercise 29.
[f A[ = [(1)
[1/x]
A[ [[(1)
[1/x]
[ [A[[ = [1 [A[[
Choose < [1 [A[[. Then > 0 ( such that [x[ < ), [f A[ > . Thus theres no value for f(0) we could choose to
make this function continuous at 0.
Exercise 30. Since
[f(x)[ = [x[[(1)
[1/x]
[ = [x[
So , let = .
Exercise 31. f continuous at x
0
.
Choose some
0
, 0 <
0
< min (b x
0
, x
0
a). Then
0
= (x
0
,
0
).
Consider
1
=
0
2
and
1
= (x
0
,
1
)
Consider x
1
(x
0

1
, x
0
+
1
), so that [f(x
1
) f(x
0
)[ <
1
.
Proceed to construct a for x
1
, some (x
1
;
0
)
[x x
1
[ = [x x
0
+x
0
x
1
[ < [x x
0
[ +[x
0
x
1
[
Without loss of generality, we can specify x
1
such that [x
0
x
1
[ <
1
2
. Also, pick only the xs such that
[x x
0
[ <

1
2
<
1
=[x x
1
[ <

1
2
+

1
2
=
1
Thus, for these xs
[f(x) f(x
1
)[ = [f(x) f(x
0
) +f(x
0
) f(x
1
)[ < [f(x) f(x
0
)[ +[f(x
1
) f(x
0
)[ <
1
+
1
=
0
So
0
,
1
for x
1
. f is continuous at x
1
(a, b). Thus, there must be innitely many points that are continuous in (a, b),
and at the very least, some or all are clustered around some neighborhood about the one point given to make f continuous.
Exercise 32. Given =
1
n
, [f(x)[ = [xsin
1
x
[ = [x[[ sin 1/x[ < [x[(1).
Let = (n) =
1
n
, so that [x[ <
1
n
.
=[f(x)[ <
1
n
Exercise 33.
(1) Consider x
0
[a, b].
Choose some
0
, 0 <
0
< min (b x
0
, x
0
a) ,= 0 , ( x
0
could be a or b )
Consider, without loss of generality, only xs such that x [a, b].
[f(x) f(x
0
)[ [x x
0
[
Let
0
= (
0
, x
0
) =
0
=[f(x) f(x
0
)[ <
0
.
Since we didnt specify x
0
, x
0
[a, b], f is continuous at x
0
.
(2)

_
b
a
f(x)dx (b a)f(a)

_
b
a
(f(x) f(a))dx

_
b
a
[f(x) f(a)[dx

_
b
a
[x a[dx = (
1
2
x
2
ax)

b
a
=
1
2
(b a)(b +a) a(b a) =
(b a)
2
2
39
(3)

_
b
a
f(x)dx (b a)f(c)

_
b
a
(f(x) f(c))dx

_
b
a
[f(x) f(c)[dx
_
b
a
[x c[dx =
=
_
c
a
(c x)dx +
_
b
c
(x c)dx = c(c a)
1
2
(c a)(c +a) +
1
2
(b c)(b +c) c(b c) =
=
1
2
((c a)
2
+ (b c)
2
)
Draw a gure for clear, geometric reasoning.
Consider a square of length (b a) and a 45 45 right triangle inside. From the gure, its obvious that right
triangles of c a length and (b c) length lie within the (b a) right triangle.
Compare the trapezoid of c a, b a bases with the b a right triangle.
1
2
(b c)(b a +c a) =
1
2
(b c)(b c + 2(c a)) >
1
2
(b c)
2
Indeed, the trapezoid and c a right triangle equals the b a trapezoid since
1
2
(b c)(b a +c a) +
1
2
(c a)
2
=
1
2
(b
2
c
2
2ab + 2ac +c
2
2ca +a
2
) =
1
2
(b a)
2
=
1
2
(b a)
2
>
1
2
(b c)
2
+
1
2
(c a)
2
so then

_
b
a
f(x)dx (b a)f(c)

(b a)
2
2
3.11 Exercises - Bolzanos theorem for continuous functions, The intermediate-value theorem for continuous func-
tions. These theorems form the foundation for continuity and will be valuable for differentiation later.
Theorem 10 (Bolzanos Theorem).
Let f be cont. at x [a, b].
Assume f(a), f(b) have opposite signs.
Then at least one c (a, b) s.t. f(c) = 0.
Proof. Let f(a) < 0, f(b) > 0.
Want: Fine one value c (a, b) s.t. f(c) = 0
Strategy: nd the largest c.
Let S = all x [a, b] s.t. f(x) 0 .
S is nonempty since f(a) < 0. S is bounded since all S [a, b].
=S has a suprenum.
Let c = supS.
If f(c) > 0, (c , c +) s.t. f > 0
c is an upper bound on S
but c is a least upper bound on S. Contradiction.
If f(c) < 0, (c , c +) s.t. f < 0
c + is an upper bound on S
but c is an upper bound on S. Contradiction.
Theorem 11 (Sign-preserving Property of Continuous functions).
Let f be cont. at c and suppose that f(c) ,= 0.
then (c , c +) s.t. f be on (c , c +) has the same sign as f(c).
Proof. Suppose f(c) > 0.
> 0, > 0 s.t. f(c) < f(x) < f(c) + if c < x < c + (by continuity).
Choose for =
f(c)
2
. Then
f(c)
2
< f(x) <
3f(c)
2
x (c , c +)
Then f has the same sign as f(c).
40
Theorem 12 (Intermediate value theorem).
Let f be cont. at each pt. on [a, b].
Choose any x
1
, x
2
[a, b] s.t. x
1
< x
2
. s.t. f(x
1
) ,= f(x
2
).
Then f takes on every value between f(x
1
) and f(x
2
) somewhere in (x
1
, x
2
).
Proof. Suppose f(x
1
) < f(x
2
)
Let k be any value between f(x
1
) and f(x
2
)
Let g = f k
g(x
1
) = f(x
1
) k < 0
g(x
2
) = f(x
2
) k > 0
By Bolzano, c (x
1
, x
2
) s.t. g(c) = 0 =f(c) = k
Exercise 1. f(0) = c
0
. f(0) 0.
Since lim
x
c
k
x
k
c
k1
x
k1
= lim
x
c
k
c
k1
x = M > 0 such that [c
n
M
n
[ > [

n1
k=0
c
k
M
k
. So then
f(M) = c
n
M
n
+
n1

k=0
c
k
M
k
c
n
By Bolzanos theorem b (0, M) such that f(b) = 0.
Exercise 2. Try alot of values systematically. I also cheated by taking the derivatives and feeling out where the function
changed direction.
(1) If P(x) = 3x
4
2x
3
36x
2
+ 36x 8, P(4) = 168, P(3) = 143, P(0) = 8, P(
1
2
) =
15
16
, P(1) = 7,
P(3) = 35, P(4) = 200
(2) If P(x) = 2x
4
14x
2
+ 14x 1, P(4) = 231, P(3) = 7, P(0) = 1, P(
1
2
) =
1
8
, P(
3
2
) =
11
8
, P(2) = 2
(3) If P(x) = x
4
+ 4x
3
+ x
2
6x + 2, P(3) = 2, P(
5
2
) =
3
16
, P(2) = 2, P(
1
3
) =
22
81
, P(
1
2
) =
3
16
, P(
2
3
) =

14
81
, P(1) = 2.
Exercise 3. . Consider f(x) = x
2j+1
a. f(0) = a > 0.
Since a is a constant, choose M < 0 such that M
2j+1
a < 0. f(M) < 0.
By Bolzanos theorem, there is at least one b (M, 0) such that f(b) = b
2j+1
a = 0.
Since x
2j+1
a is monotonically increasing, there is exactly one b.
Exercise 4. tan x is not continuous at x = /2.
Exercise 5. Consider g(x) = f(x) x. Then g(x) is continuous on [0, 1] since f is.
Since 0 f(x) 1 for each x [0, 1], consider g(1) = f(1) 1, so that 1 g(1) 0. Likewise 0 g(0) 1.
If g(1) = 0 or g(0) = 0, were done (g(0) = f(0) 0 = 0. f(0) = 0. Or g(1) = f(1) 1 = 0, f(1) = 1 ).
Otherwise, if 1 g(1) < 0 and 0 < g(0) 1, then by Bolzanos theorem, at least one c such that
g(c) = 0 (g(c) = f(c) c = 0. f(c) = c).
Exercise 6. Given f(a) a, f(b) b,
Consider g(x) = f(x) x 0. Then g(a) = f(a) a 0, g(b) = f(b) b 0.
Since f is continuous on [a, b] (so is g) and since g(a), g(b) are of opposite signs, by Bolzanos theorem, at least one c
such that g(c) = 0, so that f(c) = c.
41
3.15 Exercises - The process of inversion, Properties of functions preserved by inversion, Inverses of piecewise mono-
tonic functions. Exercise 1. D = R, g(y) = y 1
Exercise 2. D = R, g(y) =
1
2
(y 5)
Exercise 3. D = R, g(y) = 1 y
Exercise 4. D = R, g(y) = y
1/3
Exercise 5. D = R,
g(y) =
_

_
y if y < 1

y if 1 y 16
_
y
8
_
2
if y > 16
Exercise 6. f(M
f
) = f(f
1
_
1
n

n
i=1
f(a
i
)
_
) =
1
n

n
i=1
f(a
i
)
Exercise 7. f(a
1
)
1
n

n
i=1
f(a
i
) f(a
n
). Since f is strictly monotonic.
g preserves monotonicity.
=a
1
M
f
a
n
Exercise 8. h(x) = af(x) +b, a ,= 0
M
h
= H
_
1
n
n

i=1
h(a
i
)
_
= H
_
1
n
n

i=1
(af(a
i
) +b)
_
= H
_
a
1
n
n

i=1
f(a
i
) +b
_
The inverse for h is g
_
hb
a
_
= H(h) = h
1
. So then
M
h
= g
_
1
n
n

i=1
f(a
i
)
_
= M
f
The average is invariant under translation and expansion in ordinate values.
3.20 Exercises - The extreme-value theorem for continuous functions, The small-span theorem for continuous func-
tions (uniform continuity), The integrability theorem for continuous functions.
Since for c [a, b], m = min
x[a,b]
f f(c) max
x[a,b]
f = M
and

b
a
f(x)g(x)dx

b
a
g(x)dx
= f(c)
Exercise 1.
g = x
9
> 0 for x [0, 1]; f =
1

1 +x
m =
1

2
, M = 1
_
1
0
x
9
=
1
10
x
10

1
0
=
1
10
1
10

_
1
0
x
9

1 +x
dx
1
10
Exercise 2.
_
1 x
2
=
1 x
2

1 x
2
. f =
1

1 x
2
g = (1 x
2
) M =
2

3
, m = 1
_
1/2
0
(1 x
2
)dx = (x
1
3
x
3
)

1/2
0
=
11
24
11
24

_
1/2
0
_
1 x
2
dx
11
24
_
4
3
Exercise 3.
42
f =
1
1 +x
6
g = 1 x
2
+x
4
_
a
0
1 x
2
+x
4
=
_
x
1
3
x
3
+
1
5
x
5
_

a
0
= a
a
3
3
+
a
5
5
m =
1
1 +a
6
M = 1
1
1 +a
6
_
a
a
3
3
+
a
5
5
_

_
a
0
1
1 +x
2
dx
_
a
a
3
3
+
a
5
5
_
So if a =
1
10
, (a a
3
/3 +a
5
/5) = a 0.333 . . . a
3
+ 0.2a
5
= 0.099669
Exercise 4. (b) is wrong, since it had chosen g = sin t, but g needed to be nonnegative.
Exercise 5. At worst, we could have utilized the fundamental theorem of calculus.
_
sin t
2
dt =
_ _
1
2t
_
(2t sin t
2
)dt =
1
2c
(cos t
2
)

(n+1)

n
=
=
1
2c
((1)
n+1
(1)
n
) =
1
c
(1)
n
Exercise 6.
_
b
a
(f)(1) = f(c)
_
b
a
1 = f(c)(b a). Then f(c) =

b
a
f
ba
= 0 for some c [a, b] by Mean-value theorem for
integrals.
Exercise 7. f nonnegative. Consider f at a point of continuity c, and suppose f(c) > 0. Then
1
2
f(c) > 0.
[f(x) f(c)[ < =f(c) < f(x) < f(c) +
Let =
1
2
f(c) > 0 for =
1
2
f(c)
_
c+
c
f(x)dx >
1
2
f(c)(2) = f(c) > 0
But
_
b
a
f(x)dx = 0 and f is nonnegative. f(c) = 0.
Exercise 8.
m
_
g
_
fg M
_
g =m
_
g 0 M
_
g g
m 0 M for
_
g = 1 but also
m 0 M =m 0 M 0 for
_
g = 1
So because of this contradiction, m = M = 0. By intermediate value theorem, f = 0, x [a, b].
4.6 Exercises - Historical introduction, A problem involving velocity, The derivative of a function, Examples of deriva-
tives, The algebra of derivatives. Exercise 1. f

= 1 2x, f

(0) = 1, f

(1/2) = 0, f

(1) = 1, f

(10) = 19
Exercise 2. f

= x
2
+x 2
(1) f

= 0, x = 1, 2
(2) f

(x) = 2, x = 0, 1
(3) f

= 10, x = 4, 3
Exercise 3. f

= 2x + 3
Exercise 4. f

= 4x
3
+ cos x
Exercise 5. f

= 4x
3
sin x +x
4
cos x
Exercise 6. f

=
1
(x+1)
2
Exercise 7. f

=
1
(x
2
+1)
2
(2x) + 5x
4
cos x +x
5
(sin x)
Exercise 8. f

=
x1(x)
(x1)
2
=
1
(x1)
2
43
Exercise 9. f

=
1
(2+cos x)
2
(sin x) =
sin x
(2+cos x)
2
Exercise 10.
(2x + 3)(x
4
+x
2
+ 1) (4x
3
+ 2x)(x
2
+ 3x + 2)
(x
4
+x
2
+ 1)
2
=
2x
5
9x
4
+ 12x
3
3x
2
2x + 3
(x
4
+x
2
+ 1)
2
Exercise 11.
f

=
(cos x)(2 cos x) (sin x)(2 sin x)
(2 cos x)
2
=
2 cos x 2 sin x + 1
(2 cos x)
2
Exercise 12.
f

=
(sin x +xcos x)(1 +x
2
) 2x(xsin x)
(1 +x
2
)
2
=
sin x +xcos x +x
3
cos x x
2
sin x
(1 +x
2
)
2
Exercise 13.
(1)
f(t +h) f(t)
h
=
v
0
h 32th 16h
2
h
= v
0
+ 32t 16h
f

(t) = v
0
32t
(2) t =
v0
32
(3) v
0
(4) T =
v0
16
, v
0
= 16 for 1sec. v
0
= 160 for 10sec.
v0
16
for Tsec.
(5) f

= 32
(6) h = 20t
2
Exercise 14. V = s
3
,
dV
dS
= 3s
2
Exercise 15.
(1)
dA
dr
= 2r = C
(2)
dV
dr
= 4r
2
= A
Exercise 16. f

=
1
2

x
Exercise 17. f

=
1
(1+

x)
2
_
1
2

x
_
Exercise 18. f

=
3
2
x
1/2
Exercise 19.
3
2
x
5/2
Exercise 20. f

=
1
2
x
1/2
+
1
3
x
2/3
+
1
4
x
3/4
x > 0
Exercise 21. f

=
1
2
x
3/2
+
1
3
x
4/3

1
4
x
5/4
Exercise 22. f

=
1
2
x
1/2
(1+x)

x
(1+x)
2
=
1
2

x(1+x)
2
Exercise 23. f

=
(1+

x)x
1
2
1

x
(1+

x)
2
=
1+
1
2

x
(1+

x)
2
Exercise 24.
44
g = f
1
f
2
g

= f

1
f
2
+f
1
f

2
g

g
=
f

1
f
1
+
f

2
f
2
g = f
1
f
2
. . . f
n
f
n+1
g

= (f
1
f
2
. . . f
n
)

f
n+1
+ (f
1
f
2
. . . f
n
)f

n+1
;
g

g
=
(f
1
f
2
. . . f
n
)

f
1
f
2
. . . f
n
+
f

n+1
f
n+1
=
f

1
f
1
+
f

2
f
2
+ +
f

n
f
n
+
f

n+1
f
n+1
Exercise 25.
(tan x)

=
_
cos x
sin x
_

=
cos
2
x (sin x) sin x
cos
2
x
= sec
2
x
(cot x)

=
_
cos x
sin x
_

=
sin xsin x cos xcos x
sin
2
x
= csc
2
x
(sec x)

=
1
cos
2
x
(sin x) = tan xsec x
(csc x)

=
1
sin
2
x
cos x = cot xcsc x
Exercise 35.
f

=
(2ax +b)(sin x + cos x) (cos x sin x)(ax
2
+bx +c)
(sin x + cos x)
2
=
=
(2ax +b)(sin x + cos x) (cos x sin x)(ax
2
+bx +c)
(sin x + cos x)
2
Exercise 36.
f

= a sin x + (ax +b) cos x +c cos x + (cx +d)(sin x) = axcos x + (b +c) cos x + (a d) sin x cxsin x
So then a = 1, d = 1, b = d, c = 0.
Exercise 37.
g

= (2ax +b) sin x + (ax


2
+bx +c) cos x + (2dx +e) cos x + (dx
2
+ex +f)(sin x) =
= ax
2
cos x dx
2
sin x + (2a e)xsin x + (b + 2d)xcos x + (b f) sin x + (c +e) cos x
g = x
2
sin x. So d = 1, b = 2, f = 2, a = 0, e = 0, c = 0.
Exercise 38. 1 +x +x
2
+ +x
n
=
x
n+1
1
x1
(1)
(1 +x +x
2
+ +x
n
)

= 1 + 2x + +nx
n1
=
(n + 1)x
n
(x 1) (1)(x
n+1
1)
(x 1)
2
=
(n + 1)(x
n+1
x
n
) x
n+1
+ 1
(x 1)
2
=
nx
n+1
(n
1
)x
n
+ 1
(x 1)
2
x(1 + 2x + +nx
n1
) = x + 2x
2
+ +nx
n
=
nx
n+2
(n + 1)x
n+1
+x
(x 1)
2
(2)
(x + 2x
2
+ +nx
n
)

= (1 + 2
2
x
1
+ +n
2
x
n1
) =
=
(n(n + 2)x
n+1
(n + 1)
2
x
n
+ 1)(x 1)
2
2(x 1)(nx
n+2
(n + 1)x
n+1
+x)
(x 1)
4
x + 2
2
x
2
+ +n
2
x
n
=
(n(n + 2)x
n+2
(n + 1)
2
x
n+1
+x)(x 1) 2(nx
n+3
(n + 1)x
n+2
+x
2
)
(x 1)
3
=
=
n
2
x
n+3
+ (2n
2
2n + 1)x
n+2
+ (n + 1)
2
x
n+1
x
2
x
(x 1)
3
45
Exercise 39.
f(x +h) f(x)
h
=
(x +h)
n
x
n
h
(x +h)
n
=
n

j=0
_
n
j
_
x
nj
h
j
(x +h)
n
x
n
h
=

n
j=1
_
n
j
_
x
nj
h
j
h
=
n

j=1
x
nj
h
j1
_
n
j
_
lim
h0
(x +h)
n
x
n
h
=
_
n
1
_
x
n1
= nx
n1
4.9 Exercises - Geometric interpretation of the derivative as a slope, Other notations for derivatives. Exercise 6.
(1)
f = x
2
+ax +b
f(x
1
) = x
2
1
+ax
1
+b
f(x
2
) = x
2
2
+ax
2
+b
f(x
2
) f(x
1
)
x
2
x
1
=
x
2
2
x
2
1
+a(x
2
x
1
)
x
2
x
1
= x
2
+x
1
+a
(2)
f

= 2x +a
m = x
2
+x
1
+a = 2x +a x =
x
2
+x
1
2
Exercise 7. The line y = x as slope 1.
y = x
3
6x
2
+ 8xy

= 3x
2
12x + 9
3x
2
12x + 8 = 1 =x = 3, 1
The line and the curve meet under the condition
x = x
3
6x
2
+ 8x =x = 3; f(3) = 3
At x = 0, the line and the curve also meet.
Exercise 8. f = x(1 x
2
). f

= 1 3x
2
.
f

(1) = 2 = y = 2x 2
For the other line,
f

(a) = 1 3a
2
=y(1) = 0 = (1 3a
2
)(1) +b =b = 1 3a
2
Now f(a) = a(1 a
2
) = a a
3
at this point. The line and the curve must meet at this point.
y(a) = (1 3a
2
)a + (1 3a
2
) =
= a 3a
3
+ 1 3a
2
= a a
3
=2a
3
+ 1 3a
2
= 0 = a
3

1
2
+
3
2
a
2
The answer could probably be guessed at, but lets review some tricks for solving cubics.
First, do a translation in the x direction to center the origin on the point of inection. Find the point of inection by taking
the second derivative.
f

= 6a + 3 =a =
1
2
So
a = x
1
2
=(x
1
2
)
3
+
3
2
(x
1
2
)
2

1
2
= x
3
=
3
4
x
1
4
= 0
46
Then recall this neat trigonometric fact:
cos 3x = cos 2xcos x sin 2xsin x = 4 cos
3
x 3 cos x
=cos
3
x =
3
4
cos x
cos 3x
4
= 0
Particularly for this problem, we have cos 3x = 1. So x = 0, 2/3, 4/3. cos x = 1,
1
2
. Plugging cos x x back into
what we have for a, a = 1, which we already have in the previous part, and a =
1
2
. So
f
_
1
2
_
=
3
8
y(x) =
_
1
4
x
_
+
1
4
Exercise 9.
f(x) =
_
x
2
if x c
ax +b if x > c
f

(x) =
_
2x if x c
a if x > c
a = 2c; b = c
2
Exercise 10.
f(x) =
_
1
|x|
if [x[ > c
a +bx
2
if [x[ c
Note that c 0 since [x[ c, for the second condition.
f

(x) =
_

1
x
2
if x > c
1
x
2
if x < c
2bx if [x[ c
So b =
1
2c
3
, a =
3
2c
.
Exercise 11.
f

=
_
cos x if x c
a if a > c
Exercise 12. f(x) =
_
1

2
1+

2
_
=
1A
1+A
A =

xA

= a =
1
2
x
1/2
=
1
2A
; A

=
1
4
x
3/2
=
1
4A
3
f

=
A

(1 +A) A

(1 A)
(1 +A)
2
=
2A

(1 +A)
2
=
1

x(1 +

x)
2
f

=
1
(A(1 +A
2
))
2
(A

(1 +A)
2
+A(2)(1 +A)A

) =
3

x + 1
2x
3/2
(1 +

x)
3
f

=
1
2
_
1
A
2
A

(A
2
(1 +A)
3
) (2AA

(1 +A)
3
+ 3A
2
(1 +A)
2
A

)(3 +
1
A
)
(A
2
(1 +A)
3
)
2
_
=
3
4
_
1
A
+ 4 + 5A
A
4
(1 +A
4
)
_
=
3
4
(1 + 4

x + 5x)

x(x +

x)
4
Exercise 13.
47
P = ax
3
+bx
2
+cx +d
P

= 3ax
2
+ 2bx +c
P

= 6ax + 2b
P

(0) = 2b = 10 =b = 5
P

(0) = c = 1 P(0) = d = 2
P(1) = a + 5 +1 +2 = a + 2 = 2 =a = 4
Exercise 14.
fg = 2,
f

= 2
f

g
= 4
g

g
= 2 f =
1
2
, g = 4
(1)
h

=
f

g g

f
g
2
=
f

g

g

g
f
g
= 4 2(
1
8
) =
15
4
(2)
k

= f

g +fg

= 4g
2
+f2g = 64 + 4 = 68
(3)
lim
x0
g

(x)
f

(x)
=
lim
x0
g

(x)
lim
x0
f

(x)
=
1
2
Exercise 15.
(1) True, by denition of f

(a).
(2)
lim
h0
f(a) f(a h)
h
= lim
h0
f(a h) f(a)
h
= lim
h0
f(a h) f(a)
h
= f

(a)
True, by denition of f

(a).
(3)
lim
t0
f(a + 2t) f(a)
t
= 2 lim
2t0
f(a + 2t) f(a)
2t
= 2f

(a)
False.
(4)
lim
t0
f(a + 2t) f(a) +f(a) f(a +t)
2t
=
lim
2t0
f(a + 2t) f(a)
2t
+
1
2
lim
t0
f(a +t) f(a)
t
=
f

(a)
1
2
f

(a) =
1
2
f

(a)
False.
Exercise 16.
(1)
D

(f +g) = lim
h0
(f(x +h) +g(x +h))
2
(f(x) +g(x))
2
h
= lim
h0
(F +G)
2
(f +g)
2
h
=
= D

f +D

g + lim
h0
2FG2fg
h
lim
h0
2FG2fg
h
= lim
h0
(2(FG) 2fG)(F +f)
(F +f)h
+
(2fG2fg)(G+g)
(g +G)h
=
= lim
h0
2g
F +h
lim
h0
F
2
f
2
h
+ lim
h0
2f
G+g
lim
h0
G
2
g
2
h
=
=
g
f
D

f +
f
g
D

g
48
D

(f g) = lim
h0
(f(x +h) g(x +h))
2
(f(x) g(x))
2
h
=
= lim
h0
(F G)
2
(f g)
2
h
=
= D

f +D

g + lim
h0
2FG2fg
h
= D

f +D

g
g
f
D

f +
f
g
D

g
D

(fg) = lim
h0
((fg)(x +h))
2
((fg)(x))
2
h
=
= lim
h0
(f
2
(x +h))(g
2
(x +h)) f
2
(x)g
2
(x +h) + (g
2
(x +h) g
2
(x))f
2
(x)
h
=
= g
2
D

f +f
2
D

g
D

(f/g) = lim
h0
f
2
(x+h)
g
2
(x+h)

f
2
(x)
g
2
(x)
h
= lim
h0
f
2
(x+h)f
2
(x)
g
2
(x+h)
+
f
2
(x)
g
2
(x+h)

f
2
(x)
g
2
(x)
h
=
=
D

f
g
2
+
f
2
g
4
(D

g) when g(x) ,= 0
(2)
(3)
4.12 Exercises - The chain rule for differentiating composite functions, Applications of the chain rule. Related rates
and implicit differentiation. Exercise 1. 2 sin 2x 2 cos x
Exercise 2.
x

1+x
2
Exercise 3. 2xcos x
2
+ 2x(x
2
2) sin x
2
+ 2 sin x
3
+ 6x
3
cos x
3
Exercise 4.
f

= cos (cos
2
x)(2 cos xsin x) cos (sin
2
x) + sin (cos
2
x) sin (sin
2
x)(2 sin xcos x) =
= sin 2x(cos (cos 2x))
Exercise 5.
f

= nsin
n1
xcos xcos nx +nsin nxsin
n
x
Exercise 6.
f

= cos (sin (sin x))(cos (sin x))(cos x)


Exercise 7.
f

=
2 sin xcos xsin x
2
2xcos x
2
sin
2
x
sin
2
x
2
=
sin 2xsin x
2
2xsin
2
xcos x
2
sin
2
x
2
Exercise 8. f

=
1
2
sec
2 x
2
+
1
2
csc
2 x
2
Exercise 9. f

= 2 sec
2
xtan x +2 csc
2
xcot x
Exercise 10. f

1 +x
2
+
x
2

1+x
2
=
1+2x
2

1+x
2
Exercise 11. f

=
4
(4x
2
)
3/2
Exercise 12.
f

=
1
3
_
1 +x
3
1 x
3
_
2/3
_
3x
2
(2)
(1 x
3
)
2
_
=
2x
2
(1 x
3
)
2
_
1 +x
3
1 x
3
_
2/3
Exercise 13. This exercise is important. It shows a neat integration trick.
49
f(x) =
1

1 +x
2
(x +

1 +x
2
)
=
1

1 +x
2
(x +

1 +x
2
)
_
x

1 +x
2
x

1 +x
2
_
=
=
x

1 +x
2

1 +x
2
= 1
x

1 +x
2
f

1 +x
2

x
2

1+x
2
1 +x
2
=
1
(1 +x
2
)
3/2
Exercise 14.
1
2
(x +
_
x +

x)
1/2
(1 +
1
2
(x +

x)
1/2
_
1 +
1
2

x
_
)
Exercise 15.
f

= (2 +x
2
)
1/2
(3 +x
3
)
1/3
+ (1 +x)x(2 +x
2
)
1/2
(3 +x
2
)
1/3
+ (1 +x)(2 +x
2
)
1/2
(3 +x
3
)
2/3
x
2
Exercise 16.
f

=
1
_
1 +
1
x
_
2
_
1
x
2
_
=
1
(x + 1)
2
g

=
1
_
1 +
1
f
_
2
_
1
f
2
_
f

=
f

(f + 1)
2
g

=
(x + 1)
2
_
x
x+1
+ 1
_
2
=
1
(2x + 1)
2
Exercise 17. h

= f

x h h

k k

0 f(2) = 0 2(5) = 10 g(1) = 0 1(5) = 5


1 f(0) = 1 5(1) = 5 g(3) = 1 6(2) = 12
2 f(3) = 2 4(1) = 4 g(0) = 2 5(2) = 10
3 f(1) = 3 2(6) = 12 g(2) = 3 1(4) = 4
Exercise 18.
g(x) = xf(x
2
)
g

(x) = f(x
2
) +x(2x)f

(x
2
) = f(x
2
) + 2x
2
f

(x
2
)
g

(x) = 2xf

(x
2
) + 4xf

(x
2
) + 2x
2
(2x)f

(x
2
) = 6xf

(x
2
) + 4x
3
f

(x
2
)
x g(x) g

(x) g

(x)
0 0 0 0
1 1 3 10
2 12 6 + 8(3) = 30 12(3) + 32(0) = 36
Exercise 19.
(1)
g

=
df(x
2
)
dx
2
2x = 2xf

(2)
g

= 2 sin xcos xf

2 cos xsin xf

= (sin 2x)(f

(sin
2
x) f

(cos
2
x))
(3)
g

=
df(f(x))
d(f(x))
f

(4)
g

=
df(f(f(x)))
d(f(f(x)))
d(f(f(x)))
d(f(x))
df
dx
50
Exercise 20. V = s
3
, s = s(t)
dV
dt
= 3s
2 ds
dt
.
s = 5cm75cm
3
/sec
s = 10cm300cm
3
/sec
s = xcm3x
2
cm
3
/sec
Exercise 21.
l =
_
x
2
+h
2
dl
dt
=
1
l
x
dx
dt
dx
dt
=
l
x
dl
dt
=
10mi

10
2
8
2
(4mi/sec) =
20
3
mi
sec
_
3600sec
1hr
_
Exercise 22.
l
2
= x
2
+s
2
2l
dl
dt
= 2x
dx
dt
dl
dt
=
x
l
dx
dt
dl
dt
_
x =
s
2
_
= 20

5
dl
dt
(x = s) = 50

2
Exercise 23.
dl
dt
=
x
l
dx
dt
=
3
5
12 =
36
5
mi/hr
Exercise 24. Given the preliminary information
r
h
=
2
5
= , V =
1
3
r
2
h =
1
3

2
h
3
(1)
V =
r
2
h
2
(h
2
y hy
2
+
1
3
y
3
)
dV
dt
=
r
2
h
2
(h
2
2hy +y
2
)
dy
dt
dy
dt
=
h
2
r
2
_
1
h
2
2hy +y
2
_
dV
dt
=
10
2
4
2
_
1
10
2
2(10)5 + 25
_
5 =
5
4
(2)
dV
dt
=
2
h
2
dh
dt
,
dh
dt
=
1

2
h
2
dV
dt
=
5
4
Exercise 25.
=
r
h
=
3
2
dV
dt
=
2
h
2
dh
dt
c 1
9
4
(2
2
)4 = 36 = c = 36 + 1
Exercise 26. The constraint equation, using Pythagorean theorem on the geometry of a bottom hemisphere, is
r
2
= R
2
(R h)
2
= 2Rh h
2
So then
r
dr
dt
= (R h)
dh
dt
V =
_
r
2
dh =
dV
dh
= r
2
= (2Rh h
2
)
=
dV
dh
= (2(10(5) 25)) = 50
51
dV
dt
=
dV
dh
dh
dt
, =
dh
dt
=
dV
dt
_
1
(2Rh h
2
)
_
_
r
R h
_
dr
dt
=
dV
dt
_
1
(2Rh h
2
)
_
=
dr
dt
=
dV
dt
_
R h
r(2Rh h
2
)
_
=
= (5

3)
_
10 5
(2(10)5 25)
3/2
_
=
1
15
Exercise 27. I suppose the area of the triangle is 0 at t = 0.
Now the point on vertex B moves up along the y axis according to y = 1 + 2t. y
_
7
2
_
= 8.
A =
1
2
_
(y 1)
36
7
y
dA
dt
=
1
2
_
_
36
y
1
2
1

y 1
y +
_
(y 1)
36
7
_
dy
dt
=
=
1
2
_
6
2(7)
8 + 6
_
(2) =
66
7
Exercise 28. From the given information, h = 3r + 3. The volume formula is V =
R
2
3
H. So then
V = /3r
2
(3r + 3) = r
3
+r
2
dV
dr
= r(3r + 2)
dr
dt
With the given information, we get
dr
dt
=
1
(6)(20)
Using this, we can plug this back in for the different case:
dV
dt
= n = (36)(110)/(120) = 33
Exercise 29.
(1)
dy
dt
= 2x
dx
dt
; when x =
1
2
, y =
1
4
,
dy
dt
=
dx
dt
(2) t =

6
Exercise 30.
(1) 3x
2
+ 3y
2
y

= 0 =x
2
+y
2
y

= 0
(2)
2x + 2yy
2
+y
2
y

= 0 =y
2
y

= 2(x +yy
2
)
=y

= 2
_
xy
4
+yx
4
y
6
_
= 2xy
5
Exercise 31.
1
2
1

x
+
1
2

y
y

= 0 y

x
< 0
Exercise 32.

_
12 3x
2
4
52
6x + 8yy

= 0 =y

=
3x
4y
3 + 4(y
2
+yy

) = 0
y

=
_

3
4
y
2
_
1
y
=
9
4y
3
Exercise 33.
sin xy +xcos
2
xy(y +xy

) + 4x = 0
y

x
2
cos xy +xy cos xy + sin xy + 4x = 0
Exercise 34. y = x
4
. y
n
= x
m
.
y
n
= x
m
, y

ny
n1
= mx
m1
; y

=
mx
m1
ny
n1
=
m
n
x
m1
x
m(11/n)
=
y

=
m
n
x
m/n1
4.15 Exercises - Applications of differentiation to extreme values of functions, The mean-value theoremfor derivatives.
Lets recap what was shown in the past two sections:
Theorem 13 (Theorem 4.3).
Let f be dened on I.
Assume f has a rel. extrema at an int. pt. c I.
If f

(c), f

(c) = 0; the converse is not true.


Proof. Q(x) =
f(x)f(c)
xc
if x ,= c, Q(c) = f

(c)
f

(c), so Q(x) Q(c) as x c so Q is continuous at c.


If Q(c) > 0,
f(x)f(c)
xc
> 0. For x c 0, f(x) f(c), thus contradicting the rel. max or rel. min. (no neighborhood
about c exists for one!)
If Q(c) < 0,
f(x)f(c)
xc
< 0. For x c 0, f(x) f(c), thus contradicting the rel. max or rel. min. (no neighborhood
about c exists for one!)
Converse is not true: e.g. saddle points.
Theorem 14 (Rolles Theorem).
Let f be cont. on [a, b], f

(x) x (a, b) and let


f(a) = f(b)
then at least one c (a, b), such that f

(c) = 0.
Proof. Suppose f

(x) ,= 0 x (a, b).


By extreme value theorem, abs. max (min) M, m somewhere on [a, b].
M, m on endpoints a, b (Thm 4.3).
F(a) = f(b), so m = M. f constant on [a, b]. Contradict f

(x) ,= 0
Theorem 15 (Mean-value theorem for Derivatives). Assume f is cont. everywhere on [a, b], f

(x) x (a, b).


at least one c (a, b) such that
(6) f(b) f(a) = f

(c)(b a)
Proof.
h(x) = f(x)(b a) x(f(b) f(a))
h(a) = f(a)b f(a)a af(b) +af(a)
h(b) = f(b)(b a) b(f(b) f(a)) = bf(a) af(b) = h(a)
=c (a, b), such that h

(c) = 0 = f

(c)(b a) (f(b) f(a))

Theorem 16 (Cauchys Mean-Value Formula). Let f, g cont. on [a, b], f

, g

x (a, b)
Then c (a, b). x
(7) f

(c)(g(b) g(a)) = g

(c)(f(b) f(a)) (note how its symmetrical)


53
Proof.
h(x) = f(x)(g(b) g(a)) g(x)(f(b) f(a))
h(a) = f(a)(g(b) g(a)) g(a)(f(b) f(a)) = f(a)g(b) g(a)f(b)
h(b) = f(b)(g(b) g(a)) g(b)(f(b) f(a))
=h

(c) = f

(c)(g(b) g(a)) g

(c)(f(b) f(a)) = 0 (by Rolles Thm.)

Exercise 1. For any quadratic polynomial y = y(x) = Ax


2
+Bx +C,
y(a) = Aa
2
+Ba +C
y(b) = Ab
2
+Bb +C
y(b) y(a)
b a
=
A(b a)(b +a) +B(b a)
b a
= A(b +a) +B
y

= 2Ax +B
y

_
a +b
2
_
= A(a +b) +B
Thus the chord joining a and b has the same slope as the tangent line at the midpt.
Exercise 2. The contrapositive of a theorem is always true. So the contrapositive of Rolles Theorem is
If at least one c (a, b) s.t. f

(c) = 0,
then f(a) ,= f(b).
g

= 3x
2
3 = 3(x
2
1) =g

(1) = 0
Suppose g(B) = 0, B (1, 1)
then x (1, 1), x ,= B, g(x) ,= g(B), so g(x) ,= 0 for x ,= B
so only at most one B (1, 1) s.t. g(B) = 0
Exercise 3. f(x) =
3x
2
2
if x 1, f(x) =
1
x
if x 1.
(1) See sketch.
(2)
f(x) =
_
3x
2
2
if x 1
1/x if x 1
f(1) = 1 = f(1) = 1/1
f

(x) =
_
x; f

(1) = 1 for x 1
1/x
2
; f

(1) = 1 for x > 1


Then f(x) is cont. and diff. on [0, 2].
For 0 a < b 1
3b
2
2

_
3a
2
2
_
b a
=
(a +b)
2
= c
Note that 1 f

0 for 0 x 1
For 1 a < b 2
1
b

1
a
b a
=
1
ab
=
1
c
2
=c =

ab
Note that 1 f

1/4
For 0 a 1 , 1 b 2
1
b

_
3a
2
2
_
b a
=
2 (3 a
2
)b
2b(b a)
= c or
1
c
2
depending upon if 0 c 1 or 1 c 2, respectively
54
For instance, for a = 0, b = 2, then
f(b)f(a)
ba
= 1/2, so c = 1/2 or c =

2
Exercise 4.
f(1) = 1 1
2/3
= 0 = f(1) = 1 ((1)
2
)
3
= 0
f

=
2
3
x
1/3
,= 0 for [x[ 1
This is possible since f is not differentiable at x = 0.
Exercise 5. x
2
= xsin x + cos x. g = xS + C x
2
. g

= S + xC S 2x = xC 2x = x(C 2). Since [C[ 1 then


(C 2) is negative for all x. Then for x 0, g

0. Since g(0) = 1 and for x , g , then we could conclude


that g must become zero between 0 and and and 0.
Exercise 6.
f(b) f(a)
b a
= f

(c)
b = x +h
b a = h
a = x
x < x +h < x +h
=f(x +h) f(x) = hf

(x +h)
(1) f(x) = x
2
, f

= 2x.
(x +h)
2
x
2
= 2xh +h
2
= h(2(x +h))
2x +h
2
x = h = =
1
2
so then lim
h0
=
1
2
(2) f(x) = x
3
, f

= 3x
2
.
(x +h)
3
x
3
= 3x
2
h + 3xh
2
+h
3
= h3(x +h)
2
=
__
3x
2
+ 3xh +h
2
3
x
_
/h =
=
_
3x
2
+ 3xh +h
2
3h
2

x
h
=
_
x
2
+xh +
h
2
3
x
h
_
_
_
x
2
+xh +
h
2
3
+x
_
x
2
+xh +
h
2
3
+x
_
_
=
=
x +
h
3
x +
_
x
2
+hx +
h
2
3
= lim
h0
=
1
2
Notice the trick of multiplying by the conjugate on top and bottom to get a way to evaluate the limit.
Exercise 7. f(x) = (x a
1
)(x a
2
) . . . (x a
r
)g(x).
(1) a
1
< a
2
< < a
r
.
Since f(a
1
) = f(a
2
) = 0. f

(c) = 0 for c
1
(a
1
, a
2
).
Consider that f(a
2
) = f(a
3
) = 0 as well as f

(c
2
) = 0 for c (a
2
, a
3
).
Indeed, since f(a
j
) = f(a
j+1
) = 0, f

(c) = 0 for c (a
j
, a
j+1
).
Thus, r 1 zeros.
f
(k)
has r k zeros in [a, b].
f
(k)
= (x a
1
)(x a
2
) . . . (x a
rk
)g
k
(x)
Since f(a
1
) = f(a
2
) = 0, f
(k+1)
(c
1
) = 0 for c
1
(a
1
, a
2
).
f
(k)
(a
j
) = f
(k)
(a
j+1
) = 0, f
(k+1)
(c
j
) = 0 for c
j
(a
j
, a
j+1
)
=f
(k)
(x) has at least r k zeros in [a, b]
We had shown the above by induction.
(2) We can conclude that theres at most r + k zeros for f (since f
(k)
has exactly r zeros, the intervals containing the r
zeros are denite).
55
Exercise 8. Using the mean value theorem
(1)
sin x sin y
x y
= cos c =

sin x sin y
x y

= [ cos c[ 1
=[ sin x sin y[ [x y[
(2) x y > 0.
f(z) = z
n
is monotonically increasing for n Z.
By mean-value theorem,
x
n
y
n
x y
= nc
n1
for y < c < x
Since 0 < y < c < x; ny
n1

x
n
y
n
xy
nx
n1
.
Exercise 9. Let g(x) =
_
f(b)f(a)
ba
_
x +
_
bf(a)af(b)
ba
_
.
f g = h
h(a) = h(c)
h(c) = h(b)
so c
1
(a, c), c
2
(c, b) s. t. h

(c
1
) = h

(c
2
) = 0 by Rolles Thm.
Let h

= H
since H(c
1
) = H(c
2
) = 0 and H is cont. diff. on (c
1
, c
2
). then
c
3
(c
1
, c
2
) s.t. H

(c
3
) = h

(c
3
) = 0
Now h

= (f g)

= f

so f

(c
3
) = 0
Weve shown one exists; thats enough.
Exercise 10. Assume f has a derivative everywhere on an open interval I.
g(x) =
f(x) f(a)
x a
if x ,= a; g(a) = f

(a)
(1) g =
_
1
xa
_
f
1
xa
f(a). f is cont. on (a, b] since f

x (a, b).
1
xa
is cont. on (a, b]. Then g is cont. on (a, b] (remember, you can add, subtract, multiply, and divide cont. functions
to get cont. functions because the rules for taking limits allow so).
g is cont. at a since lim
xa
g = lim
xa
f(x)f(a)
xa
= f

(a).
By mean value theorem,
_
f(x) f(a)
x a
_
= f

(c) = g(x) c (a, x) x (a, b]


Then c [a, b], f

(c) ranges from f

(a) to g(b) since f

(c) = g(x) so whatever g(x) ranges from and to, so does


f

(c).
(2) Let h(x) =
f(x)f(b)
xb
if x ,= b; h(b) = f

(b).
h is cont. on [a, b) since
1
xb
is cont., f(x) is cont.
lim
xb
h = lim
xb
f(x)f(b)
xb
= f

(b) so h is cont. at b.
h is cont. on [a, b] h takes all values from h(a) to f

(b) on [a, b] (by intermediate value theorem).


By mean value theorem,
h(x) =
f(b) f(x)
b x
= f

(c
2
) for c
2
(x, b) x [a, b]
So then f

ranges from h(a) to f

(b) just like h.


h(a) = g(b). So then f

must range from f

(a) to f

(b)
56
4.19 Exercises - Applications of the mean-value theorem to geometric properties of functions, Second-derivative test
for extrema, Curve sketching.
Exercise 1. f(x) = x
2
3x + 2
(1) f

(x) = 2x 3 x
0
=
3
2
.
(2) f

(x) 0 for x
3
2
(3) f

= 2 > 0 for x R
(4) See sketch.
Exercise 2. f(x) = x
3
4x
(1) f

= 3x
2
4 x
c
=
2

3
(2) f

0 when [x[
2

3
(3) f

= 6x f

0 when x 0
(4) See sketch.
Exercise 3. f(x) = (x 1)
2
(x + 2)
(1) f

= 3(x 1)(x + 1) f

(x) = 0 when x = 1
(2) f

when [x[ 1
(3) f

= 3(2x) = 6x f

0 when x 0
(4) See sketch.
Exercise 4. f(x) = x
3
6x
2
+ 9x + 5
(1) f

= 3x
2
12x + 9 = 3(x 3)(x 1) f

(x) = 0 when x = 3, 1
(2)
f

(x) > 0 when x < 1, x > 3


f

(x) < 0 when 1 < x < 3


(3) f

= 6x 12 = 6(x 2) f

0 when x 2
(4) See sketch.
Exercise 5. f(x) = 2 + (x 1)
4
(1) f

(x) = 4(x 1)
3
. f

(0) = 0 when x = 1
(2) f

(x) 0 when [x[ 1


(3) f

(x) = 12(x 1)
2
> 0 x ,= 1
(4) See sketch.
Exercise 6. f(x) = 1/x
2
(1) f

=
2
x
3
f

(x) = 0 for no x
(2) f

0 when x 0
(3) f

=
6
x
4
> 0 x ,= 0
(4) See sketch.
Exercise 7. f(x) = x + 1/x
2
(1) f

= 1 +
2
x
3
f

(x) = 0 = 1
2
x
3
=x
c
= 2
1/3
(2)
f

(x) > 0 when x < 0, 0 < x < 2


1/3
f

(x) < 0 when x > 2


1/3
(3) f

=
6
x
4
> 0 x ,= 0
(4) See sketch.
Exercise 8. f(x) =
1
(x1)(x3)
(1) f

=
1
(x1)
2
(x3)
2
((x 3) +x 1) =
(2)(x2)
(x1)
2
(x3)
2
f

(x) = 0 when x = 2
(2) f

0 when x 2
57
(3)
f

= (2)
_
(x 1)
2
(x 3)
2
(x 2)(2(x 1)(x 3)
2
+ 2(x 3)(x 1)
2
)
(x 1)
4
(x 3)
4
_
=
= (6)
_
x
2
4x +
13
3
(x 1)
3
(x 3)
3
_
x
2
4x +
13
3
> 0 since 144 4(3)(13) = 144 + 12(13) < 0 so
f

> 0 if x > 3, x < 1


f

< 0 if 1 < x < 3


(4) See sketch.
Exercise 9. f(x) = x/(1 +x
2
)
(1)
f

=
(1 +x
2
) x(2x)
(1 +x
2
)
2
=
1 x
2
(1 +x
2
)
2
f

(x) = 0 when x = 1
(2) f

0 when [x[ 1
(3)
f

=
2x(1 +x
2
)
2
2(1 +x
2
)(2x)(1 x
2
)
(1 +x
2
)
4
=
2x(x
2
3)
(1 +x
2
)
3
f

> 0 when x >

3
f

< 0 when 0 < x <

3
f

> 0 when

3 < x < 0
f

< 0 when x <

3
(4) See sketch.
Exercise 10. f(x) = (x
2
4)/(x
2
9)
(1)
f

=
2x(x
2
9) (x
2
4)(2x)
(x
2
9)
2
=
10x
(x
2
9)
2
f

(0) = 0
(2) f

0 when x 0, x ,= 3
(3)
f

= (10)
_
(x
2
9)
2
2(x
2
9)(2x)x
(x
2
9)
2
_
= (30)
(x
2
+ 3)
(x
2
9)
3
f

0 when [x[ 3
(4) See the sketch.
Exercise 11. f(x) = sin
2
x
(1) f

= sin 2x So then f

= 0 when x =

2
n
(2)
f

> 0 when
0 < x <

2
n < x <

2
+n
f

< 0 when

2
+n < x < (n + 1)
(3)
f

= 2 cos 2x
f

> 0 when

4
+n < x <

4
+n
f

< 0 when

4
+n < x <
3
4
+n
(4) See sketch.
58
Exercise 12. f(x) = x sin x
(1) f

= 1 cos x f

= 0 when x = 2n
(2) f

> 0 if x ,= 2n
(3)
f

= sin x
f

> 0 when 2n < x < 2n +


f

< 0 when 2n + < x < 2(n + 1)


(4) See sketch.
Exercise 13. f(x) = x + cos x
(1) f

= 1 sin x x =

2
+ 2n f

(x) = 0
(2) f

> 0 if x ,=

2
+ 2n
(3)
f

= cos x
f

> 0 when

2
+ 2n < x <

2
+ 2n
f

< 0 when

2
+ 2n < x <
3
2
+ 2n
(4) See sketch.
Exercise 14. f(x) =
1
6
x
2
+
1
12
cos 2x
(1) f

=
1
3
x +
sin 2x
6
f

(0) = 0
(2) f

0 when x 0
(3) f

=
1
3

cos 2x
3
=
1cos 2x
3
x = n for f

= 0. Otherwise f

> 0 for x ,= n
(4) See sketch.
4.21 Exercises - Worked examples of extremum problems.
Exercise 1.
A = xy
P = 2(x +y) = 2(x +
A
x
)
P

= 2(1
A
x
2
) = 0
x =

A
P

=
4A
x
3
> 0 for x > 0 so x =

A minimizes P
Exercise 2.
A = xy L = 2x +y
A = x(L 2x) = Lx 2x
2
=
dA
dx
= L 4x = 0 when x =
L
4
y =
L
2
A

= 4 so x =
L
4
maximizes A
Exercise 3.
A = xy L = 2x +y = 2x +
A
x
dL
dx
= 2 +
A
x
2
= 0 when x =

2
y =

A
L

=
2A
x
3
> 0 for x =
_
A
2
so x minimizes L
Exercise 4. f = x
2
+y
2
= x
2
+ (S x)
2
f

= 2x + 2(S x)(1) = 2S + 4x =x =
S
2
f

= 4 > 0 so x =
S
2
minimizes f
Exercise 5. x
2
+y
2
= R > 0
59
f = x +y
f

= 1 +y

= 0 = 1 +
x
y
= 0 = y = x
f

= y

=
1 y
2
y
for y > 0, f

< 0 so that f is max. when y = x


Note that
2x + 2yy

= 0
y

=
x
y
x +yy

= 0
1 +y
2
+yy

= 0 =yy

= 1 y
2
Exercise 6.
l
2
= (L x)
2
+x
2
= L
2
2Lx + 2x
2
= A
dA
dx
= 2L + 4x = 0 =x =
L
2
d
2
A
dx
2
= 4 > 0 =A minimized
l(x =
L
2
) =
L

2
2
Exercise 7.
(x +
_
L
2
x
2
)
2
= A
A

= 2(x +
_
L
2
x
2
)(1 +
x

L
2
x
2
) = 0 when L
2
x
2
= x
2
or x =
L

2
so then the side of the circumscribing and area-maximized square is
L

2
+
_
L
2

L
2
2
=
2L

2
Exercise 8.
A = (2x)(2
_
R
2
x
2
) = 4x
_
R
2
x
2
A

= 4(
_
R
2
x
2
+
x
2

R
2
x
2
) = 4
_
R
2
2x
2

R
2
x
2
_
=x =
R

2
since A

0 when x
R

2
, so A is maximized at x =
R

2
2x =
2R

2
; 2

R
2
x
2
=
2R

2
so then the rectangle that has maximum size is a square.
Exercise 9. Prove that among all rectangles of a given area, the square has the smallest circumscribed circle.
A
0
= (2x)(2

r
2
x
2
) = 4x

r
2
x
2
(x the area to be A
0
)
_
A0
4x
_
2
= r
2
x
2
=x
4
x
2
r
2
+
A
2
0
16
= 0
=0 = 2xr
2
+x
2
2r
dr
dx
4x
3
dr
dx
= 0 (for extrema) =x =
r

2
and
_
r
2
x
2
=
r

2
We could argue that we had found a minimum because at the innity boundaries, the circumscribing circle would be
innitely large.
Exercise 10. Given a sphere of radius R, nd the radius r and altitude h of the right circular cylinder with the largest lateral
60
surface area 2rh that can be inscribed in the sphere.
R
2
=
_
h
2
_
2
+r
2
A = 2rh = 2r
_
4(R
2
r
2
) = 4r
_
R
2
r
2
dA
dr
= 4
_
_
R
2
r
2
+
r
2

R
2
r
2
_
= 4
_
R
2
2r
2

R
2
r
2
_
= r =
R

2
= h =

2R
Exercise 11. Among all right circular cylinders of given lateral surface area, prove that the smallest circumscribed sphere has
radius

2 times that of the cylinder.


A
0
= 2RH (A
0
is the total lateral area of the cylinder)
r
2
= R
2
+
_
H
2
_
2
= R
2
+
_
A
0
4R
_
2
= R
2
+
A
2
0
16
2
R
2
2r
dr
dR
= 2R +
A
2
0
16
2
_
2
R
3
_
=
dr
dR
= 0 =R =

A
0
2

=
H
2
= R
r
2
= R
2
+R
2
= 2R
2
= r =

2R
Exercise 12. Given a right circular cone with radius Rand altitude H. Find the radius and altitude of the right circular cylinder
of largest lateral surface area that can be inscribed in the cone.
h
Rr
=
H
R
= is the constraint (look, directly at the side, at the similar triangles formed)
A = 2rh = 2r(R r) = 2(rR r
2
)
dA
dr
= 2(R 2r) = 0 = r =
R
2
; h =
H
2
A

= 2(R 2r)
since
dA
dr
0 when r
R
2
, r =
R
2
maximizes lateral surface area
Exercise 13. Find the dimensions of the right circular cylinder of maximum volume that can be inscribed in a right circular
cone of radius R and altitude H.
Constraint:
h
Rr
=
H
R
=
V = r
2
h = r
2
(R r) = r
2
(R r) = (Rr
2
r
3
)
dV
dr
= (2Rr 3r
2
) = r(2R 3r) r =
2R
3
since
dV
dr
0 when r
2R
3
, r =
2R
3
maximizes volume
h =
1
3
H
Exercise 14. Given a sphere of radius R. Compute, in terms of R, the radius r and the altitude h of the right circular cone of
maximum volume that can be inscribed in this sphere.
V =
r
2
3
_
R +
_
R
2
r
2
_
dV
dr
=

3
_
2rR + 2r
_
R
2
r
2
+
r
2
(r)

R
2
r
2
_
=

3
r
(2R

R
2
r
2
+ 2R
2
3r
2
)

R
2
r
2
= 0
= r =
2

2R
3
; h =
4R
3
61
Considering the geometric or physical constraints, since lim
V
V = lim
h
V = 0, so then r =
2

2R
3
must maximize
V .
Exercise 15. Find the rectangle of largest area that can be inscribed in a semicircle, the lower base being on the diameter.
A =
_
R
2
x
2
x
A

=
_
R
2
x
2
+
x
2

R
2
x
2
= 0 = x =
R

2
; h =
R

2
Exercise 16. Find the trapezoid of largest area that can be inscribed in a semicircle, the lower base being on the diameter.
A =
1
2
h(2
_
R
2
h
2
+ 2R)
dA
dh
=
_
R
2
h
2
+R +h
_
h

R
2
h
2
_
dA
dh
= 0 = h =

3R
2
=A =
5

3R
2
8
_
R
2
h
2
= 2
_
R
2

3
4
R
2
= 2
R
2
= R
Exercise 17. An open box is made from a rectangular piece of material by removing equal squares at each corner and turning
up the sides. Find the dimensions of the box of largest volume that can be made in this manner if the material has sides (a)
10 and 10; (b) 12 and 18
(1)
(x 2r)(Y 2r)r = (xy 2rx 2ry + 4r
2
)r = xyr 2r
2
x 2r
2
y + 4r
3
= V
dV
dr
= xy 4rx 4ry + 12r
2
= 0
= r =
4(x +y)
_
16(x +y)
2
4(12)xy
2(12)
=
(x +y)
_
x
2
+y
2
xy
6
d
2
V
dr
2
= 4x 4y + 24r = 4(x +y) + 24r
We can plug in our expression for r into the second derivative of V , the volume of the box, to nd out that we want
to pick the negative root from r, in order to maximize the box volume.
Then for x = 10; y = 10, we have r =
5
3
, so that the box dimensions are
5
3

20
3

20
3
.
(2) 12 and 18
=5

7 2 + 2

7 8 + 2

7
Exercise 18. If a and b are the legs of a right triangle whose hypotenuse is 1, nd the largest value of 2a +b.
L = 2a +b = 2a +
_
1 a
2
L

= 2 +
a

1 a
2
L

= 0 =
_
a
2
_
2
= 1 a
2
=a =
2

5
L

= (1)
_
1 a
2

1a
2
a
1 a
2
_
= (1)
_
1
(1 a
2
)
3/2
_
< 0 (so a =
2

5
maximizes L )
Exercise 19. 2 +
x
2
600
gallons per hour. l
0
= 300 mi x = constant speed.
l0
x
= time spent. K = gas cost = 0.30.
62
C = gas cost + driver labor cost = l
0
_
2K
x
+
Kx
600
+
D
x
_
dC
dx
= l
0
_
2K
x
2
+
K
600

D
x
2
_
= 0
dC
dx
=0
x =
_
2K +D
K
10

6
d
2
C
dx
2
= l
0
_
(2K D)
_
2
x
3
__
= l
0
_
2(2K +D)
x
3
_
> 0
Thus, C is minimized if x =
_
2K +D
K
10

6
=C
min
= (300)
_

2K +D

K
10

6
+

2K +D10

6
600
_
= 3

6 + 10D
Remember that there is a speed limit of 60 mi/hr.
(1) D = 0, x = 20

3 C = 6

3 10.39
(2) D = 1, x = 40

2 C = 12

2 16.97
(3) D = 2, x = 60 (because of the speed limit) C = 300
_
2K
60
+
K(60)
600
+
D
60
_
= 5(2.4 +D) = 22.00
(4) D = 3, x = 60 C = 27.00
(5) D = 4, x = 60 C = 32.00
Exercise 20. y =
x
x
2
+1
Suppose the rectangle starts at x
0
on the x axis. Then its y coordinate intersecting the curve, and thus
the height of rectangle, must be y
0
=
x0
x
2
0
+1
=x
0
=
1
2y
0

1
(2y
0
)
2
1
x
2
x
1
=

1
y
2
0
4
where x
2
x
1
is going to be the base of the rectangle. The volume of the cylinder, V , which is obtained from revolving the
rectangle about the x axis, is going to be
V = y
2
0
(x
2
x
1
) = y
2
0
_
1
y
2
0
4
_
= y
0
_
1 4y
2
0
dV
dy
0
=
_
_
1 4y
2
0
+
y
0
2
_
1 4y
2
0
(8y
0
)
_
=
_
1 8y
2
0
_
1 4y
2
0
_
=y
0
=
1
2

2
We could argue that V is maximized, since the innite boundaries would yield a volume of 0 (imagine stretching and
squeezing the rectangle inside the curve).
Then V
max
=
1
8
2 =

4
Exercise 21. Draw a good diagram. Note how the right triangle that you folded is now reected backwards , so that this
triangles right angle is on the left-hand side now.
The constraint is that the crease touches the left edge.
w
0
= l sin +l sin cos (2) = l sin (1 + cos (2)) =
= 2l sin cos
2

Note that we will obtain a minimum crease because by considering the physical innite boundary, we could make a big
crease along the vertical half of the paper or the horizontal half of the paper.
63
So, isolating l, the length of the crease, and then taking the derivative,
l =
w
0
2 sin () cos
2

=
w
0
2
csc () sec
2
()
dl
d
=
w
0
2
_
cot csc sec
2
+ csc 2 sec sec tan
_
=
=
w
0
2
_
C
S
_
1
S
__
1
C
2
_
+
1
S
2
_
1
C
2
__
S
C
__
=
w
0
2
_
1
S
2
C
+
2
C
3
_
dl
d
=0
sin =
1

3
or tan =
1

2
where is the angle of the crease. The corresponding minimum length of the crease will be
l =
w
0
2
1
1

3
2
3
=
9

3
2
Exercise 22.
(1) Consider the center of the circle O, the apex of the isosceles triangle that makes an angle 2, A, and one of its other
vertices, B. Draw a line segment from O to B and simply consider the two triangles making up one half of the
isosceles triangle. Find all the angles.
Angle AOB is 2 by the geometry or i.e. inspection of the gure. The complement of that angle is 2.
Beforehand, we can get the length of the isosceles triangle leg from the law of cosines.
cos ( 2) = cos (2)
s
2
= R
2
+R
2
2R
2
cos ( 2) + 2R
2
(1 + cos (2)) = 2R
2
(2 cos
2
) = 4R
2
cos
2

s = 2Rcos
The constraint equation is
(8) P = 4Rcos + 2Rsin (2)
So then
P

= 4R(sin ) + 4Rcos (2) = 0 =cos 2 = sin


sin =

1
2

_
1
4
4(1)(
1
2
)
2(1)
=
1
2
> 0
= P = 3

3R
P = 3

3 is a max because
Look at the boundary conditions imposed on P by the physical-geometry. = 0, triangle is completely attened,
= , triangle completely disappears.
(2) I had originally thought to Reuse the constraint equation, Eqn. ( ??). This is wrong!
Think about the problem directly and for what it actually is; less wishful thinking.
Consider a xed perimeter L and imagine L to be a string that can be stretched into an isoceles triangle. A trivial
isoceles triangle is a collapsed triangle with two sides of length L/2 only. Then the radius of the disk needs to be L/4.
Consider a general isosceles triangle with 2 as the vertex angle and isosceles sides of h. The perimeter for this
triangle, P, is then
P = 2h + 2hsin = 2h(1 + sin )
=h =
P
2(1 + sin )
h
2
= Rcos
64
We could try to extremize this equation.
dR
d
(4 cos + 2 sin (2)) +R(4 sin + 4 cos (2)) = 0
dR
d
= 0 =cos (2) = sin =sin =
1

3
cos =

3
R =
3P
4

2(

3 + 1)
However, this is the minimized R, minimized radius for the smallest circle tting a particular isosceles triangle of a
xed perimeter. We want to smallest circle with a radius big enough to t all the possible triangles. Thus R =
L
4
Exercise 23. The constraint equation on perimeter is
P = 2h +W +
_
W
2
_
= 2h +W(1 +

2
)
Then intensity function, normalized is given by
I = Wh +

2
_
W
2
_
2
_
1
2
_
=
Wp
2

W
2
2

3
16
W
2
So then
dI
dW
=
P
2
W
3
8
W = 0 =W =
P
2 +
3
4
The height of the rectangle is
h =
P
P
2+
3
4
(1 +

2
)
2
= P
_
4 +
16 + 6
_
Exercise 24. A log 12 feet long has the shape of a frustum of a right circular cone with diameters 4 feet and (4 +h) feet at its
ends, where h 0. Determine, as a function of h, the volume of the largest right circular cylinder that can be cut from the
log, if its axis coincides with that of the log.
Remember to label your diagram carefully.
y
x
=
h
2
y
l
0
x
=
h/2
l
0
=
h/2
_
xh
2l0
_
l
0
x
V = (H +y)
2
(l
0
x) = (H +
xh
2l
0
)
2
(l
0
x)
Note that V (x = 0) = H
2
l
0
= 4(12)
dV
dx
= (2(H +
hx
2l
0
)
h
2l
0
(l
0
x) + (H +
xh
2l
0
)
2
(1)) =
= (H +
hx
2l
0
)(h H
3xh
2l
0
)
dV
dx
=0
x =
(h H)2l
0
3h
V (x =
(h H)2l
0
3h
) =
_
l
0

(h H)2l
0
3h
__
H +
h
2l
0
(h H)2l
0
3h
_
2
= l
0
(h + 2H)
3
27h
where H = 2, l
0
= 12
Exercise 25.
S =
n

k=1
(x a
k
)
2
=
dS
dx
=
n

k=1
2(x a
k
) = 0
=nx =
n

k=1
a
k
=x =

n
k=1
a
k
n
Since lim
x
S = +, x =

n
k=1
a
k
n
minimizes S.
Exercise 26. Hint: draw a picture . Then observe that for f(x) 24, A must be greater than 0 (well show that explicitly
soon) and that f must have a minimum somewhere.
65
If A < 0, then consider f(x) =
5x
7
+A
x
5
. Consider x =
A
1/7
6
1/7
> 0.
f(x =
A
1/7
6
1/7
=
A
6x
5
< 0
Thus, A > 0.
df
dx
= 10x 5Ax
6
= 5
_
2x
7
A
x
6
_
= 0
x =
_
A
2
_
1/7
d
2
f
dx
2
= 10 + 30Ax
7
= 10 + 3 A
_
2
A
_
= 70 > 0
Thus x =
_
A
2
_
1/7
minimizes f for A > 0.
f(x =
_
A
2
_
1/7
) =
5
_
A
2
_
+A
_
A
2
_
5/7
= 24
= A = 2
_
24
7
_
7/2
Exercise 27. Consider f(x) =
x
3
3
+t
2
x over 0 x 1.
f(0) = 0, f(1) =
1
3
+t
2
=f(1) 0 if t
2

1
3
f

(x) = x
2
+t
2
= 0
=x
2
= t
2
but x 0, so x = [t[
f(x
2
= t
2
) =
1
3
t
2
(x) +t
2
x =
2
3
t
2
x > 0 for 1 x 0
So the minimum isnt in the interior of [0, 1]. Its on the end points.
m(t) = 0 for [t[ >
1
3
m(t) =
1
3
+t
2
for [t[ <
1
3
Exercise 28.
(1)
E(x, t) =
[t x[
x
M(t) = max
[t x[
x
as x = a x = b
[t x[
x
=
_
tx
x
if t x
xt
x
if t < x
d
dt
_
[t x[
x
_
=
_

t
x
2
if t x
t
x
2
if t < x
Now t, x a > 0 (this is an important, given, fact ). So x = t should be a relative minimum.
So the maximum occurs at either endpoints
t a
a
= E(a, t),
b t
b
= E(b, t)
By monotonicity on [a, t) , (t, b], and having shown the relative minimum of
|tx|
x
at x = t, the maximum occurs
at x = a or x = b, depending upon the relationship E(a, t) E(b, t).
66
(2)
M(t) =
_
ta
a
if
ta
a
>
bt
b
i.e. t
_
b+a
ab
_
> 2
bt
b
if
bt
b
>
ta
a
dM
dt
=
_
1
a
if t > 2
ab
a+b
1
b
if t <
_
ab
a+b
_
2
Since
dM
dt
0 when t
2ab
a+b
, M is minimized for t =
2ab
a +b
4.23 Exercises - Partial Derivatives.
Exercise 8. f(x, y) =
x

x
2
+y
2
.
f
x
=
1
_
x
2
+y
2
+
x
2
(x
2
+y
2
)
3/2
=
y
2
(x
2
+y
2
)
3/2
f
y
=
xy
(x
2
+y
2
)
3/2
f
xx
=
3y
2
x
(x
2
+y
2
)
5/2
f
yy
= (x)
_
x
2
2y
2
(x
2
+y
2
)
5/2
_
f
xy
= (y)
_
(x
2
+y
2
)
3/2
x
3
2
(x
2
+y
2
)
1/2
(2x)
(x
2
+y
2
)
3
_
=
= (y)
_
2x
2
+y
2
(x
2
+y
2
)
5/2
_
f
yx
=
2y
(x
2
+y
2
)
3/2
+
3y
2
y
(x
2
+y
2
)
5/2
=
(y)(2x
2
+y
2
)
(x
2
+y
2
)
5/2
Exercise 9.
(1)
z = (x 2y)
2
z
x
= 2(x 2y) = 2

z
z
y
= 2(x 2y)(2) = 4

z
x(2z) 4zy = (x 2y)2

z = 2z
(2)
z = (x
4
+y
4
)
1/2
z
x
=
1
z
2x
3
z
y
=
2y
3
z
x(2z) 4zy = (x 2y)2

z = 2z
Exercise 10.
f =
xy
(x
2
+y
2
)
2
, f
x
=
y
(x
2
+y
2
)
2
+
4x
2
y
(x
2
+y
2
)
3
=
y
3
3x
2
y
(x
2
+y
2
)
3
So
f
xx
=
6xy(x
2
+y
2
)
3
3(x
2
+y
2
)
2
(2x)(y
3
3x
2
y)
(x
2
+y
2
)
6
=
=
12xy(x
2
y
2
)
(x
2
+y
2
)
4
By label symmetry,
f
xx
+f
yy
=
12xy(x
2
y
2
)
(x
2
+y
2
)
4
+
12yx(y
2
x
2
)
(x
2
+y
2
)
4
= 0
67
5.5 Exercises - The derivative of an indenite integral. The rst fundamental theorem of calculus, The zero-derivative
theorem, Primitive functions and the second fundamental theorem of calculus, Properties of a function deduced from
properties of its derivatve.
Review the fundamental theorems of calculus, Thm. 5.1 and Thm. 5.3. Note the differences between the two.
Theorem 17 (First fundamental theorem of calculus).
Let f be integrable on [a, x] x [a, b]
Let c [a, b] and
(9) A(x) =
_
x
c
f(t)dt if a x b
Then A

(x) x (a, b) where f is continuous at x and


(10) A

(x) = f(x)
Theorem 18 (Second fundamental theorem of calculus).
Assume f continuous on open interval I
Let P be any primitive of f on I, i.e. P

= f x I
Then c, x I
(11) P(x) = P(c) +
_
x
c
f(t)dt
Exercise 6.

2
2
3
x
3/2
+
_
1
2
2
3
x
3/2
=

2x
3/2
_
b
a
f =

2(b
3/2
a
3/2
)
Exercise 7. f = x
3/2
3x
1/2
+
7
2
x
1/2
;
P =
2
5
x
5/2
2x
3/2
+ 7x
1/2
_
b
a
f =
2
5
(b
5/2
a
5/2
) 2(b
3/2
a
3/2
) + 7(b
1/2
a
1/2
)
Exercise 8. P =
3
2
x
4/3

3
2
x
2/3
; x > 0
Exercise 9. P = 3 cos x +
x
6
3
Exercise 10. P =
3
7
x
7/3
5 sin x
Exercise 11. f

(x) =
1
x
f =

k=
a
k
x
k
f

k=
ka
k
x
k1
=
1
x
Comparing terms, only k = 0 would work, but the coefcient is unequivocally 0
Exercise 12.
_
x
0
[t[dt =
_
_
x
0
tdt if x 0
_
x
0
tdt if x < 0
=
_
1
2
x
2
if x 0
1
2
x
2
if x < 0
=
1
2
x[x[
Exercise 13.
_
x
0
(t +[t[)
2
dt =
_
_
x
0
(2t)
2
dt if x 0
0 if x < 0
=
_
4
3
x
3
if x 0
0 if x < 0
=
2x
2
3
(x +[x[)
Exercise 14. Using 1st. fund. thm. of calc.
68
_
x
0
f(t)dt = A(x) A(0)
A

(x) = f(x)
=2x + sin 2x + 2xcos 2x +sin 2x
f

= 2 + 4 cos 2x +4xsin 2x 2 cos 2x


= 2 + 2 cos 2x 4xsin 2x
f
_

4
_
=

2
f

4
_
= 2
Exercise 15.
_
x
c
f(t)dt = cos x
1
2
f(x) = sin x c =

6
.
Exercise 16. Suppose f(x) = sin x 1 and c = 0.
_
x
0
t sin t t =
_
t cos t + sin t
1
2
t
2
_

x
0
= sin x xcos x
1
2
x
2
So c = 0.
Exercise 17. For f(x) = x
2
f(x) + 2x
1
5 + 2x
1
7 (found by taking the derivative of
_
x
0
f =
_
1
x
t
2
f +
x
16
8
+
x
18
9
+ C,)
Suppose that f = 2x
15
.
=
x
16
8
=
x
18
9
+
1
9
+
x
16
8
+
x
18
9
+C
=C =
1
9
Exercise 18. By plugging in x = 0 into the dened f(x), f(x) = 3 +
_
x
0
1+sin t
2+t
2
dt, we get for p(x) = a +bx +cx
2
,
a = 3
Continuing on,
f

=
1 + sin x
2 +x
2
;
f

(0) =
1
2
= b
f

=
(cos x)(2 +x
2
) 2x(1 + sin x)
(2 +x
2
)
2
f

(0) =
1
2
+ 2c; c =
1
4
Exercise 19.
f(x) =
1
2
_
x
0
(x t)
2
g(t)dt =
1
2
_
x
0
(x
2
2xt +t
2
)g(t)dt =
=
1
2
_
x
2
_
x
0
g 2x
_
x
0
tg +
_
x
0
t
2
g
_
f

= x
_
x
0
g +
x
2
2
g(x)
_
x
0
tg x(xg(x)) +
1
2
x
2
g(x) =
= x
_
x
0
g
_
x
0
tg
f

=
_
x
0
g +xg xg =
_
x
0
g f

(1) = 2
f

= g f

(1) = 5
Exercise 20.
(1) (
_
x
0
(1 +t
2
)
3
dt)

= (1 +x
2
)
3
(2) (
_
x
2
0
(1 +t
2
)
3
dt)

= (1 +x
4
)
3
(2x) =
2x
(1+x
4
)
3
(3) (
_
x
2
x
3
(1 +t
2
)
3
dt)

= (1 +x
4
)
3
(2x) (1 +x
6
)
3
(3x
2
) =
2x
(1+x
4
)
3

3x
2
(1+x
6
)
3
69
Exercise 21.
f

(x) =
_
_
x
2
x
3
t
6
1 +t
4
dt
_

=
_
x
12
1 +x
8
_
(2x)
_
x
1
8
1 +x
12
_
3x
2
Exercise 22.
(1)
f(x) = 2x(1 +x) +x
2
= 2x + 3x
2
f(2) = 16
(2)
d
dx
_
_
b(x)
a(x)
f(t)dt
_
= f(b)b

f(a)a

2x + 3x
2
= f(x
2
)(2x)
f(x
2
) = (1 +
3x
2
)
f(2) = 1 +
3

2
2
(3)
_
f(x)
0
t
2
dt = x
2
(1 +x)
2x + 3x
2
= (f(x))
2
f

(x)
=f
3
(2) = 3(4)(3) = 9(4) = 36
f(2) = 36
1/3
(4)
d
dx
_
_
x
2
(1+x)
0
f(t)dt
_
= 1 = f(x
2
(1 +x))(2x(1 +x) +x
2
)
x = 1 (f(2))(5) = 1 =f(2) =
1
5
Exercise 23.
a
3
2a cos a + (2 a
2
) sin a =
_
a
0
f
2
(t)dt
3x
2
2 cos x + 2xsin x +2xsin x + (2 x
2
) cos x = 3x
2
x
2
cos x = f
2
(x)
f(x) = x

3 cos x
f(a) = a

3 cos a
Exercise 24. f(t) =
t
2
2
+ 2t sin t
(1)
f

= 2t + 2 sin t + 2t cos t f

() = 2 2 = 0
(2)
f

= 2 + 2 cos t + 2 cos t +2t sin t = 2 + 4 cos t 2t sin t


f

2
_
= 2
(3) f

_
3
2
_
= 0
(4) f
_
5
2
_
=
25
2
8
+ 5
(5) f() =

2
2
Exercise 25.
70
(1)
df
dt
=
1 + 2 sin t cos t
1 +t
2
= v(t)
a(t) =
2(cos (2t))(1 +t
2
) 2t(sin (2t))
(1 +t
2
)
2
a(t = 2) = a(t = 1) =
4
4
=
(2) v(t = 1)
1
2
(3) v(t) = (t 1) +
1
2
; t > 1
(4)
f(t) f(1) =
_
t
1
v(t)dt =
_
t
1
(t 1) +
1
2
=
_
t
2
2
t +
1
2
t
_

t
1
=
t
2
2
+t +
t
2
+

2

1
2
Exercise 26.
(1)
f

(x) > 0 x f

(0) = 1; f

(1) = 0
_
1
0
f

(t)dt = f

(1) f

(0) = 0 1 < 0
Thus, its impossible, since f

(x) > 0, so
_
1
0
f

(t)dt > 0
(2)
_
1
0
_
3

2
sin
x
2
_
dx =
_
3x + cos
x
2
_

1
0
= 3 1 = 2
f(x) =
3x
2
2
+
2

sin
x
2
+C
(3) f

(0) > 0 x f

(0) = 1; f(x) 100 x > 0


_
b
a
f

(t)dt = f

(b) f

(a);
_
k
c
f

(t)dt = f(k) f(c)


_
b
0
f

= f

(b) f

(0) = f

(b) 1 0 if b 0
_
k
c
(f

(b) 1)db = f(k) f(c) (k c) > 0 if k > c > 0


f(k) f(c) > k c
f(k) f(0) > k 0
f(100) f(0) > 100
f(x) 100 is untrue for all x > 0
(4) f

(x) = e
x
> 0 f

(x) = e
x
; f

(0) = 1 f(x) = e
x
x < 0, e
x
< 1
Exercise 27. f

(t) 6. b a =
1
2
. f

(0) = 0
_
b
a
f

= f

(b) f

(a) 6(b a) = 3 since b a =


1
2
_
a
0
f

= f

(a) f

(0) = f

(a) 6(a 0) = 6a
If a =
1
2
, f

(1/2) 3
Then by intermediate value theorem, with f being continuous and f

(0) = 0, f

(1/2) 3, f

must take on the value of 3


somewhere between 0 and 3. Thus there is an interval [a, b] of length 1/2 where f

3.
71
5.8 Exercises - The Leibniz notation for primitives, Integration by substitution.
Exercise 1.
_
2x + 1dx =
1
3
(2x + 1)
3/2
.
Exercise 2.
_
x

1 + 3x =
2x
9
(1 + 3x)
3/2
+
4
135
(1 + 3x)
5/2
Exercise 3.
_
x
2

x + 1 =
2x
2
(x + 1)
3/2
3

8x(x + 1)
5/2
15
+
16(x + 1)
7/2
105
since
_
2x
2
(x + 1)
3/2
3
_
= x
2
(x + 1)
1/2
+
4x(x + 1)
3/2
3
_
8x(x + 1)
5/2
15
_
=
4
3
x(x + 1)
3/2
+
8(x + 1)
5/2
15
_
16(x + 1)
7/2
105
_
=
8(x + 1)
5/2
15
Exercise 4.
_
xdx

2 3x
=
2x(2 3x)
1/2
3
+
4(2 3x)
3/2
27
_
1/3
2/3
xdx

2 3x
= 2/9 4/27 (8/9 32/27) = 2/27
Exercise 5.
_
(x + 1)dx
((x + 1)
2
+ 1)
3
=
((x + 1)
2
+ 1)
2
4
Exercise 6.
_
sin
3
x =
_
sin x(1 cos
2
x) = cos x +
1
3
cos
3
x
Exercise 7.
_
x
1/3
(1 +x) =
3
4
x
4/3
+
3
7
x
7/3
=
3
4
(z 1)
4/3
+
3
7
(z 1)
7/3
Exercise 8.
sin
2
x
2
Exercise 9.
(4sin 2x)
3/2
3

/4
0
=
3
3/2
8
3
Exercise 10. (3 + cos x)
1
Exercise 11. 2 cos
1/2
x
Exercise 12. 2 cos

x + 1

8
3
= 2(cos 3 cos 2)
Exercise 13.
cos x
n
n
Exercise 14.
(1x
6
)
1/2
3
Exercise 15.
_
t(1 +t)
1/4
dt =
_
(x 1)x
1/4
dx =
4
9
x
9/4

4
5
x
5/4
=
4
9
(1 +t)
9/4

4
5
(1 +t)
5/4
Exercise 16.
_
(x
2
+ 1)
3/2
dx =?
_
x

x
2
+ 1
_

x
2
+ 1 x
2
/

x
2
+ 1
x
2
+ 1
=
1
(x
2
+ 1)
3/2
Exercise 17.
72
(8x
3
+ 27)
5/3
_
3
5
__
1
24
_
=
1
40
(8x
3
+ 27)
5/3
Exercise 18.
3
2
(sin x cos x)
2/3
Exercise 19.
_
xdx
_
1 +x
2
+ (1 +x
2
)
3/2
=
_
1
2
du

u +u
3/2
=
=
1
2
_
du

1 +u
1/2
= 2(1 +u
1/2
)
1/2
=
= 2(1 +
_
1 +x
2
)
1/2
+C
Exercise 20.
_
(x
2
2x + 1)
1/5
dx
1 x
=
_
(x 1)
2/5
x 1
dx =
_
(x 1)
3/5
dx = 5/2(x 1)
2/5
Exercise 21. Thm. 1.18. invariance under translation.
_
b
a
f(x)dx =
_
b+c
a+c
f(x c)dx.
Thm. 1.19. expansion or contraction of the interval of integration.
_
b
a
f(x)dx =
1
k
_
kb
ka
f
_
x
k
_
dx
yx +c
dy = dx
_
b
a
f(x)dx =
_
b+c
a+c
f(y c)dy
y = kx
dy = kdx
_
b
a
f(x)dx =
1
k
_
kb
ka
f
_
y
k
_
dy
Exercise 22.
F
_
x
a
, 1
_
=
_
x/a
0
u
p
(u
2
+ 1
2
)
q
du
u =
t
a
du =
dt
a
F
_
x
a
, 1
_
=
1
a
_
x
0
(t/a)
p
dt
_
_
t
a
_
2
+ 1
2
_
q
=
= a
p1+2q
_
x
0
t
p
(t
2
+ 1
2
)
q
dt = a
p1+2q
F(x, a)
Exercise 23.
_
1
x
dt
1 +t
2
= F(1) F(x)
_
x
1
dt
1 +t
2
= F(x) F(1)
_
1/x
1
dt
1 +t
2
= F
_
1
x
_
F(1)
u =
1
t
du =
1
t
2
dt,
1
u
2
du = dt
_
x
1
dt
1 +t
2
=
_
1/x
1
du
u
2
_
1 +
1
u
2
_ =
=
_
1/x
1
du
u
2
+ 1
=
_
1
1/x
dt
t
2
+ 1
Exercise 24.
_
1
0
x
m
(1 x)
n
dx =
_
0
1
(1 u)
m
(u
n
)du =
_
1
0
(1 x)
m
x
n
dx using
u = 1 x
x = 1 u
Exercise 25.
73
cos
m
xsin
m
x =
_
sin 2x
2
_
m
= 2
m
sin
m
2x
_
2
0
cos
m
xsin
m
xdx = 2
m
_
/2
0
sin
m
2xdx = 2
m
_

0
1
2
sin
m
xdx = 2
m1
_

0
sin
m
xdx =
= 2
m1
_
/2
/2
sin
m
_

2
x
_
dx = 2
m1
_
/2
/2
cos
m
xdx = 2
m
_
/2
0
cos
m
xdx
Exercise 26.
(1)
u = x
x = u
_

0
xf(sin x)dx =
_
0

( u)f(sin ( u))(du) =
_

0
( u)f(sin u)du =
=
_

0
f(sin x)dx
_

0
xf(sin x)dx
=
_

0
xf(sin x)dx =

2
_

0
f(sin x)dx
(2)
u = cos x
du = sin xdx
_

0
xsin x
1 + cos
2
x
dx =
_

0
xsin x
2 sin
2
x
=

2
_

0
sin x
2 sin
2
x
dx =

2
_

0
sin x
1 + cos
2
x
dx =
=

2
_
1
1
du
1 +u
2
=

2
_
1
1
du
1 +u
2
=
_
1
1
dx
1 +x
2
Exercise 27.
x = sin u
dx = cos u
_
1
0
(1 x
2
)
n
1
2
dx =
_
/2
0
(cos
2
u)
n
1
2
cos udu =
_
/2
0
cos
2n
udu
5.10 Exercises - Integration by Parts.
Exercise 1.
_
xsin x = xcos x + sin x
Exercise 2.
_
x
2
sin x = x
2
cos x + 2xsin x + 2 cos x
Exercise 3.
_
x
3
cos x = x
3
sin x + 3x
2
cos x 6xsin x +6 cos x
Exercise 4.
_
x
3
sin x = x
3
cos x + 3x
2
sin x + 6xcos x 6 sin x
Exercise 5.
_
sin xcos x =
1
4
cos 2x =
1
4
(cos
2
x sin
2
x)
Exercise 6.
_
xsin xcos xdx =
_
x
2
sin 2x =
x cos 2x
4
+
sin 2x
8
Exercise 7.
_
sin
2
x =
_
sin xsin x = sin xcos x +
_
cos
2
x
_
sin
2
xdx =
1
4
sin 2x +
x
2
Exercise 8.
_
sin
n
xdx = cos xsin
n1
x +
_
(n 1) sin
n2
xcos
2
x
u = sin
n1
x
dv = sin xdx
_
sin
n
x = cos xsin
n1
x +
_
(n 1) sin
n2
x(1 sin
2
x) =
= cos xsin
n1
x +
_
(n 1) sin
n2
x (n 1)
_
sin
n
x
_
sin
n
x =
1
n
sin
n1
xcos x +
(n 1)
n
_
sin
n2
x
Exercise 9.
74
(1)
_
sin
2
x =
1
2
sin xcos x +
1
2
_
1 =
1
2
sin xcos x +
1
2
x
_
/2
0
sin
2
xdx =

4
(2)
_
/2
0
sin
4
x =
1
4
sin
3
xcos x

/2
0
+
3
4
_
/2
0
sin
2
x =
3
16
(3)
_
/2
0
sin
6
x =
5
6
_
/2
0
sin
4
x =
5
32
Exercise 10.
(1)
_
sin
3
xdx =
1
3
sin
2
xcos x +
2
3
_
sin x =
1
6
sin 2xcos x
2
3
cos x =
3
4
cos x +
1
12
cos 3x since
3
4
cos x +
1
12
cos 3x =
3
4
cos x +
1
12
(cos xcos 2x sin 2xsin x) =
=
3
4
cos x +
1
12
(cos x(1 2 sin
2
x) +2 sin
2
xcos x) =
2
3
cos x
1
3
sin
2
xcos x
(2)
_
sin
4
xdx =
1
4
sin
3
xcos x +
3
4
_
sin
2
x =
1
4
sin
3
xcos x +
3
4
(
x
2

sin 2x
4
) =
1
4
sin
3
xcos x +
3x
8

3 sin 2x
16
Now
1
32
sin 4x =
1
32
(2 sin 2xcos 2x) =
1
8
(sin xcos x(1 2 sin
2
x) =
sin 2x
16

1
4
sin
3
xcos x
=
1
4
sin
3
xcos x +
3x
8

3 sin 2x
16
=
3x
8

1
4
sin 2x +
1
32
sin 4x
(3)
_
sin
5
xdx =
_
sin
4
xsin xdx = cos xsin
4
x +
_
cos
2
x4 sin
3
x =
= cos xsin
4
x + 4(
_
sin
3
x
_
sin
5
x) = cos xsin
4
x + 4
_
sin
3
x 4
_
sin
5
x
5
_
sin
5
dx = cos xsin
4
x + 4
_
sin
3
x
5
_
sin
5
dx = cos x(1 cos
2
x)
2
+ 4(
3
4
cos x +
1
12
cos 3x)
= cos x(1 2 cos
2
x + cos
4
x) +3 cos x +
1
3
cos 3x
= cos x + 2 cos
3
x cos
5
x 3 cos x +
1
3
(cos xcos 2x sin xsin 2x) =
= 4 cos x + 2 cos
3
x cos
5
x +
1
3
(4 cos
3
x 3 cos x) = 5 cos x +
10 cos
3
x
3
cos
5
x
_
sin
5
xdx = cos x +
2 cos 3x
3

1
5
cos
5
x
My solution to the last part of this exercise conicts with whats stated in the book.
Exercise 11.
(1)
_
xsin
2
xdx = (
_
sin
2
x)x
_
(sin
2
t) =
x
2
2

xsin 2x
4

_
x
2
4
+
cos 2x
8
_
=
=
x
2
8

xsin 2x
4

cos 2x
8
we had used
_
sin
2
x =
x
2

sin 2x
4
75
(2)
_
xsin
3
x =
3x
4
cos x +
x
12
cos 3x
_

3
4
cos x +
1
12
cos 3x =
=
3x
4
cos x +
x
12
cos 3x +
3
4
sin x +
sin 3x
36
_
sin
3
x =
3
4
cos x +
1
12
cos 3x
(3)
_
x
2
sin
2
xdx = x
2
_
x
2

sin 2x
4
_

_
2x
_
x
2

sin 2x
4
_
=
x
3
2

x
2
sin 2x
4

1
3
x
3
+
_
xsin 2x
2
=
=
x
3
6

x
2
sin 2x
4
+
1
2
_
xcos 2x
2
+
sin 2x
4
_
=
=
x
3
6

x
2
sin 2x
4

xcos 2x
4
+
sin 2x
8
Exercise 12.
_
cos
n
xdx =
_
cos
n1
xcos xdx = cos
n1
xsin x +
_
(n 1) cos
n2
xsin
2
x =
= cos
n1
xsin x + (n 1)
_
cos
n2
x
_
(n 1) cos
n
x
=
_
cos
n
x =
cos
n1
xsin x
n
+
_
n 1
n
__
cos
n2
x
Exercise 13.
(1)
_
cos
2
x =
sin 2x
5
+
1
2
x
(2)
_
cos
3
x =
cos
2
x sin x
3
+
2
3
sin x =
3
4
sin x +
1
12
sin 3x since
1
12
sin 3x =
1
12
(sin 2xcos x + sin xcos 2x) =
1
6
sin xcos
2
x +
1
12
sin x(2 cos
2
x 1) =
=
1
3
sin xcos
2
x
1
12
sin x
(3)
_
cos
4
xdx =
cos
3
xsin x
4
+
3
4
_
1
2
x +
1
4
sin 2x
_
=
3
8
x +
3
16
sin 2x +
cos
3
xsin x
4
sin 4x = 2 sin 2xcos 2x = 4 sin xcos x(2 cos
2
x 1) = 8 sin xcos
3
x 2 sin 2x then
_
cos
4
xdx =
3
8
x +
1
4
sin 2x +
1
32
sin 4x
Exercise 14.
_
_
1 x
2
dx = x
_
1 x
2
+
_
x
2

1 x
2
dx
_
x
2

1 x
2
dx =
_
x
2
+1

1 x
2
+ 1 1
=
_
_
1 x
2
+
1

1 x
2
x
2
= x
2
1 + 1
=
_
_
1 x
2
dx =
1
2
x
_
1 x
2
+
1
2
_
1

1 x
2
Exercise 15.
76
(1)
_
(a
2
x
2
)
n
dx = x(a
2
x
2
)
n

_
n(a
2
x
2
)
n1
(2x)xdx = x(a
2
x
2
)
n
+ 2n
_
x
2
(a
2
x
2
)
n1
dx
_
x
2
(a
2
x
2
)
n1
dx =
_
((x
2
a
2
) +a
2
)(a
2
x
2
)
n1
dx =
_
(a
2
x
2
)
n
+a
2
(a
2
x
2
)
n1
dx
=
_
(a
2
x
2
)
n
dx =
x(a
2
x
2
)
n
2n + 1
+
2a
2
n
2n + 1
_
(a
2
x
2
)
n1
dx
(2)
_
(a
2
x
2
)dx =
x(a
2
x
2
)
3
+
2a
2
3
x =
x
3
3
+a
2
x
_
(a
2
x
2
)
5/2
dx =
x(a
2
x
2
)
5/2
6
+
a
2
5
6
_
(a
2
x
2
)
3/2
dx
_
(a
2
x
2
)
3/2
dx =
x(a
2
x
2
)
3/2
4
+
3a
2
4
_
(a
2
x
2
)
1/2
dx
_
(a
2
x
2
)
1/2
= a
_
_
1
_
x
a
_
2
dx = a
2
_
cos
2
d =
= a
2
_
1 + cos 2
2
= a
2
_

2
+
sin 2
4
_
= a
2
_
arcsin
x
a
+
1
2
x
a
_
1
_
x
a
_
2
_
sin =
x
a
cos d =
dx
a
_
a
0
_
a
2
x
2
= a
2
_

2
0
_
=
a
2
2
_
a
0
(a
2
x
2
)
3/2
dx =
3a
2
4
_
a
2
2
_
=
3a
4
8
_
a
0
(a
2
x
2
)
5/2
dx =
5a
2
6
_
3a
4
8
_
=
5
16
a
6
Exercise 16. I
n
(x) =
_
x
0
t
n
(t
2
+a
2
)
1/2
dt
(1)
I
n
(x) = (t
2
+a
2
)
1/2
t
n1

_
(n 1)t
n2
(t
2
+a
2
)
1/2
= t
n1
(t
2
+a
2
)
1/2
(n 1)
_
t
n2
(t
2
+a
2
)
(t
2
+a
2
)
1/2
(n)I
n
= x
n1
(x
2
+a
2
)
1/2
a
2
(n 1)
_
t
n2
(t
2
+a
2
)
1/2
= x
n1
_
x
2
+a
2
(n 1)a
2
I
n2
(2) n = 5; x = 2; a =

5.
I
1
(2) =
_
2
0
x(x
2
+ 5)
1/2
dx = (x
2
+ 5)
1/2

2
0
= 3

5
5I
5
(2) =
_
2
0
t
5
(t
2
+ 5)
1/2
dt = 2
51
(4 + 5)
1/2
5(5 1)I
3
(2) = 48 20I
3
(2)
3I
3
(2) = 2
2

4 + 5 5(3 1)I
1
(2) = 12 10(3

5)
I
5
(2) =
1
5
(48 20(6 +
10

5
3
)) =
168
5

40

5
3
Exercise 17.
_
t
3
(c +t
3
)
1/2
dt =
t2(c +t
3
)
1/2
3

_
2(c +t
3
)
1/2
3
_
3
1
t
3
(4 +t
3
)
1/2
dt =
t2(4 +t
3
)
1/2
3

3
1

2
3
_
3
1
(4 +t
3
)
1/2
= 2

31 +
2

3
3

2
3
(11.35)
Exercise 18.
77
_
sin
n+1
x
cos
m+1
x
dx =
_
sin
n
x
_
sin x
cos
n+1
x
_
dx =
sin
n
x
mcos
m
x

_
nsin
n1
x
mcos
m1
x
=
_
sin
n+1
x
cos
m+1
x
dx =
sin
n
x
mcos
m
x

n
m
_
sin
n1
x
cos
m1
x
Exercise 19.
_
cos
m+1
x
sin
n+1
x
dx =
_
cos
m
x
_
cos xdx
sin
n+1
x
_
= cos
m
x
1
nsin
n
x

_
mcos
m1
x
nsin
m
x
=
=
cos
m
x
nsin
n
x
+
m
n
_
cos
m1
x
sin
n1
x
_
cot
2
x =
_
cos
1+1
x
sin
1+1
x
=
1
1
cos
1
x
sin
1
x

1
1
_
dx = cot x x
_
cot
4
xdx =
_
cos
3+1
x
sin
3+1
x
=
1
3
cos
3
x
sin
3
x

_
cos
31
x
sin
31
x
=
1
3
cot
3
x (cot x x) =
1
3
cot
3
x + cot x +x
Exercise 20.
(1)
_
2
0
tf

(t)dt = 2
_
1
0
tf

(2t)dt n = 2
(2)
_
1
0
xf

(2x)dx =
1
2
_
2
0
tf

(t)dt =
1
2
_
tf

(t)[
2
0

_
2
0
f

(t)dt
_
=
1
2
(2f

(2) (f(2) f(0))) = 4


Exercise 21.
(1) Recall Theorem 5.5, the second mean-value theorem for integrals:
_
b
a
f(x)g(x)dx = f(a)
_
c
a
g(x)dx +f(b)
_
b
c
g(x)dx
_
b
a
sin (t)
_

(t)

(t)
_
dt =
1

(a)
_
c
a

(t) sin (t) +


1

(b)
_
b
a

(t) sin (t) =


=
1

(a)
cos (t)[
c
a
+
1

(b)
(cos (b) cos (a))
4
m
where
1
m

1

(t)
t [a, b]
(2) (t) = t
2
;

(t) = 2t > 2a if t > a

_
x
a
sin t
2
dt

4
2a
= 2a
5.11 Miscellaneous review exercises.
Exercise 1. g(x) = x
n
f(x); f(0) = 1
g

(x) = nx
n1
f(x) + x
n
f

(x) ; g

(0) = 0 especially if n Z
+
(just note that if negative integer values are included,
g

(0) easily blows up)


g
j
(x) =

h
k=0
_
j
k
_
n!
(nk)!
x
nk
f
jk
(x)
If j < n , then g
j
(0), since each term contains some power of x
If j n,
g
j
(x) =
n

k=0
_
j
k
_
n!
(n k)!
x
nk
f
(jk)
(x)
g
j
(0) =
j!
(j n)!
f
(jn)
(0)
78
If j = n, g
n
(0) = n!
Exercise 2.
P(x) =
5

j=0
a
j
x
j
P

(x) =
5

j=1
ja
j
x
j1
P

(x) =
5

j=2
j(j 1)a
j
x
j2
P(0) = 1 = a
0
P

(0) = 0 = a
1
P

(0) = 0 = 2(1)a
2
a
1
= a
2
= 0
=
P(x) = a
5
x
5
+a
4
x
4
+a
3
x
3
+ 1
P

(x) = 5a
5
x
4
+ 4a
4
x
3
+ 3a
3
x
2
P

(x) = 20a
5
x
3
+ 12a
4
x
2
+ 6a
3
x
P(1) = a
5
+a
4
+a
3
+ 1 = 2
P

(1) = 5a
5
+ 4a
4
+ 3a
3
= 0
P

(1) = 20a
5
+ 12a
4
+ 6a
3
= 0
Solve for the undetermined coefcients by Gauss-Jordan elimination process
_
_
5 4 3
20 12 6
1 1 1
_
_
_
_
a
5
a
4
a
3
_
_
=
_
_
0
0
1
_
_
_
_
5 4 3
20 12 6
1 1 1

0
0
1
_
_
=
_
_
1 0
0 1
1 0 0

15
10
6
_
_
=a
5
= 6 a
4
= 15 a
3
= 10 P(x) = 6x
5
15x
4
+ 10x
3
+ 1
Exercise 3. If f(x) = cos x and g(x) = sin x, Prove that f
(n)
= cos (x +
n
2
) and g
(n)
(x) = sin (x +
n
2
)
f
(n)
(x) = cos (x +
n
2
) =
_
sin x(1)
j+1
if n = 2j + 1
cos x(1)
j
if n = 2j
g
(n)
(x) = sin (x +
n
2
) =
_
cos x(1)
j
if n = 2j + 1
sin x(1)
j
if n = 2j
f(x) = cos x
f

(x) = sin x
f

(x) = cos x
f

(x) = sin x
f

(x) = cos x
f
(2j)
(x) = cos x(1)
j
f
(2(j+1))
(x) = (cos x(1)
j
)

= cos x(1)
j+1
f
(2j+1)
(x) = sin x(1)
j+1
f
(2j+3)
(x) = (sin x(1)
j+1
) = sin x(1)
j+2
g(x) = sin x
g

(x) = cos x
g

(x) = sin x
g

(x) = cos x
g

(x) = sin x
g
(2j)
(x) = sin x(1)
j
g
(2(j+1))
(x) = (sin x(1)
j
)

= sin x(1)
j+1
g
(2j+1)
(x) = cos x(1)
j
g
(2j+3)
(x) = (cos x(1)
j+1
)
Exercise 4.
79
h

(x) = f

g +fg

(x) = f

g + 2f

+fg

h
(n)
=
n

k=0
_
n
k
_
f
(k)
g
(nk)
h
(n+1)
=
n

k=0
_
n
k
_
_
f
(k+1)
g
(nk)
+f
(k)
g
(nk+1)
_
=
= f
(1)
g
(n)
+fg
(n+1)
+
n1

k=1
_
n
k
_
_
f
(k+1)
g
(nk)
+f
(k)
g
(nk+1)
_
+f
(n+1)
g +f
(n)
g
(1)
= f
(1)
g
(n)
+fg
(n+1)
+
n

k=2
n!
(n k + 1)!(k 1)!
_
f
(k)
g
(nk+1)
+
n1

k=1
n!
(n k)!k!
f
(k)
g
(nk+1)
_
+
+f
(n+1)
g +f
(n)
g
(1)
Now
n!
(n k + 1)!(k 1)!
+
n!
(n k)!k!
= (k + (n k + 1))
_
n!
(n + 1 k)!(k)!
_
=
_
n + 1
k
_
so then
h
(n+1)
= fg
(n+1)
+
n

k=1
_
n + 1
k
_
f
(k)
g
(n+1k)
+f
(n+1)
g =
n+1

k=0
_
n + 1
k
_
f
(k)
g
(n+1k)
By induction, this formula is true.
Exercise 5.
(1)
f
2
+g
2
= f(g

) +gf

Y = f
2
+g
2
Y

= 2ff

+ 2gg

= 2(gf + 2g(f)) = 0 =Y = C
Y = C = f
2
+g
2
f(0) = 0; g(0) = 1 = C = 1
(2)
h = (F f)
2
+ (Gg)
2
= f

(x) = g(x), g

(x) = f(x); f(0) = 0; g(0) = 1


h

= 2(F f)(F

) + 2(Gg)(G

); h

(0) = 2(0) + 2(0) = 0


= 2(F f)(Gg) + 2(Gg)(F +f) = 0 x
h(x) = C =h(x) = (F(x) f(x))
2
+ (G(x) g(x))
2
h(0) = 0 so C = 0
=F = f; G = g
Exercise 6.
df
du
2x = 3x
2
f

(4) =
3x
2
= 3 where we had used the substitution
u = x
2
u = 4; x = 2
Exercise 7.
dg
du
= u
3/2
; g(u) =
2
5
u
5/2
g(4) =
2
5
2
5
=
64
5
Exercise 8.
_
x
0
sin t
t + 1
dt =
1
0 + 1
_
c
0
sin tdt +
1
x + 1
_
x
c
sin tdt =
= cos t[
c
0
+
1
x + 1
cos t[
x
c
= cos c + 1 +
1
x + 1
(cos x cos c) =
=
xcos c +x cos c + 1 cos x + cos c
x + 1
=
x(1 cos c) + (1 cos c)
x + 1
> 0
Exercise 9. y = x
2
is the curve for C. y =
1
2
x
2
is the curve for C
1
.
80
_
b
0
(x
2

1
2
x
2
) =
1
6
b
3
P : (b, b
2
)
Assume C
2
is of the form kx
2
_
c
0
(kx
2
x
2
) +
_
b
c
(b
2
x
2
) =
(k 1)
3
c
3
+b
2
(b c) +
1
3
(b
3
c
3
)
kc
2
= b
2
k =
b
2
c
2
=
(b
2
c
2
)c
3
+b
3
cb
2

b
3
3
+
c
3
3
=
2b
3
3

2
3
cb
2
=
2
3
b
2
(b c)
Now A(A) = A(B)
=
2
3
b
2
(b c) =
1
6
b
3
=b =
4
3
c
so then kx
2
=
16
9
x
2
Exercise 10.
(1) [Q(h) 0[ =

f(h)
h

=
_
h
2
|h|
if x is rational
0 if x is irrational
.
For now, consider 0 < h < ; let (; h = 0) =
[Q(h) 0[ =

f(h)
h

=
_
h if x is rational
0 if x is irrational
<
(2)

f(h) f(0)
h
0

= =f

(0) = 0
Exercise 11.
_
(2 + 3x) sin bxdx =
2
5
cos 5x +
3x
5
cos 5x +
3
25
sin 5x
Exercise 12.
4
3
(1 +x
2
)
3/2
Exercise 13.
(x
2
1)
10
20

1
2
=
3
10
20
Exercise 14.
1
3
_
1
0
6x + 7 + 2
(6x + 7)
3
dx =
1
3
_
(6x + 7)
1
6
+
(6x + 7)
2
6
_

1
0
=
1
3
_

_
1
13
__
1
6
_
+
1
42
+
1
(6)13
2
+
1
(6)49
_
=
=
37
8281
Exercise 15.
_
x
4
(1 +x
5
)
5
dx =
(1+x
5
)
6
30
Exercise 16.
_
1
0
x
4
(1 x)
20
dx =
u = 1 x
x = 1 u
=
_
0
1
(1 u)
4
u
20
du =
_
1
0
(1 u)
4
u
20
du =
_
1
0
(1 + 4(u) + 6u
2
+4u
3
+u
4
)u
20
du =
=
1
21
21
+
41
2
2
22
+
61
23
23
+
41
24
24
+
1
25
25
=
=
1
265650
Make sure to check your arithmetic.
Exercise 17.
_
2
1
x
2
sin
1
x
dx =
_
cos
1
x
_

2
1
= cos
1
2
cos 1
81
Exercise 18.
_
sin (x 1)
1/4
dx
u = (x 1)
1/4
du =
1
4
(x 1)
3/4
dx =
1
4
1
u
3
dx
=
_
sin (x 1)
1/4
dx =
_
(sin u)4u
3
du = 4
_
u
3
sin udu
_
u
3
sin udu = u
3
cos u + 3u
2
sin u + 6ucos u + 06 sin u
= (x 1)
3/4
cos (x 1)
1/4
+ 3(x 1)
1/2
sin (x 1)
1/4
+ 6(x 1)
1/4
cos (x 1)
1/4
6 sin (x 1)
1/4
sin (x 1)
1/4
dx =
= 4(x 1)
3/4
cos (x 1)
1/4
+ 24(x 1)
1/4
cos (x 1)
1/4
+ 12(x 1)
1/4
sin (x 1)
1/4
24 sin (x 1)
1/4
Exercise 19.
_
xsin x
2
cos x
2
dx = (1/4) sin
2
x
2
+C
Exercise 20.
_
1 + 3 cos
2
xsin 2xdx =
_
1 + 3 cos
2
x2 sin xcos xdx =
_
u
1/2 du
3
=
2u
3/2
9
=
2
9
(1 + 3 cos
2
x)
3/2
,
where we had used this substitution:
u = 1 + 3 cos
2
x
du = 6 cos xsin xdx
du
3
= 2 cos xsin xdx
Exercise 21.
_
2
0
375x
5
(x
2
+ 1)
4
dx
u = x
2
+ 1
du = 2xdx
(u 1)
2
= x
4
=
_
2
0
375
2
du(u 1)
2
u
4
=
375
2
_
5
1
du
(u
2
2u + 1)
u
4
=
375
2
_
5
1
du
_
1
u
2

2
u
3
+
1
u
4
_
=
=
375
2
_
1
u
+
1
u
2
+
1
3u
3
_

5
1
= 64 = 2
6
Exercise 22.
_
1
0
(ax +b)(x
2
+ 3x + 2)
2
dx =
3
2
Since
_
1
x
2
+3x+2
_

1
0
=
1
6
+
1
2
=
2
3
,
then if a = 9/2, b =
27
2
, well obtain 3/2
Exercise 23. I
n
=
_
1
0
(1 x
2
)
n
dx
I
n
=
_
1
0
(1 x
2
)
n
dx = x(1 x
2
)
n

1
0

_
1
0
xn(1 x
2
)
n1
(2x)dx =
= 2
_
1
0
x
2
n(1 x
2
)
n1
dx = 2n
_
1
0
((x
2
1) + 1)(1 x
2
)
n1
dx =
= 2nI
n1
2nI
n
=I
n
=
_
2n
2n + 1
_
I
n1
I
2
=
_
1
0
(1 x
2
)
2
dx =
_
1
0
dx(1 2x
2
+x
4
) =
_
x
2x
3
3
+
1
5
x
5
_

1
0
=
8
15
I
3
=
6
7
8
15
16
35
I
4
=
8
9
I
3
=
128
315
I
5
=
10
11
I
4
=
256
693
Exercise 24. F(m, n) =
_
x
0
t
m
(1 +t)
n
dt; m > 0, n > 0
82
F(m, n) =
t
m+1
m+ 1
(1 +t)
n

x
0

_
x
0
t
m+1
m+ 1
n(1 +t)
n1
dt =
x
m+1
(1 +x)
n
m+ 1

n
m+ 1
F(m+ 1, n 1)
(m+ 1)F(m, n) +nF(m+ 1, n 1) = x
m+1
(1 +x)
n
F(11, 1) =
_
x
0
t
11
(1 +t)
1
dt =
_
x
0
t
1
1 +t
12
=
_
t
12
12
+
t
13
13
_

x
0
=
x
12
12
+
x
13
13
11F(10, 2) + 2
_
x
12
12
+
x
13
13
_
= x
11
(1 +x)
2
F(10, 2) =
x
13
13
+
x
12
6
+
x
11
11
Exercise 25. f(n) =
_
/4
0
tan
n
xdx
(1)
Use this extremely important fact:
_
b
a
fg = f(b)
_
b
c
g +f(a)
_
c
a
g
f(n + 1) =
_
/4
0
tan
n
xtan x =
_
/4
c
tan
n
x <
_
/4
0
tan
n
x = f(n)
(2)
f(n + 2) +f(n) =
_
/4
0
tan
n
xtan
2
x +
_
/4
0
tan
n
x =
_
/4
0
tan
n
x(sec
2
x) =
=
tan
n+1
x
n + 1

/4
0
=
1
n + 1
(3)
f(n + 2) +f(n) =
1
n + 1
< f(n + 1) +f(n) < 2f(n)
1
n 1
= f(n 2) +f(n) > f(n 1) +f(n) > 2f(n)
=
1
n + 1
< 2f(n) <
1
n 1
Exercise 26. f(0), f() = 2
_

0
(f(x) +f

(x)) sin xdx = 5


_

0
f

sin x = f

sin x
_
f

cos x =
_
f

cos x = f cos x
_
f sin x
_

0
(f +f

) sin xdx =
_
f sin x +(f()(1) f(0))
_
f sin x = 2 +f(0) = 5 = f(0) = 3
Exercise 27.
_
/2
0
sin xcos x
x + 1
dx =
_
/2
0
sin 2x
2x + 2
dx =
1
2
_

0
sin x
x + 2
dx =
1
2
_
cos x
x + 2

_
cos x
(x + 2)
2
_
=
=
1
2
_
1
+ 2
+
1
2
A
_
=
4 +
4( + 2)

A
2
Exercise 28.
_
dx
x

a +bx
=
2

a +bx
bx
+
2
b
_

a +bx
x
2
_

a +bx
x
=
2
3b
(a +bx)
3/2
x
+
_
2
3b
(a +bx)
3/2
x
2
=
2
3b

a +bx
_
a
x
+b
_
+
2
3b
_

a +bx
_
a
x
2
+
b
x
_
=
_

a +bx
x
= a
_
dx
x

a +bx
+ 2

a +bx
83
Exercise 29.
_
x
n
_
(ax +b)dx =
2x
n
(ax +b)
3/2
3a

_
nx
n1
2(ax +b)
3/2
3a
=
2x
n
(ax +b)
3/2
3a

2n
3a
_
x
n1
(ax +b)

ax +b =
=
2x
n
(ax +b)

ax +b
3a

2n
3
_
x
n

ax +b
2nb
3a
_
x
n1

ax +b
_
x
n

ax +bdx =
2
(2n + 3)a
_
x
n
(ax +b)
3/2
nb
_
x
n1

ax +b
_
+C n ,=
3
2
Exercise 30.
_
x
m

a +bx
dx =
2x
m
(a +bx)
1/2
b

_
mx
m1
2(a +bx)
1/2
b
=
2
b
x
m
(a +bx)
1/2

2m
b
_
x
m1
(a +bx)

a +bx
=
2
b
x
m
(a +bx)
1/2
2m
_
x
m

a +bx

2ma
b
_
x
m1

a +bx
_
x
m

a +bx
dx =
1
2m+ 1
2
b
x
m
(a +bx)
1/2

2ma
b(2m+ 1)
_
x
m1

a +bx
Exercise 31.
_
dx
x
n

ax +b
= 2

ax +b
ax
n
+
_
n2

ax +b
ax
n+1
_
ax +b
ax +b
=
2

ax +b
ax
n
+
2n
a
_
ax +b
x
n+1

ax +b
=
=
2

ax +b
ax
n
+ 2n
_
1
x
n

ax +b
+
2nb
a
_
1
x
n+1

ax +b
=(1 2n)
_
dx
x
n

ax +b

ax +b
ax
n
=
_
b
_
2n
a
_
x
n+1

ax +b
_
1
x
n

ax +b
=

ax +b
(n 1)bx
n1

(2n 3)a
(2n 2)b
_
1
x
n1

ax +b
Exercise 32. I derived the formulas for this and Exercise 33 by doing the following trick.
(C
m+1
S
1n
)

= (m+ 1)C
m
(S
2n
) +C
m+2
(1 n)S
n
= (m+ 1)C
m
S
2n
+ (1 n)C
m
S
n
(1 S
2
) =
= (m+ 1 + 1 n)C
m
S
2n
+ (1 n)C
m
S
n
= (mn + 2)C
m
S
2n
+ (1 n)C
m
S
n
=
_
C
m
S
n
=
(C
m+1
S
1n
)
n 1

(mn + 2)
n 1
_
C
m
S
2n
Exercise 33.
(C
m1
S
1n
)

= (m1)C
m2
(S
2n
) + (1 n)C
m
S
n
= (m1)C
m2
(S
n
)(1 C
2
) + (1 n)C
m
S
n
=
= (m1)C
m2
S
n
+ (m1)C
m
S
n
+ (1 n)C
m
S
n
C
m1
S
1n
= (m1)
_
C
m2
S
n
+ (mn)
_
C
m
S
n
m1
mn
_
C
m2
S
n
+
C
m1
S
1n
mn
=
_
C
m
S
n
Exercise 34.
(1) P

(x) 3P(x) = 4 5x + 3x
2
P =
n

j=0
a
j
x
j
P

=
n

j=1
a
j
jx
j1
=
n1

j=0
a
j+1
(j + 1)x
j
=
n1

j=0
(a
j+1
(j + 1) 3a
j
)x
j
= 4 5x + 3x
2
84
Generally, we can say
a
j+1
=
3
j + 1
a
j
if j 3
We also have
a
1
(1) 3a
0
= 4
a
2
(2) 3a
1
= 5
a
3
(3) 3a
2
= 3 =a
3
a
2
= 1
Then let a
2
= 1 and a
3
= 0. So we have a
1
= 1 and a
0
= 1. P(x) = 1 +x +x
2
is one possible polynomial
and we were only asked for one.
Suppose Q s.t. Q

3Q = 4 5x + 3x
2
(another solution). Then
(P Q)

3(P Q) = 0 x
=
(P Q)

P Q
= 3 =ln (P Q) = 3x =ke
3x
= P Q = k

j=0
(3x)
j
j!
Q = k

j=0
(3x)
j
j!
+P
Since we didnt specify what Q has to be, we nd that, in general, any Q is P plus some amount of the homoge-
neous solution, ke
3x
.
(2) If Q(x) is a given polynomial, and suppose P is a polynomial solution to P

(x) 3P(x) = Q(x). Suppose R is


another polynomial solution such that R

(x) 3R(x) = Q(x). Then just like above, P R = ke


3x
. If we wanted
polynomial answers of nite terms, then k must be zero. Thus, theres at most only one polynomial solution P.
Exercise 35. Bernoulli Polynomials.
(1) P
1
(x) = 1; P

n
(x) = nP
n1
(x);
_
1
0
P
n
(x)dx = 0, if n 1
n = 1 (1)(1) = P

1
_
1
0
(x +c) = (
1
2
x
2
+Cx)

1
0
=
1
2
+C = 0 C = 1/2
P
1
= x 1/2
n = 2 2(x 1/2) = P

2
_
1
0
(x
2
x +C) =
_
1
3
x
3

1
2
x
2
+Cx
_

1
0
=
1
6
+C = 0 C = 1/6
P
2
= x
2
x + 1/6
n = 3 3(x
2
x +
1
6
) = P

3
_
1
0
(x
3

3x
2
2
+
x
2
+C) =
1
4

1
3
2
+
1
4
+C = 0
P
3
= x
3

3x
2
2
+
x
2
n = 4 4(x
3

3
2
x
2
+
x
2
) = P

4
_
1
0
(x
4
2x
3
+x
2
+C) =
1
5

1
4
2
+
1
3
1
3
+C = 0 C =
1
30
P
4
= x
4
2x
3
+x
2
+
1
30
n = 5 5(x
4
2x
3
+x
2

1
30
) = P

5
_
1
0
(x
5

5x
4
2
+
5x
3
3

x
6
+C) =
1
6

(1)
5
2
+
5(1)
4
12
+
(1)
2
12
+C = 0; C = 0
P
5
= x
5

5x
4
2
+
5x
3
3

x
6
(2) The rst, second, and up to fth case has already been proven.
Assume the nth case, that P
n
(t) = t
n
+

n1
j=0
a
j
t
j
(the general form of a polynomial of degree n).
P

n+1
= (n + 1)(t
n
+
n1

j=0
a
j
t
j
) =P
n+1
= t
n+1
+ (n + 1)
n1

j=0
a
j
t
j+1
j + 1
+C
85
(3) The rst, second, and up to fth case has already been proven.
Assume the nth case, that P
n
(0) = P
n
(1).
_
P

n+1
= P
n+1
(1) P
n+1
(0) (by the second fundamental theorem of calculus)
P

n+1
= (n + 1)P
n
_
1
0
(n + 1)P
n
(t) = 0 (by the given properties of Bernoulli polynomials)
=P
n+1
(1) = P
n+1
(0)
(4) P
n
(x + 1) = P
n
(x) = nx
n1
is true for n = 1, 2, by quick inspection (and doing some algebra mentally).
P

n+1
= (n + 1)P
n
=
_
P

n+1
= (n + 1)
_
P
n
P
n+1
(x + 1) P
n+1
(x) = P
n+1
(a
1
) + (n + 1)
_
x+1
a1
P
n
(t) (P
n+1
(a
2
) + (n + 1)
_
x
a2
P
n
(t))
a
1
= 1; a
2
= 0; so P
n+1
(1) P
n+1
(0) = 0 (from previous problems)
=
P
n+1
(x + 1) P
n+1
(x) = (n + 1)(
_
x+1
1
P
n
(t)
_
x
0
P
n
(t)) =
= (n + 1)
_
x
0
P
n
(t + 1) P
n
(t) = (n + 1)
_
x
0
nt
n1
= (n + 1)x
n
(5)
_
k
0
P
n
=
_
k
0
P

n+1
n + 1
=
P
n+1
(k) P
n+1
(0)
n + 1
P
n
(x + 1) P
n
(x) = nx
n1
P
n
(x + 1) P
n
(x)
n
= x
n1
=
k1

x=1
P
n+1
(x + 1) P
n+1
(x)
n + 1
=
k1

x=1
x
n
=
k1

r=1
P
n+1
(r + 1) P
n+1
(r)
n + 1
=
=
k1

r=1
r
n
=
P
n+1
(k) P
n+1
(0)
n + 1
(telescoping series and P
n+1
(1) = P
n+1
(0) )
(6) This part was fairly tricky. A horrible clue was that this part will rely directly on the last part (because of the way
this question is asked), which gave us

x1
j=1
j
n
=
_
x
0
P
n
(t)dt =
Pn+1(x)Pn+1(0)
n+1
.
Use induction. It can be easily veried, plugging in, that P
n
(1 x) = (1)
n
P
n
(x) is true for n = 0 . . . 5.
Assume the nth case is true.
u = 1 t
du = dt
_
x
0
P
n
(t)dt =
_
1x
1
P
n
(1 u)du =
_
1x
1
P
n
(u)(1)
n
du
(since P
n
(1 x) = (1)
n
P
n
(x), assumed nth case is true)
= (1)
n+1
_
1x
1
P
n
(t)dt =
= (1)
n+1
_
1x
0
P
n
(t)dt (since
_
1
0
P
n
= 0 )
== (1)
n+1
_
P
n+1
(1 x) P
n+1
(0)
n + 1
_
=
P
n+1
(x) P
n+1
(0)
n + 1
=P
n+1
(1 x) = (1)
n+1
P
n+1
(x)
In the second to last and last step, we had used (1)
n+1
P
n+1
(0) = P
n+1
(0). For n + 1 even, this is denitely true.
If n + 1 was odd,
Doing some algebra for the rst ve cases, we can show that P
2j1
(0) = 0 for j = 2, 3. Assume the jth case is
86
true. Since P
2j1
is a polynomial and P
2j1
(0) = 0, then the form of P
2j1
must be P
2j1
=

2j1
k=1
a
k
x
j
. Using
P

n+1
= (n + 1)P
n
,
P
2j
(x) P
2j
(0) = 2j
_
x
0
P
2j1
= 2j
2j1

k=1
_
x
0
a
k
t
k
= 2j
2j1

k=1
a
k
1
k + 1
t
k+1

x
0
=
= 2j
2j1

k=1
a
k
k + 1
x
k+1
= 2j
2j

k=2
a
k1
x
k
k
+P
2j
(0)
P
2j+1
(x) P
2j+1
(0) = (2j + 1)
_
x
0
_

k=2
(2j)
a
k1
k
t
k
+P
2j
(0)
_
= (2j + 1)
2j+1

k=3
2ja
k2
x
k
k(k 1)
+ (2j + 1)P
2j
(0)x
If we take the integral from 0 to 1, then we nd that P
2j+1
(0) = 0
(7) Using P
n
(1 x) = (1)
n
P
n
(x), derived above,
P
2j+1
(0) = (1)
2j+1
P
2j+1
(1) = (1)P
2j+1
(0)
=P
2j+1
(0) = 0
P
2j1
(1
1
2
) = (1)
2j1
(P
2j1
(
1
2
))
=P
2j1
(
1
2
) = 0
Exercise 36. Theres a maximum at c for f, so f

(c) = 0
_
x
a
f

(t)dt = f

(x) f

(a)
_
c
0
f

(t)dt = f

(c) f

(0) = f

(0)
_
a
c
f

(t)dt = f

(a) f

(c) = f

(a)
[f

(0)[ = [
_
0
c
f

(t)dt[
_
c
0
[f

[dt mc
[f

(a)[ m(a c)
[f

(0)[ +[f

(a)[ ma
6.9 Exercises - Introduction, Motivation for the denition of the natural logarithm as an integral, The denition
of the logarithm. Basic properties; The graph of the natural logarithm; Consequences of the functional equation
L(ab) = L(a) +L(b); Logarithm referred to any positive base b ,= 1; Differentiation and integration formulas involv-
ing logarithms;Logarithmic differentiation.
Exercise 1.
(1)
log x = c + (ln [t[)[
x
e
= c + ln [x[ 1 =ln
_
x
[x[
_
= c 1
c = 1
(2)
f(x) = ln
1 +a
1 a
+ ln
1 +b
1 b
= ln
_
(1 +a)(1 +b)
(1 a)(1 b)
_
= ln
_
1 +x
1 x
_
=
1 +x
1 x
=
(1 +a)(1 +b)
(1 a)(1 b)
=
1 +a +b +ab
1 b a +ab
x =
a +b
1 +ab
Exercise 2.
(1) log (1 +x) = log (1 x)
= x = 0
87
(2) log (1 +x) = 1 + log (1 x)
ln (1 +x) = 1 + ln (1 x) = ln (e) + ln (1 x) ln e(1 x)
=1 +x = e ex = x =
e 1
1 +e
(3) 2 log x = xlog 2
ln x
2
+ ln 2
x
= 0 = ln 1 = ln x
2
2
x
= x = 2
(4) log (

x +

x + 1) = 1

x + 1 = e

x =x + 1 = e
2
2

xe +x
2

xe = e
2
1 = x =
_
e
2
1
2e
_
2
Exercise 3.
f =
ln x
x
f

=
1 ln x
x
2
f

=
2
x
3

_
1
x
x
2
2xln x
x
4
_
=
2 ln x 3
x
3
f

< 0 when x > e f

> 0 when 0 < x < e


for x
3
> 0, 2 ln x 3 > 0 =f

(x) < 0 (concave), when 0 < x < e


3/2
;
f

(x) > 0 (convex), when x > e


3/2
Exercise 4. f(x) = log (1 +x
2
)
f

=
2x
1 +x
2
Exercise 5. f(x) = log

1 +x
2
f

=
x
1 +x
2
Exercise 6. f(x) = log

4 x
2
f

=
1
2
2x
4 x
2
=
x
4 x
2
Exercise 7. f(x) = log (log x)
f

=
1
ln x
_
1
x
_
=
1
xln x
Exercise 8. f(x) = log x
2
log x
f

= (2 log x + log log x)

=
2
x
+
1
xlog x
Exercise 9. f(x) =
1
4
log
x
2
1
x
2
+1
f

=
1
4
_
log x
2
1 log x
2
+ 1
_

=
1
4
_
2x
x
2
1

2x
x
2
+ 1
_
= x
_
1
x
4
1
_
Exercise 10. f(x) = (x +

1 +x
2
)
n
ln f = nln (x +
_
1 +x
2
)
f

f
= n
_
1
x +

1 +x
2
__
1 +
x

1 +x
2
_
=
n

1 +x
2
f

= (x +
_
1 +x
2
)
n
n

1 +x
2
Exercise 11. f(x) =

x + 1 log (1 +

x + 1)
88
f

=
1
2

x + 1

1
1 +

x + 1
_
1
2

x + 1
_
=
1
2(1 +

x + 1)
Exercise 12. f(x) = xlog (x +

1 +x
2
)

1 +x
2
f

= log (x +
_
1 +x
2
) +
x
x +

1 +x
2
_
1 +
x

1 +x
2
_

1 +x
2
Note to self: Notice how this had made some of the square root terms disappear.
Exercise 13. f(x) =
1
2

ab
log

a+x

ax

b
f =
1
2

ab
(ln (

a +x

b) ln (

a x

b))
f

= frac12

ab
_
1

a +x

b
1

a x

b
(

b)
_
=
x

a(a bx
2
)
Exercise 14. f(x) = x(sin (log x) cos log x)
f

= sin (ln x) cos (ln x) + (cos (ln x) + sin (ln x)) = 2 sin (ln x)
Exercise 15. f(x) = log
1
x
f

=
1
x(ln x)
2
Exercise 16.
_
dx
2+3x
=
1
3
ln (2 + 3x)
Exercise 17.
_
log
2
xdx
(xln
2
x)

= ln
2
x + 2 ln x
(xln x x)

= ln x + 1 1 = ln x
=xln
2
x 2(xln 2 x) = xln
2
x 2xln x + 2x
Exercise 18.
_
xlog xdx
_
x
2
ln x
2
_

= xln x +
x
2
_
xln x =
x
2
ln x
2

x
2
4
Exercise 19.
_
xlog
2
xdx
_
x
2
ln
2
x
2
_
= xln
2
x +x
2
ln x
_
1
x
_
=
_
xlog
2
x =
x
2
ln
2
x
2

x
2
ln x
2

x
2
4
Exercise 20.
_
e
2
1
0
dt
1+t
_
e
3
1
0
dt
1 +t
= ln (1 +t)[
e
3
1
0
= 3
Exercise 21.
_
cot xdx
_
cos x
sin x
dx = ln [ sin x[
Exercise 22.
_
x
n
log (ax)dx Solve the problem directly.
89
_
x
n
log ax =
_
x
n
log a +
_
x
n
log x =
x
n+1
n + 1
log a +
_
x
n
log x
_
x
n
ln x =
x
n+1
n + 1
ln x
_
x
n+1
n + 1
1
x
=
x
n+1
n + 1
log x
x
n+1
(n + 1)
2
=
_
x
n
log ax =
x
n+1
n + 1
log a +
x
n+1
n + 1
log x
x
n+1
(n + 1)
2
Exercise 23.
_
x
2
log
2
xdx
_
x
2
log
2
x =
1
3
x
3
ln
2
x
_
x
2
3
2 ln x =
x
3
ln
2
x
3

2
3
_
x
3
ln x
3

x
3
9
_
=
x
3
ln
2
x
3

2x
3
ln x
9
+
2x
3
27
Exercise 24.
_
dx
x log x
_
dx
xln x
= ln (ln x) +C
Exercise 25.
_
1e
2
1
log (1t)
1t
dt
_
1e
2
0
ln (1 t)
1 t
dt =
1
2
(ln (1 t))
2

1e
2
0
= 2
Exercise 26.
_
log |x|
x

1+log |x|
dx
_
log x
x

1 + log x
=
_
(2(1 + log x)
1/2
)

log x = 2(1 + log x)


1/2
log x
_
2(1 + log x)
1/2
x
=
= 2 log x(1 + log x)
1/2

4
3
(1 + log x)
3/2
Exercise 27. Derive
_
x
m
log
n
xdx =
x
m+1
m+ 1
ln
n
x
n
m+ 1
_
x
m
ln
n1
x
By inspection, we just needed integration by parts.
_
x
3
ln
3
x =
x
4
4
ln
3
x
3
4
_
x
3
ln
2
x =
x
4
4
ln
3
x
3
4
_
x
4
4
ln
2
x
2
4
_
x
3
ln x
_
=
x
4
ln
3
x
4

3x
4
ln
2
x
16
+
3x
4
ln x
32

3x
4
128
Exercise 28. Given x > 0, f(x) = x 1 ln x; g(x) = ln x 1 +
1
x
(1)
f

= 1
1
x
g

=
1
x

1
x
2
=
1
x
f

xg

= f

so then if f

> 0, g

> 0; f

< 0, g

< 0
For f

< 0 0 < x < 1 f

> 0 x > 1 f

(1) = g

(1) = 0
f(1) = g(1) = 0
x 1 ln x > 0 since f(1) = 0 is a rel. min.
0 < ln x 1 +
1
x
since g(0) is a rel. min.
(2) See sketch.
Exercise 29. lim
x0
log (1+x)
x
= 1
(1) L(x) =
_
x
1
1
t
dt ; L

(x) =
1
x
; L

(1) = 1
(2) Use this theorem.
90
Theorem 19 (Theorem I.31).
If 3 real numbers a, x, and y satisfy the inequalities
a x a +
y
n
n 1, n Z, then x = a
1
1
x
< ln x < x 1 =1
1
x + 1
< ln x + 1 < x
1
x
1 +x
<
ln x + 1
x
< 1 =
ln (1 +x)
x
= 1
Exercise 30. Using f(xy) = f(x) +f(y),
r =
p
q
=f(a
p/q
) = f((a
1/q
)
p
) = pf(a
1/q
) =
p
q
f(a)
since
f(a) = f(a); f(a
2
) = f(aa) = 2f(a)
f(a
p+1
) = f(a
p
) +f(a) = (p + 1)f(a)
If f(a) = f((a
1/q
)q) = qf(a
1/q
)
then f(a
1/q
) =
1
q
f(a)
Exercise 31. ln x =
_
|x|
1
1
t
dt
(1) ln x =
_
|x|
1
1
t
dt From this denition, then for n partitions, a
0
= 1, a
1
= 1 +
b1
n
, . . . , a
n
= b = x
ba
n
=
b1
n
; so if a
k
= 1 +k
_
b1
n
_
n

k=1
_
a
k
a
k1
a
k
_
< log x <
n

k=1
_
a
k
a
k1
a
k1
_
(2) log x is greater than the step function integral consisting of rectangular strips within
1
x
and less than rectangular strips
covering over
1
x
(3) a
k
= 1 +k =
a
k
a
k1
a
k1
=
1
k
n

k=1
1
1 +k
< ln (n + 1) <
n

k=1
1
k
=
n+1

k=2
1
k
< ln (n + 1) <
n

k=1
1
k
=
n

k=2
1
k
< ln (n) <
n1

k=1
1
k
Exercise 32.
(1) L(x) =
1
ln b
ln x = log
b
x
log
a
x = c log
b
x =log
a
a = 1 (There must be a unique real number s.t. L(a) = 1 )
log
a
x =
1
log
b
a
log
b
x =log
b
x = log
b
a log
a
x
(2) Changing labels for a, b: log
b
x =
log
a
x
log
a
b
Exercise 33. log
e
10 = 2.302585
log
10
e =
log
e
e
log
e
10
=log
e
10 =
1
log
10
e
=
1
2.302585
0.43429
Exercise 34. Given
_
xy
x
f(t)dt = B(y) f(2) = 2, , We Want A(x) =
_
x
1
f(t)dt
91
_
xy
x
f(t)dt = B(xy) B(x)
d
dx
_
xy
x
f(t)dt =
d
dx
(B(xy) B(x)) =
dB(xy)
d(xy)
y
dB(x)
dx
= f(xy)y f(x) = 0
=f(xy) =
f(x)
y
A(x) =
_
x
1
f(t)dt =
_
x
1
f((2)
_
t
2
_
)dt =
_
x
1
f(2)
(t/2)
dt = 4 ln x
Exercise 35. Given
_
xy
1
f(t)dt = y
_
x
1
f(t)dt +x
_
y
1
f(t)dt, and letting F be the antiderivative of f,
F(xy) F(1) = y(F(x) F(1)) +x(F(y) F(1))
d/dx
f(xy)(y) = y(f(x)) +
_
y
0
f(t)dt
d/dx
f

(xy)y
2
= y(f

(x))
x=1
f

(y) =
(f

(1))
y

f(y) = k ln y +C
y=1
f(1) = 0 +C = 3
_
xy
1
(k ln t + 3) = y
_
x
1
(k ln t + 3) +x
_
y
1
(k ln t + 3)
k ((xy) ln xy (xy) + 1) + 3(xy 1) = y (k ((x) ln x x + 1) + 3(x 1)) + x(k (y ln y y + 1) + 3(y 1))
=k 3 = ky 3y kxy +kx + 3xy 3x
y(3 k) +xy(k 3) +k 3 + (3 k)x = 0 =k = 3
=f(x) = 3 ln x + 3
Exercise 36.
6.11 Exercises - Polynomial approximations to the logarithm.
Exercise 1.
Theorem 20 (Theorem 6.5). If 0 < x < 1 and if m 1,
ln
1 +x
1 x
= 2(x +
x
3
3
+ +
x
2m1
2m1
) +R
m
(x)
where
x
2m+1
2m+ 1
< R
m
(x)
2 x
1 x
x
2m+1
2m+ 1
R
m
(x) = E
2m
(x) E
2m
(x)
where E
2m
(x) is the error term for log 1 x
m = 5, x =
1
3
ln
_
4/3
2/3
_
= ln 2 2
_
1
3
+
_
1
3
_
3
3
+
_
1
3
_
5
5
+
_
1
3
_
7
7
+
_
1
3
_
9
9
_
0.693146047
The error for m = 5, x =
1
3
is
_
1
3
_
11
11
R
5
(x)
5
2
_
1
3
_
11
11
Exercise 2. ln
_
1+x
1x
_
= ln
3
2
= ln 3 ln 2
92
x =
1
5
m = 5
ln
_
1 +x
1 x
_
= 2
_
1
5
+
(1/5)
3
3
+
(1/5)
7
7
+
(1/5)
9
9
_
= 0.405465104
(1/5)
11
11
0.000000002 < R
5
(x)
9
4
_
1
5
_
11
11
0.000000004
=
log 3 1.098611
1.098611666 < log 3 < 1.098612438
Exercise 3. x =
1
9
ln
_
1+
1
9
1
1
9
_
= ln
_
10
8
_
= ln
5
4
= ln 5 2 ln 2
For m = 2
2
_
1
9
+
_
1
9
_
5
3
+
_
1
9
_
5
5
_
= 0.2231435
_
1
9
_
7
7
0.00000003 < R
3
(x)
17
9
8
9
_
1
9
_
7
7
0.000000063
1.609437 < log 5 < 1.609438
Exercise 4. x =
1
6
ln
_
1+
1
6
1
1
6
_
= ln
_
7
5
_
= ln 7 ln 5.
For m = 3,
ln
1 +x
1 x
2
_
1
6
+
_
1
6
_
3
3
+
_
1
6
_
5
5
_
= 0.336471193
The error bounds are
_
1
6
_
2(3)+1
2(3) + 1
= 0.00000051
_
1
6
_
2(3)+1
2(3) + 1
_
2
1
6
1
1
6
_
= 0.000001123
1.945908703 < ln 7 < 1.945910316
Exercise 5.
0.6931460 < ln 2 < 0.6931476
ln 3 = 1.098614
ln 4 = 1.386293
ln 5 = 1.609436 ln 6 = ln 2 + ln 3 = 1.791700
ln 7 = 1.945909
ln 8 = 3 ln 2 = 2.0794404
ln 9 = 2 ln 3 = 2.197228
ln 10 = ln 5 + ln 2 = 1.302577
6.17 Exercises - The exponential function, Exponentials expressed as powers of e, The denition of e
x
for arbitrary
real x, The denition of a
x
for a > 0 and x real, Differentiation and integration formulas involving exponentials.
Exercise 1. f

= 3e
3x1
Exercise 2. 8xe
4x
2
Exercise 3. 2xe
x
2
Exercise 4.
1
2

x
e

x
Exercise 5.
1
x
2
e
1/x
Exercise 6. ln 22
x
Exercise 7. (2xln 2)2
x
2
93
Exercise 8. cos xe
sin x
Exercise 9. 2 cos xsin xe
cos
2
x
Exercise 10.
1
x
e
log x
Exercise 11. e
x
e
e
x
Exercise 12. e
e
e
x
(e
x
e
e
x
)
Exercise 13.
_
xe
x
dx = xe
x
e
x
Exercise 14.
_
xe
x
dx = xe
x
+e
x
Exercise 15.
_
x
2
e
x
= x
2
e
x
2xe
x
+ 2e
x
Exercise 16.
_
x
2
e
2x
dx =
x
2
e
2x
2
+
xe
2x
2
+
e
2x
4
Exercise 17.
_
e

x
= e

x
(2

x)
_
1

x
e

x
= e

x
(2

x) 2e

x
= 2(

xe

x
e

x
)
Exercise 18.
_
x
3
e
x
2
.
x
2
e
x
2
= 2x
3
e
x
2
+ 2xe
x
2
e
x
2
= 2xe
x
2
1
2
(x
2
e
x
2
+e
x
2
)

=
1
2
_
2xe
x
2
+2x
3
e
x
2
+2xe
x
2
_
=
x
2
e
x
2
+e
x
2
2
Exercise 19. e
x
= b +
_
b
a
e
t
dt = b +e
x
e
a
, e
a
= b.
Exercise 20.
A =
_
e
ax
cos bxdx
B =
_
e
ax
sin bxdx
A =
e
ax
a
cos bx
_

b sin bxe
ax
a
=
e
ax
a
cos bx +
b
a
B =aA+bB = e
ax
cos bx +C
B =
e
ax
a
sin bx
_
e
ax
a
b cos bx =aB +bA = e
ax
sin bx +C
A =
1
a
2
+b
2
(ae
ax
cos bx +be
ax
sin bx)
B =
be
ax
cos bx +ae
ax
sin bx
a
2
+b
2
Exercise 21.
ln f = xln x;
f

f
= ln x + 1; f

= x
x
(ln x + 1)
Exercise 22.
ln
f

f
=
1
1 +x
+
1
1 +e
x
2
(2xe
x
2
), f

= 1 +e
x
2
+ 2(1 +x)xe
x
2
Exercise 23. f =
e
x
e
x
e
x
+e
x
.
94
ln f = ln (e
x
e
x
) ln (e
x
+e
x
)
f

f
=
1
e
x
e
x
(e
x
+e
x
)
1
e
x
+e
x
(e
x
e
x
)
f

= 1
_
e
x
e
x
e
x
+e
x
_
2
Exercise 24. f

= (x
a
a
)

+ (a
x
a
)

+ (a
a
x
)

.
f
1
= x
a
a
ln f
1
= a
a
ln x
f

1
f
1
=
a
a
x
f

1
= x
a
a
1
a
a
f
2
= a
x
a
ln f
2
= x
a
ln a
f

2
f
2
= ax
a1
ln a
f

2
= a
x
a
+1
x
a1
ln a
f
3
= a
a
x
ln f
3
= a
x
ln a
f

3
f
3
= (ln a)
2
a
x
f

3
= (ln a)
2
a
x+a
x
=f

= x
a
a
1
a
a
+a
x
a
+1
x
a1
ln a + (ln a)
2
a
x+a
x
Exercise 25.
e
f
= ln (ln x);
f

e
f
=
1
ln x
_
1
x
_
f

=
1
ln (ln x)
_
1
ln x
__
1
x
_
Exercise 26. e
f
= e
x
+

1 +e
2x
(e
f
)f

= e
x
+
e
2x

1 +e
2x
f

1 +e
2x
e
x
+e
2x

1 +e
2x
(e
x
+

1 +e
2x
)
=
e
x

1 +e
2x
Exercise 27. ln f = x
x
ln x
f

f
= (x
x
)

ln x +
x
x
x
= (x
x
(ln x + 1)) ln x +x
x1
f

= x
x+x
x
(ln x + 1) ln x +x
x
x
x
+x1
Exercise 28. ln f = xln (ln x)
f

f
= ln (ln x) +
1
ln x
f

= (ln x)
x
(ln (ln x) +
1
ln x
)
Exercise 29. ln f = (ln x) ln x
f

f
=
2 ln x
x
f

= 2x
log x1
ln x
Exercise 30. ln f = xln (ln x) ln xln x = xln ln x (ln x)
2
f

f
= ln (ln x) +
1
ln x
2
ln x
x
f

=
(ln x)
x
x
ln x
_
ln ln x +
1
ln x

2 ln x
x
_
Exercise 31.
95
ln f
1
= cos xln sin x
f

1
f
1
= sin xln sin x +
cos
2
x
sin x
f

1
= sin xcos x(ln sin x)
2
+
cos
3
xln sin x
sin x
ln f
2
= sin xln cos x
f

2
f
2
= cos xln cos x +
sin
2
x
cos x
f

2
= sin xcos x(ln cos x)
2

sin
3
xln cos x
cos x
=f = sin xcos x((ln sin x)
2
+ (ln cos x)
2
) +
cos
3
xln sin x
sin x
+
sin
3
xln cos x
cos x
Exercise 32. ln f =
1
x
ln x
f

f
=
1
x
2
ln x +
1
x
2
=f

= x
1/x2
ln x +x
1/x2
Exercise 33. ln f = 2 ln x +
1
3
ln (3 x) ln (1 x) +
2
3
ln (3 +x)
f

f
=
2
x
+
1
3(3 x)

1
1 x

2
3
1
3 +x
f

= 2
x(3 x)
1/3
(1 x)(3 +x)
2/3
+
1
3
_
x
2
(3 x)
2/3
(1 x)(3 +x)
2/3
_
+
x
2
(3 x)
1/3
(1 x)
2
(3 +x)
2/3

2
3
x
2
(3 x)
1/3
(1 x)(3 +x)
5/3
=
x(18 12x +
4
3
x
2
+
2
3
x
3
)
(1 x)
2
(3 +x)
5/3
(3 x)
2/3
Exercise 34. ln f =

n
i=1
b
i
ln (x a
i
)
f

f
=
n

i=1
b
i
x a
i
= f

=
n

j=1
b
j
x a
j
n

i=1
(x a
i
)
bi
Exercise 35.
(1) Show that f

= rx
r1
for f = x
r
holds for arbitrary real r.
x
r
= e
r ln x
(e
r ln x
)

= e
r ln x
r
x
= rx
r1
(2) For x 0, by inspection of x
r
= e
r log x
, then if x
r
> 0, then the equality would remain valid. So then x
r
= [x
r
[ =
[x[
r
and so
ln [f(x)[ = r ln [x[
f

(x)
f(x)
= r
1
x
=f

(x) = rx
r1
Exercise 36. Use the denition a
x
= e
x log a
(1) log a
x
= xlog a
Taking the exponential is a well-dened inverse function to log so taking the log of both sides of the denition, we
get log a
x
= xlog a
(2) (ab)
x
= a
x
b
x
(ab)
x
= e
x log ab
= e
x(log a+log b)
= a
x
b
x
(3) a
x
a
y
= a
x+y
a
x+y
= e
(x+y) log a
= e
x log a
e
y log a
= a
x
a
y
(4) (a
x
)
y
= (a
y
)
x
= a
xy
(a
x
)
y
= e
xy log a
= (a
y
)
x
= a
xy
96
(5) If x = log
a
y,
Using the denition
log
b
x =
log x
log b
if b > 0, b ,= 1, x > 0
so then
log
a
y =
log y
log a
= x =
log y = xlog a
e
x log a
= e
log y
= y = a
x
If y = a
x
,
log y = xlog a =x =
log y
log a
= log
a
y
Exercise 37. Let f(x) =
1
2
(a
x
+a
x
) if a > 0.
f(x +y) =
1
2
(a
x+y
+a
(x+y)
)
f(x +y) +f(x y) =
1
2
_
a
x+y
+a
(x+y)
+a
xy
+a
(xy)
_
f(x)f(y) =
1
4
(a
x
+a
x
)(a
y
+a
y
) =
1
4
_
a
x+y
+a
xy
+a
(xy)
+a
(xy)
_
Exercise 38.
f(x) = e
cx
; f

(x) = ce
cx
; f

(0) = c
lim
x0
e
cx
1
x
= c
_
lim
x0
e
cx
1
cx
_
= c
_
lim
cx0
e
cx
1
cx
_
= c
df(cx)
d(cx)
(0) =
d
dx
(e
cx
)(0)
lim
x0
e
cx
1
x
= f

(0) = c
Exercise 39.
g(x) = f(x)e
cx
g

(x) = f

e
cx
+cg = cg cg = 0
f = Ke
kx
Exercise 40. Let f be a function dened everywhere on the real axis. Suppose also that f satises the functional equation
f(x +y) = f(x)f(y) for all x and y
(1)
f(0) = f(0)f(0) = f
2
(0)
If f(0) = 0, then were done
If f(0) ,= 0 then f(0) = 1 (by dividing both sides by f(0) )
(2) Take the derivative with respect to x on both sides of the functional equation.
df(x +y)
d(x +y)
d(x +y)
dx
=
df(x)
dx
f(y) =
d(f(x +y))
d(x +y)
= f

(x)
f(x +y)
f(x)
Let y = x +y
df(y)
dy
= f

(x)
f(y)
f(x)
=f

(x)f(y) = f

(y)f(x)
(3)
f

(x)
f(x)
=
f

(y)
f(y)
x, y
The only way they could do that for any arbitrary x, for any arbitrary y they one could choose on either side, is for
them to both equal a constant
=
f

(y)
f(y)
= c
(4) Referring to Exercise 39 of the same section, f = e
cx
since f

(0) = 1
Exercise 41.
97
(1)
f = e
x
1 x
f

= e
x
1 0 if x 0
f(0) = e
0
1 0 = 0
_
e
x
> 1 +x for x > 0
e
x
> 1 x for x < 0
(2)
_
x
0
e
t
= e
x
1 > x +
1
2
x
2
=e
x
> 1 +x +
1
2
x
2
e
x
> 1 +x
x
2
2
=e
x
< 1 x +
x
2
2
(3)
_
x
0
e
t
= e
x
1 > x +
1
2
x
2
+
1
3 2
x
3
=e
x
> 1 +x +
1
2
x
2
+
1
3 2
x
3
e
x
> 1 +x
x
2
2
+
x
3
3 2
=e
x
< 1 x +
x
2
2

x
3
3 2
(4) Suppose the nth case is true.
e
x
>
n

j=0
x
j
j!
e
x
=
_
>

2m+1
j=0
x
j
j!
<

2m
j=0
x
j
j!
e
x
> 1 +
n

j=0
x
j+1
(j + 1)!
= 1 +
n+1

j=1
x
j
j!
=
n+1

j=0
x
j
j!
e
x
+ 1
_
>

2m+1
j=0
x
j+1
(j+1)!
<

2m
j=0
x
j+1
(j+1)!
= e
x
_
<

2m+2
j=0
x
j
j!
>

2m+1
j=0
x
j
j!
Exercise 42. Using the result from Exercise 41,
_
1 +
x
n
_
n
=
n

j=0
_
n
j
_
1
nj
_
x
n
_
j
=
n

j=0
n!
(n j)!j!
x
j
n
j
=
n

j=0
n(n 1) . . . (n j + 1)
j!
x
j
n
j
<
n

j=0
x
j
j!
< e
x
If you make this clever observation, the second inequality is easy to derive.
x > 0
x
n
> 0
e

x
n
> 1
x
n
=
_
e
x/n
_
n
>
_
1
x
n
_
n
e
x
> (1 x/n)
n
=e
x
< (1 x/n)
n
Exercise 43. f(x, y) = x
y
= e
y ln x

x
f = x
y
y/x

y
f = x
y
ln x
6.19 Exercises - The hyperbolic functions.
Exercise 7.
2 sinh xcosh x = 2
e
x
e
x
2
e
x
+e
x
2
=
1
2
(e
2x
e
x
) = sinh 2x
Exercise 8.
cosh
2
x + sinh
2
x =
_
e
x
+e
x
2
_
2
+
_
e
x
e
x
2
_
2
=
1
4
(e
2x
+ 2 +e
2x
+e
2x
2 +e
2x
) = cosh 2x
Exercise 9.
cosh x + sinh x =
e
x
+e
x
2
+
e
x
e
x
2
= e
x
Exercise 10.
cosh x sinh x =
e
x
+e
x
2

_
e
x
e
x
2
_
= e
x
Exercise 11. Use induction.
98
(cosh x + sinh x)
2
= cosh
2
x + 2 sinh xcosh x + sinh
2
x = cosh 2x + sinh 2x
(cosh x + sinh x)
n+1
= (cosh x + sinh x)(cosh nx + sinh nx) =
= cosh nxcosh x + cosh nxsinh x + sinh nxcosh x + sinh xsinh nx =
=
e
nx
+e
nx
2
e
x
+e
x
2
+
e
nx
+e
nx
2
e
x
e
x
2
+
+
e
nx
e
nx
2
e
x
+e
x
2
+
e
nx
e
nx
2
e
x
e
x
2
= cosh (n + 1)x + sinh (n + 1)x
Exercise 12.
cosh 2x = cosh
2
x + sinh
2
x = 1 + 2 sinh
2
x
6.22 Exercises - Derivatives of inverse functions, Inverses of the trigonometric functions.
Exercise 1.
(cos x)

= sin x =
_
1 cos
2
xDarccos x =
1

1 x
2
1 < x < 1
Exercise 2.
(tan x)

= sec
2
x =
sin
2
x + cos
2
x
cos
2
x
= tan
2
x + 1
Darctan x =
1
1 +x
2
Exercise 3.
(cot x)

= csc
2
x =
(sin
2
x + cos
2
x)
sin
2
x
= (1 + cot
2
x) =arccotx =
1
1 +x
2
Exercise 4.
(sec y)

= tan y sec y =
_
sec
2
y 1 sec y; [ sec y[ > 1 y R
If we choose to restrict y such that 0 y , then (sec y)

> 0. Then we must make sec y [ sec y[.


Darcsecx =
1
|x|

x
2
1
Exercise 5.
(csc y)

= cot xcsc x = csc y(


_
csc
2
y 1)
Let y such that

2
< y <

2
(csc y) < 0
Darccscx =
1
[x[

x
2
1
Exercise 6.
(xarccotx)

= arccotx
x
1 +x
2
_
1
2
ln (1 +x
2
)
_

=
1x
(1 +x
2
)
_
arccotx = xarccotx +
1
2
ln (1 +x
2
) +C
Exercise 7.
(xarcsecx)

= arcsecx +
x
[x[

x
2
1
_
x
[x[
ln [x +
_
x
2
1[
_

=
_

_
1+
x

x
2
1
|x+

x
2
1|
x > 1

1+
x

x
2
1
|x+

x
2
1|
x < 1
=
x
[x[

x
2
1
=
_
arcsecxdx = xarcsecx
x
[x[
log [x +
_
x
2
1[ +C
99
Take a note of this exercise. When dealing with (x
2
1)
2j+1
2
; j Z; try x

x
2
1 combinations. Itll work out.
Exercise 8.
(xarccscx)

= arccscx +
x
[x[

x
2
1
_
x
[x[
ln [x +
_
x
2
1[
_

=
_
_
_
1
x+

x
2
1
_
1 +
x

x
2
1
_
=
1

x
2
1
x > 1
1

x
2
1
x < 1
=
_
arccscx = xarccscx +
x
[x[
ln [x +
_
x
2
1[
Exercise 9.
(x(arcsin x)
2
)

= (arcsin x)
2
+
2xarcsin x

1 x
2
_
_
1 x
2
arcsin x
_

=
x

1 x
2
arcsin x + 1
_
(arcsin x)
2
= x(arcsin x)
2
+ 2
_
1 x
2
arcsin x 2x
Exercise 10.
_
arcsin x
x
_

=
1
x
2
arcsin x
1
x

1 x
2
I would note how x is in the denominator of the second term. Again, reiterating,
(
_
1 x
2
)

=
x

1 x
2
(y
_
1 x
2
)(y
_
1 x
2
) = y
2
(1 x
2
)
Multiply by its conjugate. As we see, choose y appropriately to get the desired denominator (thats achieved after differen-
tiation). Here, pick y = 1.
(ln (1 +
_
1 x
2
))

=
1
1 +

1 x
2
_
x

1 x
2
_
=
x(1

1 x
2
)

1 x
2
(x
2
)
=
(1

1 x
2
)
x

1 x
2
=
_
arcsin x
x
2
= ln

1 x
2
x

arcsin x
x
+C
Exercise 11.
(1)
D
_
arccotx arctan
1
x
_
=
1
x
2
+ 1

1
1 +
_
1
x
_
2
_
1
x
2
_
= 0
(2) arccotx arctan
1
x
= C
Now arccotx =

2
arctan x.

2
arctan x arctan
1
x
= C =

2
C = arctan x + arctan
1
x
x =

2
C =

2
+ 0 =C = 0
but x

2
C =

2
+ 0 =C =
There are problems with the choice of brances for arccotx, arctan
1
x
, even though the derivatives work in all cases.
100
Exercise 12.
f

=
1
_
1
_
x
2
_
2
_
1
2
_
Exercise 13.
f

=
1
_
1
_
1x

2
_
2
_
1

2
_
=
1

1 + 2x x
2
Exercise 14. f = arccos
1
x
.
f

=
1
_
1
_
1
x
_
2
_
1
x
2
_
=
1

x
2
1[x[
Exercise 15.
f(x) = arcsin (sin x) =
1
_
1 sin
2
x
cos x =
cos x
[ cos x[
Exercise 16.
1
2 sqrtx

1
x + 1
1
2

x
=

x
2(x + 1)
Exercise 17.
1
1 +x
2
+
x
2
1 +x
6
Exercise 18.
1
_
1
__
1x
2
1+x
2
_
2
_
2x(2)
(1 +x
2
)
2
_
=

1 +x
2
_
1 +x
2
(1 x
2
)
4x
(1 +x
2
)
2
=
4
(1 +x
2
)
3/2

2
Exercise 19. f = arctan tan
2
x
1
1 + tan
4
x
_
2 tan xsec
2
x
_
=
2 tan xsec
2
x
1 + tan
4
x
Exercise 20.
f

=
1
1 + (x +

1 +x
2
)
2
(1 +
x

1 +x
2
) =

1 +x
2
+x
1 + (x +

1 +x
2
)
2
Exercise 21.
f

=
1
_
1 (sin
2
x + cos
2
x 2 sin xcos x)
(cos x + sin x) =
cos x + sin x

2 sin xcos x
Exercise 22.
f

= (arccos
_
1 x
2
)

=
1

_
1 (1 x
2
)
=
1
[x[
Exercise 23.
f

=
1
1 +
_
1+x
1x
_
2
_
2
(1 x)
2
_
=
2
(1 x)
2
+ (1 +x)
2
=
2
(1 2x +x
2
+ 1 + 2x +x
2
)
=
1
1 +x
2
Exercise 24. f = (arccos (x
2
))
2
f

= 2(arccos x
2
)
3
_
1

1 x
2
_
(2x) =
4x(arccos x
2
)
3

1 x
4
Exercise 25.
101
f

=
_
1
arccos
1

x
_
_
_
1
_
1
1
x
_
_
_
1
2x
3/2
_
=
1
2 arccos
1

x
3
x
2
Exercise 26.
dy
dx
=
x+y
xy
.
_
arctan
y
x
_

=
1
1 +
_
y
x
_
2
_
y

x y
x
2
_
=
_
1
2
ln (x
2
+y
2
)
_

=
1
2
1
(x
2
+y
2
)
(2x + 2yy

)
=y

=
x +y
x y
Exercise 27.
ln y = ln (arcsin x)
1
2
ln (1 x
2
)
y

y
=
1
arcsin x
_
1

1 x
2
_

1
2
1
1 x
2
(2x) =
1
arcsin x

1 x
2
+
x
1 x
2
y =
arcsin x

1 x
2
y

=
_
1
1 x
2
_
+
(arcsin x)x
(1 x
2
)
3/2
=

1 x
2
+x(arcsin x)
(1 x
2
)
3/2
ln y

= ln (
_
1 x
2
+xarcsin x)
3
2
ln (1 x
2
)
y

=
1

1 x
2
+xarcsin x
_
x

1 x
2
+ arcsin x +
x

1 x
2
_

3
2
(2x)
1 x
2
y

= y

arcsin x

1 x
2
+xarcsin x
+
y

3x
1 x
2
=
arcsin x
(1 x
2
)
3/2
+
(

1 x
2
+xarcsin x)(3x)
(1 x
2
)
3/2
Exercise 28.
f

=
1
1 +x
2
1 +x
2
=
1 1 x
2
+x
4
1 +x
2
=
x
4
1 +x
2
0 x
since f(0) = arctan 0 0 + 0 = 0, arctan x > x
x
3
3
, x > 0
Exercise 29.
_
dx

a
2
x
2
, a ,= 0 = arcsin
x
a
Exercise 30.
_
dx
_
2 (x + 1)
2
=
_
dx

2
_
1
_
x+1

2
_
2
= arcsin
x + 1

2
Exercise 31.
_
dx
a
2
_
1 +
_
x
a
_
2
_ =
1
a
arctan
x
a
Exercise 32.
dx
a(1 +
_

ba

a
_
2
)
=
1

ba
arctan

bx

a
Exercise 33.
_
dx
_
x
1
2
_
2
+
7
4
=
4
7
_
dx
_
2

7
_
x
1
2
_
_
2
+ 1
=
2

7
arctan
2
_
x
1
2
_

7
102
Exercise 34.
_
x
2
arctan x
2
_

= xarctan x +
x
2
2
1
1 +x
2
= xarctan x +
1
2
_
1
1
1 +x
2
_
_
1
2
(x arctan x)
_

=
1
2
_
1
1
1 +x
2
_
_
xarctan x = xarctan x +
1
2
(x arctan x)
Exercise 35.
_
x
3
3
arccos x
_

= x
2
arccos x +
x
3
3
1

1 x
2
(x
2
_
1 x
2
)

= 2x
_
1 x
2
+
x
3

1 x
2
((1 x
2
)
3/2
)

=
3
2
(2x)(1 x
2
)
1/2
= 3x(1 x
2
)
1/2
_
x
2
arccos x =
x
3
3
arccos x
1
3
x
2
_
1 x
2

9
2
(1 x
2
)
3/2
Exercise 36.
_
x
2
(arctan x)
2
2
_

= x(arctan x)
2
+x
2
_
1
1 +x
2
_
arctan x = x(arctan x)
2
+
_
1
_
1
1 +x
2
__
arctan x
_
(arctan x)
2
2
_

=
arctan x
1 +x
2
(xarctan x)

= arctan x +
x
1 +x
2
_
x(arctan x)
2
dx =
x
2
(arctan x)
2
2

_
xarctan x
ln (1 +x
2
)
2
_
+
(arctan x)
2
2
Exercise 37.
(arctan

x)

=
_
1
1 +x
__
1
2

x
_
(xarctan

x)

= arctan

x +

x
2(1 +x)
= arctan

x +
1
2
_
1

x

1

x(1 +x)
_
(xarctan

x + arctan

x +x
1/2
)

= arctan

x + 0
_
arctan

x = xarctan

x + arctan

x x
1/2
Exercise 38. From the previous exercise,
_
arctan

x(1 +x)
dx = (arctan

x)
2
Exercise 39. Let x = sin u
_
_
1 x
2
dx =
_
cos
2
udu =
u
2
+
sin 2u
4
=
arcsin x
2
+
x

1 x
2
4
Exercise 40.
103
_
xe
arctan x
(1 +x
2
)
3/2
_
e
arctan x

1 +x
2
_

=
xe
arctan x
(1 +x
2
)
3/2
+
e
arctan x
(1 +x
2
)
3/2
_
xe
arctan x

1 +x
2
_

=
e
arctan x

1 +x
2
+
x
2
e
arctan x
(1 +x
2
)
3/2
+
xe
arctan x
(1 +x
2
)
3/2
=
e
arctan x
(1 +x
2
)
3/2
+
xe
arctan x
(1 +x
2
)
3/2
1
2
_
xe
arctan x

1 +x
2

e
arctan x

1 +x
2
_

=
xe
arctan x
(1 +x
2
)
3/2
Exercise 41. From the previous exercise,
1
2
_
xe
arctan x

1 +x
2
+
e
arctan x

1 +x
2
+C
_
Exercise 42.
Since
_
x(1 +x
2
)
1
2
_

=
x
2
(1 +x
2
)
2

1
2(1 +x
2
)
_
x
2
(1 +x
2
)
2
dx =
x
2(1 +x
2
)
+
1
2
arctan x
Exercise 43. arctan e
x
.
Exercise 44.
_
arccote
x
e
x
dx
(arccote
x
)

=
e
x
1 +e
3x
(e
x
arccote
x
)

= e
x
arccote
x
+
e
x
(1)e
x
1 +e
2x
= e
x
arccote
x
+
_
1
e
2x
1 +e
2x
_
(ln (1 +e
2x
))

=
2e
2x
1 +e
2x
(e
x
arccote
x
+x
1
2
ln (1 +e
2x
))

= e
x
arccote
x
Exercise 45.
_
_
a +x
a x
dx =
_
a +x

a
2
x
2
dx =
_
_
_
1a
_
1
_
x
a
_
2
+
x

a
2
x
2
_
_
dx = a arcsin
x
a
+
_
a
2
x
2
Exercise 46.
104
_

x a

b xdx =
_
_
bx ab x
2
+axdx =
=
_

_
x
_
a +b
2
___
x
_
a +b
2
__
+
a
2
+b
2
4

2ab
4
=
=
_

_
a b
2
_
2

_
x
_
a +b
2
__
2
=
_
a b
2
__

_
1
_
x
_
a+b
2
_
_
ab
2
_
_
2
dx =
=
_
a b
2
_
2
_
_
1 u
2
=
=
_
a b
2
_
2
arcsin
_
2x(a+b)
ab
_
2
+
2x (a +b)
2(a b)
2
_
(a b)
2
(2x (a +b))
2
Since, recall,
_
arcsin x
2
+
1
2
x
_
1 x
2
_

=
1
2
1

1 x
2
+

1 x
2
2
+
1
4
x(2x)

1 x
2
=
_
1 x
2
Exercise 47. Wow!
_
dx
_
(x a)(b x)
x a = (b a) sin
2
u
dx = (b a)(2) sin ucos udu
b x = (a b) sin
2
u +b a = (b a)(cos
2
u)
_
dx
_
(x a)(b x)
=
_
(b a)(2) sin ucos udu

b a cos u

b a sin u
= 2u = 2 arcsin
_
x a
b a
6.25 Exercises - Integration by partial fractions, Integrals which can be transformed into integrals of rational func-
tions.
Exercise 1.
_
2x+3
(x2)(x+5)
=
_
_
1
x2
_
+
_
1
x+5
_
= ln (x 2) + ln (x + 5)
Exercise 2.
_
xdx
(x+1)(x+2)(x+3)
A
x + 1
+
B
x + 2
+
C
x + 3
= A(x
2
+ 5x + 6) +B(x
2
+ 4x + 3) +C(x
2
+ 3x + 2)
=
_
_
1 1 1
5 4 3
6 3 2
_
_
_
_
A
B
C
_
_
=
_
_
0
1
0
_
_
=
_
_
1 1 1
5 4 3
6 3 2

0
1
0
_
_
=
_
_
1 0
1 0
0 1

1/2
2
3/2
_
_
A = 1/2, B = 2, C = 3/2
=
1
2
ln (x + 1) + 2 ln (x + 2) +
3
2
ln (x + 3)
Exercise 3.
_
x
(x2)(x1)
=
_
2
x2
+
1
x1
= 2 ln x 2 ln (x 1)
Exercise 4.
_
x
4
+2x6
x
3
+x
2
2x
dx
105
x
4
+ 2x 6
x
3
+x
2
2x
= x 1 +
3(x
2
2)
x
3
+x
2
2x
(do long division)
_
x 1 +
3(x
2
2)
x(x + 2)(x 1)
=
1
2
x
2
x + 3
_
x
2
2
x(x + 2)(x 1)
_
x
2
2
x(x + 2)(x 1)
=
_
1
x
+
1/3
x + 2
+
1/3
x 1
= ln x +
1
3
ln x + 2
1
3
ln x 1
=
1
2
x
2
x + 3 ln x + ln x + 2 ln x 1
Exercise 5.
_
8x
3
+7
(x+1)(2x+1)
3
dx
8x
3
+ 7
(x + 1)(2x + 1)
3
=
A
(2x + 1)
3
+
B
(2x + 1)
2
+
C
(2x + 1)
+
D
(x + 1)
8x
3
+ 7 = A(x + 1) +B(2x
2
+ 3x + 1) + (4x
3
+ 8x
2
+ 5x + 1)C +D(8x
3
+ 12x
2
+ 6x + 1)
=
_

_
0 0 4 8
0 2 8 12
1 3 5 6
1 1 1 1
_

_
_

_
A
B
C
D
_

_
=
_

_
8
0
0
7
_

_
=
_

_
1 0 0 0
1 0 0
1 0
0 1
[
12
6
0
1
_

_
A = 12, B = 6, C = 0, D = 1
_
12
(2x + 1)
3
+
6
(2x + 1)
2
+
1
x + 1
=
6(2x + 1)
2
2
+
6(2x + 1)
1
2
+ ln (x + 1)
Exercise 6.
_
4x
2
+x+1
(x1)(x
2
+x+1)
4x
2
+x + 1
(x 1)(x
2
+x + 1)
=
A
x 1
+
Bx +C
x
2
+x + 1
=A(x
2
+x + 1) + (Bx +C)(x 1) = 4x
2
+x + 1
=
_
_
1 1
1 1 1
1 0 1
_
_
_
_
A
B
C
_
_
=
_
_
4
1
1
_
_
=A = 2, B = 2, C = 1
=
_
2
x 1
+
2x + 1
x
2
+x + 1
= 2 ln [x 1[ + ln [x
2
+x + 1[
Exercise 7.
_
x
4
dx
x
4
+5x
2
+4
Doing the long division,
x
4
x
4
+5x
2
+4
= 1 +
_
5x
2
+4
(x
2
+1)(x
2
+4)
_
Ax +B
x
2
+ 1
+
Cx +D
x
2
+ 4
=
5x
2
+ 4
(x
2
+ 1)(x
2
+ 4)
It could be seen that A+C = 0, 4A+C = 0 so A = C = 0
B +D = 5
4B +D = 4
B =
1
3
D =
16
3
=
_
1
5x
2
+ 4
(x
2
+ 1)(x
2
+ 4)
= x
_
1/3
x
2
+ 1
+
16/3
x
2
+ 4
= x +
1
3
arctan x + 4/3 arctan x/2 +C
Exercise 8.
_
x+2
x(x+1)
dx =
_
1
x+1
+
2
x(x+1)
= ln [x + 1[ + 2
_
1
x

1
x+1
= ln [x + 1[ + 2 ln x
Exercise 9.
_
dx
x(x
2
+1)
2
=
_
A
x
+
Bx+C
(x
2
+1)
+
Dx+E
(x
2
+1)
2
106
A(x
4
+ 2x
2
+ 1) +x(Bx +C)(x
2
+ 1) +Dx
2
+Ex
x(x
2
+ 1)
2
=
A(x
4
+ 2x
2
+ 1) +Bx
4
+Cx
3
+Bx
2
+Cx +Dx
2
+Ex
x(x
2
+ 1)
2
=A = 1; B = 1; D = 1; C = 0; E = 0
_
1
x
+
x
x
2
+ 1
+
x
(x
2
+ 1)
2
= ln x +
ln [x
2
+ 1[
2
+
(x
2
+ 1)
1
2
Exercise 10.
_
dx
(x+1)(x+2)
2
(x+3)
3
Exercise 11.
_
x
(x+1)
2
dx
x
(x + 1)
2
=
A
x + 1
+
B
(x + 1)
2
=
x = A(x + 1) +B
A = 1; B = 1
_ _
1
x + 1
+
1
(x + 1)
2
_
dx = ln x + 1 +
1
x + 1
+C
Exercise 12.
_
dx
x(x
2
1)
=
_
dx
x(x1)(x+1)
=
_
A
x
+
B
x1
C
x+1
A(x
2
1) +Bx(x + 1) +Cx(x 1) = Ax
2
A+Bx
2
+Bx +Cx
2
Cx =A = 1, B =
1
2
= C
_
1
x
+
1/2
x 1
+
1/2
x + 1
= ln x +
1
2
ln [x 1[ +
1
2
ln [x + 1[
Exercise 13.
_
x
2
dx
x
2
+x6
=
_
x
2
dx
(x+3)(x2)
The easiest way to approach this problem is to notice that this is an improper fraction and to do long division:
x
2
x
2
+x6
= 1 +
6x
x
2
+x6
A
x + 3
+
B
x 2
=
6 = A(x 2) +B(x + 3)
A =
6
5
; B =
6
5
x = (A+B)x 2A+ 3B
2A = 3B
A =
3B
2
=
B = 2/5
A3/5
=
_
1 +
6/5
x + 3
+
6/5
x 2
+
3/5
x + 3
+
2/5
x 2
=
9
5
ln [x + 3[ +
4
5
ln [x 2[ +x +C
Exercise 14.
_
x+2
(x2)
2
=
_
x2+4
(x2)
2
= ln [x 2[ +
_
4
(x2)
2
= ln [x 2[ +4(x 2)
1
Exercise 15.
_
dx
(x2)
2
(x
2
4x+5)
Consider the denominator with its x
2
4x+5. Usually, we would try a partial fraction form such as
A
x2
+
B
(x2)
2
+
Cx+D
x
2
4x+5
,
but the algebra will get messy. Instead, it helps to be clever here.
1
(x 2)
2
(x
2
4x + 4 + 1)
=
1
(x 2)
2
((x 2)
2
+ 1)
=
1
(x 2)
2

1
(x 2)
2
+ 1
=
_
dx
(x 2)
2
(x
2
4x + 5)
=
_
1
(x 2)
2

1
(x 2)
2
+ 1
= (x 2)
1
arctan (x 2) +C
Exercise 16.
_
(x3)dx
x
3
+3x
2
+2x
=
_
(x3)dx
x(x+2)(x+1)
_
(x 3)dx
x(x + 2)(x + 1)
=
_
1
(x + 2)(x + 1)
+3
_
1
x(x + 2)(x + 1)
1
(x + 2)(x + 1)
=
1
x + 2
+
1
x + 1
1
x(x + 2)(x + 1)
=
A
x
+
B
x + 2
+
C
x + 1
107
Now to solve for A, B, C in the last expression, it is useful to use Gaussian elimination for this system of three linear
equations:
_
_
1 1 1
3 1 2
2 0 0
_
_
_
_
A
B
C
_
_
=
_
_
0
0
1
_
_
_
_
1 1 1
3 1 2
2

0
0
1
_
_
=
_
_
0 1 0
0 0 1
1 0 0

1/2
1
1/2
_
_
=
1
x(x + 2)(x + 1)
=
1/2
x
+
1/2
x + 2
+
1
x + 1
=ln [x + 2[ + ln [x + 1[ +3/2 ln x +3/2 ln [x + 2[ + 3 ln [x + 1[ = 5/2 ln [x + 2[ + 4 ln [x + 1[ 3/2 ln x
Exercise 17. Use partial fraction method to integrate
_
1
(x
2
1)
2
. Then build the sum.
A
(x 1)
2
+
B
(x + 1)
2
+
C
(x 1)
+
D
(x + 1)
Now
(x
2
1)(x 1) = x
3
x
2
x + 1
(x
2
1)(x + 1) = x
3
+x
2
x 1
=(x
2
1)(x + 1) (x
2
1)(x 1) = 2x
2
2
x
2
+ 2x + 1
x
2
2x + 1
= (summing the above two expressions we obtain) 2x
2
+ 2
1
x 1
+
1/4
x + 1
+
1/4
(x 1)
2
+
1/4
(x + 1)
2
_
dx
(x
2
1)
2
=
1
4
ln

x + 1
x 1

+
1
2
_
x
x
2
1
_
Exercise 18. Use the method of partial fractions, where we nd that
_
(x + 1)
x
3
1
dx =
_
x + 1
(x 1)(x
2
+x + 1)
dx =
_

2
3
x
1
3
x
2
+x + 1
+
2
3
x 1
=
=
1
3
ln [x
2
+x + 1[ +
2
3
ln [x 1[
where we had used the following partial fraction decomposition for the given integrand
Ax +B
x
2
+x + 1
+
C
x 1
=
x + 1
x
3
1
Ax
2
+Bx Ax B +Cx
2
+Cx +C = x + 1
2Ax +B A+ 2Cx +C = 1 (where we used the trick to take the derivative of the above equation)
=A = C B AA = 1
B +C = 1; A =
2
3
, C =
2
3
B =
1
2
Exercise 19.
_
x
4
+1
x(x
2
+1)
2
Again, it helps to be clever here.
_
x
4
+ 1
x(x
2
+ 1)
2
=
_
x
4
+ 2x
2
+ 1 2x
2
x(x
2
+ 1)
2
=
_
(x
2
+ 1)
2
x(x
2
+ 1)
2
+
2x
(x
2
+ 1)
2
=
= ln x + (x
2
+ 1)
1
+C
Exercise 20.
_
dx
x
3
(x2)
108
Working out the algebra for the partial fractions method, we obtain
1
x
3
(x 2)
=
_
1/2
x
3
_
+
1/4
x
2
+
1/8
x
+
1/8
x 2
So then
_
dx
x
3
(x 2)
=
1
4x
2
+
1
4x
+
1
8
ln x +
1
8
ln [x 2[ +C
Exercise 21.
_
1 x
3
x(x
2
+ 1)
=
_
x
3
1
x(x
2
+ 1)
=
_
x
2
x
2
+ 1
+
_
1
x(x
2
+ 1)
=
=
_ _
1
1
x
2
+ 1
_
+
_
1
x
+
x
x
2
+ 1
=
= x + arctan x + ln x ln [x
2
+ 1[ +C
Exercise 22.
_
dx
x
4
1
=
_ _
1
x
2
+ 1
__
1
x
2
1
_
=
_
1/2
x
2
1

1/2
x
2
+ 1
=
=
_
1
2
_
1/2
x 1

1/2
x + 1
_

1/2
x
2
+ 1
=
=
1
4
ln (x 1)
1
4
ln (x + 1)
1
2
arctan x +C
Exercise 23.
_
dx
x
4
+ 1
I had to rely on complex numbers.
Notice that with complex numbers, you can split up polynomial power sums
x
4
+ 1 = (x
2
+i)(x
2
i) = (x +ie(i

4
))(x ie(i

4
))(x +e(i

4
))(x e(i

4
)) =
= (x +e(i
3
4
))(x e(i
3
4
))(x +e(i

4
))(x e(i

4
))
A
(x +e(i
3
4
))
+
B
(x e(i
3
4
))
+
C
(x +e(i

4
))
+
D
(x e(i

4
))
=
1
x
4
+ 1
A(x
2
i)(x e(i
3
4
)) +B(x
2
i)(x +e(i
3
4
)) +C(x
2
+i)(x e(i

4
)) +D(x
2
+i)(x +e(i

4
)) = 1
do the algebra

x
3
: A+B +C +D = 0
x
2
: Ae(
3
4
) +Be(i
3
4
) Ce(i

4
) +De(i

4
) = 0
x
1
: iAiB +iC +iD = 0
x
0
: e(i

4
)A+Be(i

4
) +C(e(i
3
4
)) +D(e(i
3
4
)) = 1
=
_

_
1 1 1 1
e(i
3
4
) e(i
3
4
) e(i

4
) e(i

4
)
i i i i
e(i

4
) e(i

4
) e(i
3
4
) e(i
3
4
)
_

_
_

_
A
B
C
D
_

_
=
_

_
0
0
0
1
_

_
To do the complex algebra for the desired Gaussian elimination procedure, I treated the complex numbers as vectors and
added them and rotated them when multiplied.
109
_

_
1 1 1 1


0
0
0
1
_

_
=
_

_
1 1 0 0
2 2
0 0 1 1
0 0 2 2

0
0
0
1
_

_
=
_

_
1 1 0 0
2 2
0 0 1 1
0 4

0
1
_

_
=
_

_
1

0 0 1 0
0 1

1
4

1
4

1
4

1
4

_

_
=
=
_

_
1
1
1
1

1
4

1
4

1
4

1
4

_

_
=
A =
1
4
e(
i3
4
)
B =
1
4
e(
i3
4
)
C =
1
4
e(i

4
)
D =
1
4
e(i

4
)
=
_
1/4e(
i3
4
)
x +e(i
3
4
)
+
1/4e(i
3
4
)
x e(i
3
4
)
+
1/4e(i

4
)
x +e(i

4
)
+
1/4e(i

4
)
x e(i

4
)
_
1
4
__
e(i
3
4
) ln (x +e(i
3
4
)) e(i
3
4
) ln (x e(i
3
4
)) +e(i

4
) ln (x +e(i

4
)) e(i

4
) ln (x e(i

4
))
_
After doing some complex algebra,
=
1
4

2
_
ln

x
2
+

2x + 1
x
2

2x + 1

2 arctan
_
1

2x 1
_
2 arctan
_
1

2x + 1
_
_
The computation could be done to do the derivative on this, so to check our answer and reobtain the integrand.
Is there a way to solve this without complex numbers?
Exercise 24.
_
x
2
dx
(x
2
+2x+2)
2
_
x
2
dx
(x
2
+ 2x + 2)
2
=
_
x
2
+ 2x + 2 2x 2
(x
2
+ 2x + 2)
2
=
__
1
x
2
+ 2x + 2
_
+ (x
2
+ 2x + 2)
1
_
1
x
2
+ 2x + 2
=
_
1
(x + 1)
2
+ 1
= arctan (x + 1)
_
x
2
dx
(x
2
+ 2x + 2)
2
= arctan (x + 1) +
1
x
2
+ 2x + 2
+C
Exercise 25.
_
4x
5
1
(x
5
+x+1)
2
dx
((x
5
+x + 1)
1
)

= (x
5
+x + 1)
2
(5x
4
+ 1) (doesnt work)
(x(x
5
+x + 1)
1
)

= (x
5
+x + 1)
2
(5x
5
+x) +(x
5
+x + 1)
1
= (x
5
+x + 1)
2
(5x
5
+x x
5
x 1)
=
_
4x
5
1
(x
5
+x + 1)
2
dx = x(x
5
+x + 1)
2
Exercise 26.
_
dx
2 sin xcos x+5
(good example of the use of half angle substitution )
_
dx
2 sin x +cos x + 5
=
_
dx
4SC +C
2
+S
2
+ 5
_
1
C
2
1
C
2
_
=
_
sec
2 x
2
dx
4T 1 +T
2
+ 5(1 +T
2
)
=
=
_
sec
2 x
2
dx
6T
2
+ 4T + 4
=
_
sec
2 x
2
dx
6(T +
1
3
)
2
+
10
3
=
_
2du
6(u +
1
3
)
2
+
10
3
(where
u = tan
x
2
du =
sec
2 x
2
2
dx
)
=
3
5
_
du
9(u+
1
3
)
2
5
+ 1
=
1

5
arctan
_
3(tan
x
2
+
1
3
)

5
_
110
Exercise 27.
_
dx
1+a cos x
(0 < a < 1) Again, using the half-angle substitution,
u = tan
x
2
du =
sec
2 x
2
2
dx
,
1
a
_
dx
1
a
+ cos x
=
1
a
_
dx
1
a
+C
2
S
2
=
1
a
_
sec
2 x
2
dx
1
a
sec
2
x
2
+ 1 T
2
=
=
1
a
_
sec
2 x
2
dx
1
a
+ 1 +T
2
(
1
a
1)
=
1
a
_
2du
1
a
+ 1 +u
2
_
1
a
1
_ =
2
1 +a
_
du
1 +
_
u
_
1a
1+a
_
2
=
2
1 +a
arctan
_
1a
1+a
u
_
1a
1+a
==
2

1 a
2
arctan
_
_
1 a
1 +a
tan
x
2
_
Exercise 28.
_
dx
1+a cos x
Half-angle substitution.
_
dx
1 +a cos x
=
_
dx
1 +a(C
2
S
2
)
=
_
sec
2 x
2
dx
sec
2
x
2
+a(1 T
2
)
=
u = tan /2 = T
du = sec
2
/2
_
1
2
_
d
==
_
2du
1 +T
2
+a(1 T
2
)
= 2
_
du
(1 a)T
2
+ (1 +a)
=
2
1 a
_
du
u
2

a+1
a1
=
=
2
1 a
_
_
_
1
u
_
a+1
a1

1
u +
_
a+1
a1
_
_
1
2
_
a+1
a1
=
=
_
a 1
a + 1
_
1
1 a
_
_
ln (u
_
a + 1
a 1
) ln (u +
_
a + 1
a 1
)
_
=
=
1

a
2
1
_
_
ln
_
_
tan
x
2

_
a+1
a1
tan
x
2
+
_
a+1
a1
_
_
_
_
Exercise 29.
_
sin
2
x
1+sin
2
x
dx
_
s
2
1 +s
2
dx =
_
s
2
+ 1 1
1 +s
2
dx = x +
_
dx
1 + sin
2
x
_
dx
1 + sin
2
x
=
_
dx
1 +
_
1cos 2x
2
_ =
_
2dx
3 cos 2x
=
2
3
_
dx
1
cos 2x
3
=
2
3
_
dx
1
_
c
2
s
2
3
_ =
2
3
_
sec
2
xdx
sec
2
x
_
1T
2
3
_ =
u = tan x
du = sec
2
xdx
==
2
3
_
du
1 +u
2

_
1u
2
3
_ =
2
3
_
du
2
3
+
4
3
u
2
=
_
du
1 + (

2u)
2
=
=
1

2
arctan

2 tan x
=
_
sin
2
x
1 + sin
2
x
dx = x
1

2
arctan (

2 tan x)
It seems like for here, when dealing with squares of trig. functions, step up to double angle.
Exercise 30.
_
dx
a
2
sin
2
x+b
2
cos
2
x
(ab ,= 0) Take note, we need not change the angle to half-angle or double-angle.
111
1
a
2
s
2
+b
2
c
2
=
1
a
2
(1 c
2
) +b
2
c
2
=
1
a
2
+ (b
2
a
2
)c
2
=
1
a
2
(1 + (kc)
2
)
=
sec
2
a
2
(sec
2
+k
2
)
=
sec
2
a
2
(1 +T
2
+k
2
)
=
u = tan x
du = sec
2
xdx
==
du
a
2
(1 +u
2
+k
2
)
=
1/a
2
du
(1 +k
2
)(1 +
u
2
1+k
2
)
=

1 +k
2
a
2
(1 +k
2
)
arctan
_
u

1 +k
2
_
=
_
dx
a
2
sin
2
x +b
2
cos
2
x
=
1
ab
arctan
_
a tan x
b
_
Exercise 31.
_
dx
(a sin x+b cos x)
2
(a ,= 0)
Note its a good idea to simplify, cleverly, your constants as much as you can.
_
dx
(a sin x +b cos x)
2
=
1
a
2
_
dx
(sin x +k cos x)
2
Thus, only one constant, k, is only worried about.
1
(s +kc)
2
=
1
s
2
+ 2ksc +k
2
c
2
=
1/c
2
t
2
+ 2kt +k
2
=
sec
2
(t +k)
2
=
u = tan x
du = sec
2
xdx
==
du
(u +k)
2
=
1
a
2
_
1
(s +
b
a
c)
2
=
1
(a
2
tan x +ab)
Again, note, we need not always step up or step down a half angle in the substitution.
Exercise 32. Note that we have a rational expression consisting of single powers of sin and cos. Then use the tan

2
substitution.
_
sin x
1 + cos x + sin x
=
_
2CS
1 + 2CS +C
2
S
2
=
_
CS
C(S +C)
=
_
T
(T + 1)
where
C = cos x/2
S = sin x/2
u = tan /2
du =
sec
2
/2d
2
_
u
(u + 1)
_
2du
u
2
+ 1
_
=
_
2udu
(u
2
+ 1)(u + 1)
A
u + 1
+
Bu +C
u
2
+ 1
Au
2
+A+Bu
2
+Cu +Bu +C = u
A = B C +B = 1 A+C = 0 =C =
1
2
; B =
1
2
; A =
1
2
2
_ _
1/2
u + 1
+
1
2
(u + 1)
u
2
+ 1
_
du =
_
1
u + 1
+
u + 1
u
2
+ 1
= ln [u + 1[ +
1
2
ln [u
2
+ 1[ + arctan u =
= ln [ tan x/2 + 1[ +
1
2
ln [ sec
2
x/2[ +
x
2
=(ln [2[) +
1
2
ln [2[ +

4
=
1
2
ln [2[ +

4
Exercise 33.
_
3 x
2
dx
112
_
(x)

_
3 x
2
dx = x
_
3 x
2

_
x(x)

3 x
2
= x
_
3 x
2

_
x
2
+ 3 3

3 x
2
= x
_
3 x
2

_
_
3 x
2
+ 3
_
1

3 x
2
=2
_
_
3 x
2
= x
_
3 x
2
+

3
_
1
_
1
_
x

3
_
2
=
_
_
3 x
2
=
x
2
_
3 x
2
+
3
2
arcsin
x

3
Exercise 34.
_
1

3x
2
dx = (3 x
2
)
1/2
+C.
_
arccos
x

3
_

=
1

3
1
_
1
x
2
3
=
1

3 x
2
_
x
_
3 x
2
_

=
_
3 x
2
+
x
2

3 x
2
x

3 x
2
2
+
3
2
arccos
x

3
Exercise 35.
_

3x
2
x
dx =
_
_
3
x
2
1dx.

3
x
= sec

3 cos = x
dx = sin

3
_
_
sec
2
1(sin )

3 =
_
tan sin (

3) =

3
_
(sec cos ) =
=

3 ln [ sec + tan [ +

3 sin =
=

3 ln

3
x
+
_
3
x
2
1

3
_
1
x
2
3
Exercise 36.
_
_
1 +
1
x
dx
_
x
_
1 +
1
x
_

=
_
1 +
1
x
+
x
2
_
1 +
1
x
_
1
x
_
=
_
1 +
1
x
+
1/2

x
2
+x
_
ln
_
x +
1
2
+
_
x
2
+x
__

=
1
x +
1
2
+

x
2
+x
_
1 +
x +
1
2

x
2
+x
_
=
1

x
2
+x
=
_ _
1 +
1
x
_
dx = x
_
1 +
1
x
+
1
2
ln
_
x +
1
2
+
_
x
2
+x
_
Exercise 37.
(x
_
x
2
+ 6)

=
_
x
2
+ 5 +
x
2

x
2
+ 5
(ln (x +
_
x
2
+b))

=
_
1
x +

x
2
+b
__
1 +
x

x
2
+b
_
=
1

x
2
+b
_
_
x
2
+ 5 =
1
2
_
x
_
x
2
+ 5 + 5 ln (x +
_
x
2
+ 5)
_
Exercise 38.
113
_
ln (x +
1
2
+
_
x
2
+x + 1)
_

=
1
x +
1
2
+

x
2
+x + 1
_
1 +
x +
1
2

x
2
+x + 1
_
=
1

x
2
+x + 1
_
x

x
2
+x + 1
=
_
x +
1
2

1
2

x
2
+x + 1
= (x
2
+x + 1)
1/2

1
2
ln (x +
1
2
+
_
x
2
+x + 1)
The trick is to note how I formed a conjugate-able sum from x
2
+x + 1s derivative.
Exercise 39.
_
dx

x
2
+x
=
_
dx
_
_
x +
1
2
_
2

1
4
=
_
2dx
_
_
2
_
x +
1
2
__
2
1
_
ln
_
_
(2(x + 1/2))
2
1 + 2(x + 1/2)
__

=
=
1
_
(2(x + 1/2))
2
1 + 2(x + 1/2)
_
2 +
2(x + 1/2)2
_
(2(x + 1/2))
2
1
_
=
=
2
_
(2(x + 1/2))
2
1
_
dx
_
_
x +
1
2
_
2

1
4
= ln
_
2
_
x +
1
2
_
+
_
(2(x + 1/2))
2
1
_
+C
Exercise 40.
6.26 Miscellaneous review exercises. Exercise 1.
f(x) =
_
x
1
log t
t + 1
f
_
1
x
_
=
_ 1
x
1
log t
t + 1
dt =
_
x
1
ln (u)
1
u
+ 1
_
1
u
2
du
_
=
=
_
x
1
ln (u)
u +u
2
du
u =
1
t
du =
1
t
2
dt
f(x) +f
_
1
x
_
=
_
x
1
t ln t + ln t
t(t + 1)
dt =
_
x
1
ln t
t
dt =
(ln t)
2
2

x
1
=
(ln x)
2
2
f(2) +f
_
1
2
_
=
1
2
(ln 2)
2
Exercise 2. Take the derivative of both sides, using the (rst) fundamental theorem of calculus.
2ff

= f(x)
sin x
2 + cos x
; =2f

=
sin x
2 + cos x
At this point, it could be very easy to evaluate the integral by guessing at the solution.
(ln 2 + cos x)

=
sin x
2 + cos x
= f =
ln [2 + cos x[
2
+C
114
Otherwise, remember that for rational expressions involving single powers of sin and cos, we can make a u = tan /2
substitution.
u = tan
x
2
2du = sec
2
x
2
dx
C = cos x/2, S = sin x/2
_
sin x
2 + cos x
dx =
_
2SCdx
2 +C
2
S
2
=
_
2T
2 sec
2
x/2 + 1 T
2
_
2du
sec
2
x/2
_
=
= 4
_
udu
(1 +u
2
)(3 +u
2
)
= 2
_
T
T
2
+ 1

T
T
2
+ 3
= 2
_
1
2
ln T
2
+ 1
1
2
ln T
2
+ 3
_
=
_
ln
_
T
2
+ 1
T
2
+ 3
__
= ln
_
2
4 + 2 cos x
_
where tan
2
x
2
=
sin
2 x
2
cos
2
x
2
=
1 cos x
1 + cos x
Exercise 3.
_
e
x
x
dx = e
x

_
e
x
(x 1)
x
2
xdx = e
x

_
e
x
(x 1)
x
dx. . .
No way.
Exercise 4.
_
/2
0
ln (e
cos x
)dx = cos x[
3/2
0
= 1 .
Exercise 5.
(1)
f =

4x + 2x(x + 1)(x + 2)
ln f =
1
2
ln
_
4(x + 2)
x(x + 1)(x + 2)
_
=
1
2
(ln (4x + 2) ln x ln (x + 1) ln (x + 2))
f

f
=
1
2
__
4
4x + 2
_

1
x

1
x + 1

1
x + 2
_
=f

=
1
2

4x + 2
x(x + 1)(x + 2)
__
4
4x + 2
_

1
x

1
x + 1

1
x + 2
_
f

(1) =
7
12
(2)
_
4
1

4x + 2
x(x + 2)(x + 1)
dx = 2
_
4
1
(2x + 1)
x(x + 2)(x + 1)
dx =
= 2
_
1
2
ln x +
3
2
ln [x + 2[ + ln [x + 1[
_

4
1
= ln
25
8
since we can nd the antiderivative through partial fractions:
A
x
+
B
x + 2
+
C
x + 1
=
2x + 1
x(x + 2)(x + 1)
A(x
2
+ 3x + 2) +B(x
2
+x) +C(x
2
+ 2x) = 2x + 1
_
_
1 1 1
3 1 2
2 0 0
_
_
_
_
A
B
C
_
_
=
_
_
0
2
1
_
_
=
_
_
0 1 0
0 0 1
1 0 0

3/2
1
1/2
_
_
Exercise 6.
115
(1)
log x =
_
x
1
1
t
dt F(x) =
_
x
1
e
t
t
dt;
if x > 0
e
t
> 1 for t > 0
If 0 < x < 1
log x =
_
x
1
1
t
dt =
_
1
x
1
t
dt >
_
1
x
e
t
t
= F(x)
log x F(x) for x 1
(2)
F(x +a) F(1 +a) =
_
x+a
1
e
t
dt
t

_
a+1
1
e
t
dt
t
=
_
x
1a
e
t+a
dt
t +a

_
1
1a
e
t+a
dt
t +a
= e
a
_
x
1
e
t
t +a
dt
(3)
_
x
1
e
at
t
dt =
_
ax
a
e
t
t
dt =
_
ax
1
e
t
t
+
_
1
a
e
t
t
=
= F(ax) F(a)
_
x
1
e
t
t
2
=
1
t
e
t

_

1
t
e
t
=
_
e
x
x
e
_
+F(x)
_
x
1
e
1/t
dt =
_
1/x
1
e
u
u
2
du =
_
e
u
u

_

e
u
u
_
=
= xe
1/x
e F(1/x) where we used the substitution
u =
1
t
du =
1
t
2
dt
Exercise 7.
(1)
e
x
= F(x) F(0); F(x) = e
x
+F(0) =F(0) = 1 +F(0)
0 ,= 1. False.
(2)
d
dx
_
x
2
0
f(t)dt = f(x
2
)(2x) = (2x) ln 2e
x
2
ln 2
f(x) = ln 2e
x ln 2
_
x
2
0
ln 2e
ln 2
dt = e
t ln 2

x
2
0
= e
x
2
ln 2
+ 1
(3)
f(x) = 2f(x)f

(x); =f(x) =
x
2
+C
_
x
0
_
1
2
t +c
_
dt =
_
t
2
4
+ct
_

x
0
=
x
2
4
+cx
f
2
(x) 1 =
x
2
4
+Cx +C
2
1
=C = 1, f(x) =
x
2
+1
Exercise 8.
(1)
f(x +h) f(x)
h
=
f(x)f(h) f(x)
h
=
f(x)(hg(h))
h
= f(x)g(h)
g(h) 1 as h 0 so =f

(x) = f(x)
116
(2) Since for f(x) = e
x
, we dened e
x
such that f

= f, if
(e
x
+g)

= e
x
+g

= e
x
+g
=e
x
+g = Ce
x
=g = (C 1)e
x
but f

(0) = 1 so g = e
x
Exercise 9.
(1)
g(2x) = 2e
x
g(x)
g(3x) = e
x
g(2x) +e
2x
g(x) = e
x
2e
x
g +e
2x
g = 3e
2x
g
(2)
Assume g(nx) = ne
(n1)x
g
g((n + 1)x) = e
x
g(nx) +e
(n+1)x
g(x) = ne
nx
g(x) +e
nx
g = (n + 1)e
nx
g
(3) From g(x +y) = e
y
g(x) +e
x
g(y),
g(0) = g(0) +g(0) =g(0) = 0
g(x +h) g(x)
h
=
e
h
g(x) +e
x
g(h) g(x)
h
= g(x)
_
e
h
1
h
_
+
e
x
g(h)
h
f

(0) = 2 = lim
h0
g(x +h) g(x)
h
= limh 0
g(h)
h
(4) g

(x) = g(x) + 2e
x
C = 2
Exercise 10.
x R, f(x +a) = bf(x); f(x + 2a) = bf(x +a) = b
2
f(x)
f(x + (n + 1)a) = f(x +na +a) = bf(x +na) = b
n+1
f(x)
f(x +na) = b
n
f(x)
f(x) = b
x/a
g(x) where g is periodic in a
Exercise 11.
(ln (fg))

=
f

f
+
g

g
=(fg)

= f

g +fg

_
ln
_
f
g
__

=
f

f

g

g
=
f

g g

f
g
2
=
_
f
g
_

Exercise 12. A =
_
1
0
e
t
t+1
dt
(1)
_
a
a1
e
t
t a 1
dt
u = t a
=
_
0
1
e
ta
t 1
dt =
_
0
1
e
ta
t 1
dt = e
a
_
1
0
e
t
t + 1
= e
a
A
(2)
_
1
0
te
t
2
t
2
+1
dt =
_
1
0
1
2
due
u
u+1
=
1
2
A
(3)
_
1
0
e
t
(t+1)
2
dt =
e
t
(t+1)

1
0

_
1
0
e
t
t+1
=
e
1
2
+ 1 +A
(4)
_
1
0
e
t
ln (1 +t)dt = e
t
ln (1 +t)
_
e
t
1+t
= e ln 2 A
Exercise 13.
117
(1) p(x) = c
0
+c
1
x +c
2
x
2
; f(x) = e
x
p(x) p

= c
1
+ 2c
2
x p

= 2C
2
f

= f +e
x
p

f
(n)
(x) =
n

j=0
_
n
j
_
e
x
(p(x))
j
= f +e
x
(c
1
+ 2c
2
x) +
_
n!
(n 1)!
_
+e
x
(2c
2
)
_
n!
(n 2)!2!
_
f
(n)
(0) = c
0
+c
1
n +n(n 1)c
2
(2) See generalization below.
(3)
p =
m

j=0
a
j
x
j
; p(0) = a
0
; p
(k)
(x) =
m

j=0
a
j
j!
(j k)!
x
jk
= p
(k)
(0) = a
k
k!
f
(n)
(x) =
n

j=0
_
n
j
_
e
x
p
(j)
(x)
f
(n)
(0) =
n

j=0
_
n
j
_
p
(j)
(0) =
m

j=0
_
n
j
_
a
j
j! =
m

j=0
n!
(n j)!
a
j
So for m = 3, then f
(n)
(0) = a
0
+na
1
+n(n 1)a
2
+n(n 1)(n 2)a
3
Exercise 14. f(x) = xsin ax ; f
(2)
= a
2
xsin ax + 2a cos ax
f
(2n)
(x) = (1)
n
(a
2n
xsin ax 2na
2n1
cos ax)
f
(2n+1)
(x) = (1)
n
(a
2n+1
xcos ax +a
2n
sin ax + 2na
2n
sin ax)
f
(2n+2)
(x) = (1)
n
(a
2n+2
xsin ax +a
2n+1
cos ax(2n + 2))
Exercise 15.
n

k=0
(1)
k
_
n
k
_
1
k +m+ 1
=
n

k=0
(1)
k
_
n
k
__
1
0
t
k+m
dt =
_
1
0
n

k=0
(1)
k
_
n
k
_
t
k+m
dt =
=
_
1
0
t
m
n

k=0
_
n
k
_
(t)
k
dt =
_
1
0
t
m
(1 t)
n
dt =
=
_
0
1
(1 u)
m
u
n
du =
_
1
0
(1 u)
m
u
n
du =
=
_
1
0

j=0
6m
_
m
j
_
(u)
j
u
n
du =
m

j=0
(1)
j
_
m
j
__
1
0
t
j+n
dt
u1 t
du = dt
Exercise 16. F(x) =
_
x
0
f(t)dt
(1)
F(x) =
_
x
0
(t +[t[)
2
=
_
_
x
0
(2t)
2
dt =
4
3
x
3
if t, x 0
_
x
0
0dt = 0 if t, x < 0
(2)
F(x) =
_
x
0
f(t)dt =
_
_
x
0
(1 t
2
)dt if [t[ 1
_
x
0
(1 [t[)dt if [t[ > 1
=
=
_

_
_
t
1
3
t
3
_

x
0
= x
x
3
3
if [x[ 1
_
2
3
+
_
x
1
(1 t)dt = x
x
2
2
+
1
6
if x > 1
2
3
+
_
x
1
(1 +t)dt = x +
x
2
2

1
6
if x < 1
if [x[ 1
118
(3) f(t) = e
|t|
F(x) =
_
x
0
f(t)dt =
_
x
0
e
|t|
dt =
=
_
_
x
0
e
t
dt = e
t
[
0
x
= 1 e
x
if x 0
_
x
0
e
t
dt = e
t
[
x
0
= e
x
1 if x < 0
(4) f(t) = max. of 1 and t
2
F(x) =
_
x
0
f(t)dt =
_

_
_
x
0
1dt = x if [x[ 1
1 +
_
x
1
t
2
dt if x > 1

_
x
0
f if x < 1
=
=
_

_
x if [x[ 1
1 +
1
3
t
3

x
1
=
x
3
3
+
2
3
if x > 1

_
1
x
t
2
+
_
0
1
1 =
1
3
t
3

x
1
1 =
x
3
3
+
2
3
if x > 1
Exercise 17.
_
f
2
=
_
a
0
f
2
= a
2
+a.
_
x
2
+x

=
_
2x + 1

for
_
a
0
2x + 1

=
_
x
2
+x
_

a
0
= a
2
+a
Exercise 18. f(x) = e
2x
.
(1) A(t) =
_
t
0
e
2x
dx =
e
2x
2

t
0
=
e
2t
1
2
(2) V (t) =
_
t
0
e
4x
dx =

4
(e
4t
1)
(3)
y = e
2x
=
ln y
2
= x
W(t) =
_
1
e
2t
_
ln y
2
_
2
dy =

4
(y(ln y)
2
(2(y ln y y)))

1
e
2t
=
=

4
_
2
_
e
2t
4t
2
(2(e
2t
(2t) e
2t
))
__
=
=
_

2
te
2t


2
e
2t
_
where the antiderivative used was (y(ln y)
2
)

= (ln y)
2
+ 2 ln y
(4)

4
(1 e
4t
)
_
1e
2t
2
_ =

2
_
e
4t
1
t
_
e
2t
1
t
=
where we used the limit lim
x0
e
cx
1
x
= c
Exercise 19. sinh c =
3
4
(1)
e
c
= e
x
+
_
e
2x
+ 1
e
c
e
c
2
=
e
x
+

e
2x
+ 1
1
e
x
+

e
2x
+1
2
=
=
e
2x
+ 2e
x

e
2x
+ 1 +e
2x
+ 1 1
2(e
x
+

e
2x
+ 1)
= e
x
=
3
4
x = ln 3 2 ln 2
119
(2)
e
c
e
c
2
=
e
x

e
2x
1
1
e
x

e
2x
1
2
=
=
e
2x
2e
x

e
2x
1 +e
2x
1 1
2(e
x

e
2x
1)
=
e
x
+
1
e
x

e
2x
1
= e
x

1
e
c
=
3
4
= x = ln 5 2 ln 2
Exercise 20.
(1) True. ln (2
log 5
) = ln 5
ln 2
= (ln 2) ln 5.
(2) log
3
5 =
log
2
5
log
2
3
This is a true fact.
log
3
5
log
2
3
=
log
2
5
(log
2
3)
2
= log
2
5
=1 = log
2
3 False
(3) Use induction
n = 1 1
1/2
< 2

1
n = 2 1 +
1

2
< 2

2
n + 1 case
n+1

k=1
k
1/2
=
n

k=1
k
1/2
+
1
_
(n + 1)
<
< 2

n
_
_
(n + 1)
_
(n + 1)
_
+
1
_
(n + 1)
Now (n +
1
2
)
2
= n
2
+n +
1
2
> n
2
+n, certainly. So then
n +
1
2
>
_
n
2
+n =n + 1 >
_
n
2
+n
=
n+1

k=1
k
1/2
< 2

n + 1
(4)
f = (cosh x sinh x 1) =
e
x
+e
x
e
x
+e
x
2
1 = e
x
1 < 0 for x > 0
False.
Exercise 21. For 0 < x <

2
,
(sin x)

= cos x > 0 for 0 < x <



2
(x sin x)

= 1 cos x 0 for 0 < x <



2
(x sin x)(x = 0) = 0 =sin x < x
Exercise 22.
1
t
<
1
t
_
x + 1
t
_
if 0 < x < t < x + 1
_
x+1
x
1
t
dt = ln (x + 1) ln x;
_
x+1
x
x + 1
t
=
1
x
So ln
x + 1
x
<
1
x
Exercise 23.
120
(x sin x)

= 1 cos x 0 x > 0
since (x sin x)(x = 0) = 0; (x sin x)

(x = 0) = 0, then x sin x > 0 in general for x > 0


(sin x
_
x
x
3
6
_
)

= cos x 1 +
x
2
2
> 0 x > 0
=x
x
3
6
< sin x < x
Exercise 24. (x
b
+y
b
)
1/b
< (x
a
+y
a
)
1/a
if x > 0, y > 0 and 0 < a < b
(x
n
+y
n
)
1/n
= x(1 +
_
y
x
_
n
)
1/n
. Without loss of generality, assume x < y.
Consider (1 +A
n
)
1/n
, A constant.
((1 +A
n
)
1/n
)

= (exp
_
1
n
ln (1 +A
n
)
_
)

= (1 +A
n
)
1/n
_
1
n
2
ln (1 +A
n
) +
1
n
_
1
1 +A
n
_
(ln A)A
n
_
=
= (1 +A
n
)
1/n
_
(1 +A
n
) ln (1 +A
n
) +n(ln A)A
n
n
2
(1 +A
n
)
_
=
= (1 +A
n
)
1/n
_
ln (1 +A
n
) A
n
ln (1 +A
n
) +A
n
ln A
n
n
2
(1 +A
n
)
_
=
=
(1 +A
n
)
1n
n
n
2
_
_
ln (1 +A
n
) +A
n
ln
_
A
n
1+A
n
_
n
2
(1 +A
n
)
_
_
< 0 since ln
_
A
n
1 +A
n
_
< 0
=(x
b
+y
b
)
1/b
< (x
a
+y
a
)
1/a
if b > a
Exercise 25.
(1)
_
x
0
e
t
t =
_
te
t
e
t
_

x
0
= xe
x
e
x
+ 1
(2)
_
t
0
t
2
e
t
dt = t
2
e
t

x
0

_
e
t
(2t)dt = x
2
e
x
+ 2
_
te
t
dt =
= x
2
e
x
+2xe
x
2e
x
+ 2
(3)
_
x
0
t
3
e
t
dt = t
3
e
t

x
0
3
_
x
0
t
2
(e
t
)dt = x
3
e
x
+ 3
_
x
0
t
2
e
t
dt =
= x
3
e
x
+ 3(2)(e
x
)
_
e
x
1 x
x
2
2!
_
(4) Assume the induction hypothesis, that
_
x
0
t
n
e
t
dt = n!e
x
_
_
e
x

j=0
x
j
j!
_
_
_
x
0
t
n+1
e
t
dt = t
n+1
e
t

x
0

_
(n + 1)t(e
t
) = x
n+1
e
x
+ (n + 1)n!e
x
_
_
e
x

j=0
x
j
j!
_
_
= (n + 1)!e
x
_
_
e
x

n+1

j=0
x
j
j!
_
_
Exercise 26. Consider the hint a
1
sin x + b cos x = A(a sin x + b cos x) + B(a cos x b sin x). Solve for A, B in terms of
121
a
1
, b
1
, a, b. Matching up term by term the coefcients for sin and cos separately,
Aa
2
abB = aa
1
Ab
2
+Bab = b
1
b
A =
aa
1
+bb
1
a
2
+b
2
Aab +Bb
2
= a
1
b
Aab +Ba
2
= ab
1
B =
ab
1
a
1
b
a
2
+b
2
So if not both a, b = 0 ,
_
a
1
sin x +b
1
cos x
a sin x +b cos x
=
_
A(a sin x +b cos x) +B(a cos x b sin x)
a sin x +b cos x
=
= Ax +Bln [a sin x +b cos x[ +C
Exercise 27.
(1)
f

(x
2
) =
1
x
df
du
= u
1/2
f(x
2
) = 2x 1
(2)
f

(sin
2
x) = 1 sin
2
xf

(u) = 1 uf = u
1
2
u
2
+C = f(x) = x
x
2
2
+
1
2
(3)
f

(sin X) = (1 sin
2
x) f(u) = u
1
3
u
3
+C f(x) = x
x
3
3
+
1
3
(4)
f

(ln x) =
_
1 for x 1
x for x > 1
=
_
1 for 0 < x 1
e
ln x
x > 1
f(y) =
_
y for y < 0
e
y
1 for y > 0
Exercise 28.
(1)
Li(x) =
_
x
2
dt
ln t
if x 2
Li(x) =
x
ln x
2
1
ln 2

_
x
2
1
(ln x)
2
dt =
x
ln x
+
_
x
2
dt
(ln x)
2

2
ln 2
(2)
Li(x) =
x
ln x

2
ln x

2
(ln 2)
2
+
x
(ln x)
2

_
x
a
2
(ln t)
3
dt
Li(x) =
x
ln x
+
n1

k=1
k!x
ln
k+1
x
+n!
_
x
ln
n+1
x

_
x
2
(n + 1)dt
ln
n+2
t
_
Li(x) =
x
ln x
+
n

k=1
k!x
ln
k+1
x
+ (n + 1)!
_
x
2
dt
ln
(n+1)+1
t
C
2
=
2
ln x
+
2

j=2
2(j 1)!
(ln 2)
j
C
n
= 2
1
ln 2

n

j=2
2(j 1)!
(ln 2)
j
C
n+1
= 2
1
ln 2

n+1

j=2
2(j 1)!
(ln 2)
j
122
(3)
Li(x) =
_
x
2
dt
ln t
u = ln t
du =
1
t
dt
e
u
du = dt e
u
= t
Li(x) =
_
ln x
ln 2
e
t
dt
t
(4)
c = 1 +
1
2
ln 2
t = u + 1 =
_
x1
c1
e
2(u+1)
du
u
= e
2
_
x1
c1
e
2u
u
du =
= e
2
1
2
_
2(x1)
3
2(c1)
3
e
t
t
2
dt =
= e
2
Li(e
2(x1)
)
(5)
f(x) = e
4
Li(e
2(x2)
) e
2
Li(e
2(x1)
)
=
_
x
c
e
2t
t 2

_
x
c
e
2t
(t 1)
= f

(x) =
e
2x
t 2
+
e
2x
t 1
= e
2x
_
1
t
2
3t + 2
_
Exercise 29. f(x) = log [x[ if x < 0. x < 0 uniquely ln [x[ since f

=
1
x
< 0 x < 0.
e
y
= x(y) = g(y) D = R
Recall Theorem 3.10.
Theorem 21. Assume f is strictly increasing and continuous on an interval [a, b]. Let c = f(a), d = f(b) and let g be the
inverse of f.
That is y [c, d], Let g(y) be that x [a, b], such that y = f(x).
Then
(1) g is strictly increasing on [c, d]
(2) g is continuous on [c, d]
Exercise 30. f(x) =
_
x
0
(1 +t
3
)
1/2
dt if x 0.
(1)
f

(x) =
1

1 +x
3
> 0 for x > 0
(2)
g

(x) =
1
f

(x)
=
_
1 +x
3
g

(x) =
3x
2
2

1 +x
3
7.4 Exercises - Introduction, The Taylor polynomials generated by a function, Calculus of Taylor polynomials. Use
the following theorems for the following exercises.
Theorem 22 (Properties of Taylor polynomials, Apostol Vol. 1. Theorem 7.2.). (1) Linearity T
n
(c
1
f+c
2
g) = c
1
T
n
(f)+
c
2
T
n
(g)
(2) Differentiation (T
n
f)

= T
n1
(f

)
(3) Integration. If g(x) =
_
x
a
f(t)dt
T
n+1
g(x) =
_
x
a
T
n
f(t)dt
Theorem 23 ( Substitution Property, Apostol Vol. 1. Theorem 7.3. ). Let g(x) = f(cx), c is a constant.
(12) T
n
g(x; a) = T
n
f(cx; ca)
This theorem is useful for nding new Taylor polynomials without having to nd the jth derivatives of the desired function.
123
Theorem 24. P
n
is a polynomial of degree n 1.
Let f, g be 2 functions with derivatives of order n at 0.
(13) f(x) = P
n
(x) +x
n
g(x)
where g(x) 0 as x 0.
Then P
n
= T
n
(f, x = 0).
Exercise 3.
T
n
f(x) =
n

j=0
f
(j)
(a)
j!
(x a)
j
a
x
= e
x ln a
(a
x
)

= (a
x
) ln a
(a
x
)
(n+1)
= (a
x
(ln a)
n
)

= a
x
(ln a)
n+1
T
n
(a
x
) =
n

j=0
(ln a)
j
j!
x
j
Exercise 4.
_
1
1 +x
_

=
1
(1 +x)
2
;
_
1
1 +x
_

=
(1)
2
2
(1 +x)
3
_
1
1 +x
_
(n+1)
=
_
(1)
n
n!
(1 +x)
n+1
_

=
(1)
n+1
(n + 1)!
(1 +x)
n+2
T
n
_
1
1 +x
_
=
n

j=0
(1)
j
x
j
Exercise 5. Use Theorem 7.4. . Theorem 7.4 says for f(x) = P
n
(x) +x
n
g(x), P
n
(x) is the Taylor polynomial.
1
1 x
2
=
_
_
n

j=0
(x
2
)
j
_
_
+
(x
2
)
n+1
1 x
2
=
_
_
n

j=0
x
2j
_
_
+
x
n
x
n+2
1 x
2
x
1 x
2
=
_
_
n

j=0
x
2j+1
_
_
+
(x
2n+3
)
1 x
2
T
2n+1
_
x
1 x
2
_
=
n

j=0
x
2j+1
Exercise 6.
(ln (1 +x))

=
1
1 +x
T
n
_
1
1 +x
_
=
n

j=0
(x)
j
T
n
(ln 1 +x) =
n

j=0
(1)
j
x
j+1
j + 1
=
n

j=1
(1)
j+1
x
j
j
Exercise 7.
_
log
_
1 +x
1 x
_

=
_
1 x
1 +x
_
1 x
1 +x
1
(1 x)
2
=
1
(1 +x)(1 x)
=
1
1 x
2
_
1
1 x
2
= log
_
1 +x
1 x
so
_
n

j=0
x
2j
=
n

j=0
x
2j+1
2j + 1
= T
2n+1
_
ln
_
1 +x
1 x
_
Exercise 8.
T
n
_
1
2 x
_
= T
n
_
1/2
1 x/2
_
=
1
2
T
n
_
1
1
_
1
2
_
x
_
=
1
2
_
_
n

j=0
(
1
2
x)
j
_
_
=
n

j=0
x
j
2
j+1
Exercise 9. We can show this in two ways.
124
We could write out the actual polynomial expansion.
(1 +x)

=
n

j=0
_

j
_
1
j
x
j
=
n

j=0
_

j
_
x
j
or determine each of the coefcients of the Taylor polynomial.
((1 +x)

= (1 +x)
1
; ((1 +x)

= ( 1)(1 +x)
2
((1 +x)

)
(n+1)
=
_
!
( n)!
(1 +x)
n
_

=
!
( (n + 1))!
(1 +x)
(n+1)
Exercise 10. Use the substitution theorem, Apostol Vol.1. Thm. 7.3., to treat cos 2x.
T
2n
(cos x) =
n

j=0
(1)
j
x
2j
(2j)!
; T
2n
(cos 2x) =
n

j=0
(1)
j
(2x)
2j
(2j)!
T
2n
(sin x
2
) = T
2n
(
1
2
(1 cos 2x)) =
1
2
_
_
1
n

j=0
(1)
j
(2x)
2j
(2j)!
_
_
=
n

j=1
(1)
j+1
2
2j1
x
2j
(2j)!
7.8 Exercises - Taylors formula with remainder, Estimates for the error in Taylors formula, Other forms of the
remainder in Taylors formula. We will use Theorem 7.7, which we learn in the preceding sections, extensively.
Theorem 25. If for j = 1, . . . , n + 1, m f
(j)
(t) < t I, I containing a,
m
(x a)
n+1
(n + 1)!
E
n
(x) M
(x a)
n+1
(n + 1)!
if x > a (14)
m
(a x)
n+1
(n + 1)!
(1)
n+1
E
n
(x) M
(a x)
n+1
(n + 1)!
if x < a (15)
E
n
(x) =
1
n!
_
x
a
(x t)
n
f
(n+1)
(t)dt (16)
Exercise 1. For a = 0, [ sin
(j)
(x)[ 1 for x R.
E
n
(x)
(x)
2n+1
(2n + 1)!
if x > 0; (1)
2n+1
E
2n
(x) (+1)
(x)
2n+1
(2n + 1)!
= E
2n
(x)[
[x[
2n+1
(2n + 1)!
Exercise 2.
cos x =
n

k=0
(1)
k
x
2k
(2k)!
+E
2n+1
(x) [ cos
(j)
(x)[ 1
E
2n+1
(x)
x
2n+2
(2n + 2)!
; (1)
2n+2
E
2n+1
(x) (1)
(x)
2n+2
(2n + 2)
=[E
2n+1
(x)[
[x[
2n+2
(2n + 2)!
Exercise 3.
arctan x =
n1

k=0
(1)
k
x
2k+1
2k + 1
+E
2n
(x)
n1

k=0
(1)
k
x
2k+1
2k + 1
_
(2k)!
(2k)!
_
=
n1

k=0
(1)
k
(2k)!x
2k+1
(2k + 1)!
=f
(2k+1)
(0) = (1)
k
(2k)!
f
(2n+1)
(0)x
2n+1
(2n + 1)!
=
(1)
n
(2n)!x
2n+1
(2n + 1)!
=
(1)
n
x
2n+1
2n + 1

x
2n+1
2n + 1
Note how jth derivative (arctan x)
(j)
changes sign with each differentiation for f
(2j+1)
(0). Then we can always pick a
small enough closed interval with a = 0 as a left or right end point to make the f
(2j+1)
(0) value the biggest for f
(2j+1)
(t).
125
Exercise 4.
(1)
x
2
= sin x = x
x
3
6
=
x
3
6
+x
2
x =
x
6
_
x
_
3 +

15
___
x (3

15)
_
x =

15 3
(2)
E
4
(r; 0) =
1
4!
_
r
0
(r t)
4
cos tdt > 0 sin r r
2
= 0 +E
4
(r)
r
5
5!
<
r
5!
=
3
5(2)(5)(4)(3)(2)1
=
3
4
(5)(4)(2)5
<
1
200
Exercise 5.
arctan r r
2
= r
r
3
3
r
2
+E
4
(r; 0) = 0 +E
4
(r; 0)
E
4
(r, 0) =
1
4!
_
r
0
(x t)
4
f
(5)
(t)dt
M(r
5
)
5!
r
5
5!
= 0.065536 <
7
100
E
4
(r, 0) < E
j
(r, 0); j > 4
the 5th degree term is
f
(5)
(0)
5!
r
5
=
24
5!
r
5
> 0
so r
2
arctan r = E
4
(r, 0) < 0
Exercise 6. Apply long division on the fraction in the integrand.
_
1
0
1 +x
30
1 +x
60
dx =
_
1
0
1 +
x
30
x
60
1 +x
60
dx = 1 +
_
1
0
x
30
_
1 x
30
1 +x
60
_
dx =
= 1 +c
1
31
x
31

1
0
= 1 +
c
31
Exercise 7.
_
1/2
0
1
1+x
4
dx.
1
1 +x
4
=

j=0
(x
4
)
j
=
n1

j=0
(x
4
)
j
+E
n
= 1 +x
4
+x
8
. . .
16
17
x
4n+1
(n + 1)!
E
n
(x; 0)
1
(n + 1)!
(x
4
)
n+1
=
_
1/2
0
E
n
=
_
1
2
_
4n+5
(n + 1)!
_
1
4n + 5
_
_
1/2
0
1
1 +x
4

1
2
+
1
5
_
1
2
_
5
0.493852 < 0.49375 < 0.493858
Exercise 8.
(1)
0 x
1
2
sin x = x
x
3
3!
+E
4
(x)
[E
4
(x)[
M[x[
5
5!
=
sin
1
3
[x[
5
5!

1
_
1
2
_
5
5!
126
(2)
sin x
2
= x
2

x
6
6
+E
4
(x
2
)
_

2
2
0
sin x
2
=
_
1
3
x
3

x
7
42
_

2/2
0
=

2
_
1
12

1
42(16)
_
E
4
(x
2
)

2
64(5!)
_

2
2
0
sin x
2

2
_
55
672
+
1
64(5!)
_
= 0.1159
Exercise 9.
sin x = x
x
3
6
+
x
5
5!
E
6
(x; 0)
(1)x
7
7!
sin x
x
= 1
x
2
6
+
x
4
5!
+
E
6
(x; 0)
x
1
x
2
6
+
x
4
5!
+
x
6
7!
_
1
0
sin x
x
dx = 1
1
3
_
1
6
_
+
1
5(5!)
+
1
7(7!)
= 0.9461 +
1
7(7!)
= 0.9461 + 0.0000283
Exercise 10. = arctan
1
5
, = 4

4
.
(1)
tan (A+B) =
(tan A+ tan B)
1 tan Atan B
; A = B = ; tan 2 =
2 tan
1 tan
2

=
2/5
24/25
= 5/12
tan 4 =
2(tan 2)
1 tan
2
(2)
=
2(5/12)
1 (5/12)
2
=
10/12
119/144
=
120
119
A = 4, B =

4
tan (4

4
) = tan =
tan 4 + tan
_

4
_
1 tan 4tan
_

4
_ =
120
119
+
119
119
1
120
119
(1)
=
1
239
4 arctan
1
5
=

4
+ arctan
1
239
This is incredible.
(2)
T
11
(arctan x) =
61

k=0
(1)
k
x
2k+1
2k + 1
+E
2(6)
(x)
= x +
x
3
3
+
x
5
5

x
7
7
+
x
9
9

x
11
11
. . .
; [E
2(6)
(x)[
x
2(6)+1
2(6) + 1
=3.158328957 < 16 arctan
1
5
< 3.158328958
(3)
T
3
(arctan x); x =
1
239
0.016736304 < 4 arctan
1
239
< 0.016736304
(4)
3.141592625
3.158328972 0.016736300 = 3.141592672
7.11 Exercises - Further remarks on the error in Taylors formula. The o-notation; Applications to indeterminate
forms.
Exercise 1.
2
x
= exp xln 2 = 1 + (xln 2) +
x
2
(ln 2)
2
2!
+o(x
2
)
Exercise 2.
127
x(cos x) = ((x 1) + 1)(cos x) = (x 1) cos 1 + (sin 1)(x 1)
2

cos 1(x 1)
3
2
+ cos 1+
+ (sin 1)(x 1)
cos 1(x 1)
2
2
+
sin 1(x 1)
3
3!
=
= cos 1 + (cos 1 sin 1)(x 1) +
_
sin 1
cos 1
2
_
(x 1)
2
+
_
sin 1 3 cos 1
3!
_
(x 1)
3
+o(x 1)
3
Exercise 3. Just treat the argument of sin x x
2
just like u with u 0.
sin (x x
2
) = (x x
2
)
(x x
2
)
3
3!
+
(x x
2
)
5
5!
+o(x x
2
)
5
=
= (x x
2
)
1
6
_
x
3
3x
4
+ 3x
5
x
6
_
+
1
120
_
x
5
5x
6
+ 10x
7
10x
8
+ 5x
9
+x
1
0
_
=
= (x x
2
)
1
6
x
3

1
2
x
4
+
61
120
x
5

25
120
x
6
Exercise 4.
log x = log (1 + (x 1)) = (x 1)
(x 1)
2
2
+
(x 1)
3
3
= a = 0; b = 1, c =
1
2
Exercise 5.
cos x = 1
1
2
x
2
+o(x
3
) as x 0
1 cos x =
1
2
x
2
+o(x
3
)
1 cos x
x
2
=
1
2
+
o(x
3
)
x
2
since
1 cos x
x
2
=
1
2
+o(x),
1 cos x
x
2

1
2
as x 0
cos x = 1
1
2
x
2
+
x
4
4!
+o(x
5
) =cos 2x = 1 2x
2
+
2
3
x
4
+o(x
5
)
1 cos 2x 2x
2
x
4
=

2
3
x
4
o(x
5
)
x
4
=
2
3
o(x)
2
3
as x 0
Exercise 6.
lim
x0
sin ax
sin bx
= lim
x0
ax
(ax)
3
3!
+o(x
4
)
bx +o(x
2
)
=
a
b
Exercise 7.
lim
x0
sin 2x
cos 2xsin 3x
= lim
x0
(2x) +
(2x)
3
3!
+o(x
4
)
_
1
(2x)
2
2!
+
(2x)
4
4!
+o(x
5
)
__
(3x)
(3x)
3
3!
+o(x
4
)
_ =
2
3
Exercise 8.
lim
x0
sin x x
x
3
=
1
6
Exercise 9.
lim
x0
ln 1 +x
e
2x
1
= lim
x0
x o(x)
2x +o(x)
=
1
2
Exercise 10. Dont do the trig. identity.
lim
x0
1 cos
2
x
xtan x
= lim
x0
1
_
1
x
2
2!
+o(x
2
)
_
2
x
_
x +
x
3
6!
+o(x
3
)
_ = lim
x0
1 (1 +x
2
+o(x
2
))
x
2
+o(x
2
)
= 1
Exercise 11.
128
lim
x0
x +
x
3
6
+o(x
4
)
x
x
3
3
= 1
Exercise 12.
lim
x0
e
x ln a
1
e
x ln b
1
= lim
x0
xln a +o(x)
xln b +o(x)
= ln a/b
Exercise 13.
lim
x1
(x 1)
(x1)
2
2
+
(x1)
3
3
+o(x 1)
4
(x + 2)(x + 1)
=
1
3
Exercise 14. 1 .
Exercise 15.
lim
x0
x(e
x
+ 1) 2(e
x
1)
x
3
= lim
x0
x(2 +x +
x
2
2
) 2(x +x
2
/2 +x
3
/6)
x
3
= lim
x0
x
3
(
1
6
)
x
3
=
1
6
Exercise 16.
lim
x0
ln (1 +x) x
1 cos x
= lim
x0
x
x
2
2
+
x
3
3
+o(x
3
) x
x
3
/2
= 1
Exercise 17.
lim
x/2
cos x
x

2
=
0 +1(x

2
)
x

2
= 1
Exercise 18. 1/6
Exercise 19.
lim
x0
cosh x cos x
x
2
= lim
x0
e
x
+e
x
2
cos x
x
2
=
= lim
x0
1 +x +
x
2
2
2
_
1
x
2
2
_
+o(x
3
)
x
2
= 2
Exercise 20.
lim
x0
3 tan 3x 12 tan x
3 sin 4x 12 sin x
= lim
x0
(4x) +
(4x)
3
3
4
_
x +
x
3
3
_
+o(x
4
)
(4x)
(4x)
3
3!
4
_
x
x
3
3!
_
+o(x
4
)
=
=
4
3
4
4
3
+4
2
= 2
Exercise 21.
lim
x0
a
x
a
sinx
x
3
= lim
x0
e
x ln a
e
sin x ln a
x
3
=
= lim
x0
1 +xln a +
(x ln a)
2
2!
+
(x ln a)
3
6

_
1 + sin xln a +
sin
2
x(ln a)
2
2
+
sin
3
x(ln a)
3
3!
_
+o(x
4
)
x
3
=
= lim
x0
ln a(x
_
x
x
3
3!
_
) +
(x
2
ln a)
2
2!

(ln a)
2
2
(x
2
) +
(lna)
3
6
(x
3
x
3
) +o(x
4
)
x
3
=
ln a
6
Exercise 22.
lim
x0
cos sin x cos x
x
4
= lim
x0
1
sin
2
x
2!
+
sin
4
x
4!
+o(x
5
)
_
1
x
2
2!
+
x
4
4!
_
x
4
=
= lim
x0
1
2
_
x
2
(x
2

x
4
3
)
_
+
_
x
4
x
4
4!
_
+o(x
5
)
x
4
=
1
6
129
Exercise 23.
limx 1x
1
1x
= limx 1e
1
1x
ln x
= exp
_
limx 1
ln x
1 x
_
= exp
_
limx 1
(x 1) +o(x 1)
2
1 x
_
= e
1
Exercise 24.
lim
x0
(x +e
2x
)
1/x
= exp lim
x0
1
x
ln (x +e
2x
)
lim
x0
ln (x +e
2x
)
x
= lim
x0
ln (1 +x +e
2x
1)
x
=
= lim
x0
x +e
2x
1 +o(x
2
)
x
= lim
x0
3x +o(x
2
)
x
= 3
= lim
x0
(x +e
2x
)
1/x
= e
3
Exercise 25.
lim
x0
(1 +x)
1/x
e
x
= lim
x0
e
1
x
ln (1+x)
e
x
= lim
x0
e
x+
x
2
2
+o(x
2
)
x
e
x
=
= lim
x0
e
1
x
2
+o(x)
e
x
= lim
x0
e(1 +
x
2
+o(x)) e
x
=
e
2
Exercise 26.
lim
x0
_
(1 +x)
1/x
e
_1/x
= lim
x0
_
exp
_
1
x
ln (1 +x)
_
1
_
1/x
= lim
x0
e
x
x
2
2
+o(x
3
)x
x
2
= e
1/2
Exercise 27.
(arcsin x)

=
1

1 x
2
(arcsin x)

=
x
(1 x
2
)
3/2
(arcsin x)

=
1
(1 x
2
)
3/2
+
3x
2
(1 x
2
)
5/2
exp
_
lim
x0
1
x
2
_
ln
_
arcsin x
x
___
= exp lim
x0
1
x
2
ln
_
1 +
_
arcsin x
x
1
__
=
= exp
_
lim
x0
1
x
2
ln
_
1 +
x +x
3
/6 +o(x
4
) x
x
__
= exp
_
lim
x0
1
x
2
ln
_
1 +
x
2
6
+o(x
3
)
__
=
= exp
_
lim
x0
1
x
2
_
x
2
+o(x
3
)
6
__
= e
1/6
Exercise 28. lim
x0
_
1
x

1
e
x
1
_
= lim
x0
_
e
x
1x
x(e
x
1)
_
= lim
x0
x
2
2
+o(x
3
)
x
2
+o(x
3
)
=
1
2
Exercise 29.
lim
x1
_
1
log x

1
x 1
_
= lim
x1
_
(x 1) log x
(x 1) log x
_
=
= lim
x1
(x 1) ((x 1)
(x1)
2
2
+o(x 1)
3
)
(x 1)((x 1) +o(x 1)
2
)
=
1
2
Exercise 30.
lim
x0
e
ax
e
x
x
x
2
= lim
x0
1 +ax +
(ax)
2
2
+o(x
3
) 1 x
x
2
2
x
x
2
if a = 2, the limit is 2
Exercise 31.
130
(1) Prove
_
x
0
f(t)dt = o
__
x
0
g(t)dt
_
as x 0, given f(x) = o(g(x)).
Consider lim
x0

x
0
f(t)dt

x
0
g(t)dt
.
Since f, g have derivatives in some interval containing 0, f, g continuous and differentiable for [t[ x.
lim
x0
_
x
0
f(t)dt
lim
x0
g(t)dt
= lim
x0
_
A(x)A(0)
x
_
_
B(x)B(0)
x
_ =
f(0)
g(0)
=
lim
x0
f(x)
lim
x0
g(x)
= 0
We can do the second to last step since f, g have derivatives at 0 and thus are continuous about 0.
(2) Consider lim
x0
x
e
x
= 0. However, lim
x0
1
e
x
= 1.
Exercise 32.
(1) Use long division to nd that
1
1 +g(x)
= 1 g(x) +g
2
(x) +
g
3
(x)
1 +g(x)
= 1 g(x) +g
2
(x) +o(g
2
(x))
(2)
Exercise 33. Given lim
x0
_
1 +x +
f(x)
x
_
1/x
= e
3
, use the hint.
lim
x0
g(x) = A, then G(x) = A+o(1) as x 0
Then
g(x) = e
3
+o(1) =
_
1 +x +
f
x
_
1/x
x
_
e
3
+o(1)
_
x
= x +x
2
+f(x) = f(0) = 0
xexp xln (e
3
+o(1)) = x +x
2
+f(x)
1 exp xln (e
3
+o(1)) +xexp xln (e
3
+o(1))
_
ln e
3
+o(1) +
x
e
3
+o(1)
(o

(1))
_
= 1 + 2x +f

(0)
1 + 3(0) + 0 = 1 + 0 +f

(0) =f

(0) = 0
We need to assume that in general o(1) = x +kx
2
+o(x
2
).
2 exp xln (e
3
+o(1))
_
ln (e
3
+o(1)) +
x
e
3
+o(1)
(o

(1))
_
+
+xexp xln (e
3
+o(1))
_
ln (e
3
+o(1)) +
x
e
3
+o(1)
o

(1)
_
+
xexp xln (e
3
+o(1))
_
2o

(1)
(e
3
+o(1))
+
xo

(1)
e
3
+o(1)
+
x
(e
3
+o(1))
(o

(1))
2
_
= 2 +f

(x)
x0
2(3) = 2 +f

(0) =f

(0) = 4
To evaluate lim
x0
_
1 +
f(x)
x
_
1/x
, consider a Taylor series expansion of f.
lim
x0
_
1 +
f(x)
x
_
1/x
= lim
x0
_
1 +
0 + 0 + 4
x
2
2
+o(x
3
)
x
_
1/x
= lim
x0
(1 +x(2 +o(x)))
1/x
=
= lim
y0
lim
x0
(1 +x(2 +o(y)))
1/x
= lim
y0
exp 2 +o(y) = e
y
131
7.13 Exercises - LHopitals rule for the indeterminate form 0/0. Exercise 1.
lim
x2
3x
2
+ 2x 16
x
2
x 2
= lim
x2
(3x + 8)(x 2)
(x 2)(x + 1)
=
14
3
Exercise 2.
lim
x3
x
2
4x + 3
2x
2
13x + 21
= lim
x3
(x 3)(x 1)
(2x 1)(x 3)
= 2
Exercise 3.
lim
x0
sinh x sin x
x
3
=
0
0
= lim
x0
_
cosh x cos x
3x
2
_
=
0
0
= lim
x0
_
sinh x + sin x
6x
_
= lim
x0
_
cosh x + cos x
6
_
=
1
3
Exercise 4.
lim
x0
(2 x)e
x
x 2
x
3
= lim
x0
e
x
+ (2 x) 1
3x
2
= lim
x0
(2 x)(1 +x +x
2
/2 +x
3
/6 +o(x
3
))
x
3
=
1
6
Exercise 5.
lim
x0
log (cos ax)
log (cos bx)
= lim
x0
cos bxsin axa
cos axsin bxb
= lim
x0
cos axa
2
sin bxb
2
=
a
2
b
2
Exercise 6. When it doubt, Taylor expand.
lim
x0
+
x sin x
(xsin x)
3/2
= lim
x0
+
1 cos x
3
2
(xsin x)
1/2
(sin x +xcos x)
=
2
3
lim
x0
+
1 (1
x
2
2
+
x
4
24
+o(x
4
))
e
1
2
ln (x sin x)
(x
x
3
6
+o(x
3
) +x
x
3
2
+o(x
3
))
=
=
2
3
lim
x0
+
_
_
x
2
2

x
4
24
+o(x
4
)
_
x(x +
x
3
6
+o(x
3
))(2x
2
3
x
3
+o(x
3
))
_
_
=
=
2
3
lim
x0
+
_
_
1
2

x
2
24
+o(x
2
)
_
1 +
x
2
6
+o(x
2
)(2 +
2
3
x
2
+o(x
2
))
_
_
=
2
3
lim
x0
+
_
1/2
2
_
=
1
6
Notice in the third step how in general we deal with powers, (xsin x)
1/2
, is to convert it into exponential form, e
1
2
ln (x sin x)
,
but it wasnt necessary.
Exercise 7. Do LHopitals rst.
lim
xa
+

a +

x a

x
2
a
2
= lim
xa
+
1
2

x
+
1
2

xa
x

x
2
a
2
=
1
2
lim
xa
+

x
2
a
2
x
3/2
+

x +a
x
=
=
1
2
lim
xa
+

x
2
a
2
+x
1/2

x +a
x
3/2
=
1
2

2a
a
3/2
=

2
2

a
Exercise 8. Do LHopitals at the second step.
lim
x1
+
exp (xln x) x
1 x + ln x
= lim
x1
+
exp xln x(ln x + 1) 1
1 +
1
x
= lim
x1
+
exp (xln x)(ln x + 1)
2
+
1
x
exp (xln x)
1/x
2
=
= lim
x1
+
x
2
exp (xln x)(ln x + 1)
2
+xexp (xln x) = 1 + 1 = 2
Exercise 9. Keep doing LHopitals.
132
lim
x0
arcsin 2x 2 arcsin x
x
3
= lim
x0
2

1(2x)
3
2
1

1x
2
3x
2
=
=
2
3
lim
x0

1
2
(1 (2x)
2
)
3/2
(8x) (
1
2
)(1 x
2
)
3/2
(2x)
6x
=
=
1
9
lim
x0
4(1 (2x)
2
)
3/2
+ 4x(
3
2
)(1 (2x)
2
)
5/2
(8x) +(1 x
2
)
3/2
x(
3
2
)(1 x
2
)
5/2
(2x)
1
=
=
1
9
4 1
1
=
1
3
Exercise 10.
lim
x0
xcot x 1
x
2
= lim
x0
xcos x sin x
x
2
sin x
= lim
x0
cos x xsin x cos x
2xsin x +x
2
cos x
= lim
x0
sin x
2 sin x +xcos x
=
= lim
x0
cos x
2 cos x + cos x +xsin x
= lim
x0
cos x
3 cos x xsin x
=
1
3
Exercise 11.
lim
x1

n
k=1
x
k
n
x 1
= lim
x1

n
k=1
kx
k1
1
=
n

k=1
k =
n(n + 1)
2
Exercise 12.
lim
x0+
1
x

x
_
a arctan

x
a
b arctan

x
b
_
= lim
x0+
_
a
1
1+
x
a
2
_
1
2a

x
_
b
1
1+
x
b
2
_
1
2b

x
__
3
2
x
1/2
=
=
1
3
lim
x0+
a
2
a
2
+x
_
1
x
_

b
2
b
2
+x
_
1
x
_
=
1
3
lim
x0+
(b
2
+x)a
2
b
2
(a
2
+x)
(a
2
+x)(x +b
2
)x
=
=
1
3
lim
x0+
(a
2
b
2
)
(a
2
x)(b
2
+x)
=
1
3
a
2
b
2
a
2
b
2
Exercise 13.
(sin 4x)(sin 3x)
xsin 2x
=
(2 sin 2xcos 2x)(sin 3x)
xsin 2x
=
2(2 sin 2xsin 3x + cos 2x3 cos 3x)
1
= 6 as x 0 otherwise
2 cos 2xsin 3x
x

4

as x

2
We used LHopitals at the second to last step for x 0.
Exercise 14.
lim
x0
(x
3
sin 3x +ax
2
+b) = 0
lim
x0
sin 3x +ax +bx
3
x
3
=
3 cos 3x +a + 3bx
2
3x
2
=
9 sin 3x + 6bx
6x
=
27 cos 3x + 6b
6
=
27 + 6b
6
= 0
So b =
9
2
, a = 3 .
Exercise 15.
lim
x0
1
bx sin x
_
x
0
t
2
dt

a +t
=
x
2

a+x
b cos x
=
2x
1
2

a+x
(1 cos x) +

a +x(sin x)
=
2x

a +x
1
2
(1 cos x) + (a +x) sin x
=
= 2 lim
x0

a +x +
x
2

a+x
sin x
2
+ sin x + (a +x) cos x
= 2

a
a
= 1
= a = 4
Note that we had dropped the limit notation in some earlier steps and applied LHopitals a number of times, and we also
rearranged the denominator and numerator cleverly at each step.
133
Exercise 16.
(1)
angle ABC is
x
2
, length BC is tan
x
2
2 tan
x
2
cos
x
2
= 2 sin
x
2
is the base length of ABC
tan
x
2
sin
x
2
is the height of triangle ABC
=T(x) = tan
x
2
sin
2
x
2
=
1 cos
2 x
2
cos
x
2
sin
x
2
= tan
x
2

1
2
sin x
(2)
S(x) =
_
x
2
_
((1))
1
2
cos
x
2
(2 sin
x
2
) =
x
2

sin 2
2
(3) Use LHopitals theorem.
T(x)
S(x)
=
tan
x
2

1
2
sin x
xsin x
2
d
dx

1
2
sec
2 x
2

cos x
2
1cos x
2
d
dx

sec
2 x
2
tan
x
2
+ sin x
sin x
d
dx

sec
2 x
2
tan
2 x
2
+
1
2
sec
4 x
2
+ cos x
cos x

x0

3
2
Exercise 17. Use LHopitals rule.
I(t) =
E
R
(1 e

Rt
L
)
lim
R0
I(t) = lim
R0
E(1)(e
Rt/L
)
_
t
L
_
1
=
Et
L
Exercise 18.
c k 0 since c k
k c = u
k u = c
f(t) =
A(sin kt sin ct)
c
2
k
2
=
A(sin (kt) sin (k u)t)
u(2k u)
=
=
A(cos (k u)t)(t)
(2k u) +u

At cos kt
2k
7.17 Exercises - The symbols +and . Extension of LHopitals rule; Innite limits; The behavior of log x and
e
x
for large x.
Exercise 15. Use LHopitals at the second to last step.
lim
x1
1
(ln x)(ln (1 x)) = lim
x1
1
ln (1 x)
1
ln x
= lim
x1
1
_
1
1x
_
(1)
1
(ln x)
2
_
1
x
_ = lim
x1
1
x
1 x
(ln x)
2
= lim
x1
1
2 ln x
_
1
x
_
(1)
= 0
Exercise 16. Persist in using LHopitals and trying all possibilities systematically.
lim
x0
+
x
x
x
1
= lim
x0
+
e
(x
x
1) ln x
= lim
x0
+
e
(e
x ln x
1) ln x
= exp lim
x0
+
ln x
(e
x ln x
1)
1
=
= exp lim
x0
+
1/x
(1)(e
x ln x
1)
2
(e
x ln x
(ln x + 1))
= exp lim
x0
+
_
e
2x ln x
2e
x ln x
+ 1
e
x ln x
(xln x +x)
_
=
= exp lim
x0
+
e
x ln x
(ln x + 1) +e
x ln x
(ln x 1)
(ln x + 1 + 1)
= exp lim
x0
+
e
x ln x
e
x ln x
1 +
1
ln x+1
= 1
Exercise 17.
134
lim
x0
+
(x
x
x
1) = lim
x0
+
_
e
x
x
ln x
1
_
= lim
x0
+
_
e
e
x ln x
ln x
1
_
=
_
e
lim
x0
+ e
x ln x
ln x
1
_
=
= e
e
lim
x0
+
x ln x
lim
x0
+ ln x
1 = 0 1 = 1
We used
lim
x0
+
x

log x = 0 > 0
since
t =
1
x
, x

log x =
log t
t

0 as t .
Exercise 18.
lim
x0

e
sin x ln (12
x
)
= exp
_
lim
x0

ln (1 e
x ln 2
)
1/ sin x
_
= exp
_
lim
x0

1
1e
x ln 2
_
ln 2e
x ln 2
_
1
sin
2
x
cos x
_
=
= exp
_
lim
x0

(sin
2
x) ln 2e
x ln 2
(1 e
x ln 2
) cos x
_
= exp
_
(ln 2) lim
x0

2 sin xcos x
ln 2e
x ln 2
_
= 1
Exercise 19.
lim
x0
+
e
1
ln x
ln x
= e
Exercise 20. At the end, LHopitals could be used to verify that indeed sin xln sin x 0 as x 0.
lim
x0
+
e
sin x ln cot x
= e
lim
x0
+ sin x(ln cos xln sin x)
= e
lim
x0
+ sin x ln sin x
= 1
Exercise 21. Rewrite tan into sin and cos and use LHopitals.
lim
x

4
(tan x)
tan 2x
= lim
x

4
e
tan 2x ln tan x
= exp lim
x

4
1
cos 2x
(ln sin x ln cos x) =
= exp lim
x

4
1
sin x
cos x
1
cos x
(sin x)
2 sin 2xcos 2x
= exp lim
x

4
1
sin
2
2x
= e
1
Exercise 22.
Exercise 23. Use LHopitals theorem, taking derivatives of top and bottom.
lim
x0
+
exp
e
1 + ln x
ln x = exp e lim
x0
ln x
1 + ln x
= exp e lim
x0
1/x
1/x
= e
e
Exercise 24. Rewrite tan into sin and cos and take out sin since we could do the limit before doing LHopitals.
lim
x1
(2 x)
tan (x/2)
= lim
x1
e
tan
x
2
ln (2x)
= e
limx1
sin x/2 ln (2x)
cos x/2
= exp lim
x1
(1)
2x

2
sin x/2
= exp
2

Exercise 26.
lim
x+
exp
_
xln
_
x +c
x c
__
= exp lim
x+
ln
_
1+c/x
1c/x
_
1/x
= exp lim
x+
1
(
1+c/x
1c/x
)
_
(xc)(x+c)
(xc)
2
_
1
x
2
=
= exp (2c) = 4 = c = ln 2
Exercise 27.
(1 +x)
c
= exp (c ln (1 +x)) = exp (c(x o(x))) = 1 +c(x o(x)) +o(x o(x)) = 1 +cx +o(x)
x
2
_
1 +
1
x
2
_
1/2
x
2
= x
2
_
1 +
1
x
2
_
1/2
x
2
Let x
2
=
1
t
. So t 0 as x +.
=
(1 +t)
1/2
1
t
=
1 +
1
2
t 1 +o(t)
t
=
1
2
135
Exercise 28.
(x
5
+ 7x
4
+ 2)
c
x = x
5
_
1 +
7
x
+
2
x
5
_
c
x
Let
1
t
= x and guess that c =
1
5
_
1
t
5
+
7
t
4
+ 2
_
c

1
t
=
_
1 + (7t + 2t
5
)
_
1/5
/t
1
t
=
=
1 +
1
5
(7t + 2t
5
) +o(t) 1
t
=
7
5
Exercise 29.
g(x) = xe
x
2
g

(x) = e
x
2
+ 2x
2
e
x
2
g

(x) = 2xe
x
2
+ 4xe
x
2
+ 4x
3
e
x
2
= 6xe
x
2
+ 4x
3
e
x
2
f(x) =
_
x
1
g(t)
_
t +
1
t
_
dt
f

(x) = g(x)
_
x +
1
x
_
g(1)2;
f

(x) = g

(x)(x + 1/x) +g(x)(1 1/x


2
)
f

(x)
g

(x)
=
(e
x
2
+ 2x
2
e
x
2
)(x + 1/x) +xe
x
2
(1
1
x
2
)
6xe
x
2
+ 4x
3
e
x
2
=
2xe
x
2
+ 2x
3
e
x
2
+ 2xe
x
2
(6xe
x
2
+ 4x
3
e
x
2
)
=
=
4x + 2x
3
6x + 4x
3
=
1
2
as x
Exercise 30.
g(x) = x
c
e
2x
g

(x) = cx
c1
e
2x
+ 2x
c
e
2x
f(x) =
_
x
0
e
2t
(3t
2
+ 1)
1/2
dt
f

(x) = e
2x
(3x
2
+ 1)
1/2
1
Guessing that c = 1
f

(x)
g

(x)
=
e
2x
(3x
2
+ 1)
1/2
1
2x
c
e
2x
+cx
c1
e
2x
=

3x(1 +
1
3x
2
)
1/2
e
2x
2x + 1
=

3
2
So c = 1 .
Exercise 31.
Exercise 32.
(1)
P
_
1 +
r
m
_
, P
_
1 +
r
m
_
2
, . . . P
_
1 +
r
m
_
m
For each year, there are the just previously shown m compoundings, so for n years,
P
_
1 +
r
m
_
mn
(2)
2 = e
rt
ln 2
r
= t = 11.55years
(3)
2P
0
= P
0
_
1 +
r
m
_
mt
ln 2 = mt ln (1 +r/m)
t =
ln 2
mln (1 +r/m)
=
ln 2
4 ln (1 + 0.06/4)
= 11.64years
136
7.17 Exercises - The symbols +and . Extension of LHopitals rule; Innite limits; The behavior of log x and
e
x
for large x. Exercise 1.
lim
x0
e
1/x
2
x
1000
= lim
u
e
u
u
5
00 = lim
u
u
500
e
u
= 0
u =
1
x
2
x
2
=
1
u
x
1000
=
1
u
5
00
Where we had used Theorem 7.11, which are two very useful limits for log and exp.
Theorem 26. If a, b > 0,
lim
x+
(log x)
b
x
a
= 0 (17)
lim
x+
x
b
e
ax
= 0 (18)
Proof. Trick - use the denition of the logarithm as an integral.
If c > 0, t 1, t
c
1 =t
c1
t
1
.
0 < ln x =
_
x
1
1
t
dt
_
x
1
t
c1
dt =
1
c
(x
c
1) <
x
c
c
=0 <
(ln x)
b
x
a
<
x
cba
c
b
Choose c =
a
2b
,
x
cba
c
b
=
x
a/2
c
b
0 as x
then
(ln x)
b
x
a
0 as x 0
For exp, Let t = e
x
. ln t = x.
x
b
e
ax
=
(ln t)
b
t
a
0 as t as x .
Exercise 2.
lim
x0
sin
1
x
arctan
1
x
= lim
x0
1
x
o
_
1
x
_
1
x
o
_
1
x
_ = 1
Exercise 3. Use LHopitals.
lim
x

2
tan 3x
tan x
= lim
x

2
cos x
cos 3x
= lim
x

2
sin x
3 sin 3x
=
1
3
Exercise 4. Use LHopitals.
lim
x
ln (a +be
x
)

a +bx
2
= lim
x
1
a+be
x
(be
x
)
bx

a+bx
2
= lim
x
1
ae
x
+b
1

a
x
2
+b
=
1

b
Exercise 5. Make the substitution x =
1
t
.
lim
x
x
4
_
cos
1
x
1 +
1
2x
2
_
= lim
t
1
t
4
_
cos t 1 +
t
2
2
_
= lim
t
t
4
/4! +o(t
4
)
t
4
=
1
4!
=
1
120
Exercise 6.
lim
x
ln [ sin x[
ln [ sin 2x[
= lim
x
1
sin x
cos x
2
sin 2x
cos 2x
=
1
2
lim
x
sin 2x
sin x
=
1
2
lim
x
2 cos 2x
cos x
= 1
Exercise 7.
lim
x
1
2

ln (1 2x)
tan x
= lim
x
1
2

_
1
12x
_
(2)
(sec
2
x)
= lim
x
1
2

2(cos
2
x)
(1 2x)
=
=
2

lim
x
1
2

2 cos x sin x
2
= 1
137
Exercise 8.
lim
x
cosh x + 1
e
x
= lim
x
e
x+1
+e
x1
2e
x
=
1
2
lim
x
e
1
+
1
e
2x+1
=
e
2
Exercise 9.
lim
x
a
x
x
b
= lim
x
e
x ln a
x
b
; a > 1
since lim
x
x
b
e
ax
(in this case, ln a > 0 )
Exercise 10.
lim
x

2
tan x 5
sec x + 4
= lim
x

2
sec
2
x
tan xsec x
= lim
x

2
sec x
tan x
= lim
x

2
1
cos x
cos x
sin x
= 1
8.5 Exercises - Introduction, Terminology and notation, A rst-order differential equation for the exponential func-
tion, First-order linear differential equations.
The ordinary differential equation theorems we will use are
y

+P(x)y = 0 (19)
A(x) =
_
x
a
P(t)dt
y = be
A(x)
(20)
Consider y

+P(x)y = Q(x); A(x) =


_
x
a
P(t)dt.
Let h(x) = g(x)e
A(x)
; g a solution.
h

(x) = (g

+Pg)e
A
= Qe
A
2nd. fund. thm. of calc.
h(x) = h(a) +
_
x
a
Q(t)e
A(t)
dt
since h(a) = g(a)
(21) y = g(x) = e
A(x)
__
x
a
Q(t)e
A(t)
dt +b
_
We had done some of these problems previously, using an integration constant C, but following Apostols notation for
y(a) = b for initial conditions is far more advantageous and superior as we seem clearly the dependence upon the initial
conditions - so some of the solutions for the exercises will show corrections to the derived formula using Apostols notation
for y(a) = b initial conditions.
Exercise 1. A(x) =
_
x
0
(3)dt = 3x
y = e
3x
__
x
0
e
2t
e
3t
dt + 0
_
= e
3x
(e
t
)

x
0
= e
2x
+e
3x
Exercise 2. y

2
x
y = x
4
. A(x) =
_
x
1
_
2
t
_
dt = 2 ln x.
y = e
A(x)
__
x
a
Q(t)e
A(t)
dt +b
_
= e
2 ln x(

x
1
t
4
e
2 ln t
dt+1)
= x
2
__
x
1
t
2
dt + 1
_
=
= x
2
+
x
2
3
(x
3
1) =
x
5
3
+
2x
2
3
Exercise 3. y

+y tan x = sin 2x on
_

2
,

2
_
, with y = 2 when x = 0.
138
A(x) =
_
x
0
P(t)dt =
_
x
0
tan tdt = ln [ cos t[[
x
0
= ln cos x
y = e
A(x)
__
x
a
Q(t)e
A(t)
dt +b
_
= cos x
_
_
b
a
sin 2te
ln cos t
dt + 2
_
=
= cos x
__
x
a
2 sin tdt + 2
_
= 2 cos
2
x + 4 cos x
Exercise 4. y

+xy = x
3
. y = 0, x = 0.
A(x) =
1
2
x
2
y = e

1
2
x
2
__
x
0
t
3
e
t
2
2
dt + 0
_
= e
x
2
2
_
t
2
e
t
2
2
2e
t
2
2
_

x
0
=
= x
2
2 + 2e

x
2
2
Exercise 5. y

+y = e
2t
. y = 1, t = 0.
A = x
y(x) = e
x
__
x
0
e
2t
e
t
dt + 1
_
= e
x
_
e
3t
3

x
0
+ 1
_
=
e
2x
3
+
2
3
e
x
Exercise 6. y

sin x +y cos x = 1; (0, ). =y

+ cot xy = csc x
A(x) =
_
x
a
cot tdt = ln
_
sin x
sin a
_
y(x) = e
ln (
sin x
sin a
)
__
(csc t)e
ln (
sin t
sin a
)
dt +b
_
=
_
sin a
sin x
__
x a
sin a
+b
_
indeed y =
x a
sin x
+
b sin a
sin x
x 0 for y =
x a
sin x
+
b sin a
sin x
=
x +b sin a a
sin x
b sin a = a for x 0
a b sin a = for x
Exercise 7.
x(x + 1)y

+y = x(x + 1)
2
e
x
2
=y

+
1
x(x + 1)
y = (x + 1)e
x
2
A(x) =
_
x
a
_
1
t

1
t + 1
_
dt = ln
x
a
ln
x + 1
a + 1
e
A(x)
=
(a + 1)x
a(x + 1)
y =
a(x + 1)
(a + 1)x
__
x
a
(t + 1)e
t
2
_
(a + 1)t
a(t + 1)
_
dt +b
_
=
(x + 1)
x
__
x
a
te
t
2
dt +b
a(x + 1)
(a + 1)x
_
=
=
x + 1
2x
_
e
a
2
e
x
2
_
+
a(x + 1)b
(a + 1)x
Its easy to see that the last equation above goes to 0 as x 1.
y = (e
a
2
e
x
2
)(1/2)(1 +
1
x
) +
ab
a + 1
(1 +
1
x
) =
lim
x0
y =
1
x
_
e
a
2
e
x
2
2
+
ab
a + 1
_
a = 0
139
Exercise 8. y

+y cot x = 2 cos x on (0, ).


A(x) =
_
x
a
cot tdt = ln
_
sin x
sin a
_
e
A(x)
=
sin x
sin a
y =
sin a
sin x
__
2 cos t
sin t
sin a
+b
_
=
sin a
sin x
_

cos 2t
2 sin a

x
a
+b
_
=
=
cos (2a) + cos 2x
2 sin x
+
sin a
sin x
y = sin x y =
cos (2a) cos (2x)
2 sin x
+
sin a
sin x
Exercise 9. (x 2)(x 3)y

+ 2y = (x 1)(x 2). y

+
2
(x2)(x3)
y =
x1
x3
.
A(x) =
_
x
a
2dt
(t 2)(t 3)
= 2
_
x
a
_
1
t 3

1
t 2
_
dt = 2 (ln [t 3[ ln [t 2[)[
x
a
= 2 ln

x 3
a 3

a 2
x 2

If (, 2), (3, +),


x 3 0
x 2 0
3 x
2 x
=
x 3
x 2
If (2, 3), x 3 < 0, but x 2 > 0
If (, 2), (3, ) y = b
_
x 2
x 3
_
2
_

a 3
a 2

_
2
+
_
x 2
x 3
_
2
_
x +
1
x 2
a
1
a 2
_
If (2, 3) y = b
_
x 2
x 3
_
2
_

a 3
a 2

_
2
+
_
x 2
3 x
_
2

a 3
a 2

2
_ _
t 1
t 3
_

a 2
a 3

2
_
3 t
t 2
_
2
=
= b
_
x 2
3 x
_
2

a 3
a 2

2
+
x 2
3 x
_

_
(t 1)(3 t)
(t 2)
2
_
=
= b
_
x 2
x 3
_
2

a 3
a 2

2
+
_
x 2
x 3
_
2
(x + (x 2)
1
a (a 2)
1
)
Exercise 10. s(x) =
sin x
x
; x ,= 0 s(0) = 1, T(x) =
_
x
0
s(t)dt f(x) = xT(x)
f

= T +x(s(x)) = T + sin x
xf

f = xsin x
y

y
x
= sin x
A(x) =
_
x
a
1
t
dt = ln
x
a
, e
A(x)
=
x
a
y =
x
a
__
x
a
sin t
a
t
+b
_
= xT +
bx
a
y(0) = 0 ,= 1 since P(x) =
1
x
is not continuous at x = 0
Exercise 11.
f(x) = 1 +
1
x
_
x
1
f(t)dt
xf(x) = x +
_
x
1
f(t)dt =f(x) +xf

= 1 +f(x) =f

=
1
x
= f(x) = ln [x[ C
ln [x[ C = 1 +
1
x
_
x
1
(ln [t[ c) dt =
= 1 +
1
x
(t ln [t[ t ct)[
x
1
= ln [x[ C +
1 +C
x
= C = 1
f(x) = ln [x[ + 1
Exercise 12. Rewrite the second property we want f to have:
_
x
1
f(t)dt =
1 f(x)
x
=f(x) =
1
x
2

_
f

x f
x
2
_
140
So then
f

+
_
x
1
x
_
f =
1
x
A(x) =
_
P(t)dt =
_ _
t
1
t
_
dt =
_
1
2
x
2
ln x
_

_
1
2
a
2
ln a
_
e
A(x)
= e
x
2
a
2
2
a
x
; e
A(x)
=
x
a
e
a
2
x
2
2
f(x) =
x
a
e
a
2
x
2
2
__
x
a
1
t
2
ae
t
2
a
2
2
dt +b
_
Exercise 13.
v = y
k
v

= ky
k1
y

+kPv = kQ = ky
k1
y

+kPy
k
= kQ where k = 1 n
=y

y
n
+Py
1n
= Q = y

+Py = Qy
n
Exercise 14. y

4y = 2e
x
y
1/2
n =
1
2
k = 1
1
2
=
1
2
; v = y
1/2
; v

+
1
2
(4)v =
1
2
(2e
x
) = v

2v = e
x
A(x) =
_
x
a
P(t)dt = 2(x a) = 2x
v = e
2x
__
e
t
e
2t
dt +b
_
= e
2x
_
e
x
+b
_
= be
2x
e
x
=y = (1 +

2)
2
e
4x
2(1 +

2)e
3x
+e
2x
y(x) = b
2
2b + 1 = 2
b = 1 +

2
Exercise 15. y

y = y
2
(x
2
+x + 1), n = 2 k = 1 n = 1, v = y
k
v = y
1
.
v

+kPv = kQv

+ (1)(1)v = (1)((x
2
+x + 1)) = v

+v = x
2
+x + 1
A(x) =
_
P(t)dt =
_
x
0
1dt = x
v = e
x
__
(t
2
+t + 1)e
t
dt +b
_
= e
x
_
t
2
e
t
te
t
+ 2e
t
_

x
0
+be
x
= (x
2
x + 2) (2e
x
) +be
x
y =
1
x
2
x + 2 2e
x
Exercise 16.
v

+
1
x
v = 2x
2
v = e
ln x
__
2x
2
e
ln x
dx +C
_
= x(x
2
+C) = x
3
+Cx
Then since v = y
k
, k = 1 n,
y = (x
3
+Cx)
2
= x
2
(x
2
+C)
2
; x ,= 0
y = x
2
(x
2
1)
2
Check:
y

= 2x(x
2
1)
2
+x
3
(4)(x
2
1)
2x
2
+x
4
(4)(x
2
1) 2x
2
(x
2
1)
2
= 4x
3
Exercise 17. xy

+y = y
2
x
2
log x on (0, +) with y =
1
2
when x =
1
2
, x ,= 0.
141
y

+
y
x
= y
2
xlog x
k = 1 n = 1 2 = 1, v = y
k
= y
1
v

+kPv = kQ; v

+
_
1
x
_
v = 1xlog x
v = x
__
xlog x
x
dt +C
_
= x
__
log tdt +C
_
=
= x((xlog x x) +C) = x
2
log x +x
2
+Cx
y =
1
Cx +x
2
x
2
log x
C =
1
2
(1 log
1
2
) + 2
Check:
y

= (C + 2x 2xlog x x)(y
2
)
y
2
(C +2x + 2xlog x +x +C +x xlog x)
y =
1
2
; x = 1 y(x = 1) =
1
2
b = 2
y =
1
x(xln x x + 3)
Exercise 18. 2xyy

+ (1 +x)y
2
= e
x
on (0, +), y =

e when x = 1; y =

e when x = 1.
If x ,= 0, y ,= 0,
y

+
(1 +x)
2x
y =
e
2
2x
y
1
k = 1 n = 2; v = y
k
= y
2
v

+ 2Pv = 2Q; v

+ 2
_
1 +x
2x
_
v =
2e
x
2x
=v

+
1 +x
x
v =
e
x
x
= v

+P
v
= Q
v
A
v
=
_
x
a
P
v
=
_
x
a
1 +t
t
dt = ln
_
x
a
_
+ (x a); e

x
a
Pvdt
= e
ln x/a+xa
=
x
a
e
xa
v =
ae
x+a
x
__
x
a
e
t
t
t
a
e
ta
dt +b
v
_
=
a
x
e
x+a
_
1
2a
e
2ta

x
a
+b
v
_
=
a
x
e
x+a
_
e
2xa
2a

e
a
2a
+b
v
_
y =

a
x
e
x+a
_
e
2xa
2a

e
a
2a
+b
v
_
Now y
k
= v
y
2
= v = e =
1
1
e
1+1
_
e
21
2(1)

e
1
2
+b
v
_
= b
v
= e
(1) b
v
= e =y =
_
1
x
e
x+1
_
e
2x1
2

e
2
+e
_
(2) b
v
= e =y =
_
1
x
e
x+1
_
e
2x1
2

e
2
+e
_
(3) If we could take a = 0, then lim
x0
y =
_
e
x
2x

e
x+2a
2x
+
bvae
x+a
x
=
_
1+xe
2(0)
(1x)+2bv0e
x+a
2x
= 1
If we consider lim
x0
y
2
, and let a go to 0, then
v =
1
x
_
e
x
e
x
_
=
sinh x
x
Exercise 19. The Ricatti equation is y

+P(x)y +Q(x)y
2
= R(x).
142
If u is a known solution, y = u +
1
v
is also a solution if v satises a rst-order ODE.
(u + 1/v)

= u

+
1
v
2
v

+Py +Qy
2
= R =u

+
v

v
2
+P(u +
1
v
) +Q(u
2
+
2u
v
+
1
v
2
) = R
= v

Pv = Q(2uv + 1)
Exercise 20. y

+y +y
2
= 2, y = 1, 2.
(1) If 2 b < 1,
y

+y +y
2
= 2 P = 1, Q = 1, R = 2
v

+ (P 2Qu)v = Qy = u +
1
v
u = 1
v

+ (1 2(1)(1))v = v

3v = Q = 1
v = e
3x
__
1e
3t
dt +b
_
= e
3x
_
e
3t
3

x
a
+b
_
= be
3x

1
3
_
1 e
3x3a
_
y = 1 +
3
3be
3x
(1 e
3x3a
)
y(0) = 1 +
3
3b (1 e
3a
)
=
b = 1 +
1
b
b
2
b 1 = 0
b =
1

5
2
y = 1 +
3
3be
3x
(1 e
3x
)
b =
1

5
2
(2)
u = 2
v

+ (1 2(1)(2))v = v

+ 3v = 1
v = e
3x
__
x
a
e
3t
dt +b
_
= e
3x
_
e
3x
e
3a
3
_
+be
3x
=
1 e
3a3x
3
+be
3x
y = 2 +
3
1 e
3a3x
+ 3be
3x
; y(0) = 2 +
3
1 e
3a
+ 3b
a=0
y(0) = 2 +
3
3b
=b = 1

2
b 1 or b < 2, y = 2 +
3
1 e
3x
+ 3be
3x
b = 1

2
8.7 Exercises - Some physical problems leading to rst-order linear differential equations.
Exercise 3.
(1) y

= y(t). y(T) = y
0
e
T
=
y0
n
. n = e
T
so the relationship between T and n doesnt depend upon
y
0
.
1
k
ln e = T .
143
(2) f(a) = y
0
e
ka
; f(b) = y
0
e
kb
.
f(a)
f(b)
= e
ka+kb
=ln
f(a)
f(b)
= k(a b);
1
a b
ln
f(a)
f(b)
= k
f(t) = y
0
exp
_
_
ln
f(a)
f(b)
t
a b
_
_
; f(a) = y
0
_
f(b)
y
0
_
a/b
; =
f(a)
(f(b))
a/b
= y
1
a
b
0
f(t) =
_
f(a)
(f(b))
a/b
_ b
ba
_
f(a)
f(b)
_ t
ab
=
(f(a))
b
ba
f(b)
a
ba
f(a)

t
ba
f(b)
t
ab
=
=
f(a)
bt
ba
(f(b))
at
ba
= f(a)
bt
ba
f(b)
ta
ba
w(t) =
b t
b a
; 1 w(t) =
b a (b t)
b a
=
t a
b a
Exercise 4. F = mv

= w
0

3
4
v
w
0
= 192
w
0
g
= 6 = m
v

=
w
0
m

1
8
v =v

+
1
8
v =
w
0
m
= 32
v(t) = e

t
8
__
t
0
w
0
m
e
t
8
dt +b
_
= e
t
8
_
8w
0
m
(e
t/8
1) + 0
_
=
_
256(1 e
t/8
)
_
v(10) = 256(1 e
5/4
) = 256(1 37/128) = 182
F = mv

= w
0
12v v

+
12
m
v =
w
0
m
= v

+ 2v = 32
v(t) = e
2t
__
t
t0
e
2x
32dx +b
_
= e
2t
_
16(e
2t
e
2t0
) +b
_
=
_
16(1 e
2(t0t)
) +be
2t
_
v(10) = be
2(10)
= 182 =b = 182e
20
v(t) = 16 + 166e
202t
So then
v(t) =
_
256(1 e
t/8
) if t < 10
16 + 166e
202t
if t > 10
Exercise 7.
(1)
y

(t) = (y M)k = ky kM y

+ky = kM
y = e
kt
(
_
kMe
kt
dt +b) = e
kt
(M(e
kt
1) +b) M = 60

y(0) = b = 200

y
f
= e
kT
(M(e
kT
1) + 200) = M +e
kT
(200 M)
1
T
ln
_
y
f
M
b M
_
= k =
1
T
(ln (60) ln (140)) =
1
T
ln
_
3
7
_
=
1
30
(ln 3 ln 7)
y(t) = 60 + 140e
ln 3ln 7
30
t
(2)
y(t) = 60 + 140e
kt
; k =
(ln 7 ln 3)
30
ln
_
Y 60
140
_
= kt =t
f
=
(ln 140 ln (T 60))
k
for 60 < T 200
(3) t
f
=
ln (140)ln (30)
k
=
30
ln 7/3
ln
14
3
= 54 minutes
144
(4) M = M(t) = M
0
t =
1
10
y = e
kt
__
kMe
kt
dt +b
_
= e
kt
__
t
0
k(M
0
u)e
ku
du +b
_
=
= ke
kt
_
t
0
(M
0
e
ku
ue
ku
)du +be
kt
=
= ke
_
M
0
e
ku
k

t
0

_
ue
ku
k
+
e
ku
k
2
_

t
0
_
+be
kt
=
= ke
kt
_
M
0
k
(1 e
kt
)
_
te
kt
k

e
kt
k
2
+
1
k
2
__
+be
kt
=
y(t) = M
0
e
kt
+M
0
t /k +e
kt
/k +be
kt
= (M
0
+/k +b)e
kt
+ (M
0
t /k) =
y(t) = (140 +
3
(ln 3 ln 7)
)e
kt
+ (60
t
10

_
3
ln 3/7
_
)
Exercise 8. y

(t) = k(y M
0
); y

+ky = kM
0
.
y = e
kt
__
t
f
ti
kM
0
e
ku
du +b
_
= e
kt
f
_
M
0
(e
kt
f
e
kti
) +b
_
= M
0
(1 e
k(t
f
ti)
) +be
kt
f
=
y(t
f
) M
0
= M
0
e
k(t
f
ti)
+be
kt
f
65 M
0
= M
0
e
k(5)
+ 75e
k(5)
= (75 M
0
)e
5k
=ln
_
65 M
0
75 M
0
_
= 5k; k =
1
5
ln
_
75 M
0
65 M
0
_
60 M
0
= M
0
e
k(5)
+ 65e
k(5)
= (65 M
0
)e
5k
=ln
_
60 M
0
65 M
0
_
= 5k
=
65 M
0
75 M
0
=
60 M
0
65 M
0
M
0
= 55
Exercise 9.
Let y(t) = absolute amount of salt.
Water is leaving according to w(t) = w
0
+ (3 2)t = w
0
+t.
Salt leaving =
_
2 gal
min.
__
y(t) salt
w(t)
_
So then
y

=
2y
w
0
+t
=ln y = 2(ln (w
0
+t) ln w
0
) = 2 ln
_
w
0
+t
w
0
_
is the equation of motion given by the problem.
y(t) = Ce
ln

w
0
+t
w
0

2
= C
_
w
0
+t
w
0
_
2
y(t) = 50
_
100
100 +t
_
2
y(t = 60 min.) = 50
_
100
160
_
2
= 50
25
64
=
625
32
19.53
Exercise 10.
Let y be the dissolved salt (total amount of) at t time. The (total) amount of water at any given time in the tank is w = w
0
+t.
There is dissolved salt in mixture that is leaving the tank at any minute. There is also salt from undissolved salt in the tank
that is coming into the dissolved salt, adding to the amount of dissolved salt in the mixture. Thus
y

(t) = (2)
_
y
w
_
+
_
y
w
3
_
; =
1 gal
3 min
We obtained easily by considering only the dissolving part and how it dissolves 1 pound of salt per minute if the salt
concentration,
y
w
was zero, i.e. water is fresh.
145
y

(t) =
7
3
y
w
+ 1; y

=
7
3
y
w
0
+t
= 1
P =
7/3
w
0
+t
_
P =
_
7/3
w
0
+t
=
7
3
ln (w
0
+t)[
t
0
=
7
3
ln
_
w
0
+t
w
0
_
y = e
ln

w
0
+t
w
0

7/3
__
t
0
(1)e
ln

w
0
+u
w
0

7/3
du +b
_
=
=
_
w
0
w
0
+t
_
7/3
_
_
t
0
_
w
0
+u
w
0
_
7/3
du +b
_
=
_
w
0
w
0
+t
_
7/3
_
3w
0
10
_
_
w
0
+t
w
0
_
10/3
1
_
+b
_
=
y =
_
100
100 + 60
_
7/3
_
3(100)
10
_
_
100 + 60
100
_
10/3
1
_
+ 50
_
54.78 lbs.
Exercise 11. LI

(t) +RI(t) = V (t) V (t) = E sin t.


I(t) = I(0)e
Rt/L
+e
Rt/L
_
t
0
V (x)
L
e
Rx/L
dx
I(t) = I(0)e
Rt/L
+e
Rt/L
E
L
_
t
0
sin xe
Rx/L
dx
Using
_
e
ax
sin bxdx = ae
ax
sin bx be
ax
cos bx
I(t) = I(0)e
Rt
L
+
Ee
Rt/L
L
_
R
L
e
Rt
L
sin t e
Rt
L
cos t
_
R
L
_
2
+
2
+

_
R
L
_
2
+
2
_
I(0) = 0 So
I(t) =
E
L
R
L
sin t cos t
_
R
L
_
2
+
2
+
EL
R
2
+ (L)
2
e
Rt/L
=
E(Rsin t Lcos t)
R
2
+ (L)
2
+
EL
R
2
+ (L)
2
e
Rt/L
=
sin =
L
_
R
2
+ (L
2
)
=I(t) =
E sin (t )
_
R
2
+ (L)
2
+
EL
(R
2
+
2
L
2
)
e
Rt/L
L = 0 sin = 0
Exercise 12.
E(t) =
_
E if 0 < a t < b
0 otherwise
I(t) = e
Rt/L
_
t
0
0 = 0 for t < a
I(t) = e
Rt/L
_
t
a
E
L
e
Rx/L
dx =
E
L
e
Rt/L
L
R
_
e
Rt/L
e
Ra/L
_
=
E
R
_
1 e
R(at)
L
_
I(b) =
E
R
(1 e
R(ab)/L
)
for t > b, I(t) = Ke
Rt/L
=I(t) =
Ee
Rt/L
R
(e
Rb
L
= e
Ra
l
) for I(b) = I(b)
Exercise 13. From Eqn. 8.22,
dx
dt
= kx(M x)
146
dx
dt
= kx(M x) = kMx kx
2
; =
dx
kMx kx
2
= dt =
1/kdx
x(M x)
= dt
=kdt =
_
1
x
+
1
M x
_
1
M
dx =
ln x +ln (M x)
M
Mk(t t
i
) = ln
x
M x
; e
Mk(tti)
(M x) = x
x(t) =
Me
Mk(tti)
1 +e
Mk(tti)
=
M
1 +e
Mk(tti)
Exercise 14. Note that we are given three equally spaced times.
M = x
2
+x
2
e
(t2t0)
;
M x
2
x
2
= e
(t2t0)
M x
2
x
2
_
x
1
M x
1
_
= e
t2+t0+t1t0
= e
(t2t1)
M x
3
x
3
_
x
2
M x
2
_
= e
(t3t2)
=
M x
2
x
2
_
x
1
M x
1
_
(M x
3
)(M x
1
)x
2
2
= x
1
x
3
(M x
2
)
2
= x
1
x
3
(M
2
2Mx
2
+x
2
2
) = x
2
2
(M
2
M(x
1
+x
3
) +x
1
x
3
)
(x
2
2
x
1
x
3
)M
2
= M(x
2
2
(x
1
+x
3
) +2x
2
x
1
x
3
) = (x
1
(x
3
x
2
) +x
3
(x
2
x
1
))x
2
= M = x
2
(x
3
(x
2
x
1
) x
1
(x
3
x
2
))
x
2
2
x
1
x
3
Exercise 15.
dx
dt
= k(t)Mx k(t)x
2
dx
Mx x
2
= k(t)dt =M
_
t
ti
k(u)du = ln
_
x
M x
_
x
M x
= e
M

t
t
i
k(u)du
x =
Me
M

t
t
i
k(u)du
1 +e
M

t
t
i
k(u)du
=
M
1 +e
M

t
t
i
k(u)du
Exercise 16.
(1) M = 23
92(233.9)3.9(9223)
23
2
3.9(92)
= 201
(2)
M = 122
_
150(122 92) 92(150 122)
(122)
2
92(150)
_
= 122
_
150(30) 92(28)
(122)
2
92(150)
_
= 216
(3) Reject.
8.14 Exercises - Linear equations of second order with constant coefcients, Existence of solutions of the equation
y

+ by = 0, Reduction of the general equation to the special case y

+ by = 0, Uniqueness theorem for the equation


y

+by = 0, Complete solution of the equation y

+by = 0, Complete solution of the equation y

+ay

+by = 0.
Exercise 1. y

4y = 0 y = c
1
e
2x
+c
2
e
2x
.
Exercise 2. y

+ 4y = 0 y = c
1
cos (2x) +c
2
sin (2x).
Use Theorem 8.7.
Theorem 27. Let d = a
2
4b be the discrimnant of y

+ay

+by = 0.
Then solutions on (, ) has the form
(22) y = e
ax/2
(c
1
u
1
(x) +c
2
u
2
(x))
where
(1) If a = 0, then u
1
(x) = 1 and u
2
(x) = x
(2) If d > 0, then u
1
(x) = e
kx
and u
2
(x) = e
kx
, where k =

d
2
(3) If d < 0, then u
1
(x) = cos kx and u
2
(x) = sin kx; where k =
1
2

d
147
Exercise 3. y

4y

= 0; a = 4.
y = e
2x
(c
1
e
2x
+c
2
e
2x
) = c
1
e
4x
+c
2
Exercise 4. y

+ 4y

= 0
y = e
2x
(c
1
e
2x
+c
2
e
2x
) = c
1
+c
2
e
4x
Exercise 5. y

2y

+ 3y = 0 d = 4 4(3) = 8 = y = e
x
(c
1
sin

2x +c
2
cos

2x)
Exercise 8. y

2y

+ 5y = 0 d = 16 y = e
x
(c
1
cos 2x +c
2
sin 2x)
Exercise 9. y

+ 2y

+y = 0 d = 4 4(1)(1) = 0 y = e
x
(1 +x)
Exercise 10. y

2y

+y = 0 d = 4 4(1)(1) = 0 y = e
x
(1 +x)
Exercise 11. y

+
3
2
y

= 0 y = 1, y

= 1; x = 0 d =
9
4
> 0
y = e
3
4
x
(c
1
e
3x
4
+c
2
e
3x
4
) = c
1
+c
2
e
3x
2
c
2
=
2
3
=y =
5
3
+
2
3
e
3x
2
Exercise 12. y

+ 25y = 0; y = 1, y

= 0, x = 3.
y = c
1
sin 5x +c
2
cos 5x
y

= 5c
1
cos 5x +5c
2
sin 5x
0 = 5c
1
cos 15 5c
2
sin 15c
2
sin 15 = c
1
cos 15
1 = c
1
sin 15 +c
2
cos 15
1c
1
(sin 15 +
cos
2
15
sin 15
) = c
1
_
1
sin 15
_
c
1
= sin 15
c
2
= cos 15
y = sin 15 sin 5x cos 15 cos 5x
Exercise 13. y

4y

y = 0; y = 2; y

= 1 when x = 1
d = 10 4(1)(1) = 20
y = c
1
e
(2+

5)x
+c
2
e
(2

5)x
y(x = 1) = c
1
e
2+

5
+c
2
e
2

5
= 2
y

(x = 1) = (2 +

5)c
1
e
2+

5
+ (2

5)c
2
e
2

5
= 1
=(5 + 2

5)c
1
e
2+

5
+ (5 2

5)c
2
e
2

5
= 0
c
1
=
2

5 5
5 + 2

5
c
2
e
2

5
_
2

5 5
5 + 2

5
_
c
2
e
2

5
+c
2
e
2

5
= 2 =
4

5c
2
e
2

5
5 + 2

5
=
5 + 2

5
2

5
e

52
= c
2
c
1
=
2

5 5
2

5
e
2

5
y =
2

5 5
2

5
e
2

5
e
(2+

5)x
+
5 + 2

5
2

5
e

52
e
(2

5)x
Exercise 14. y

+ 4y

+ 5y = 0, with y = 2 and y

= y

when x = 0
16 4(1)(5) = 4
y = e
2x
(c
1
sin 2x +c
2
cos 2x) y(x = 0) = c
2
= 2
y

= 2e
2x
(c
1
sin 2x + 2 cos 2x) + 2e
2x
(c
1
cos 2x 2 sin 2x)
y

= 4e
2x
(c
1
sin 2x + 2 cos 2x) 8e
2x
(c
1
cos 2x 2 sin 2x) + 4e
2x
(c
1
sin 2x 2 cos 2x)
y

(0) = 2(2) + 2(c


1
) = 4 + 2c
1
y

(0) = 4(2) + (c
1
) + 4(2) = c
1
= 4 + 2c
1
c
1
=
4
3
y = e
2x
(
4
3
sin 2x + 2 cos 2x)
148
Exercise 15. y

4y

+ 29y = 0
d = 16 4(1)(29) = 100 =u = e
2x
(c
1
sin 5x +c
2
cos 5x).
v : y

+ 4y

+ 13y = 0
d = 10 4(13) = 36 =v = e
2x
(b
1
sin 3x +b
2
cos 3x)
v(0) = b
2
u(0) = 1(0 +c
2
) = c
2
= 0
u = e
2x
c
1
sin 5x
u

= 2e
2x
c
1
sin 5x +e
2x
c
1
5 cos 5x
u

2
_
= 1 = 2e

c
1
(1) c
1
=
1
2e

(0) =
1
2e

5
u

(0) = v

(0) =b
1
=
5
6e

v = e
2x
b
1
sin 3x
v

(0) = 3b
1
u =
1
2e

e
2x
sin 5x
v = e
2x
5
6e

sin 3x
Exercise 16.
y

3y

4y = 0 u 9 4(1)(4) = 25 u = e
3x
2
(c
1
e
5x
2
+c
2
e

5x
2
)
y

+ 4y

5y = 0 v 16 4(1)(5) = 36 v = e
2x
(b
1
e
3x
+b
2
e
3x
)
u(0) = c
1
+c
2
= 0v(0) = b
1
+b
2
= 0 =
u = 2e
3x
2
c
1
(sinh
_
5x
2
_
)
v = 2b
1
e
2x
(sinh (3x))
u

= c
1
3
2
e
3x
2
(e
5x
2
e
5x
2
) + 2e
3x
2
c
1
5
2
cosh
_
5x
2
_
u

(0) = 5c
1
v

= 4b
1
e
2x
sinh (3x) + 6b
1
e
2x
cosh (3x)
v

(0) = 6b
1
c
1
=
6b
1
5
Exercise 17.
y

+ky = 0
d = 4(k)

d
2
=

4k
2
=

k
Assume k > 0
y = c
1
sin

kx +c
2
cos

kx
y(0) = c
2
= 0
y(1) = c
1
sin

k1 = 0 =

k = n
k < 0; k = > 0
y = c
1
e

x
+c
2
e

x
; y(0) = c
1
+c
2
= 0 y = c
1
sinh

x
y = c
1
sinh

1 = 0 c
1
= 0
so if k < 0, there are no nontrivial solutions satisfying f
k
(0) = f
k
(1) = 0
Exercise 18. y

+k
2
y = 0 d = 4k
2
< 0

d
2
=
2k
2
= k > 0
149
y = c
1
sin kx +c
2
cos kx
y

= kc
1
cos kx c
2
k sin kx
y(a) = b = c
1
sin ka +c
2
cos ka
y

(a) = m = kc
1
cos ka c
2
k sin ka
kb cos ka = kc
1
cos ka sin ka +c
2
k cos
2
ka
msin ka = kc
1
cos ka sin ka c
2
k sin
2
ka
kb cos ka msin ka = c
2
k =c
2
=
kb cos ka msin ka
k
c
1
sin ka = b c
2
cos ka =
kb
k

_
kb cos ka msin ka
k
_
cos ka =
=
kb(1 cos
2
ka) +msin ka cos ka
k
=
kb sin
2
ka +msin ka cos ka
k
c
1
=
kb sin ka +mcos ka
k
y =
_
kb sin ka +mcos ka
k
_
sin kx +
_
kb cos ka msin ka
k
_
cos kx
k = 0 =y = mx ma +b
Exercise 19.
(1) y = k
1
sin x +k
2
cos x
b
2
= k
1
sin (a
2
) +k
2
cos (a
2
) = k
1
s
2
+k
2
c
2
b
1
= k
1
sin (a
1
) +k
2
cos (a
1
) = k
1
s
1
+k
2
c
1
b
2
c
1
= k
1
s
2
c
1
+k
2
c
2
c
1
(b
1
c
2
= k
1
s
1
c
2
+k
2
c
1
c
2
)
=b
2
c
1
b
1
c
2
= k
1
(s
2
c
1
s
1
c
2
)
=k
1
=
b
2
c
1
b
1
c
2
s
2
c
1
s
1
c
2
s
1
b
2
= k
1
s
1
s
2
+k
2
s
1
c
2
(s
2
b
1
= k
1
s
1
s
2
+k
2
c
1
s
2
)
=k
2
=
s
1
b
2
s
2
b
1
s
1
c
2
c
1
s
2
y =
_
b
2
cos a
1
b
1
cos a
2
sin a
2
cos a
1
sin a
1
cos a
2
_
sin x +
_
b
2
sin a
1
b
1
sin a
2
sin a
1
cos a
2
cos a
1
sin a
2
_
cos x
y =
b
2
cos a
1
b
1
cos a
2
sin (a
2
a
1
)
sin x +
b
2
sin a
1
b
1
sin a
2
sin (a
1
a
2
)
cos x if a
2
a
1
,= n
otherwise, if a
2
a
1
= n;
b
2
c
1
b
1
c
2
= 0
b
2
s
1
b
1
s
2
= 0
b
2
c
1
= b
1
(1)
n
c
1
; if c cos (a
1
) ,= 0, b
2
= b
1
(1)
n
(2) Its true if a
1
= a
2
=

4
; b
1
= b
2
.
(3) y

+k
2
y = 0
y = Asin kx +Bcos kx
y(a
1
) = Asin ka
1
+Bcos ka
1
= b
1
= AS
1
+BC
1
y(a
2
) = Asin ka
2
+Bcos ka
2
= b
2
= AS
2
+BC
2
_
S
1
C
1
S
2
C
2
_ _
A
B
_
=
_
b
1
b
2
_
_
A
B
_
=
_
C
2
C
1
S
2
S
1
_ _
1
S
1
C
2
C
1
S
2
__
b
1
b
2
_
S
1
C
2
C
1
S
2
= sin ka
1
cos ka
2
cos ka
1
sin ka
2
= sin (k(a
1
a
2
))
y =
b
1
cos (ka
2
) b
2
cos (ka
1
)
sin (k(a
1
a
2
))
sin ka
1
+
b
1
sin (ka
2
) +b
2
sin (ka
1
)
sin (k(a
1
a
2
))
sin ka
2
if k(a
1
a
2
) ,= n k = 0; y =
_
b
2
b
1
a
2
a
1
_
(x a
1
) +b
1
Exercise 20.
(1)
u
1
(x) = e
x
; u
2
(x) = e
x
u

2
= e
x
u

2
= e
x
=y

y = 0
150
(2)
u
1
= e
2x
u

1
= 2e
2x
u

1
= 4e
2x
u
2
= xe
2x
u

2
= e
2x
+ 2xe
2x
u

2
= 2e
2x
+ 2e
2x
+ 4xe
2x
= 4e
2x
+ 4xe
2x
= 4e
2x
(1 +x)
u

2
4u

2
+ 4u
2
= 0 = y

4y

+ 4y = 0
(3)
u
1
(x) = e
x/2
cos x;
u

1
=
1
2
e
x/2
cos x +e
x/2
sin x
u

1
=
1
4
e
x/2
cos x +e
x/2
sin x +e
x/2
cos x
=
3
4
e
x/2
cos x +e
x/2
sin x
u
2
(x) = e
x/2
sin x
u

2
=
1
2
e
x/2
sin x +e
x/2
cos x
u

2
=
1
4
e
x/2
sin x +e
x/2
cos x e
x/2
sin x =
=
3
4
e
x/2
sin x e
x/2
cos x
u

2
+u

2
+
5
4
u
2
= 0
y

+y

+
5
4
y = 0
(4) u
1
(x) = sin (2x + 1); u
2
(x) = sin (2x + 2)
u

1
= 2 cos (2x + 1)
u

1
= 4 sin (2x + 1)
u

2
= 2 cos (2x + 2)
u

2
= 4 sin (2x + 2)
y

+ 4y = 0
(5)
u
1
= cosh x
u

1
= sinh x
u

1
= cosh x
y

y = 0
Exercise 21. w = u
1
u

2
u
2
u

1
.
(1) w = 0 x open interval I,
_
u
2
u
1
_

=
u

2
u
1
u

1
u
2
u
2
1
= 0 =
u
2
u
1
= c
If
u2
u1
is not constant, then w(0) ,= 0 for at least one c in I (otherwise, itd be constant).
(2) w

= u
1
u

2
u
2
u

1
Exercise 22.
(1) w

+aw = u
1
u

2
u
2
u

1
+a(u
1
u

2
u
2
u

1
) = u
1
(bu
2
) +u
2
(bu
1
) = 0
w(x) = w(0)e
ax
if w(0) ,= 0, then w(x) ,= 0 x.
(2) u
1
,= 0 If w(0) = 0, w(x) = 0 x, so
u2
u1
constant. If
u2
u1
constant, w(0) = 0 since from the previous part.
Exercise 23. Recall the properties of the Wronskian.
(1) If W(x) = v
1
(x)v

2
v
2
v

1
= v
1
v

2
v
2
v

1
= 0 x I,
then
v2
v1
constant on I
(2) W

= v
1
v

2
v
2
v

1
(3) W

+aW = 0 if v
1
, v
2
are solutions to y

+ay

+by = 0
W(x) = W(0)e
ax
So if W(0) ,= 0, W(x) ,= 0 x
151
Consider adding together the solutions and the solutions derivatives into some function f. By the linearity of the differential
equation, we know that f is also a solution since it is a linear superposition of solutions.
y(x) = Av
1
(x) +Bv
2
(x)
f(0) = Av
1
(x) +Bv
2
(0)
y

(x) = Av

1
(x) +Bv

2
(x)
f

(0) = Av

1
(0) +Bv

2
(0)
_
v
1
(0) v
2
(0)
v

1
(0) v

2
(0)
_ _
A
B
_
=
_
f(0)
f

(0)
_
=
1
W(0)
_
v

2
(0) v
2
(0)
v

1
(0) v
1
(0)
_ _
f(0)
f

(0)
_
=
_
A
B
_
since W(0) ,= 0, this division is allowed above
so y(x) =
_
v

2
(0)f(0) v
2
(0)f

(0)
W(0)
_
v
1
(x) +
_
v
1
(0)f

(0) v

1
(0)f(0)
W(0)
_
v
2
(x)
f(0), f

(0) are initial conditions for y. f(0), f

(0) are arbitrary.


But since W(0) ,= 0, W(0) = v
1
(0)v

2
(0) v
2
(0)v

1
(0), we can do things like
f(0) =
v

2
(0)f(0) v
2
(0)f

(0)
v
1
(0)v

2
(0) v
2
(0)v

1
(0)
v
1
(0) +
v
2
(0)f

(0) v

1
(0)f(0)
W(0)
v
2
(0)
8.17 Exercises - Nonhomogeneous linear equations of second order with constant coefcients, Special methods for
determining a particular solution of the nonhomogeneous equation y

+ay

+by = R.
Exercise 1. y

y = x homogeneous solution c
1
e
x
+c
2
e
x
. y
p
= x
y = c
1
e
x
+c
2
e
x
x
Exercise 2. y

= x
2
For the homogeneous solution
y

= 0 d = (1)
2
4(1)(0) = 1 y
h
= e
x
2
_
c
1
e
x
2
+c
2
e

x
2
_
= c
2
e
x
+c
1
y
p
= Ax
3
+Bx
2
+Cx +D
y

p
= 3Ax
2
+ 2Bx +C
y

p
= 6Ax + 2B
y = c
1
+c
2
e
x
+
1
3
x
3
+x
2
Exercise 3. y

+y

= x
2
+ 2x
e

x
2
_
c
1
e

x
2
+c
2
e
x
2
_
= c
1
e
x
+c
2
P = Ax
3
+Bx
2
+Cx +D; P

= 3Ax
2
+ 2Bx +C P

= 6Ax + 2B
3Ax
2
+ 2Bx +C + 6Ax + 2B = x
2
+ 2x A =
1
3
B = 0 C = 0
y = c
1
e
x
+c
2
+
1
3
x
3
Exercise 4. y

2y

+ 3y = x
3
u = e
x
(c
1
sin

2x +c
2
cos

2x)
3(Ax
3
+Bx
2
+Cx +D)
2(3Ax
2
+ 2Bx +C)
(6Ax + 2B)
A =
1
3
B =
2
3
C =
8
9
D =
16
27
y = C
1
e
x
sin

2x +C
2
e
x
cos

2x +
1
3
x
3
+
2
3
x
2
+
8
9
x +
16
27
Exercise 5. y

5y

+ 4y = x
2
2x + 1
y
h
= e
5x
2
_
e
3x
2
+e
3x
2
_
= c
1
e
4x
+c
2
e
x
152
d =
_
25 4(4) = 3
4(Ax
2
+Bx +C)
5(2Ax +B)
2A
4Ax
2
+ 4Bx + 4C
10Ax 5B
2A
A =
1
4
B =
1
8
1
2

5
8
+ 4C = 1
C =
9
32
y = c
1
e
4x
+c
2
e
x
+
1
4
x
2
+
1
8
x +
9
32
Exercise 6.
y

+y

6y = 2x
3
+ 5x
2
7x + 2
y
h
= e
x
2
(e
5x
2
+e
5x
2
) = e
3x
+e
2x
d =
_
1 4(6) = 5
y
p
= Ax
3
+Bx
2
+Cx +D
y

p
= 3Ax
2
+ 2Bx +C
y

p
= 6Ax + 2B
=
6Ax
3
6Bx
2
6Cx 6D
3Ax
2
+ 2Bx +C
6Ax + 2B
A =
1
3
B = 1 C =
1
2
D =
7
12
y = C
1
e
3x
+C
2
e
2x

1
3
x
3
+x
2
+
1
2
x
7
12
Exercise 7.
y

4y = e
2x
y
h
= c
1
e
2x
+c
2
e
2x
v
1
= e
2x
v

2
= 2e
2x
v

1
= 2e
2x
v
2
= e
2x
Use Theorem 8.9.
Theorem 28. Let v
1
, v
2
be solutions to L(y) = 0 where L(y) = y

+ay

+by
Let W = v
1
v

2
v
2
v

1
. Then L(y) = R where
y
p
= t
1
v
1
+t
2
v
2
(23) t
1
=
_
v
2
R
W(x)
dx; t
2
(x) =
_
v
1
R
W
dx
t
1
=
_
e
2x
e
2x
4
=
1
4
x
t
2
=
_
e
2x
e
2x
4
=
e
4x
16
w = e
2x
(2)e
2x
e
2x
2e
2x
= 4
y
p
=
x
4
e
2x
+
e
2x
16
y = c
1
e
2x
+c
2
e
2x
+
x
4
e
2x
+
e
2x
16
Exercise 8.
153
y

+ 4y = e
2x
y
h
= c
1
sin 2x +c
2
cos 2x
w = sin 2x sin 2x(2) 2 cos 2xcos 2x = 2
t
1
=
_
cos 2xe
2x
dx
2
=
1
2
_
e
2x
cos 2xdx
t
2
=
_
sin 2xe
2x
2
=
1
2
_
e
2x
sin 2xdx
(e
2x
cos 2x)

= 2e
2x
cos 2x +2e
2x
sin 2x
(e
2x
sin 2x)

= 2e
2x
sin 2x + 2e
2x
cos 2x
_
e
2x
sin 2x e
2x
cos 2x
4
_

= e
2x
cos 2x
_
e
2x
sin 2x +e
2x
cos 2x
4
_

= e
2x
sin 2x
t
1
=
e
2x
cos 2x e
2x
sin 2x
8
t
2
=
e
2x
sin 2x +e
2x
cos 2x
8
y
p
=
e
2x
sin 2xcos 2x e
2x
sin
2
2x
8
+
e
2x
sin 2xcos 2x +e
2x
cos
2
2x
8
=
e
2x
8
y = c
1
sin 2x +c
2
cos 2x +
e
2x
8
Exercise 9. y

+y

2y = e
x
d
2
= 1 (4)(1)(2) = 9
y
h
= e
1x
2
_
e
3x
2
+e
3x
2
_
= e
x
+e
2x
(xe
x
)

= e
x
+xe
x
+(xe
x
)

= +(2e
x
+xe
x
)
=3e
x
+ 2xe
x
y = c
1
e
x
+c
2
e
2x
+
1
3
xe
2
Exercise 10. y

+y

2y = e
2x
. y
h
= e

x
2
_
c
1
e
3x
2
+c
2
e

3x
2
_
= c
1
e
x
+c
2
e
2x
.
W(x) = v
1
v

2
v
2
v

1
= e
x
(2)e
2x
e
2x
e
x
= 3e
x
t
1
=
_
v
2
R
W
t
2
=
_
v
1
R
W
t
1
=
_
e
2x
e
2x
3e
x
=
1
3
e
x
t
2
=
_
e
x
e
2x
3e
x
=
1
12
e
4x
y
1
= t
1
v
1
+t
2
v
2
=
1
3
e
x
e
x
+
1
12
e
4x
e
2x
=
1
4
e
2x
y = c
1
e
x
+c
2
e
2x
+
1
4
e
2x
Exercise 11. y

+y

2y = e
x
+e
2x
.
Consider solutions to Exercise 9,10.
L(y
a
) = e
x
; L(y
b
) = e
2x
; L(y
a
+y
b
) = e
x
+e
2x
= y = c
1
e
x
+c
2
e
2x
+
1
3
xe
x
+
1
4
e
2x
Exercise 12. y

2y

+y = x + 2xe
x
. d = 4 4(1) = 0. Recall the denition to be learned for this section of exercises:
Theorem 29. Let d = a
2
4b be the discriminant of y

+ay

+by = 0. Then every solution of this equation on (, )


has the form
(24) y = e
ax/2
(c
1
u
1
(x) +c
2
u
2
(x))
(1) If d = 0 then u
1
= 1, u
2
= x
(2) If d > 0, u
1
= e
kx
; u
2
= e
kx
, k =

d
2
(3) If d < 0, u
1
= cos kx, u
2
= sin kx, k =

d
2
154
y
h
= e
x
(c
1
+c
2
x) = c
1
e
x
+c
2
xe
x
t
1
=
_
xe
x
(2xe
x
)
e
2x
=
2x
3
3
t
2
=
_
e
x
(2xe
x
)
e
2x
= x
2
W(x) = e
x
(e
x
+xe
x
) (xe
x
)(e
x
) = e
2x
y
p
=
2x
3
3
e
x
+x
3
e
x
=
x
3
e
x
3
y =
x
3
e
x
3
+c
1
e
x
+c
2
xe
x
Exercise 13. y

+ 2y

+y =
e
x
x
2
y
h
= e
x
(c
1
+c
2
x) = c
!
e
x
+c
2
xe
x
t
1
=
_
xe
x
_
e
x
x
2
_
e
2x
= ln x t
2
=
_
e
x
_
e
x
x
2
_
e
2x
=
1
x
W = e
x
(e
x
+xe
x
) (e
x
)(xe
x
) = e
2x
y
p
= ln xe
x
+xe
x
_
1
x
_
= ln xe
x
e
x
y = c
1
e
x
+c
2
xe
x
+ (ln x 1)e
x
Exercise 14. y

+y = cot
2
x. y
h
= c
1
sin x +c
2
cos x.
t
1
=
_
cos x
1
cot
2
xdx =
_
cos xcos
2
x
sin
2
x
=
_
cos x(1 sin
2
x)
sin
2
x
=
1
sin x
+sin x
t
2
=
_
sin x
1
cot
2
x =
_
cos
2
x
sin x
=
_
1 sin
2
x
sin x
= ln [ csc x + cot x[ +cos x
y
p
= 1 sin
2
x + cos xln [ csc x + cot x[ cos
2
x = 2 + cos xln [ csc x + cot x[
y = c
1
sin x +c
2
cos x 2 + cos xln [ csc x + cot x[
Exercise 15. y

y =
2
1+e
x
y
h
= c
1
e
x
+c
2
e
x
=W(x) = e
x
e
x
e
x
e
x
= 2
t
1
=
_
e
x 2
1+e
x
2
=
_
e
x
1 +e
x
=
=
_
1
e
x

1
1 +e
x
= e
x

_
1
1 +e
x
= e
x
+
_
e
x
e
x
+ 1
= e
x
+ ln (1 +e
x
)
t
2
=
_
e
x 2
1+e
x
2
= ln [1 +e
x
[
y = 1 +e
x
ln (1 +e
x
) +e
x
ln (1 +e
x
) +c
1
e
x
+c
2
e
x
Exercise 16. y

+y

2y =
e
x
1+e
x
155
Discriminant:
1

1
2
4(2)
2
= 2, 1 =y
h
= c
1
e
x
+c
2
e
2x
=W = e
x
(2)e
2x
e
2x
e
x
= 3e
x
t
1
=
_
e
2t
_
e
t
1+e
t
_
3e
t
=
1
3
_
1
1 +e
t
=
1
3
ln (1 +e
x
)
t
2
=
_
e
t
_
e
t
1+e
t
_
3e
t
=
1
3
_
e
3t
1 +e
t
u=e
t

1
3
_
u
2
du
1 +u
=
=
1
3
_
u +
u
u + 1
=
1
3
_
u +1 +
1
u + 1
=
1
3
(
1
2
u
2
u + ln u + 1)
=
1
6
e
2x
+
e
x
3
+
1
3
ln (e
x
+ 1)
y
1
=
1
3
e
x
ln (1 +e
x
) +
1
6
+
e
x
3

e
2x
3
ln (e
x
+ 1) +c
1
e
x
+c
2
e
2x
Exercise 17. y

+ 6y

+ 9y = f(x); where f(x) = 1 for 1 x 2. f(x) = 0 for all other x.


d = 36 4(1)(9) = 0
y
h
= e
3x
(c
1
+c
2
x) = c
1
e
3x
+c
2
xe
3x
W(x) = e
3x
(e
3x
3xe
3x
) (xe
3x
)(3e
3x
) = e
6x
t
1
(x) =
_

_
a < 1 < x
_
x
1
te
3t
dt =
_
te
3t
3
+
e
3t
9
_

x
1
=
3xe
3x
+e
3x
9
+
2
9
e
3
a < 1 < 2 < x
_
2
1
te
3t
dt =
6e
6
+e
6
9
+
2e
3
9
=
5e
6
9
+
2e
3
9
1 < a < x < 2
_
x
a
te
3t
dt =
_
te
3t
3
+e
3t
9
_

x
a
=
xe
3x
3
+
e
3x
9
+
ae
3a
3

e
3a
9
t
2
(x) =
_
e
3t
f(t)
e
6t
=
_
e
3t
f(t) =
_
x
a
e
3t
=
1
3
_
e
3x
e
3a
_
y
1
= e
3x
_
3xe
3x
+e
3x
9
+C
_
+
x
3
y = c
1
e
3x
+c
2
xe
3x
+
1
9
when 1 x 2; otherwise y = y
h
Exercise 18. Start from y

k
2
y = R(x). Suppose L(y
p
) = y

p
k
2
y
p
= R(x).
y
h
= c
1
sinh (kx) +c
2
cosh (kx); L(y
h
) = 0
So consider L(y
p
+y
h
) = R(x); y
p
+y
h
= y
1
is a nother particular solution.
The key to this problem is to apply the integration directly on the ODE itself, not to go the other way around by differenti-
ating the supposed particular solution.

x
0
dt sinh (k(xt))

_
x
0
dt
d
2
y
dt
2
(t) sinh (k(x t)) k
2
_
x
0
dty(t) sinh (k(x t)) =
_
x
0
dtR(t) sinh (k(x t))dt
_
x
0
y

sinh () = y

(0) sinh (kx)


_
y

cosh ()(k) =
= y

(0) sinh (kx) +k(y(x) y(0) cosh (kx) +k


_
y(t) sinh ()) =
= y

(0) sinh (kx) +ky(x) ky(0) cosh (kx) +k


2
_
y(t) sinh ()
=y(x)
y

(0) sinh (kx)


k
y(0) cosh (kx) =
1
k
_
x
0
dtR(t) sinh (k(x t))dt
Now note that L(y
h
) = 0, so applying
_
x
0
dt sinh (k(x t)) results in 0 still.
156
With y
p
(x) =
1
k
_
x
0
dtR(t) sinh (k(x t))dt +
y

(0) sinh (kx)


k
+ y(0) cosh (kx), we can add a homogeneous solution of
y

(0) sinh (kx)


k
+y(0) cosh (kx) to y
p
(x) to obtain
y
1
(x) =
1
k
_
x
0
dtR(t) sinh (k(x t))dt
Now for y

9y = e
3x
,
y
1
(x) =
1
3
_
x
0
dt(e
3t
) sinh (3(x t))dt =
1
6
_
x
0
dte
3t
_
e
3x3t
e
3x+3t
2
_
=
1
6
_
x
0
dt(e
3x
e
3x+6t
)
=
1
6
_
e
3x
x e
3x
1
6
e
6t

x
0
_
=
1
6
xe
3x
e
3x
e
6x
1
36
=
1
6
(xe
3x

e
3x
6
+
e
3x
6
)
y

1
=
1
6
(e
3x
+ 3xe
3x

3
2
e
3x

3
2
e
3x
)
y

1
=
1
6
_
3e
3x
+ 3e
3x
+ 9xe
3x

9
2
e
3x
+
9
2
e
3x
_
=
1
4
e
3x
+
3
4
e
3x
+
3
2
xe
3x
y

1
9y
1
=
1
2
e
3x
+
1
2
e
3x
Thus, we need to add homogeneous parts to our particular solution to make it work. So if
y
p
=
xe
3x
6

e
3x
y
+
e
3x
6
then it could be checked easily with some computation, that this satises the ODE.
Exercise 19. Start from y

+k
2
y = R(x)
Again, note that if L(y
p
) = y

p
+ k
2
y
p
= R(x), L(y
p
+ y
h
) = R(x) + 0 = R(x), so y
1
= y
p
+ y
h
is also a particular
solution.

x
0
dt sin k(xt)

_
x
0
dt sin k(x t)
d
2
y
dt
2
+k
2
_
x
0
dt sin k(x t)y =
_
x
0
R(t) sin k(x t)
_
x
0
dt sin k(x t)
d
2
y
dt
2
= y

(0) sin (kx) +k


_
x
0
y

(t) cos (k(x t))dt =


= y

(0) sin (kx) +k(y(x) y(0) cos (kx) k


_
x
0
y(t) sin (k(x t))dt) =
= ky(x) ky(0) cos (kx) y

(0) sin (kx) k


2
_
x
0
y(t) sin (k(x t))dt
=y(x) =
1
k
_
x
0
dtR(t) sin k(x t) +y(0) cos (kx) +
y

(0) sin (kx)


k
We can add y
h
with c
1
= y(0), c
2
=
y

(0)
k
y
1
=
1
k
_
x
0
dtR(t) sin k(x t)
Now for y

+ 9y = sin 3x, then k = 3,


y
1
=
1
3
_
x
0
sin 3t sin 3(x t)dt
_
x
0
sin 3t(sin 3xcos 3t cos 3xsin 3t)
_
x
0
sc =
_
x
0
s(6t)
2
dt =
1
12
(c(6x) 1)
_
x
0
s
2
=
_
x
0
1 cos (6t)
2
=
x
2

sin (6t)
12

x
0
=
x
2

sin (6x)
12
y
1
=
1
3
_
sin 3x
1
12
(cos (6x) 1) cos 3x
_
x
2

sin (6x)
12
__
=
sin 3x
18

xcos 3x
6
157
It could be shown with some computation that this particular solution satises the ODE without having to add or subtract
parts of a homogeneous solution.
Exercise 20. y

+y = sin x
y
h
= c
1
sin x +c
2
cos x =W(x) = s
2
c
2
= 1
t
1
=
_
cs
1
=
cos 2x
4
; t
2
=
_
ss
1
=
_
1 cos 2x
2
=
_
x
2

sin 2x
4
_
y
p
=
sin xcos 2x
4
+
_
sin 2x 2x
4
_
cos x =
sin xcos
2
x + sin
3
x 2xcos x
4
y = c
1
sin x +c
2
cos x +
sin xcos
2
x + sin
3
x 2xcos x
4
Exercise 21. y

+y = cos x
y
h
= c
1
sin x +c
2
cos x = c
1
S +c
2
C W(x) = 1
t
1
=
_
CC
1
=
_
1 + cos 2x
2
=
x +
sin 2x
2
2
; t
2
=
_
SC
1
=
cos 2x
4
y
p
=
xsin x
2
+
sin 2xsin x
4
+
cos xcos 2x
4
= y =
xsin x
2
+
sin 2xsin x
4
+
cos xcos 2x
4
+c
1
sin x +c
2
cos x
Exercise 22. y

+ 4y = 3xcos x
y
h
= c
1
sin 2x +c
2
cos 2x W(x) = sin
2
2x(2) +cos
2
2x(2) = 2
t
1
=
_
cos (2x)(3xcos x)
2
=
3
2
_
xcos xcos (2x) =
3
2
_
xc(1 2s
2
) =
3
2
_
xc 3
_
xcs
2
=
=
3
2
(xs +c) 3
_
x
_
c
3
3
_

=
3
2
(xs +c) (xs
3

_
s
3
) =
3
2
(xs +c) xs
3
+
_
s(1 c
2
) =
=
3
2
(xs +c) xs
3
+c +
1
3
c
3
=
3
2
xs +
c
2
xs
3
+
1
3
c
3
t
2
=
_
sin (2x)(3xcos x)
2
= 3
_
xsc
2
=
_
x(c
3
)

= xc
3

_
c
3
= xc
3

_
c(1 s
2
) =
= xc
3
s +
1
3
s
3
y
p
=
_
3
2
xs +
c
2
xs
3
+
1
3
c
3
_
(2sc) + (xc
3
s +
s
3
3
)(1 2s
2
) = (lots of algebra) =
= xc
2
+
2
3
s
= y = c
1
sin 2x +c
2
cos 2x +xsin x
2
3
cos x
Remember, persistence is key to work through the algebra, quickly.
Exercise 23. y

+ 4y = 3xsin x. From the work above, we could guess at the solution.


(xs)

= s +xc
(xs)

= 2c +xs
(xs)

+ 4(xs) = 2c xs + 4xs = 2c + 3xs (c)

+ 4c = 3c =
_
2
3
c
_

+ 4
_
2
3
c
_
= 2c
= y
h
= xs
2
3
c
y = xsin x
2
3
cos x +c
1
sin 2x +c
2
cos 2x
158
Exercise 24. y

3y

= 2e
2x
sin x Guessing and stitching together the solution seems easier to me.
(e
2x
c)

= 2e
2x
c +e
2x
s
(e
2x
c)

= 4e
2x
c +4e
2x
s e
2x
c = 3e
2x
c 4e
2x
s
(e
2x
c)

3(e
2x
c)

= 3e
2x
c 4e
2x
s 6e
2x
c + 3e
2x
s =
= 3e
2x
c e
2x
s
(e
2x
s)

= 2e
2x
s +e
2x
c
(e
2x
s)

= 4e
2x
s + 4e
2x
c +se
2x
=
= 3e
2x
s + 4e
2x
c
(e
2x
s)

3(e
2x
s)

= 3e
2x
s + 4e
2x
c 6e
2x
s 3e
2x
c =
= 3e
2x
s +e
2x
c
(3e
2x
s)

3(3e
2x
s)

+ (e
2x
c)

3(e
2x
c)

= 10e
2x
s
= y
p
=
e
2x
(3 sin x + cos x)
5
= y = c
1
sin

3x +c
2
cos

3x +
e
2x
(3 sin x + cos x)
5
Exercise 25. y

+y = e
2x
cos 3x.
(e
2x
c(3x))

= 4e
2x
c(3x) +12e
2x
s(3x) 9e
2x
c(3x) = 5e
2x
c(3x) 12e
2x
s(3x)
(e
2x
s(3x))

= 4e
2x
s(3x) + 12e
2x
c(3x) +9e
2x
s(3x) = 5e
2x
s(3x) + 12e
2x
c(3x)
L(e
2x
c(3x) 3e
2x
s(3x)) = 40e
2x
cos (3x)
y = c
1
sin x +c
2
cos x +
e
2x
cos (3x) 3e
2x
sin (3x)
40
8.19 Exercises - Examples of physical problems leading to linear second-order equations with constant coefcients.
In exercises 1-5, a partcile is assumed to be moving in simple harmonic motion, according to the equation y = C sin (kx +).
The velocity of the particle is dened to be the derivative y

. The frequency of the motion is the reciprocal of the period.


(Period = 2/k, frequency = k/2 )
Exercise 1. Find the amplitude C if the frequency is 1/ and if the initial values of y and y

(when x = 0) are 2 and 4,


respectively.
frequency =
k
2
=
1

=k = 2
y(x = 0) = C sin
y

(x = 0) = C cos
=
y(x = 0)
y

(x = 0)
=
1
k
tan =
1
2
=

4
and C = 2

2
Exercise 2. Find the velocity when y is zero, given that the amplitude is 7 and the frequency is 10.
y = C sin (kx +)
y

= Ck sin (kx +)
C = 7
k
2
= 10 =k = 20
y(x =

k
) = 0 =y

(x =

k
) = 140
Exercise 3.
y = Acos (mx +)
y = Acos (mx +) = Acos cos (mx) Asin sin (mx)
y = C sin kx + = C cos sin kx +C sin cos kx
= k = m (since x is arbitrary )
Asin = C cos
Acos = C sin
=tan = cot = tan
_

2

_
= tan (

2
)
= =

2
and [C[ = [A[
Exercise 4.
2
T
= 4
y = C cos (kx +) = C cos (kx) = 3 cos (4x)
159
Exercise 5. y = C cos (x +) y
0
= C cos (x
0
+)
y

= C sin (x +) = v
0
v
2
0
+y
2
0
= C
2
sin
2
(x +) +C
2
cos
2
(x
0
+) = C
2
= C =
_
v
2
0
+y
2
0
Exercise 6.
y = C cos (kx +)
y(0) = C cos () = 1
y

(0) = k
2
C cos () = 12
y

= kC sin (kx +)
y

(0) = kC sin () = 2
y

(0)
y(0)
= k
2
=
12
1
=k = 2

3
y

(0)
y(0)
=
kC sin ()
C cos ()
=
2
1
= 2 = k tan () = =

6
Exercise 7. k =
2
3
y = C sin (kx)
y = C sin
2x
3
; C > 0
Exercise 8. Lets rst solve the homogenous equation.
y

+y = 0
y
h
= C
1
sin x +C
2
cos x
W(x) = S
2
C
2
= 1
t
1
=
_
x
0
cos t(1)
1
= sin x
t
2
=
_
x
0
(sin t)(1)
1
= (cos x 1)
for 0 x 2 otherwise, for x > 2, t
1
= 0, t
2
= 0
y
1
= sin
2
x + cos
2
x + (1 cos x)
y

(x) = C
1
cos x + sin x
y(0) = 0 = c
2
y

(0) = c
1
= 1
y = sin x + (1 cos x)
= I(t) = sin t + (1 cos t) 0 t 2
Exercise 9.
(1) Consider large t. Then I(t) = F(t) +Asin (t +) Asin (t +)
I = AS(t +) = A(S(t)C() +C(t)S())
I

= AC(t +) = A(C(t)C() S(t)S())


I

=
2
AS(t +) =
2
A(S(t)C() +C(t)S())
I

+RI

+I = I

+I

+I =
= A((
2
C() +S() +C())S(t) + (
2
S() +C() +S())C(t)) = S(t)
=tan () =

1
2
With tan =

1
2
and the trig identities t
2
+1 = sec
2
,
1
C
2
= sec
2
, and S
2
+C
2
= 1, we can get
C =
1
2

1
2
+
4
S =

1
2
+
4
Note that the sign of S is xed by tan.
A(
2
C() +S() +C())S(t) = S(t) =A =
1
(1
2
)C() S()
=
D
(1
2
)
2
()
= A =
1

2
+ 1
160
We could immediately see that =
1

2
, f =
1
2

2
will maximize A.
(2) We could have, from the beginning, considered the problem with any R, in general.
A((
2
C() +RS() +C())S(t) + (
2
S() +RC() +S())C(t)) = S(t)
tan =
R
1
2
S() =
R
_
(R)
2
+ (1
2
)
2
C() =
1
2
_
(R)
2
+ (1
2
)
2
=A =
1

2
C() RS() +C()
=
1
_
(R)
2
+ (1
2
)
2
(R)
2
+ (1
2
)
2
=
4
+
2
(2 +R
2
) + 1
d
d
4
3
+ 2(2 +R
2
) = 2(2
2
+ (2 +R
2
)) = 0
=
= 0
2
2
= 2 R
2
to have resonances
R <
1

2
Exercise 10. A spaceship is returning to earth. Assume that the only external force acting on it is the action of gravity, and
that it falls along a straight line toward the center of the earth. The rocket fuel is consumed at a constant rate of k pounds per
second and the exhaust material has a constant speed of c feet per second relative to the rocket.
Let M(t) = M be the mass of the rocket +fuel combination at time t. With +y direction being towards earth, then the
equation of motion is F
g
= +M(t)g, where g = 9.8m/s
2
.
M(t)v(t) = Mv
R
is the momentum of the rocket.
M(t +h) = M(t) m = M m is the change in mass of the rocket due to spent fuel.
v
e
= velocity of the exhaust in the lab frame = c +v
R
(t)
p = m(c +v
R
) + (M m)v
R
(t +h) Mv
R
= M(v
R
(t +h) v
R
) +m(v
R
(t +h) v
R
) + mc
p
t
= M
_
v
R
(t +h) v
R
t
_
+m
_
v
R
(t +h) v
R
t
_
+
_
m
t
_
c = M(t)g
Mv

R
+
kc
g
= M(t)g
Now M(t) = M
0

kt
g
=v

R
= g
kc/g
M
= g
kc/g
M
0

kc
g
v
R
= gt
kc
g
g
k
ln (M
0

kt
g
) = gt +c ln (M
0

kt
g
)
y
R
=
gt
2
2
+c
g
k
__
k
g
t M
0
_
ln
_
M
0

kt
g
_

kt
g
_
+
M
0
cg
k
ln M
0
Exercise 11.
Mv

R
=
kc
g
v

R
=
kc
g
_
1
M
0

kt
g
_
v
R
= c ln (M
0

kt
g
)
=y
R
= c
g
k
__
kt
g
M
0
_
ln
_
M
0

kt
g
_

kt
g
_
+
M
0
cg ln M
0
k
M
0
g = w = y
R
= c
g
k
_
kt w
g
ln
_
w kt
g
_

kt
g
_
+
wc
k
ln
w
g
We couldve also solved this problem with an initial velocity of v
0
and gravity. Then
v
R
(t) = gt +c ln (1
kt
M
0
g
) +v
0
y(t) = v
0
t +
1
2
gt
2
+c
__
t
M
0
g
k
_
ln
_
1
kt
M
0
g
_
t
_
161
Exercise 12.
Mv
R
= (M m)(v
R
(t +h)) + 0
M(v
R
(t +h) v
R
(t)) = (m)v
R
(t +h)
Mv

R
=
k
g
v
R
=
v

R
v
R
=
k
g(M
0

kt
g
)
=
k
M
0
g(1
kt
M0g
)
ln v
R
= (k/w) ln (1
kt
w
)
_
w
k
_
= ln (1
kt
w
)
v
R
=
v
0
1
kt
w
=x(t) = v
0
_
w
k
_
ln (1
kt
w
) =
v
0
w
k
ln (1
kt
w
)
8.22 Exercises - Remarks concerning nonlinear differential equations, Integral curves and direction elds.
Exercise 1. 2x + 3y = C =y

=
2
3
Exercise 2. y = Ce
2x
=y

= 2y
Exercise 3. x
2
y
2
= c =yy

= x =y

=
x
y
; y ,= 0
Exercise 4. xy = c =y

=
y
x
; x ,= 0
Exercise 5. y
2
= cx =
y
2
x
= c =y

=
y
2x
x ,= 0
Exercise 6. x
2
+y
2
+ 2Cy = 1
x
2
y
+y
1
y
= 2C
2xy y

x
2
y
2
+y

+
1
y
2
y

= 0
y

=
2xy
1 +y
2
x
2
Exercise 7. y = C(x 1)e
x
y
(x 1)e
x
= C
y

(x 1)e
x
(e
x
+ (x 1)e
x
)y
(x 1)
2
e
2x
= 0
y

=
xy
x 1
Exercise 8. y
4
(x + 2) = C(x 2)
y
4
(x + 2)
x 2
= C
(4y
3
y

(x + 2) +y
4
)(x 2) y
4
(x + 2)
(x 2)
2
= 0 =4y
3
y

(x + 2) +y
4
=
y
4
(x + 2)
x 2
y

=
y
(x 2)(x + 2)
Exercise 9. y = c cos x =y

= tan xy
Exercise 10. arctan y + arcsin x = C
1
1 +y
2
y

+
1

1 x
2
= 0 = y

=
(1 +y
2
)

1 x
2
Exercise 11. All circles through the points (1, 0) and (1, 0).
Start with the circle equation: (x A)
2
+ (y B)
2
= R
2
162
(1, 0): (1 A)
2
+B
2
= R
2
=(1 2A+A
2
+B
2
= R
2
)
(1, 0): (1 A)
2
+B
2
= 1 + 2A+A
2
+B
2
= R
2
=4A = 0, A = 0 1 +B
2
= R
2
x
2
+ (y
_
R
2
1)
2
= R
2
x
2
+y
2
2By + (R
2
1) = R
2
=x
2
+y
2
2By = 1
B depends upon R, the radius of the circles, so we could use B as the parameter for the family of circles.
x
2
+y
2
1 = 2By
x
2
y
+y
1
y
= 2B =
2xy y

x
2
y
2
+y

+
1
y
2
y

= 0
y

=
2xy
y
2
x
2
+ 1
Exercise 12.
(x +A)
2
+ (y +B)
2
= r
2
(1 +A)
2
+ (1 +B)
2
= 1 + 2A+A
2
+ 1 + 2B +B
2
= r
2

_
(1 +A)
2
+ (1 +B)
2
_
=
_
1 2A+A
2
+ 1 2B +B
2
= r
2
_
=4A+ 4B = 0 =A = B
(x +B)
2
+ (y +B)
2
= r
2
2(x B) + 2(y +B)y

= 0
(y +B)y

= B x
=y

=
B x
y +B
(1 B)
2
+ (1 +B)
2
= r
2
=

2
_
(1 +B
2
) = r or
_
r
2
2
1 = B
= (so B could be treated as a parameter for the family of curves)
8.24 Exercises - First-order separate equations.
Exercise 1. y

= x
3
/y
2
1
3
y
3
=
1
4
x
4
+C =y
3
=
3
4
x
4
+C
Exercise 2. tan xcos y = y

tan y
ln [ cos x[ =
1
cos y
Exercise 3. (x + 1)y

+y
2
= 0
1
y
= ln (x + 1) +c
Exercise 4. y

= (y 1)(y 2)
_
1
y 2
+
1
y 1
_
y

= 1 =ln (y 2) ln (y 1) = x
y 2
y 1
= e
x
Exercise 5. y

1 x
2
y

= x
1
2
y
2
=

1 x
2
=y
2
= 2

1 x
2
Exercise 6. (x 1)y

= xy
ln y =
_
1 +
1
x 1
= x + ln [x 1[
y = e
x
(x 1) +C
163
Exercise 7. (1 x
2
)
1/2
y

+ 1 +y
2
= 0
arctan y = arccos x +C
Exercise 8. xy(1 +x
2
)y

(1 +y
2
) = 0
1
2
ln (1 +y
2
) =
_ _
1
x

x
1 +x
2
_
+C = ln x
1
2 ln [1 +x
2
[
y
2
= k
_
x

1 +x
2
_
2
Exercise 9. (x
2
4)y

= y
ln y =
1
2
arctanh
_
x
2
_
since
_
1
x
2
4
dx =
1
4
_
dx
_
x
2
_
2
1
=
=
1
2
_
du
u
2
1
( where u =
x
2
)
(tanh (u))

=
cosh
2
u sinh
2
u
cosh
2
u
= 1 tanh
2
u
= y = k exp (
1
2
arctanh
_
x
2
_
)
Exercise 10. xyy

= 1 +x
2
+y
2
+x
2
y
2
1
2
ln (1 +y
2
) = ln x +
1
2
x
2
+C = y
2
= kx
2
e
x
2
1
Exercise 11. yy

= e
x+2y
sin x
ye
2y
2

e
2y
4
=
e
x
sin x e
x
cos x
2
+C
(2y + 1)e
2y
= 2e
x
(sin x cos x) +C
Exercise 12. xdx +ydy = xy(xdy ydx)
y(1 x
2
)dy = x(y
2
1)dx
ydy
1 +y
2
=
xdx
x
2
1
=
1
2
ln [1 +y
2
[ =
1
2
ln [x
2
1[ +C
1 +y
2
= (x
2
1)K = y
2
= K(x
2
1) 1
Exercise 13. f(x) = 2 +
_
x
1
f(t)dt
f

(x) = f(x) =f(x) = Ce


x
=Ce
x
= 2 +Ce
x
Ce
1
C =
2
e
f(x) =
2
e
e
x
Exercise 14. f(x)f

(x) = 5x f(0) = 1
f(x)
2
= 5x
2
+C =f(x) =
_
5x
2
+C
f(x) =
_
5x
2
+ 1
Exercise 15. f

(x) + 2xe
f(x)
= 0 f(0) = 0
164
e
y
y

= 2x =e
y
= x
2
+C
y = ln (x
2
+C)
1
= y = ln (x
2
+ 1)
Exercise 16. f
2
(x) + (f

(x))
2
= 1
f = 1
y
2
= 1 y
2
y

=
_
1 y
2
arcsin (y) = x +c = f(x) = sin (x +c)
Exercise 17.
_
x
a
f(t)dt = K(x a)
f > 0 x !
= f(x) = k > 0
Exercise 18.
_
x
a
f(t)dt = k(f(x) f(a))
d
dx
f(x) = kf

(x)
=f(x) = Ce
1
k
x
; C > 0
Exercise 19.
_
x
a
(f(t))dt = k(f(x) +f(a)) =f(x) = Ce
x
k
kCe
x
k
kCe
a
k
= kCe
x
k
+kCe
a/k
=2kCe
a/k
= 0 =C =
f = 0
Exercise 20.
_
x
a
f(t)dt = kf(x)f(a); f(x) = kf

(x)f(a)
1
kf(a)
=
f

(x)
f(x)
=ln f(x) =
_
1
kf(a)
_
x +C; =f(x) = C exp
_
x
kf(a)
_
_
x
a
f(t)dt =
_
kf(a)Ce
t
kf(a)
_

x
a
= kf(a)Ce
x
kf(a)
kf(a)Ce
a
kf(a)
= kCe
x
kf(a)
Ce
a
kf(a)
f(a)
_
e
x
kf(a)
e
a
kf(a)
_
= Ce
x
kf(a)
e
a
kf(a)
x=a
0 = Ce
2a
kf(a)
=C = 0
= f = 0
8.26 Exercises - Homogeneous rst-order equations.
Exercise 1. f(tx, ty) = f(x, y) homogeneity (or homogeneity of zeroth order).
y

= f(x, y) =
_
x
v
_

=
v xv

v
2
= f(x,
x
v
) = f(1,
1
v
)
v v
2
f(1,
1
v
) = xv

= ln x =
_
dv
v v
2
f(1,
1
v
)
Exercise 2. y

=
x
y
=
1
2
y
2
=
1
2
x
2
+C = y
2
= x
2
+C
Exercise 3. y

= 1 +
y
x
y
x
= v =y

= v +xv

= 1 +v =v = ln x
y = x(ln x +C)
Exercise 4. y

=
x
2
+2y
2
xy
165
y

=
x
2
+ 2y
2
xy
=
x
y
+
2y
x
v =
y
x
=v +xv

=
1
v
+ 2v =
v

1
v
+v
=
1
x
1
2
ln [1 +v
2
[ = ln x +C = y
2
= (Cx
2
1)x
2
Exercise 5. (2y
2
x
2
)y

+ 3xy = 0
if 2y
2
,= x
2
, y

=
3xy
x
2
2y
2
y = vx
y

= v

x +v
=y

= v

x +v =
3vx
2
x
2
2v
2
x
2
=
3v
1 2v
2
=
1 2v
2
2v(1 +v
2
)
v

=
1
x
1
2
_
1
v
+
3v
1 +v
2
_
v

=
1
x
=
1
2
ln v +
3
2
ln (1 +v
2
) = ln x +C =
v
(1 +v
2
)
3
= Cx
2
=
y/x
_
x
2
+y
2
x
2
_
3
yx
3
= C(x
2
+y
2
)
3
However,
y

=
3xy
x
2
2y
2
v =
y
x
y

= v

x +v
=v

x +v =
3x
2
v
x
2
2v
2
x
2
=
3v
1 2v
2
v

x =
3v
1 2v
2

(v 2v
3
)
1 2v
2
=
2(v +v
3
)
1 2v
2
=
_
1
v
+
3v
1 +v
2
_
v

=
2
x
=ln v +
3
2
ln [1 +v
2
[ = 2 ln x +C
v
(1 +v
2
)
3/2
= Cx
2
=
y
2
/x
4
(x
2
+y
2
)
3
= Cx
4
= y
2
= C(x
2
+y
2
)
3
Exercise 6. xy

= y
_
x
2
+y
2
=y

=
y
x
=
_
1 +
_
y
x
_
2
v =
y
x
vx = y
v

x +v = v
_
1 +v
2
=
v

x =
_
1 +v
2
v

1 +v
2
=
1
x
=ln (v +
_
1 +v
2
) = ln x +C since
(ln (v +
_
1 +v
2
))

=
1
v +

1 +v
2
_
1 +
v

1 +v
2
_
=
1

1 +v
2
v +
_
1 +v
2
= Cx =1 +v
2
= C
2
x
2
2vCx +v
2
=v =
Cx
2

1
2Cx
= y =
Cx
2
2

1
2C
Exercise 7. x
2
y

+xy + 2y
2
= 0
166
x
2
y

= 2y
2
xy =y

=
2y
2
x
2

y
x
v =
y
x
=
v

x +v = 2v
2
v
v

x = 2(v
2
+v)
v

v(v + 1)
=
2
x
=
_
v

_
1
v

1
v + 1
_
= 2 ln x +C =
v
v + 1
=
C
x
2
y =
Cx
C x
2
Exercise 8. y
2
+ (x
2
xy +y
2
)y

= 0
_
y
x
_
2
+
_
1
y
x
+
_
y
x
_
2
_
y

= 0
Let v =
y
x
=
v
2
1 v +v
2
= v

x +v
v

x =
v
2
1 v +v
2
v =
v(1 +v
2
)
1 v +v
2
=
v
2
v + 1
v(1 +v
2
)
v

=
1
x
=
_
1
v
+
1
v
2
+ 1
_
v

=
1
x
=ln v arctan v = ln x +C =ln (vx) = arctan x +C ln y = arctan
y
x
+C
Exercise 9. y

=
y(x
2
+xy+y
2
)
x(x
2
+3xy+y
2
)
y

=
y(x
2
+xy +y
2
)
x(x
2
+ 3xy +y
2
)
=
_
y
x
_
_
1 +
y
x
+
y
2
x
2
1 +
3y
x
+
y
2
x
2
_
v=
y
x
v

x +v = v
_
1 +v +v
2
1 + 3v +v
2
_
= v +
2v
2
v
2
+ 3v + 1
v

(1 +
3
v
+
1
v
2
) =
2
x
=v + 3 ln v +
1
v
= 2 ln x +C
y
x
+ 3 ln y
x
y
= ln x +C
Exercise 10. y

=
y
x
+ sin
y
x
y
x
= x
y

= v +v

x
=
v +v

x = v + sin v
v

sin v
=
1
x
ln csc v + cot v = ln x +C = csc v + cot v =
K
x
Exercise 11. x(y + 4x)y

+y(x + 4y) = 0
y

=
y(x + 4y)
x(y + 4x)
=

y
x
(1 +
4y
x
)
y
x
+ 4
v=
y
x
v +xv

=
v(1 + 4v)
v + 4
xv

=
5v(1 +v)
v + 4
=
5
x
=
v + 4
v(1 +v)
v

=
_
4
v
+
3
1 +v
_
v

4 ln v 3 ln (1 +v) = 5 ln x +C = (yx)
4
= (x +y)
3
C
8.28 Miscellaneous review exercises - Some geometrical and physical problems leading to rst-order equations.
Exercise 1.
2x + 3y = C y

=
2
3
g

=
3
2
=g
3
2
x = C
Exercise 2.
xy = C
d/dx
y +xy

= 0 =y

= y/xx ,= 0 =g

= x/g =
1
2
g
2
=
1
2
x
2
+C
Exercise 3. x
2
+y
2
+ 2Cy = 1
167
x +yy

+Cy

= 0 =y

(y +C) = x
y

=
x
y +C
=
x
y +
1x
2
y
2
2y
=
2xy
y
2
x
2
+ 1
orthogonalcurves
y

=
y
2
x
2
+ 1
2xy
=
_
1
2x
_
y +
1
2
_
1
x
x
_
y
1
Recognize that this is a Ricatti equation and we know how to solve them.
y

+
1
2x
y = y
1
_
x
2
+
1
2x
_
n = 1
k = 1 n = 1 (1) = 2
v = y
k
= y
2
v

+ 2
_
1
2x
_
v =
2
2
_
1
x
x
_
A(x) =
_
x
a
P(t)dt =
_
x
a
1
t
= ln
a
x
_
x
a
Qe
A
=
_
x
a
_
1
t
t
_
a
t
=
a
x
+ 1 a(x a)
y
2
= v = 1 +
x
a
x(x a) +
bx
a
Exercise 4. y
2
= Cx.
y
2
x
= C
d/dx

2yy

x y
2
x
2
= 0
y

=
y
2x
=y

=
1
_
y
2x
_ =
2x
y
= y
2
+ 2x
2
= C
Exercise 5. x
2
y = C.
2xy +x
2
y

= 0 y

=
2y
x
=y

=
x
2y
1
2
y
2
=
x
2
4
+C
2y
2
x
2
= C
Exercise 6. y = Ce
2x
e
2x
y = C =2e
2x
y +e
2x
y

= 0
y

= 2y
invert
y

=
1
2y
= y
2
= x +C
Exercise 7. x
2
y
2
= C
2x 2yy

= 0
y

=
x
y
=y

=
y
x
=ln y = ln x +C = y =
C
x
Exercise 8. y sec x = C
y

sec x +y tan xsec x = 0


y

= y tan x
(invert)
y

=
1
y tan x
1
2
y
2
= ln [ sin x[ +C
168
Exercise 9. All circles through the points (1, 0) and (1, 0) From Sec. 8.22, Ex.10, we had obtained y

=
2xy
y
2
x
2
+1
=y

=
x
2
y
2
1
2xy
=
x
2
1
2yx

y
2x
=y

+
1
2x
y =
x
2
1
2x
y
1
Recognize this is a Ricatti equation.
For y

+Py = Qy
n
, in this case, n = 1, and so k = 1 n = 1 (1) = 2.
Then v = y
k
and v

+kPv = kQ. In this case,


v

+ 2
_
1
2x
_
v = 2
_
x
2
1
2x
_
= v

+
1
x
v =
x
2
1
x
= x 1/x.
A(x) =
_
x
a
P(t)dt =
_
x
a
1
t
= ln
x
a
_
(t
1
t
) exp
_
ln
t
a
_
dt =
_ _
t
2
a

1
a
_
dt =
_
1
3
t
3
a

t
a
_

x
a
e
ln
x
a
=
a
x
=y
2
= v =
1
3
x
3
x
1
3
a
3
+a
x
+
ba
x
Exercise 10. All circles through the points (1, 1) and (1, 1).
(x +A)
2
+ (y +B)
2
= r
2
(1 +A)
2
+ (1 +B)
2
= 1 + 2A+A
2
+ 1 + 2B +B
2
= r
2

_
(1 +A)
2
+ (1 +B)
2
_
=
_
1 2A+A
2
+ 1 2B +B
2
= r
2
_
=4A+ 4B = 0 =A = B
(x +B)
2
+ (y +B)
2
= r
2
2(x B) + 2(y +B)y

= 0
(y +B)y

= B x
=y

=
B x
y +B
(1 B)
2
+ (1 +B)
2
= r
2
=

2
_
(1 +B
2
) = r or
_
r
2
2
1 = B
= (so B could be treated as a parameter for the family of curves)
y

=
1
_
Bx
y+B
_ =
y +B
x B
y

y +B
=
1
x B
= y = C(x B) B
Exercise 11. With (0, Y ) = Q the point that moves up wards along the positive y-axis and
P = (x, y) being the point P that pursues Q,
y

=
Y y
Xx
=
Y y
0x
is the slope of the tangent line on a point on the trajectory of P.
The condition given, that the distance of P from the y-axis is k the distance of Q from the origin, is
kY = x.
y

=
_
1
k
_
x y
x
= f(x, y)
f(x, y) is homogeneous of zero order =y = vx (try this substitution)
y

= v

x +v =
1
k
v =
v

1
k
2v
=
1
x
=
1
2
ln
_
1
k
2v
_
= ln x +C =y =
x
2k

1
2C
2
x
(1,0)
y =
x
2k
=
1
2kx
k =
1
2
y = x
1
x
169
Exercise 12.
y =
x
2k

1
2kx
Exercise 13. y = f(x).
n
_
x
0
f(t)dt = xf(x)
_
x
0
f(t)dt; (n + 1)
_
x
0
f(t)dt = xy
=(n
1
)y = y +xy

=
ny = xy

n
x
=
y

y
nln x = ln y
y = Cx
n
of y = Cx
1/n
Exercise 14.
n
_
x
0
f
2
(t)dt =
_
x
0
((y(x))
2
(f(t))
2
)dt
(n + 1)
_
x
0
f
2
(t)dt = xy
2
(x) = xy
2
; (n + 1)f
2
(x) = y
2
+ 2xyy

=
ny
2x
=ln y =
n
2
ln x +C
y = Cx
n/2
of y = Cx
1/2n
Exercise 15.

_
x
0
f
2
(t)dt = x
2
f(x) f
2
(x) = 2xf +x
2
f

=f

=
f
2
2xf
x
2
The left hand side of the last expression shown is homogeneous. Thus do the y = vx substitution.
v

x +v =
v
2
x
2
2x
2
v
x
2
= v
2
2v
v

x
v
2
3v
= 1 =
v

(v
2

3v

)
=
1
x
=
1
3
_
1
v
3

1
v
_
v

ln (v
3

) ln v = 3 ln x +C
ln
_
v 3/
v
_
= 3 ln x +C
vx
3

xCvx
4
y
3

x = Cyx
3
= y =
3x/
1 +
x
3
2
Exercise 16. A =
_
a
0
f; B =
_
1
a
f
AB =
_
a
0
f +
_
a
1
f = 2f(a) + 3a +b
d/dx
2f(a) = 2f

(a) + 3 =1 =
f

(a)
f(a)
3
2
So then
a +C = ln (y
3
2
); f(a) = Ce
a
+
3
2
f(1) = 0 = Ce
1
+
3
2
; =C =
3
2e
f(x) =
3
2
e
x1
+
3
2
170
To nd b,
2
_
3
2
e
a1
+
3
2
a
_
+
3
2
e
1
+
3
2

3
2
= 2f(a) + 3a +b =
= 2
_
3
2
e
a1
+
3
2
_
+ 3a +b
= b =
3
2
e
1
3
Exercise 17.
A(x) =
_
x
0
_
f(t)
__
y(x) 1
x
_
t + 1
__
dt = x
3
=
_
x
0
f +
_
y(x) 1
x
_
1
2
x
2
x = x
3
=
=
_
x
0
f +
_
y(x) 1
2
_
x x = x
3
d/dx
f(x) +
1
2
(y

x +y)
1
2
= 3x
2
y

=
2(3x
2
+
1
2
y/2)
x
= 6x x
1
+
y
x
As a leap of faith, try y = vx substitution to solve y

= 6x x
1
+
y
x
.
v

x +v = 6x x
1
+v v

= 6 x
2
; v = 6x +x
1
+C
y = 6x
2
+ 5x + 1 x
Exercise 18. Assuming no friction at the orice and energy conservation.
mgh =
1
2
mv
2
f
(imagine how the top layer of water is now at the bottom of the tank (nal potential energy congurations))
V
f
=

2gh (how fast water is rushing out)


A
0
= cross-sectional area of the orice.
dV
dt
= A
dh
dt
= c
_
2ghA
0
2h
1/2

h
f
hi
= c
_
2g
A
0
A
t
=T =

2A
c

gA
0
_
_
h
f

_
h
i
_
= 59.6sec
Note that we included the discharge coefcient C = 0.6.
Exercise 19.
dV
dt
= c

2ghA
0
+
0
= A
dh
dt
= h
1/2
+
0
. = c

2gA
0
.
171
Adh

0
h
1/2
= dt = (A/
0
)
dh
1

0
h
1/2
_
ln
_
1 ah
1/2
__

=
1
1 ah
1/2
_

a
2
1
h
1/2
_
(where a =

0
)
(h
1/2
)

=
1
2h
1/2
_
1 ah
1/2
1 ah
1/2
_
=
_
1
2

a
2
h
1/2
_
h
1/2
_
1 ah
1/2
_
=
_
(A/
0
)
dh
1

0
h
1/2
= (A/
0
)
_
2
2
0

2
ln
_
1

0
h
1/2
_
+
2
0
h
1/2

h
f
hi
=
= T
=exp
_

0
A
t
2
0

_
h
1/2
f
h
1/2
i
_
_

2
2
2
0
=
1

0
h
1/2
f
1

0
h
1/2
i
t
h
f
=

2
0

2
=
(100in
3
/s)
2
c
2
(2)(32ft/s
2
)(5/3in
2
)
2
(12in/1ft)
= (25/24)
2
Exercise 20.
V
0
=
1
3
R
2
0
H
0
V (h) = V
0

1
3
h
_
h
R
0
H
0
_
2
= V
0

1
3

R
2
0
H
2
0
h
3
= V
0
h
3
mg(H
0
h) =
1
2
mv
2
f
;
_
2g(H
0
h) = v
f
(energy conservation)
cA
0
v
f
= cA
0

2gH
0
_
1
h
H
0
_
=
_
1
h
H
0
dV
dt
=
_
1 h/H = 3h
2
dh
dt
=
dh
dt
=

3h
2
_
1
h
H
_
h
2
/H
2
0
_
1
h
H0
=
/H
2
0
3
T =
= H
0
_
u
2
du

1 u
= H
0
_
(1 y)
2
(dy)

y
= H
0
_
1 2y +y
2

y
=
= H
0
_
2y
1/2

2
3
y
3/2
+
2
5
y
5/2
_

h
f
hi
=
= H
0
_
2
_
1
h
H
0
_
1/2

4
3
_
1
h
H
0
_
3/2
+
2
5
_
1
h
H
0
_
5/2
_

h
f
hi
=
=
cA
0

2gH
0
/H
2
0
3
_
1
3

R
2
0
H
2
0
_ T
For h
i
= 0, h
f
= H,
H
0
(2(1) 4/3(1) + 2/5) = H
0
(16/15) = cA
0
_
2gH
0
T/(R
2
0
); T =
16
15

H
0
R
2
0
cA
0

2g
=
2
9
R
2
0

H
0
A
0
Exercise 21. m
2
x m+ (1 x) = 0 =(m
2
1)x + 1 m = 0, =m = 1
Exercise 24. Given f s.t. 2f

(x) = f
_
1
x
_
if x > 0, f(1) = 2 and x
2
y

+axy

+by = 0
172
(1)
2f

(x) =
d
dx
f
_
1
x
_
= (f

)
_
1
x
_
=x
2
y

=
1
2
f

=
1
4
f
_
1
x
_
a = 0; b =
1
4
(2)
f(x) = Cx
n
f

= nCx
n1
f

= n(n 1)Cx
n2
=n(n 1)Cx
n
=
1
4
Cx
n
n
2
n +
1
4
= n =
1
2
Exercise 28. Choose the units for time to be in days rst - we can convert into years later.
If no one died from accidental death, then the population will grow by e. That means, with x = x(t) being the population at
time t
dx
dt
= x =x = Ce
t
which makes sense because if C is the original population number, then after 1 year, x = Ce.
With t in days, we have a decrease of
1
100
x in population each day due to death. Add up the changes from the decrease
due to deaths and the increase due to growth for the DE:
dx
dt
=
1
365
x
1
100
x =
100 365
36500
x =
265
36500
=x = Ce
265
36500
t
Change t units to years by multiplying the time constant
265
36500
by 365 days.
x = 365 exp (2.65t)
To get the total fatalities, simply integrate the deaths during each year.
_
t
0
y =
365
100
365
2.65
(exp (2.65t) + 1)
Exercise 29. For constant gravity, K = U
=(0 mgh) =
1
2
mv
2
f
v
f
=

2gh = (6.37 10
8
cm)
_
1 m
2.54 cm
_
_
1 ft
12 in
_
= 6.93
mi
sec
= 24940
mi
hr
The constant energy formula could also be obtained by considering
F =
GMem
r
2
= m
d
r
dt
2
=
r
U
Exercise 30. Let y = f(x) be the solution to y

=
2y
2
+x
3y
2
+5
f(0) = 0
(1) y

(0) = 0 as easily seen. Now y

=
(4yy

+1)(3y
2
+5)(6yy

)(2y
2
+x)
(3y
2
+5)
2
, so then
y

(0) =
1
5
> 0. It is a minimum.
(2) f

(x) 2/3 x 10/3. a = 2/3 since f will be above this tangent line.
Suppose, in the worst case, f

(x) = 0 for 0 x 2/3. Then f(x) = 0 for 0 x 2/3. Then the tangent line
must be at y = 0 at x = 10/3 to remain below the graph of f(x).
=
2
3
x 20/9 < f(x)
173
(3) Since f

(x)
2
3
for each x
10
3
, then f for x (otherwise, f would have to decrease somewhere, which
would contradict the given fact about f). Rewrite the DE for y

to be
y

=
2y
2
+x
3y
2
+ 5
=(3y
2
+ 5)y

= 2y
2
+x =(3 +
5
y
2
)y

= 2 +
x
y
2
Consider
y

=
2y
2
+x
3y
2
+ 5
=
2 +
x
y
2
3 +
5
y
2
specically,
x
y
2
. Now y must, at the very least, have some linear increase because we had already shown that y


2
3
.
So y
2
would go to innity faster than linear x. Thus lim
x
y

=
2
3
. So then (3 +
5
y
2
)y

x
(3 + 0)
2
3
= 2 =
2 +
x
y
2
.
0 =
x
y
2
(4) y

=
2+
x
y
2
3+
5
y
2
x

2
3
. =y =
2
3
x or
y
x
=
2
3
.
Exercise 31. Given a function f which satises the differential equation xf

(x) + 3x(f

(x))
2
= 1 e
x
(1) c ,= 0 for an extrenum.
cf

(c) + 3c(f

(c))
2
= cf

(c) = 1 e
c
=f

(c) =
1e
c
c
> 0
(2) Cleverly, consider the limit.
xf

(x) + 3x(f

(x))
2
= 1 e
x
=f

(x) + 3(f

(x))
2
=
_
1 e
x
x
_
x0
f

(0) + 0 = 1
So a critical point at x = 0 would be a minimum.
(3) Well have to cheat a little and use the idea of power series early on here.
f

+ 3(f

)
2
=
1e
x
x
suggests that we consider the Taylor series of e
x
.
1 e
x
x
=

j=1
(x)
j
j!
x
=

j=0
(1)
j
x
j
(j + 1)!
This further suggests that f itself has a power series representation because its rst and second order derivatives are
simply a combination of innitely many terms containing powers of x.
Then suppose f =

j=0
a
j
x
j
.
f

+ 3(f

)
2
=
1 e
x
x
=
f

j=0
(j + 1)a
j+1
x
j
f

j=0
(j + 2)(j + 1)a
j+2
x
j
=

j=0
(j + 2)(j + 1)a
j+2
x
j
+ 3

j=0

k=0
(j + 1)(k + 1)a
j+1
a
k+1
x
j+k
=

j=0
(1)
j
x
j
(j + 1)!
If f(0) = 0 a
0
= 0
If f

(0) = 0 a
1
= 0
Then f = a
2
x
2
+

j=3
a
j
x
j
. Consider the x
0
terms in the DE. (f

)
2
doesnt contribute, because f

s leading order
term is x
1
. So then
2(1)a
2
+ 0 = 1 = a
2
=
1
2
i.e. f =
1
2
x
2
+

j=3
a
j
x
j
A =
1
2
in order for f(x) Ax
2
174
9.6 Exercises - Historical introduction, Denitions and eld properties, The complex numbers as an extension of
the real numbers, The imaginary unit i, Geometric interpretation. Modulus and argument. Exercise 6. Let f be a
polynomial with real coefcients.
(1) Since z
1
z
2
(z
n+1
1
) = z
n
1
z
1
= z
n
1
z
1
= z
n+1
1
f(z) =

a
j
z
j
=

a
j
z
j
= f(z)
(2) If f(z) = 0, then f(z) = f(z) = 0 as well.
Exercise 7. The three ordering axioms are
Ax. 7 If x, y R
+
, x +y, xy R
+
Ax. 8 x ,= 0, x R
+
or x R
+
but not both
Ax. 9 0 / R
+
x < y means y x positive.
Suppose i positive: i(i) = 1 but 1 is not positive.
Suppose i is positive. i(i) = 1 but 1 is not positive.
i is neither positive nor negative so Ax. 8 is not satised.
Exercise 8. Ax. 8 , Ax. 9 are satised.
(a +ib)(c +id) = (ac bd, ad +bc) so Ax. 7 might not be satised.
Exercise 9. Ax. 7, Ax. 8 , Ax. 9 are trivially satised (all are positive).
Exercise 10. x > y x > y is well dened. Ax. 8 is satised.
For Ax. 7, (
3
2
, 1), (1,
1
2
) contradicts Ax. 7 since we required the product to be positive as well if the factors are positive.
We found this particular counterexample by considering factors (a, b), (c, d), so the product of the two is (ac bd, ad + bc)
and so we need ac bd ad bc = a(c d) b(c +d) < 0
Exercise 11. See sketch.
Exercise 12.
w =
az +b
cz +d
w =
(az +b)(cz +d)
(cz +d)(cz +d)
=
ac[z[
2
+adz +bcz +bd
c
2
[z[
2
+cd(z +z) +d
2
w +w =
ac[z[
2
+adz +bcz +bd ac[z[
2
adz bcz bd
[cz +d[
2
=
(ad bc)(z z)
[cz +d[
2
If ad bc > 0
w w = 2Imw =
ad bc
[cz +d[
2
2Imz;
ad bc
[az +d[
2
> 0
So Imw has the same sign as Imz
175
9.10 Exercises - Complex exponentials, Complex-valued functions, Examples of differentiation and integration for-
mulas.
Exercise 7.
(1)
if m ,= n ,
_
2
0
e
ix(nm)
dx =
e
ix(nm)
i(n m)

2
0
=
1 1
i(n m)
= 0
if m = n ,
_
2
0
e
ix(0)
dx = 2
(2)
_
2
0
e
inx
e
imx
dx =
_
2
0
(cos nx +i sin nx)(cos mx i sin mx) =
=
_
2
0
cos nxcos mx + sin nxsin mx +i(sin nxcos mx sin mxcos nx)
_
2
0
e
inx
e
imx
dx =
_
2
0
(cos nx i sin nx)(cos mx i sin mx) =
=
_
2
0
cos nxcos mx sin nxsin mx +i(sin nxcos mx sin mxcos nx)
Summing the two equations above
0 =
_
2
0
2 cos nxcos mx + 2
_
2
0
i sin mxcos nx
=
_
2
0
cos nxcos mx = 0,
_
2
0
sin mxcos nx = 0
_
2
0
e
inx
e
imx
dx =
_
2
0
(cos nx +i sin nx)(cos mx i sin mx) =
=
_
2
0
cos nxcos mx + sin nxsin mx +i(sin nxcos mx sin mxcos nx)
_
2
0
e
inx
e
imx
dx =
_
2
0
(cos nx +i sin nx)(cos mx +i sin mx) =
=
_
2
0
cos nxcos mx sin nxsin mx +i(sin nxcos mx + sin mxcos nx)
Subtract the two equations above
_
2
0
sin nxsin mx i
_
2
0
sin mxcos nx = 0
=
_
2
0
sin nxsin mx = 0
176
_
2
0
e
inx
e
inx
= 2 =
_
2
0
(cos nx +i sin nx)(cos nx i sin nx) =
=
_
2
0
cos
2
nx + sin
2
nx
_
2
0
e
inx
e
inx
= 0 =
_
2
0
cos
2
nx sin
2
nx +i(2 cos nxsin nx)
=
_
2
0
cos
2
nx sin
2
nx = 0
_
2
0
cos nxsin nx = 0
Summing the two results above, we obtain
2 =
_
2
0
2 cos
2
nx
=
_
2
0
cos
2
nx =
Then also,
_
2
0
sin
2
nx =
Exercise 8.
z = re
i
= re
i(+2m)
, m Z
z
1/n
= r
1/n
e
i(/n+2m/n)
m = 0, 1, . . . n 1
=z
1/n
= Re
i

m
= z
1

m
The roots are spaced equally by an angle 2/n
i = e
i/2+i2n
=i
1/3
= e
i/6
, e
i5/6
, e
i3/2
i
1/4
= e
i/8
, e
5i/8
, e
9i/8
, e
13i/8
i = e
i/2+i2n
=(i)
1/4
= e
i/8
, e
3i/8
, e
7i/8
, e
11i/8
Exercise 9.
e
iu
e
iv
= e
i(u+v)
= cos u +v +i sin u +v =
= (cos u +i sin u)(cos v +i sin v) = cos ucos v sin usin v +i(cos v sin u + cos usin v)
=sin u +v = cos v sin u + cos usin v
=cos u +v = cos ucos v sin usin v
sin
2
z + cos
2
z =
_
e
iz
e
iz
2i
_
2
+
_
e
iz
+e
iz
2
_
2
=
=
(e
2iz
+e
2iz
2) + (e
2iz
+ 2 +e
2iz
)
4
= 1
cos iy =
e
iiy
+e
iiy
2
=
=
e
y
+e
y
2
= cosh y
sin iy =
e
iiy
e
iiy
2i
=
=
e
y
e
y
2i
= i sinh y
e
iz
= e
i(x+iy)
= e
ix
e
y
= (cos x +i sin x)e
y
e
iz
= e
i(x+iy)
= e
ix
e
y
= (cos x i sin x)e
y
Thus it is clear, by mentally adding and subtracting the above results that
=
cos z = cos xcosh y i sin xsinh y
sin z = i cos xsinh y + sin xcosh y
Exercise 10.
(1) Log(1) = i log (i) = ln 1 +i

2
= i

2
(2) Log(z
1
z
2
) = Log([z
1
[[z
2
[e
i(1+2)
) = ln [z
1
[[z
2
[ +i(
1
+
2
+ 2n) = Logz
1
+Logz
2
+i2n
177
(3) Log(z
1
/z
2
) = Log([z
1
[/[z
2
[e
i(1theta2)
) = ln
|z1|
|z2|
+i(
1

2
+ 2n) = Logz
1
Logz
2
+i2n
(4) exp (Logz) = exp (ln [z[ +i +i2n) = z
Exercise 11.
(1)
1
i
= e
iLog1
= e
i(i2n)
= e
2n
= 1 if n = 0
i
i
= e
iLogi
= e
i(i

2
+i2n)
= e

2
2n
= e
/2
if n = 0
(1)
i
= e
iLog1
= e
i(i+i2n)
= e
2n
= e

(2) z
a
z
b
= e
aLogz
e
bLogz
= e
aLogz+bLogz
= e
(a+b)Logz
= z
a+b
x
(3)
(z
1
z
2
)
w
= e
wLogz1z2
= e
w(Logz1+Logz2+2mi)
(z
w
1
z
w
2
) = e
wLogz1
e
wLogz2
= e
w(Logz1+Logz2)
m = 0 is the condition required for equality.
Exercise 12.
if L(u) = P, L(v) = Q,
L(u +iv) = (u +iv)

+a(u +iv)

+b(u +iv) = u

+au

+bu +i(v

+av

+bv) = L(u) +iL(v) = P +iQ = R


if L(f) = R
L(u +iv) = L(u) +iL(v) = P +iQ
then, equating real and imaginary parts, L(u) = P, L(v) = Q
Exercise 13.
L(y) =
2
y +aiy +by = Ae
ix
=(
2
+ai +b)B = A
We cannot let (
2
+aib) = 0 for a nontrivia solution. Thus b ,=
2
or a ,= 0.
B =
A

2
+ai +b
Exercise 14.
L( y) = ce
ix
; y = Be
ix
=
c

2
+ai +b
e
ix
= y =
c
_
(b
2
)
2
+ (a)
2
e
i(x)
where tan =
a
b
2
=1 y =
c
_
(b
2
)
2
+ (a)
2
cos (x )
Exercise 15.
( y) =
c
_
(b
2
)
2
+ (a)
2
sin (x +)
=A =
c
_
(b
2
)
2
+ (a)
2
; tan =
a
b
2
10.4 Exercises - Zenos paradox, Sequences, Monotonic sequences of real numbers. Exercise 1. Converges to 0.
f(n) =
n
n + 1

n + 1
n
=
n
2
(n
2
+ 2n + 1)
n(n + 1)
=
2n 1
n
2
+n
=
2
n

1
n
2
1 +
1
n
n
0
Exercise 2. Converges to 1.
f(n) =
n
3
(n
3
+n +n
2
+ 1)
(n + 1)n
=
n
2
n 1
n(n + 1)
=
1
1
n

1
n
2
1 +
1
n
n
1
Exercise 3. Diverges since
[cos
n
2
L[

1 cos
n
2

[L[

[1 [L[[
Choosing
1
=
|1|L||
2
, [ cos
n
2
L[ >
1
for n = 4m.
178
Exercise 4. f(n) =
1
5
+
3
5n

2
5n
2
lim
n
f(n) =
1
5
Exercise 5. f(x) =
x
2
x
=
x
exp (x ln 2)
0 since lim
x
x

(e
x
)

.
Exercise 6. f(n) = 1 + (1)
n
= 0 of 1.
Thus, choosing
1
=
|1|L||
2
;
[f(n) L[ [[f(n)[ [L[[ = [1 [L[[ >
1
for any n = 2m
Exercise 7. f(n) =
1+(1)
n
n
.
Suppose =
3
N
.
So for n > N,
1
N
>
1
n
, n N = N() = 3/.
[f(n)[ =

1 + (1)
n
n

2
n
<
3
n
<
3
N
=
Exercise 8. f(n) =
(1)
n
n
+
1+(1)
n
2
[f(n) L[ =

(1)
n
n
+
1 + (1)
n
2
L

L
(1)
n
n

1 + (1)
n
2

[L[

(1)
n
n

1 + (1)
n
2

[L[
1
n

1 + (1)
n
2

[L[
1
n

1
2

Thus, consider

[L[
1
n

1
2

>

1
N
[L[

=
0
for n > N
Exercise 9. f(x) = exp
_
1
x
ln 2
_
; lim
x
f(x) = 0.
Exercise 10.
[f(n) L[ = [n
(1)
n
L[ [[n
(1)
n
[ [L[[ = [[n[ [L[[ = [n[ [L[ > N [L[
Thus, for n > N, N() = +[L[, so then [f(n) L[ > .
Exercise 11. f(n) =
n
2/3
sin n!
n+1
.
[f(n)[ =

n
2/3
sin (n!)
n + 1

sin (n!)
n
1/3
+n
2/3

1
n
1/3

Thus, for n > N, N() =


1

3
, [f(n)[ < .
Exercise 12. Converges, since
f(n)
1
3
=
3
n+1
+ 3(2)
n
3
n+1
(2)
n+1
3(e
n+1
+ (2)
n+1
)
=
_
(2)
n
(3 + 2)
3(3
n+1
+ (2)
n+1
)
_
=
=
_
5
3
(2)
n
3
n+1
+ (2)
n+1
_

f(n)
1
3

=
5
3

(2)
n
3
n+1
+ (2)
n+1

(2)
n+1
3
n+1
+ (2)
n+1

=
=

1
_
3
2
_
n+1
+ 1

1
1 +
_
3
2
_
n+1

<
<
1
_
3
2
_
n+1
<
1
_
3
2
_
n
For n > N, consider =
_
2
3
_
N
, i.e. N =
ln
ln 2/3
= N(). Thus
179
L =
1
3
;
Exercise 13.
f(n) =

n + 1

n
f(n) =
_
n + 1

n
_
_
n + 1 +

n + 1 +

n
_
=
n + 1 n

n + 1 +

n
=

n + 1 +

n
[f(n)[ =

n + 1 +

1
2

n
;
So then , we have =
1
2

N
and for n > N,
1
2

n
<
1
2

N
= .
Thus f(n) converges to 0.
Exercise 14.
f(n) = na
n
= nexp nln a =
n
exp nln
1
a
0 since lim
x
x
a
(e
x
)
b
= 0
Exercise 15. f(n) =
log
a
n
n
, a > 1. lim
n
f(n) = 0 since lim
x
(log x)
a
x
b
= 0 for a > 0, b > 0
Exercise 16. lim
n
f(n) = 0
Exercise 17. lim
n
f(n) = e
2
.
Exercise 18.

1 +
n
n + 1
cos
n
2
L

1 +
n
n + 1
cos
n
2

[L[

1 +
n
n + 1

[L[

=
=

1
n + 1
[L[

> [L[
Choose
0
=
|L|
2
. For any N, for n > N, [f(n) L[ >
0
.
Exercise 19.
1 +
i
2
=

5
2
e
i
1 +
_
1
2
_
2
=
5
4
tan =
1
2
_

5
2
e
i
_
n
,
_
2

5
_
n
e
ni
lim
n
_
2

5
_
n
e
ni
= 0
Exercise 20. lim
n
f(n) = lim
n
e
in/2
diverges since
[e
iin/2
L[ [[e
iin/2
[ [L[[ = [1 [L[[ > [L[
So for
0
=
|L|
2
, for n > N, [f(n) [L[[ >
0
Exercise 21. f(n) =
1
n
e
in/2
[f(n)[ =
1
n
.
Suppose N() =
1

; then for n > N, [f(n)[ <


0
Exercise 22. [f(n) L[ [[ne
in/2
[ [L[[ = [n [L[[
Consider
0
= [1 [L[[ for n > N > 1, [f(n) L[ >
0
.
Exercise 23. a
n
=
1
n
.
[a
n
[ =

1
n

<
1
N
; N() =
1

= 1, 0.1, 0.01, 0.001, 0.0001


N = 1, 10, 100, 1000, 10000
Exercise 24. [a
n
1[ =

n+1n
n+1

=
1
n+1
<
1
n
180
N() =
1

= 1, 10, 100, 1000, 10000.


Exercise 25. [a
n
[ =
1
n
N() = 1, 10, 100, 1000, 10000.
Exercise 26. [a
n
[ =

1
n!

1
exp nln n
<
1
exp n
For n > N,
1
exp N
= , so that N = ln 1/.
N() = 1, 2, 4, 6, 9
Exercise 27. a
n
=
2n
n
2
+1
; [a
n
[ =

2
n
2
+1/n

2
n
2

.
N() =

= 1, 4, 14, 44, 141


Exercise 28. [a
n
[ =

9
10

n
=
_
9
10
_
n
= e
nln 9/10
N() =
ln
ln
_
9
10
_ =
ln 1/
ln (9/10)
=
= 1, 21, 43, 65, 87
Exercise 30. If > 0, N Z
+
such that n > N, [a
n
[ < .
[a
n
[
2
< [a
n
[ <
2
[a
2
n
[ <
2
So for
1
> 0,
1
=
2
and N = N() = N(
1
), so that [a
2
n
[ <
1
.
Exercise 31.
[a
n
+b
n
(A+B)[ = [[a
n
A[[b
n
B[[ [a
n
A[ +[b
n
B[ < + = 2
> 0, N
A
, N
B
Z
+
, [a
n
A[ < if n > N
A
; [b
n
B[ < if n > N
B
Consider max (N
A
, N
B
) = N
A+B
[a
n
+b
n
(A+B)[ < 2

1
> 0,
1
= 2, then N
A+B
= N
A+B
(
1
) Z
+
such that
[(a
n
+b
n
) (A+B)[ <
1
if n > N
A+B
[ca
n
cA[ = c[a
n
A[ < c

1
> 0,
1
= c; then N
cA
= N() = N(
1
) Z
+
such that
[ca
n
cA[ <
1
for n > N(
1
)
Exercise 32. Given lim
n
a
n
= A,
lim
n
(a
n
A)(a
n
+A) = lim
n
(a
n
A) lim
n
(a
n
+A) = 0(2A) = 0
2a
n
b
n
= (a
n
+b
n
)
2
a
2
n
b
2
n
2 lim
n
a
n
b
n
= lim
n
(a
n
+b
n
)
2
lim
n
a
2
n
lim
n
b
2
n
=
_
lim
n
(a
n
+b
n
)
_
2
A
2
B
2
= 2AB
= lim
n
a
n
b
n
= AB
Exercise 33.
_

n
_
=
(1)(2)...(n+1)
n!
181
(1)
_

1
2
1!
_
=
1/2
1
_

1
2
2!
_
=
_

1
2
_ _
3
2
_
2!
=
5
8
_

1
2
3!
_
=
_

1
2
_ _
3
2
_ _
5
2
_
3!
=
5
16
_

1
2
4
_
=
_

1
2
_ _
3
2
_ _
5
2
_ _
7
2
_
4!
=
35
128
_

1
2
5
_
=
_

1
2
_ _
3
2
_ _
5
2
_ _
7
2
_ _
9
2
_
5!
=
63
256
(2) a
n
= (1)
n
_1
2
n
_
.
a
1
=
1
2
> 0. a
2
=
3
8
> 0
a
n+1
= (1)
n+1
_

1
2
n + 1
_
= (1)
n+1
_

n
__

1
2
(n + 1) + 1
n + 1
_
=
a
n
(1)
_
1
2
n
_
n + 1
=
=
a
n
(n + 1/2)
n + 1
> 0
a
n+1
=
_
n + 1/2
n + 1
_
a
n
< a
n
Exercise 34.
(1)
t
n
s
n
=
1
n
n

k=1
f
_
k
n
_

1
n
n1

k=0
f
_
k
n
_
=
1
n
_
f(1) +
n1

k=1
f
_
k
n
_

_
f(0) +
1
n
n1

k=1
f
_
k
n
_
__
=
=
1
n
(f(1) f(0))
Since f
_
k
n
_
f(t) f
_
k+1
n
_
for
k
n
t
k+1
n
, by f being monotonically increasing.
=s
n

_
1
0
f(x)dx t
n
(from denition of integral)
0
_
1
0
f(x)dx s
n
t
n
s
n
=
1
n
(f(1) f(0))
(2) Use Theorem 1.9.
Theorem 30. Every function f which is bounded on [a, b] has a lower integral I(f) and an upper integral I(f)
satisfying
_
b
a
s(x)dx I(f) I(f)
_
b
a
t(x)dx
for all step functions s and t with s f t. The function f is integrable on [a, b] iff its upper and lower integrals
are equal,
_
b
a
f(x)dx = I(f) = I(f)
Since f(x) is integrable, then lim
n
s
n
= lim
n
t
n
=
_
1
0
f(x)dx
(3)
_
ba
n
_
= , s
n
=
1

n1
k=0
f(a +k), t
n
=
1

n
k=1
f(a +k)
So by increasing monotonicity of f, s
n

_
b
a
f(x)dx t
n
.
t
n
s
n
=
1

_
f(b) +
n1

k=1
f(a +k) =
n1

k=1
f(a +k) f(a)
_
=
f(b) f(a)

0
_
b
a
f(x)dx
f(b) f(a)

Exercise 35.
(1) lim
n
1
n

n
k=1
_
k
n
_
2
=
_
1
0
t
2
dt =
1
3
(2) lim
n

n
k=1
1
n+k
= lim
n
1
n

n
k=1
1
1+
k
n
=
_
1
0
1
x
dx = ln 2
182
(3) lim
n
1
n

n
k=1
1
1+(
k
n
)
2
=
_
1
0
1
1+x
2
dx = arctan x[
1
0
=

4
(4) lim
n

n
k=1
1

n
2
+k
2
= lim
n
1
n

n
k=1
1

1+(
k
n
)
2
=
_
1
0
1

1+x
2
dx = ln (x +

1 +x
2
)

1
0
= ln (1 +

2)
(5) lim
n

n
k=1
1
n
sin
k
n
= lim
n
1
n

n
k=1
sin
k
n
=
_
1
0
sin xdx =
_
cos x

1
0
=
(11)

=
2

(6) lim
n

n
k=1
1
n
sin
2 k
n
=
_
n
0
sin
2
x =
1
2
10.9 Exercises - Innite series, The linearity property of convergent series, Telescoping series, The geometric series.
Exercise 1.

n=1
1
(2n1)(2n+1)
=

n=1
1/2
2n1

1/2
2n+1
=
1
2
Exercise 2.

n=1
2
3
n1
= 2

n=0
1
3
n
= 2
1
11/3
= 3
Exercise 3.

n=2
1
n
2
1
=

n=2
1/2
n1

1/2
n+1
=

n=2
_
1/2
n1

1/2
n
_
+
_
1/2
n+1

1/2
n
_
=
1
2
+
1
4
=
3
4
Exercise 4.

n=1
2
n
+3
n
6
n
=

n=1
_
1
3
_
n
+

n=1
_
1
2
_
n
=
1/3
11/3
+
1/2
11/2
=
1
2
+ 1 =
3
2
Exercise 5.

n=1

n+1

n
2
+n
=

n=1
1

n

1

n+1
= 1
Exercise 6.

n=1
n
(n + 1)(n + 2)(n + 3)
=

n=1
3/2
(n + 2)(n + 3)
+
1/2
(n + 1)(n + 2)
=

n=1
1
(n + 2)(n + 3)
+
1
2
_
1
6
_
=
=

n=1
1
1 + 2

1
n + 3

1
12
=
1
3

1
12
=
1
4
Exercise 7.

n=1
2n+1
n
2
(n+1)
2
=

n=1
1
n
2

1
(n+1)
2
= 1
Exercise 8.

n=1
2
n
+n
2
+n
2
n+1
n(n+1)
= 1 =

n=1
1
2n(n+1)
+
1
2
n+1
=
1
2

n=1
1
n

1
n+1
+
1/4
11/2
=
1
2
+
1
2
= 1
Exercise 9.

n=1
(1)
n1
(2n+1)
n(n+1)
=

n=1
(1)
n1
_
1
n
+
1
n+1
_
.
(1)
n1
_
1
n
+
1
n + 1
_
+ (1)
n
_
1
n + 1
+
1
n + 2
_
= (1)
n
_
1
n
+
1
n + 2
_

j=1
_
1
2j 1
+
1
2j + 1
_
=
1
2

j=1
_
1
(j 1/2)
+
1
(j + 1/2)
_
=
1
2
1
1/2
= 1
Exercise 10.

n=2
log ((1 +
1
n
)
n
(1 +n))
(log n
n
)(log (n + 1)
n+1
)
=

n=2
log
__
n+1
n
_
n
(1 +n)
_
log (n + 1)
n+1
log n
n
=
=

n=2
log(n + 1)
n+1
log n
n
log (n + 1)
n+1
log n
n
=

n=2
1
log n
n

1
log (n + 1)
n+1
=
=
1
2 log 2
= log
2

e
since if
1
2 log 2
= y, then y = log
2

2.
Exercise 11.

n=1
nx
n
= x
d
dx

n=1
x
n
= x
_
x
1x
_

=
x
(1x)
2
.
Exercise 12.
183

n=1
n
2
x
n
= x
d
dx

n=1
nx
n
= x
_
x
(1 x)
2
_

=
= x
(1 x)
2
+ 2(1 x)x
(1 x)
4
=
x(1 x
2
)
(1 x)
4
=
x(1 +x)
(1 x)
3
Exercise 13.

n=1
n
3
x
n
= x
d
dx

n=1
n
2
x
n
= x
_
x +x
2
(1 x)
3
_

=
= x
(1 + 2x)(1 x)
3
+ 3(1 x)
2
(x)(x + 1)
(1 x)
6
= x
(1 + 2x)(1 x) + 3x(x + 1)
(1 x)
4
=
x(x
2
+ 4x + 1)
(1 x)
4
Exercise 14.

n=1
n
4
x
4
= x
d
dx

n=1
n
3
x
3
= x
_
x
3
+4x
2
+x
(1x)
4
_

.
ln
_
x
3
+ 4x
2
+x
(1 x)
4
_
= ln (x
3
+ 4x
2
+x) 4 ln (1 x)
(ln f)

=
1
f
f

=
3x
2
+ 8x + 1
x
3
+ 4x
2
+x
+ 4
1
1 x
;
f

=
(3x
2
+ 8x + 1)(1 x)
(1 x)
5
+
4(x
3
+ 4x
2
+x)
(1 x)
5
=
=
3x
2
+ 8x + 1 3x
3
8x
2
x + 4x
3
+ 16x
2
+ 4x
(1 x)
5
=
x
3
+ 11x
2
+ 11x + 1
(1 x)
5
=
x
4
+ 11x
3
11x
2
+x
(1 x)
5
Exercise 15.

n=1
x
n
n
= x

n=1
_
x
0
t
n1
dt =
_
x
0
dt

n=1
t
n1
=
_
x
0
1
1t
= ln (1 x).
Exercise 16.

n=1
x
2n1
2n 1
=

j=1
_
x
0
t
2j2
dt =
_
x
0
dt

j=1
(t
2
)
j1
=
=
_
x
0
dt
1 t
2
=
_
x
0
dt
_
1/2
1 t
+
1/2
1 +t
_
=
1
2
ln
_
1 +x
1 x
_
Exercise 17.

n=0
(n + 1)x
n
=

n=0
d
dx
x
n+1
=
d
dx
_
x
1x
_
=
1
(1x)
2
Exercise 18.

n=0
(n + 1)(n + 2)
2!
x
n
=

n=0
_
x
n+2
2
_

=
d
2
dx
2
x
2
2
_
1
1 x
_
=
d
dx
_
x
1 x
+
x
2
2(1 x)
2
_
=
1
1 x
+
x
(1 x)
2
+
x
(1 x)
2
+
x
3
(1 x)
3
=
1 2x +x
2
+ 2x 2x
2
+x
2
(1 x)
3
=
1
(1 x)
3
Exercise 19.

n=0
(n + 1)(n + 2)(n + 3)
3!
x
n
=

n=0
d
3
dx
3
x
n+3
3!
=
1
3
d
dx

n=0
d
2
dx
2
x
(n+1)+2
2
=
=
1
3
d
dx

n=1
d
2
dx
2
x
n+2
2
=
1
3
d
dx
_

n=0
d
2
dx
2
x
n+2
2

d
2
dx
2
_
x
2
2
_
_
=
=
1
3
d
dx
_
1
(1 x)
3
1
_
=
1
3
3(1)
(1 x)
4
=
1
(1 x)
4
Exercise 20.

n=1
n
k
x
n
=
P
k
(x)
(1x)
k+1
184

n=1
n
k+1
x
n
= x
d
dx

n=1
n
k
x
n
= x
d
dx
_
P
n
(x)
(1 x)
k+1
_
= x
_
P

k
(x)(1 x)
k+1
+ (k + 1)(1 x)
k
P
k
(x)
(1 x)
2k+2
_
=
= x
_
P

k
(x)(1 x) + (k + 1)P
k
(x)
(1 x)
k+2
_
=
(k + 1)xP
k
(x) +x(1 x)P

k
(x)
(1 x)
k+2
((k + 1)P
k
(x) + (1 x)P

k
(x))x has x as its lowest degree term from xP

x
(x) and
(k + 1)x
k+1
+kx
k+1
= x
k+1
highest degree term is obtained from (k + 1)P
k
(x) +xP

k
(x).
Exercise 21.

n=0
_
n+k
k
_
x
n
=
1
(1x)
k+1
=
d
k
dx
k

n=0
x
n+k
k!
.

n=0
_
n +k + 1
k + 1
_
x
n
=

n=0
d
k+1
dx
k+1
x
n+k+1
(k + 1)!
=
1
(k + 1)
d
dx

n=0
d
k
dx
k
x
(n+1)+k
k!
=
=
1
k + 1
d
dx

k=1
d
k
dx
k
x
n+k
k!
=
_
1
k + 1
_
d
dx

n=0
d
k
dx
k
x
n+k
k!

d
k
dx
k
x
k
k!
=
=
_
1
k + 1
_
d
dx
_
1
(1 x)
k+1
1
_
=
1
(1 x)
k+2
Exercise 22.
(1)

n=2
n1
n!
=

n=2
1
(n1)!

n=2
1
n!
=

n=1
1
n!

n=2
1
n!
= 1 .
(2)

n=2
n
n!
+

n=2
1
n!
=

n=2
1
(n1)!
+

n=0
1
n!
1 1 =

n=1
1
n!
+

n=0
1
n!
2 =

n=0
2
n!
3 = 2e 3 .
(3)

n=2
(n 1)(n + 1)
n!
=

n=2
n
2
n!
+

n=2
1
n!
=

n=2
n
(n 1)!
+

n=2
1
n!
=
=

n=1
n + 1
n!
+

n=2

1
n!
=

n=1
1
(n 1)!
+ 1 =

n=0
1
n!
+ 1 = e + 1
Exercise 23.
(1) x
d
dx
_
x
d
dx

n=1
x
n
n!
_
= x
d
dx
_
x

n=1
nx
n1
n!
_
= x
d
dx

n=1
nx
n
n!
=

n=1
n
2
x
n
n!
= x
d
dx
_
x
d
dx
e
x
_
= x
2
e
x
+xe
x
(2) x
d
dx
_

n=1
n
2
x
n
n!
_
=

n=1
n
3
x
n
n!
= x
d
dx
_
(x
2
+x)e
x
_
= x
_
(2x + 1)e
x
+ (x
2
+x)e
x
_
= (x
3
+ 3x
2
+x)e
x
x = 1 k = 5
Exercise 24.
(1)

n=2
(1)
n
=

n=2
(1)
n
(n (n 1)). Identical.
(2)

n=2
(1 1) =

n=2
(1)
n
. Not identical.
(3) Not identical.

n=2
(1)
n
vs. (

n=2
(1 + 1)) + 1.
(4) Identical.

n=0
_
1
2
_
n
= 1 +

n=1
_
_
1
2
_
n1

_
1
2
_
n
_
=

n=1
_
1
2
_
n
(2 1) =

n=0
_
1
2
_
n
Exercise 25.
(1)
1 +x
2
+x
4
+ +x
2n
+ =
1
1 x
2
if [x[ < 1
=1 + 0 +x
2
+ 0 +x
4
+ =
1
1 x
2
if [x[ < 1
(2) Thm. 10.2.

n=1
(a
n
+b
n
) =

n=1
a
n
+

n=1
b
n
. So then

j=0
x
j

j=0
x
j
+ (x)
j
2
=

j=0
x
j
(x)
j
2
=

j=0
x
2j+1
=
1
1 x

1
1 x
2
=
x
1 x
2
(3)

j=0
(x
2
)
j
+

j=0
x
j
=

j=0
(x
j
x
2j
) =
x
1 x
2
185
10.14 Exercises - Tests for convergence, Comparison tests for series of nonnegative terms, The integral test. Well be
using the integral test.
Theorem 31 (Integral Test).
Let f be a positive decreasing function, dened for all real x 1.
For n 1, let s
n
=

n
k=1
f(k) and t
n
=

n
1
f(x)dx.
Then both sequences s
n
and t
n
converge or both diverge.
Exercise 1.
3
4j 3
+
1
4j 1
=
3(4j 1) + (1)(4j 3)
(4j 3)(4j 1)
=
8j
(4j 3)(4j 1)
n

j=1
j
(4j 3)(4j 1)
=
n

j=1
_
3/8
4j 3
+
1/8
4j 1
_
_
n
1
_
3/8
4x 3
+
1/8
4x 1
_
dx =
_
(3/8)
ln (4x 3)
4
+ (1/8)
ln (4x 1)
4
_

n
1
=
=
1
32
ln
(4x 3)
3
4x 1

n
1
=
1
32
ln
_
3(4n 3)
3
4n 1
_
lim
n
1
32
ln
_
3(4n 3)
3
4n 1
_
= lim
n
_
n
1
xdx
(4x 3)(4x 1)
dx = , so
n

j=1
j
(4j 3)(4j 1)
diverges as well
Exercise 2.

j=1

2j 1 log (4j + 1)
j(j + 1)
=

j=1
a
j
a
j

4j + 1 log (4j + 1)
j(j + 1/4)
=
4 log (4j + 1)
j(4j + 1)
1/2
_
(4j + 1)
(4j + 1)
_
= 4
log (4j + 1)(4 + 1/j)
(4j + 1)
3/2

16 log (4j + 1)
(4j + 1)
3/2
= b
j
Now use the integral test on

b
j
to determine the convergence of

b
j
.
_
n
1
log (ax + 1)
(ax + 1)
3/2
dx =
_ _
(2)
a(ax + 1)
1/2
_

log (ax + 1)dx =


=
2
a(ax + 1)
1/2
log (ax + 1)
_
2
a(ax + 1)
1/2
_
a
ax + 1
_
=
=
2
a(ax + 1)
1/2
log (ax + 1) +
4
a(ax + 1)
1/2
lim
n
_
n
1
log(ax + 1)
(ax + 1)
3/2
dx = lim
n
_
2 log (an + 1)
a(an + 1)
1/2
+
2 log (a + 1)
a(a + 1)
1/2
+
4
a
_
_
1
an + 1
_
1/2

1
(a + 1)
1/2
__
=
=
2 log (a + 1)
a(a + 1)
1/2
+
4
a(a + 1)
1/2
Then by integral test,

b
j
converges. Since

b
j
converges, then

a
j
converges by comparison test.
Exercise 3.

j=1
j+1
2
j
.
_
n
1
x + 1
e
x ln 2
dx =
_
n
1
(xe
x ln 2
+e
x ln 2
)dx =
_
xe
x ln 2
ln 2
+
e
x ln 2
ln 2
+
e
x ln 2
(ln 2)
2
_

n
1
=
ne
nln 2
ln 2
+
e
ln 2
ln 2
+
_
1
(ln 2)
2
+
1
ln 2
_
e
nln 2
+
_
1
(ln 2)
2
+
1
ln 2
_
e
ln 2
lim
n
_
n
1
x + 1
e
x ln 2
dx =
_
2
ln 2
+
1
(ln 2)
2
__
1
2
_
186
By integral test,

j=1
j+1
2
j
converges.
Exercise 4.

j=1
j
2
2
j
.
_
n
1
x
2
2
x
dx =
_
n
1
x
2
e
x ln 2
=
_
x
2
e
x ln 2
ln 2
+
2xe
x ln 2
(ln 2)
2
+
2e
x ln 2
(ln 2)
3
_

n
1
lim
n
_
n
1
x
2
2
x
dx = e
ln 2
_
1
ln 2
+
2
(ln 2)
2
+
2
(ln 2)
3
_
=
1
2
_
1
ln 2
+
2
(ln 2)
2
+
2
(ln 2)
3
_
By integral test,

j=1
j
2
2
j
converges.
Exercise 5.

j=1
[ sin jx[
j
2
=

j=1
a
j

j=1
1
j
2

j=1
1
j
2
converges since
lim
n
_
n
1
1
x
s
dx = lim
n
x
s+1
s + 1

n
1
= lim
n
_
1
1 s
__
1
n
s1
1
_
=
1
s 1
if s > 1

j=1
|sinjx|
j
2
converges by comparison test and integral test.
Exercise 6.

j=1
2 + (1)
j
2
j
=

j=1
_
1
2
2j1
+
3
2
2j
_
=
2(1/4)
1 1/4
+
3(1/4)
1 1/4
=
4
3
Exercise 7.

j=1
j!
(j+2)!
.
a
j
=
j!
(j + 2)!
= a
j
=
1
(j + 1)(j + 2)

1
j
2
= b
j
Since

b
j
converges,

a
j
converges, by comparison test.
Exercise 8.

j=2
log j
j

j+1
=

j=2
a
j

j=2
log j
j
3/2
_
n
2
log x
x
3/2
dx =
_
n
2
(2x
1/2
)

log x = (2x
1/2
log x)

n
2

_
n
2

2x
1/2
x
dx =
= (2)
_
log n
n
1/2

log 2
2
1/2
_
+4x
1/2

n
2
lim
n
_
n
2
log x
x
3/2
dx = 2
1/2
log 2 +
4

2
So

a
j
converges by comparison test.
Exercise 9.

j=1
1

j(j+1)
=

j=1
a
j
. Let b
j
=
1
j
.
lim
j
a
j
b
j
= lim
j
j
_
j(j + 1)
= lim
j
1
_
1 + 1/j
= 1
By limit comparison test, since

b
j
diverges,

a
j
diverges.
Exercise 10.

j=1
1+

j
(j+1)
3
1
=

j=1
a
j
b
j
=
1
j
5/2
lim
j
a
j
b
j
= lim
j
_
1 +

j
(j + 1)
3
1
_
j
5/2
= lim
j
j
3
+j
5/2
(j + 1)
3
1
= lim
j
1 + 1/j
1/2
(1 + 1/j)
3

1
j
3
By limit comparison test, since

b
j
converges,

a
j
converges.
Exercise 11.

j=2
1
(log j)
s
=

a
j
187
If s 0,

a
j
diverges since lim
j
a
j
,= 0
If 0 < s 1,
1
(log j)
s
>
1
j
s
, and since

j=2
1
j
s
diverges for 0 < s < 1, so does

1
(log j)
s
_
(log x)
s
=
_ _
1
x
(log x)
s
_
x =
_ _
(log x)
s+1
(1 s)
_

x =
(log x)
s+1
(1 s)
x
_
(log x)
1s
1 s
Thus, if s > 1 has any decimal part, or is an integer, its integral will diverge, so that by integral test, the series diverges.
Exercise 12.

j=1
|aj|
10
j
; [a
j
[ < 10.

j=1
[a
j
[
10
j
<

j=1
10
10
j
=

j=0
1
10
j
=
1
1 1/10
=
10
9
Exercise 13.

j=1
1
1000j+1
<

j=1
1
1000j
=
1
1000

j=1
1
j
The series diverges since

1
j
diverges.
Exercise 14.

j=1
j cos
2
(j/3)
2
j

j=1
j
2
j
=

j=1
j
e
j ln 2
_

1
x
e
kx
=
_

1
xe
kx
=
_
xe
kx
k

e
kx
(k)
2
_

1
=
e
k
k
+
e
k
(k)
2
Exercise 15.

j=3
1
j log j(log (log j))
s
_
1
xlog x(log (log x))
s
=
_ _
(ln (ln x))
s+1
s + 1
_

=
_

_
(ln (ln x))
s+1
s+1
if s > 1, s < 1, s ,= 0
ln (ln (ln x)) if s = 1
ln (ln x) if s = 0
Converges, by integral test, for s > 1
Exercise 16. Converges by integral test since
_

1
xe
x
2
=
_
e
x
2
2
_

1
= 0 +
e
1
2
Exercise 17. I drew a picture to help me see whats going on.

x
1 +x
2

x
1
_
1/n
0

xdx =
2
3
x
3/2

1/n
0
=
2
3
_
1
n
_
3/2

j=2
2
3
_
1
j
_
3/2
=
2
3

j=2
1
j
3/2
So

1
n
_
1/n
0

x
1+x
2
dx converges by comparison test.
Exercise 18.
188
(e

x
)

= e

x
1
2

x
(

xe

x
)

=
e

x
2
+
1
2

x
e

x
(

xe

x
+e

x
)

=
e

x
2
_
n+1
n
e

x
dx = 2 (

x + 1)e

n+1
n
= 2
_
x
e

x
+
1
e

x
_

n+1
n
=
= 2
_
n + 1
e

n+1
+
1
e

n+1

n
e

n

1
e

n
_
2

j=1
_
j + 1
e

j+1

j
e

j
+
1
e

j+1

1
e

j
_
=
1
e
Note the use of telescoping sum in the last step. The series converges.
Exercise 19.
_
n
1
f(x)dx =
_
n
1
log xdx = (xln x x)[
n
1
= nln n n + 1.
n1

k=1
lnk
_
n
1
ln x
n

k=2
ln (k) =
n1

k=1
lnk nln n n + 1
n

k=2
ln (k)
exp
_
n1

k=1
ln k
_
= (n 1)! n
n
e
n+1
exp
_
n

k=2
ln k
_
= (n)! n
n
e
n+1
=
e
1/n
e
<
(n!)
1/n
n
<
e
1/n
n
1/n
e
10.16 Exercises - The root test and the ratio test for series of nonnegative terms. Exercise 1.

j=1
(j!)
2
(2j)!
((j + 1)!)
2
(2j + 2)
_
(2j)!
(j!)
2
_
=
(j + 1)
2
(2j + 2)(2j + 1)
=
j
2
+ 2j + 1
4j
2
+ 6j + 2
j

1
4
Converges by ratio test.
Exercise 2.

j=1
(j!)
2
2
j
2
.
((j + 1)!)
2
2
(j+1)
2
2
j
2
(j!)
2
=
(j + 1)
2
2
j
2
2
j
2
+2j+1
=
j
2
+ 2j + 1
2e
j ln 2
j
0
Converges by ratio test.
Exercise 3.

j=1
2
j
j!
j
j
2
j+1
(j + 1)!
(j + 1)
j+1
j
j
2
j
j!
=
2(j + 1)
(j + 1)
_
1
1 + 1/j
_
j
j

2
e
< 1
Converges by ratio test.
Exercise 4.

j=1
3
j
j!
j
j
3
j+1
(j + 1)!
(j + 1)
j+1
_
j
j
3
j
j!
_
= 3
_
1
(1 + 1/j)
j
_
j

3
e
> 1
Diverges by ratio test.
Exercise 5.

j=1
j!
3
j
.
(j + 1)!
3
j+1
3
j
j!
=
j + 1
3
Diverges by ratio test.
Exercise 6.

j=1
j!
2
2j
189
(j + 1)!
2
2(j+1)
2
2j
j!
=
(j + 1)
4
Diverges.
Exercise 7.

j=2
1
(log j)
1/j
Draw a picture to see whats going on.

j=2
1
(log j)
1/j
=

j=2
exp
_
1
j
ln log j
_
0 <
ln log j
j
for j > 3
ln (log j)
j
<
ln j
j
= lim
j
ln (log j)
j
< lim
j
ln j
j
= 0
= lim
j
1
j
ln (log j) = 0 so then
lim
j
exp
_
1
j
ln (log j)
_
= 1

j=2
1
(log j)
1/j
diverges because the a
j
term doesnt go to zero.
Exercise 8.

j=1
(j
1/j
1)
j
((j
1/j
1)
j
)
1/j
= (e
1
j
ln j
1)
j
0
Converges by root test.
Exercise 9.

j=1
e
j
2
(e
j
2
)
1/j
= e
j
j
0
Converges by root test.
Exercise 10. I systematically tried ratio test and then root test. Both were inconclusive.
Consider comparison with

1
j
.
_
e
j
2
j
je
j
2
_
j =
e
j
2
j
e
j
2
= 1
j
e
j
2
j
1
By limit comparison test, since

1
j
diverges, so does

_
1
j

1
e
j
2
_
.
Exercise 11.

j=1
(1000)
j
j!
= e
1000
Exercise 12.

j=1
j
j+1/j
(j+1/j)
j
.
a
1/j
j
=
j
1/j
2
1 +
1
j
2
lim
j
a
1/j
j
= lim
j
exp
_
1
j
2
ln j
_
1 +
1
j
2
= 1
Note that root test is inconclusive.
a
j
=
exp
_
1
j
ln j
_
_
1 +
1
j
_
j

exp
_
1
j
ln j
_
_
1 +
1
j
2
_
j
2
lim
j
a
j
lim
j
exp
_
1
j
ln j
_
_
1 +
1
j
2
_
j
2
=
1
e
> 0
Diverges since lim
j
a
j
> 0.
Exercise 13.

j=1
j
3
(

2+(1)
j
)
j
3
j
.
190
_
j
3
(

2 + (1)
j
)
j
3
j
_
1/j
=
j
3/j
(

2 + (1)
j
)
3
=
e
3
j
ln j
(

2 + (1)
j
)
3
j

2 + (1)
j
)
3
< 1
Converges by root test.
Exercise 14.

j=1
r
j
[ sin jx[.
If 0 < r < 1.

j=1
r
j
[ sin jx[ <

j=1
r
j
so by comparison test,

j=1
r
j
[ sin jx[ converges for 0 < r < 1
If r 1,
lim
j
r
j
[ sin jx[ , = 0 so

j=1
r
j
[ sin jx[ diverges, unless jx = j
Exercise 15.
(1) c
j
= b
j

bj+1aj+1
aj
> 0 j N. Then there must be a positive number r thats in between c
j
and 0.
a
j
b
j
a
j+1
b
j+1
ra
j
r
n

j=N
a
j

n

j=N
(a
j
b
j
a
j+1
b
j+1
) = a
N
b
N
a
n+1
b
n+1
a
N
b
N
=
n

j=N
a
j

a
N
b
N
r
(2) c
n
< 0
b
j

b
j+1
a
j+1
a
j
< 0
a
j
b
j
< b
j+1
a
j+1
=
b
j
b
j+1
<
a
j+1
a
j

1
b
j
diverges, so
lim
j
b
j
b
j+1
1 by ratio test
j
1
b
j
b
j+1
<
a
j+1
a
j
So by ratio test,

a
j
diverges.
Exercise 16. b
n+1
= n; b
n
= n 1.
c
n
= n 1
nan+1
an
r =
an+1
an
1
1
n

r
n
.
Using Exercise 15,

a
n
converges.

1
bn
diverges since

1
bn
is a harmonic series of s = 1.
n 1
na
n+1
a
n
0 =1
1
n

a
n+1
a
n
Exercise 17. For some N 1, s > 1, M > 0, and given that
a
n+1
a
n
= 1
A
n
+
f(n)
n
s
= 1
_
A
f(n)
n
s1
n
_
Consider A
f(n)
n
s1
.
Since [f(n)[ < M, f(n) is nite, so consider s larger than 1 and n going to innity so that
f(n)
n
s1
0.
Using Exercise 16, for

a
j
to converge, A
f(n)
n
s1
= 1 +r where r > 0, for all n N, where N is some positive number.
Let r =
M
N
s1
so that
A = 1 +r +
f(n)
n
s1
> 1
191
If A > 1, then

a
n
converges.
If A = 1, then consider using Exercise 15 and b
n
= nlog n.
c
n
= b
n
b
n+1
a
n+1
a
n
= (n 1) log (n 1) nlog n
_
a
n+1
a
n
_
= (n 1) log (n 1) nlog n
_
1
1
n
+
f(n)
n
s
_
= (n 1) log
_
(n 1)
n
_
nlog n
_
f(n)
n
s
_
=
= (n 1) log
_
n
(n 1)
_
nlog n
_
f(n)
n
s
_
since
log n
n
s1
n
0,
(n 1) log
_
n
(n 1)
_
nlog n
_
f(n)
n
s
_
< 0 for n large enough
since c
n
< 0 for n N for some N > 0, then by Exercise 15,

a
n
is divergent.
Given that A < 1, then for A
f(n)
n
s1
, choose N > 0 so that
M
n
s1
< < 1 and that A
f(n)
n
s1
A+
M
n
s1
= A+ 1.
We can always choose small enough because theres always a real number in between A and 1 (Axiom of Archimedes).
A
f(n)
n
s1
1 =
_
A
f(n)
n
s1
_
1
= using Exercise 16,
a
n+1
a
n
= 1
A+
f(n)
n
s1
n
1
1
n
for all n N
Exercise 18.
_
1 3 5 . . . (2n + 1)
2 4 6 . . . (2n + 2)

2 4 6 . . . (2n)
1 3 5 . . . (2n 1)
_
k
=
_
2n + 1
2n + 2
_
k
n
1
Ratio test fails.
a
n+1
a
n
=
_
2n + 1
2n + 2
_
k
=
_
1 +
1
2n + 2
_
k
=
_
1 +
1/2
n + 1
_
k
=
=
k

j=0
_
k
j
__
1/2
n + 1
_
j
= 1 +k
_
1/2
n + 1
_
+
k

j=2
_
k
j
__
1/2
n + 1
_
j
Note that for k < ,
k

j=2
_
k
j
__
1/2
n + 1
_
j
<
Let

j=2
_
k
j
__
1/2
n + 1
_
j

M
k/2 = A > 1 or k > 2 means

a
j
converges k/2 = A 1 or k 2 means

a
j
diverges
10.20 Exercises - Alternating series, Conditional and absolute convergence, The convergence tests of Drichlet and
Abel. We will be using Leibnizs test alot, initially.
Theorem 32 (Leibnizs Rule). If a
j
is a monotonically decreasing sequence with limit 0,

j=1
(1)
j1
a
j
converges.
If S =

j=1
a
j
, s
n
=

n
j=1
(1)
j1
a
j
,
0 < (1)
j
(S s
j
) < a
j+1
Exercise 1.

j=1
(1)
j+1

j
. lim
j
1

j
= 0 Converges conditionally.
Exercise 2.

j=1
(1)
j

j
j+100
lim
j

j
j+100
= 0. Converges by Leibnizs test.
1

j+
100

1
101

j
, so by comparison test, the series diverges absolutely. So the alternating series converges conditionally
by comparison test.
192
Exercise 3.

j=1
(1)
j1
j
s
If s > 1, then the series absolutely converges. lim
j
1
j
s
= 0 if s > 0. Converges conditionally
for 0 < s < 1. Otherwise, if s < 0 the series diverges absolutely.
Exercise 4.

j=1
(1)
j
_
135...(2j1)
246...(2j)
_
3
.
a
j+1
a
j
=
1 3 5 (2j + 1)
2 4 6 . . . (2j + 2)
2 4 6 . . . (2j)
1 3 5 . . . (2j 1)
=
2j + 1
2j + 2
Absolutely converges.
Exercise 5.

j=1
(1)
j(j1)/2
2
j
converges since lim
j
(1)
j(j1)/2
2
j
= 0;

j=1
1
2
j
=
1/2
11/2
= 1 . Absolutely converges.
Exercise 6.

j=1
(1)
j
_
2j+100
3j+1
_
j
.
exp
_
j ln
_
2j + 100
3j + 1
__
= exp
_
j ln
_
2
3
+
298
9j + 3
__
exp
_
j
_
ln
_
2
3
_
+
1
2
3
_
298
9j + 3
___
=
= exp
_
j ln
2
3
+
146
3 + 1/j
_
0 lim
j
exp
_
j ln
_
2j + 100
3j + 1
__
lim
j
exp
_
j ln
2
3
+
146
3 + 1/j
_
= 0
= lim
j
exp
_
j ln
_
2j + 100
3j + 1
__
= 0
So the alternating series converges.
_
2j + 100
3j + 1
_
<
2j + 100
3j
<
2.5j
3j
=
5
6
( for j 200)
=
_
2j + 100
3j + 1
_
j
<
_
5
6
_
j
for j 200
So the series absolutely converges by comparison with a geometric series.
Exercise 7.

j=2
(1)
j

j+(1)
j
.
lim
j
1

j + (1)
j
doesnt exist since ???
To show divergence, we usually think of either taking the general term and nding the limit (and if it goes to a nonzero
constant, then it diverges), or we use ratio, root, comparison test on the general term. Since this is an alternating series, Ive
observed that the general term is a sum of two adjacent terms, one even and one odd.
(1)
j

j + (1)
j
(1)
2j

2j + (1)
2j
+
(1)
2j+1

2j + 1 + (1)
2j+1
=
1

2j + 1
+
1

2j + 1 + 1
=

2j + 1 1 (

2j + 1)
(

2j + 1)(

2j + 1 1)
=
=

2j + 1

2j 2
(

2j + 1)(

2j + 1 1)
=

2j
_
1 +
1
2j

2j 2
(

2j + 1)(

2j
_
1 +
1
2j
1)
=
for j large

2j
_
1 +
1
4j
_

2j 2
(

2j + 1)(

2j
_
1 +
1
4j
_
1)
=
2
j
_
1
1
4

2j
2
1
2j
+
1
2

2j
3/2
_
Every term, since we considered any j, will contain 2. So we factor it out. Then
1
j
_
1
1
4

2j
2
1
2j
+
1
2

2jj
3/2
_
>
1
j
_
1
1
4

2j
4
1

2j
_
=
1
4j
193
By comparison test to
1
j
the series diverges.
Exercise 8. Using the theorem
Theorem 33.
Assume

[a
j
[ converges
Then

a
j
converges and [

a
j
[

[a
j
[.
So using the contrapositive,
If

a
j
diverges,

[a
j
[ diverges.
1
j
1/j
=
1
e
1
j
ln j
lim
j
1
j
1/j
=
1
exp
_
lim
j
1
j
ln j
_ = 1
Diverges absolutely.
Exercise 9.

j=1
(1)
j j
2
1+j
2
Diverges absolutely.
(2j)
2
1 + (2j)
2

(2j 1)
2
1 + (2j 1)
2
=
4j
2
1 + 4j
2
_
4j
2
4j + 2
4j
2
4j + 2
_

(4j
2
4j + 1)
(4j
2
4j + 2)
(1 + 4j
2
)
1 + 4j
2
=
4j 1
2(1 + 4j
2
)(2j
2
2j + 1)
4j 1
2(1 + 4j
2
)(2j
2
2j + 1)
(j
3
) =
4 1/j
2(4 + 1/j
2
)(2 2/j + 1/j
2
)
=
1
4
By limit comparison test, with

1
j
3
,

j=1
j
2
1+j
2
converges.
Exercise 10.

n=1
(1)
n
log (e
n
+e
n
)
lim
n
1
log (e
n
+e
n
)
= lim
n
1
n
= 0
The series converges.
lim
n
n
log (e
n
+e
n
)
= lim
n
n
log e
n
+ log (1 +e
2n
)
= lim
n
1
_
n+log (1+e
2n
)
n
_ = 1
Since

1
n
diverges,

1
log (e
n
+e
n
)
diverges.
Exercise 11.

j=1
(1)
j
j log
2
(j+1)
lim
j
1
j log
2
(j+1)
= 0 so by Leibnizs test, the alternating series.
1
nlog
2
(n + 1)
<
1
nlog
2
(n)
_
1
nlog
2
n
=
_ _
1
log n
_

=
1
log n
n

1
log 2
Converges by comparison test to
1
nlog
2
n
, which converges by integral test. So the series absolutely converges.
Exercise 12.

j=1
(1)
j
log (1+1/j)
diverges absolutely.
194
(1)
log
_
1 +
1
2j1
_ +
1
log
_
1 +
1
2j
_ =
1
log
_
2j
2j1
_ +
1
log
_
2j+1
2j
_ =
log
_
2j+1
2j
_
+ log
_
2j
2j1
_
log
_
2j
2j1
_
log
_
2j+1
2j
_ =
=
log
_
(2j)
2
4j
2
1
_
log
_
2j
2j1
__
log
_
1 +
1
2j
__ =
log
_
1 +
1
4j
2
1
_
log
_
1 +
1
2j1
_
log
_
1 +
1
2j
_ =
=
1
4j
2
1
+o
_
1
4j
2
1
_
_
1
2j1
+o
_
1
2j1
___
1
2j
+o
_
1
2j
__

4j
2
2j
4j
2
1
=
1
1
2j
1
1
4j
2
j
1
So the alternating series diverges.
Exercise 13.

j=1
(1)
j
j
37
(j+1)!
Use the ratio test.
a
j+1
a
j
=
(j + 1)
3
7
(j + 2)!
(j + 1)!
j
3
7
=
_
1
j + 2
__
1 +
1
j
_
37
0
Converges for

[a
j
[. Then

a
j
converges. The series absolutely converges.
Exercise 14.

n=1
(1)
n
_
n+1
n
e
x
x
dx
_
n+1
n
e
x
x
dx
_
n+1
n
1
e
2x
dx =
e
2x
2

n+1
n
=
_
1
2
__
1
e
2(n+1)

1
e
2n
_
=
=
e
2
1
2e
2n+2
< 1
Converges absolutely.
Exercise 15.

n
j=1
sin (log j)
lim
j
sin (log j) doesnt exist. So the series is divergent.
Exercise 16.

j=1
log
_
j sin
1
j
_
Note that
log
_
j sin
1
j
_
= log
_
sin 1/j
1/j
_
lim
j
log
_
sin 1/j
1/j
_
= log
_
lim
j
sin 1/j
1/j
_
= log 1 = 0
sin
1
j
=

k=0
_
1
j
_
2k+1
(2k + 1)!
(1)
k
log
_
_
_j

k=0
_
1
j
_
2k+1
(1)
k
(2k + 1)!
_
_
_ = log
_
_
_1 +
1
6j
2
+

k=2
_
1
j
_
2k
(1)
k
(2k + 1)!
_
_
_
log
_
1 +
1
6j
2
_

1
6j
2
The series absolutely converges.
Exercise 17.

j=1
(1)
j
_
1 j sin
1
j
_
195
_
1 xsin
1
x
_

= sin
1
x
xcos
1
x
_
1
x
2
_
=
= sin
1
x
+
1
x
cos
1
x
=
xsin
1
x
+ cos
1
x
x
sin
1
x
=

j=0
_
1
x
_
2j+1
(1)
j
(2j + 1)!
xsin
1
x
+ cos
1
x
x
=
=
1 +
(
1
x
)
3
(+1)
3!
+

j=2
(
1
x
)
2j+1
(1)
j
(2j+1)!
+ 1
_
1
x
_
2
/2 +

j=2
(
1
x
)
2j
(1)
j
(2j)!
x
< 0 for x large enough

j=1
(1)
j
_
1 j sin
1
j
_
converges since a
j
= 1 j sin
1
j
is monotonically decreasing sequence with limit 0.
1 j sin
1
j
= 1 j

k=0
_
1
j
_
2k+1
(1)
k
(2k + 1)!
= 1 j
_
_
_
1
j
+

k=1
_
1
j
_
2k+1
(1)
k
(2k + 1)!
_
_
_ =
= 1
_
_
_1 +

k=1
_
1
j
_
2k
(1)
k
(2k + 1)!
_
_
_ =

k=1
_
1
j
_
2k
(1)
k+1
(2k + 1)!

1
6j
2
The series converges absolutely since the term itself is a series that is dominated by
1
6j
2
, so that by comparison test, the series
must converge.
Exercise 18.

j=1
(1)
j
_
1 cos
1
j
_
.
(cos
1
x
)

=
_
sin
1
x
_
1
x
2
__
=
1
x
2
sin
1
x
< 0

j=1
(1)
j
(1 cos
1
j
) converges since a
j
= (1 cos
1
j
) is monotonically decreasing to 0
(1 cos
1
j
) = 1

k=0
(1/j)
2k
(1)
k
(2k)!
=

k=1
(1/j)
2k
(1)
k+1
(2k)!

1
2j
2
So the series converges absolutely, by comparison test with

1
j
2
which converges.
Exercise 19.

j=1
(1)
j
arctan
1
2j+1
.
(arctan
_
1
2j + 1
_
)

=
1
1 +
_
1
2j+1
_
2
_
1
(2j + 1)
2
_
(2) =
2
(2j + 1)
2
+ 1
< 0

j=1
(1)
j
arctan
1
2j+1
converges, since a
j
= arctan
1
2j+1
is monotonically decreasing to 0
1
1 +x
2
= (arctan x)

j=0
(x
2
)
j
=

j=0
(1)
j
x
2j
=arctan x =

j=0
(1)
j
x
2j+1
2j + 1
arctan
1
2j + 1
=

k=0
(1)
k
_
1
2j+1
_
2k+1
(2k + 1)
=
1
2j + 1
+ (1)
_
1
(2j+1)
3
3
_
+

k=2
(1)
k
_
1
2j+1
_
2k+1
2k + 1
>
>
1
2j + 1
+ (1)
1
3(2j + 1)
3
=
3(4j
2
+ 4j + 1) + (1)
3(2j + 1)
3
=
12j
2
+ 12j + 2
3(2j + 1)
3
>
2j
(2j + 1)
2
>
2
9
1
j
for j > 2
So by comparison test to

1
j
,

arctan
1
2j+1
diverges absolutely. The series is conditionally convergent.
Exercise 20.

j=1
(1)
j
_

2
arctan log j
_
196
_

2
arctan log n
_

=
_
1
1 + (log n)
2
__
1
n
_

2
arctan log n

2
arctan (n 1)

2
arctan n just change indices
_
n
0
_

2
arctan x
_
dx =

2
x
_
xarctan x
1
2
ln (1 +x
2
)
_

n
0
=

2
n
_
narctan n
1
2
_
ln
_
1 +n
2
__
_
= n
_

2
arctan n
_
+
1
2
ln (1 +n
2
)
lim
n
_
n
_

2
arctan n
_
+
1
2
ln (1 +n
2
)
_

Then by the integral test,

2
arctan log n diverges absolutely. So the alternating series is conditionally convergent.
Exercise 21.

j=1
log
_
1 +
1
| sin j|
_
lim
j
log
_
1 +
1
| sin j|
_
doesnt exist and log
_
1 +
1
| sin j|
_
> 0 j so the series diverges.
Exercise 22.

j=2
sin
_
j +
1
log j
_
sin
_
2j +
1
log 2j
_
+ sin
_
(2j + 1) +
1
log (2j + 1)
_
=
= sin (2j) cos
1
log 2j
+ sin
_
1
log 2j
_
cos (2j) + sin (2j + 1) cos
1
log (2j + 1)
+ cos (2j + 1) sin
_
1
log 2j + 1
_
=
= sin
1
log 2j
sin
1
log 2j + 1
=
=

k=0
_
1
log 2j
_
2k+1
(1)
k
(2k + 1)!

k=0
_
1
log (2j+1)
_
2k+1
(1)
k
(2k + 1)!
sin
_
1
log 2j
_
sin
_
1
log 2j + 1
_
=
=

k=0
(1)
k
(2k + 1)!
_
_
1
log 2j
_
2k+1

_
1
2j + 1
_
2k+1
_
0 < log 2j < log 2j + 1 so
_
1
log 2j
_
2k+1

_
1
log 2j + 1
_
2k+1
> 0
and since for j > 1,
_
1
log 2j
_
and
_
1
log 2j + 1
_
are < 1 and so we are adding smaller and smaller amounts
<
1
log 2j

log 2j + 1
=
=
log (2j + 1) log 2j
log 2j log 2j + 1

log
_
1 +
1
2j
_
(log (2j))
2

1
2j
(log (2j))
2
_
n
1
1
2j(log 2j)
2
=
1
log 2j

n
1
n

1
log 2
So the series converges by using integral test, showing that

1
2j(log 2j)
2
converges, so by comparison test, the series con-
verges.
Exercise 33.

n=1
n
n
z
n
197

n=1
n
n
z
n

j=1
(jz)
j

_
e
ln j
z
_
j
n

j=1
(e
ln j
z)
j
=
=
e
ln j
z (e
ln j
z)
n
1 e
ln j
z

So z = 0
Exercise 34.

j=1
(1)
j
z
3j
j!
=

j=1
(z
3
)
j
j!
= e
z
3
. C .
Exercise 35.

j=0
z
j
3
j
=

j=0
_
1
3
_
j
z
j

z
j
be convergent or

n
j=1
z
j
bounded.
[z[ < 3 and [z[ = 3 if z ,= 3
Exercise 36.

j=1
z
j
j
j
z = C since

_
z
j
_

< 1 for j N > [z[


Exercise 37.

j=1
(1)
j
z+j
By Leibnizs Rule, since
1
z+j
j
0, then the series converges. However, z cannot be equal to any negative integer since
one term in the series will then blow up.
Exercise 38.

j=1
z
j

j
log
_
2j+1
j
_
.
z
j

j
log
_
2 +
1
j
_
= z
j
log
_
2 +
1
j
_

j
Since lim
j
log (2+
1
j
)

j
= 0 so that
log (2+
1
j
)

j
is a monotonically convergent sequence.
Then by Dirchlets test,

n
j=1
z
j
must be bounded. [z[ > 1, and [z[ = 1 if z ,= 1.
Exercise 39.

j=1
_
1 +
1
5j+1
_
j
2
[z[
17j
=

j=1
_
1 +
1
5j+1
_
j
2
([z[
17
)
j
_
_
1 +
1
5j + 1
_
j
[z[
17
_
j
lim
j
_
1 +
1
5j + 1
_
j
lim
j
_
1 +
1/5
j
_
j
= e
1/5
e
1/5
[z[
17
< 1 = [z[ < e
1/85
Exercise 40.

j=0
(z1)
j
(j+2)!

j=0
(z 1)
j
(j + 2)!

j=0
[(z 1)[
j
(j + 2)!

j=0
[(z 1)[
j+2
(j + 2)!
= e
|z1|
1
[z 1[
1!
The series converges z.
Exercise 41.

j=1
(1)
j
(z 1)
j
j
=

j=1
(1 z)
j
j
= log (1 (1 z)) = log z
So the series converges z except for z = 0.
198
Exercise 42.

j=1
(2z+3)
j
j log (j+1)
lim
j
1
j log (j + 1)
= 0 so
1
j log (j + 1)
is a monotonically convergent sequence

(2z + 3)
j
[2z + 3[ < 1
[z +
3
2
[ <
1
2
then

(2z + 3)
j
converges
_
1
xlog (x + 1)
_

=
1
(xlog (x + 1))
2
_
log (x + 1) +
x
x + 1
_
< 0; for x > 0
By Dirichlets test,

j=1
(2z+3)
j
j log (j+1)
converges for [z +
3
2
[
1
2
; z ,= 1 .
Exercise 43.

j=1
(1)
j
(2j1)
_
1z
1+z
_
j
=

j=1
(1)
j
/2
j
1
2
_
1z
1+z
_
j
=

j=1
(
z1
z+1
)
j
2j1
= lim
j
1
2j1
= 0 and
1
2j1
is monotonically decreasing.
So
1
2j1
is a monotonically decreasing convergent sequence of real terms.
For

z1
z+1

< 1,

z 1
z + 1

< 1 =[z 1[ < [z + 1[


(u 1)
2
+v
2
< (u + 1)
2
+v
2
u
2
2u + 1 < u
2
+ 2u + 1
u < u
1(z) > 0; z ,= 0
Exercise 44.

j=1
_
z
2z + 1
_
j
=

j=1
_
1
2
+
1/2
2z + 1
_
j
=

j=1
_
1
2
_
j
_
1
1
2z + 1
_
j
_
1
2
_
j
is a monotonically decreasing, convergent sequence
Now we want

1
1
2z + 1

2z + 1 1
2z+

2z
2z + 1

< 1
[2z[ < [2z + 1[
[z[ < [z + 1/2[
u
2
+v
2
< (u + 1/2)
2
+v
2
= u
2
+u + 1/4 +v
2
1
4
< u = 1(z) >
1
4
if 1(z) =
1
4
2
_

1
4
+iv
_
2
_
1
4
+iv
_
+ 1
=
1
2
+ 2iv
1
2
+ 2iv
= z ,=
1
4
Exercise 45.

j=1
j
j+1
_
z
2z+1
_
j
.
_
x
x + 1
_

=
(x + 1) x
(x + 1)
2
=
1
(x + 1)
2
> 0
j
j + 1
is a monotonically increasing and convergent sequence
1
2
+
1
2(2z + 1)
=

z
2z + 1

< 1

2z + 1
z

> 1 = [2 +
1
z
[ > 1
Exercise 46.

j=1
1
(1+|z|
2
)
j
=

j=1
_
1
1+|z|
2
_
j
199
1
1 +[z[
2
< 1
1 < 1 +[z[
2
0 < [z[
2
z except z = 0
Exercise 47.

j=1
(1)
j 2
j
sin
2j
x
j
Use Dirichlets Test.
1
j
is a monotonically decreasing sequence converging to zero. Consider (2)
j
sin
2j
x. The condition
for convergence is
[(2 sin
2
x)[ < 1 for j N for some N
=x (

4
+n,

4
+n), n Z
Exercise 49.

a
j
converges.

a
j
1
aj
diverges.
Then since

a
j
is a convergent series (by Abels test),
1
aj
is a divergent sequence.
Then

1
aj
is divergent (since lim
j
1
aj
doesnt exist ).
Exercise 50.

[a
j
[ converges.

[a
j
[ converges, then

a
j
converges.
[a
j
[
2
= a
2
j
. [a
j
[ converges, then
lim
j
[a
j
[ = 0
lim
j
[a
j
[
2
= 0
lim
j
[a
j
[
2
a
2
j
= 1
By limit comparison test,

a
2
j
converges.
Counterexample:

_
1
j
_
2
converges, but

1
j
diverges.
Exercise 51. Given

a
j
, a
j
0.

a
j
converges.

a
j
1
(j)
p
lim
j
a
j
= 0
lim
j

a
j
=
_
lim
j
a
j
_
1/2
= 0
_
n1

j=0
x
j
=
n1

j=0
x
j+1
j + 1
=
n

j=1
x
j
j
=
_
1 x
n
1 x
A counterexample would be

aj
j
p
=
_
aj
j
.
Exercise 52.
(1)

a
j
converges absolutely, then if

[a
j
[ converges,

a
2
j
converges.
a
2
j
1 +a
2
j
= 1 +
1
1 +a
2
j
a
2
j
1 +a
2
j
a
2
j
since

a
2
j
converges ,

a
2
j
1 +a
2
j
converges
(2)

a
j
converges absolutely, lim
j
[a
j
[ = 0

aj
1+aj

[a
j
[
_
1
|1+aj|
_
.
By Abels test, since
lim
j
1
[1 +a
j
[
=
1
[1 + lim
j
a
j
[
= 1 shows that
1
[1 +a
j
[
0 is a monotonically convergent sequence
By Abels test,

aj
1+aj
is convergent.
200
10.22 Miscellaneous review exercises - Rearrangements of series. Exercise 1.
(1)
a
j
=
_
j + 1
_
j =
_
j
_
1 +
1
j

_
j =
_
j
_
1 +
1
2
_
1
j
_
+o
_
1
j
_
+1
_
=
=
_
j
_
1
2
_
1
j
_
+o
_
1
j
__
lim
j
a
j
= 0
(2)
a
j
= (j + 1)
c
j
c
= j
c
__
1 +
1
j
_
c
1
_
= (j
c
)
_
1 +c
_
1
j
_
+o
_
1
j
_
1
_
=
= j
c
_
c
_
1
j
_
+o
_
1
j
__
=
_
cj
c1
+j
c
o
_
1
j
__
if c > 1, a
j
diverges
if c = 1, lim
j
a
j
= 1
if c < 1, lim
j
a
j
= 0
Exercise 2.
(1)
(1 +x
n
)
1
n
= exp
_
1
n
ln (1 +x
n
)
_
= exp
_
_
1
n

j=1
(x
n
)
j
(1)
j1
j
_
_
n
1
(2) lim
n
(a
n
+b
n
)
1/n
= lim
n
a
_
1 +
_
b
n
_
n
_
1/n
= a if a > b.
Exercise 3. a
n+1
=
an+an1
2
=
an1+an2
2
2
+
an2+an3
2
2
10.24 Exercises - Improper integrals.
Exercise 1.
_

0
x

x
4
+1
dx
lim
x
_
x

x
4
+ 1
__
1
1/x
_
= lim
x
x
2
x
2
_
1 + 1/x
4
= 1
Since
_

1
1
x
diverges, so does
_

0
x

x
4
+1
dx
Exercise 2.
_

e
x
2
dx =
_

0
e
x
2
dx +
_

0
e
x
2
dx =
_

0
e
x
2
dx +
_
0

e
x
2
dx = 2
_

0
e
x
2
dx
_

0
e
x
2
dx
_

0
e
x
dx = e
x
2

0
= 1
Converges by theorem.
Exercise 3.
_

0
1

x
3
+1
dx
Exercise 4.
_

0
1

e
x
dx
Exercise 5.
_

0
+
e

x
dx
Exercise 6.
_
1
0
+
log x

x
dx
Exercise 7.
_
1
0
+
log x
1x
dx
Exercise 8.
_

x
cosh x
dx
Exercise 9.
_
1
0
+
dx

x log x
201
Exercise 10.
_

dx
x(log x)
s
11.7 Exercises - Pointwise convergence of sequences of functions, Uniform convergence of sequences of functions,
Uniform convergence of sequences of functions, Uniform convergence and continuity, Uniform convergence and inte-
gration, A sufcient condition for uniform convergence, Power series. Circle of convergence. Exercise 1.

j=0
z
j
2
j
=

j=0
_
z
2
_
j
Using the comparison test,

z
2

j
t
j
;

z
2

< 1 =[z[ < 2


Suppose [z[ = 2, z ,= 2

j=0
_
z
2
_
j
=

j=0
(e
2i
)
j
Now

n
j=0
(e
2i
)
j

1
sin
+ 1, since
n

j=1
(e
2i
)
j
=
e
2i
e
2i(n+1)
1 e
2i
=
e
in
e
in
e
in+i
e
i
+e
i
=
sin ne
in+i
sin
<
1
sin
So

j=0
_
z
2
_
j
converges for [z[ = 24, z ,= 2
Exercise 2.

j=0
z
j
(j+1)2
j
Use ratio test .
a
j+1
a
j
=
z
j+1
(j + 2)2
j+1
(j + 1)2
j
z
j
=
z
2
(j + 1)
(j + 2)
=
z
2
(1 + 1/j)
(1 + 2/j)
j

z
2
If [z[ < 2,

j=0
a
j
converges, if [z[ > 2,

a
j
diverges.
If [z[ = 2,

z
j
(j+1)2
j
=

(e
2i
)
j
_
1
j+1
_
Now
1
j+1
is a monotonically decreasing sequence of real terms.

(e
2i
)
j
is a bounded series.
By Dirichlets test,

a
j
converges if [z[ = 2, z ,= 2
Exercise 3.

j=0
(z+3)
j
(j+1)2
j
Use ratio test:
a
j+1
a
j
=
_
(z + 3)
j+1
(j + 2)2
j+1
_
(j + 1)2
j
(z + 3)
j
=
(z + 3)
2
_
j + 1
j + 2
_
=
(z + 3)
2
_
1 + 1/j
1 + 2/j
_
j

z + 3
2
Using Theorem 11.7,
Theorem 34 (Existence of a circle of convergence).
Assume

a
j
z
j
converges for at least z
1
,= 0
diverges for at least one z
2
,= 0
r > 0, such that

a
j
z
j
absolutely converges for [z[ < r
diverges for [z[ > r
We can plug in real numbers to satisfy the condition
|z+3|
2
< 1 for convergence.

a
j
converges for [z + 3[ < 2; diverges for [z + 3[ > 2.
Consider [z + 3[ = 2; z ,= 1

a
j
=

(e
2i
)
j
_
1
j+1
_
. Since
1
j+1
is a monotonically decreasing sequence of real
numbers and

(e
2i
)
j
is a bounded series, by Dirichlets test,

a
j
converges for [z + 3[ = 2; z ,= 1.
Exercise 4.

j=1
(1)
j
2
2j
z
2j
2j
=

j=1
(1)
j1
(2z)
2j
(2j)
. Look at what the terms look like.
Consider using Leibnizs Rule, Theorem 10.14.
Theorem 35 (Leibnizs rule). If a
j
is a monotonic decreasing sequence and lim
j
a
j
= 0,
then

j=1
(1)
j1
a
j
converges.
202
(1)
j
2
2j
z
2j
2j
=
(1)
j
(2z)
2j
2j
Consider

(2z)
2j
2j

=
[2z[
2j
2j
=
(2[z[)
2j
2j
Consider 2[z[ = M
1/2
<
=
(2[z[)
2j
2j
=
M
j
2j
=
e
j ln M
2j
converges for 0 < 2[z[ = M 1 =[z[
1
2
(we use Theorem 11.6 at this point, because real numbers are included in complex numbers).
Theorem 36. Assume

a
j
z
j
converges for some z = z
1
,= 0.
(1)

a
j
z
j
converges absolutely z with [z[ < [z
1
[.
(2)

a
j
z
j
converges uniformly on every circular disk with center at 0 and R < [z
1
[
We had rst used Leibnizs test to nd az
1
on the real line.
2
4j
z
4j
(4j(1
1
4z
2
) 2)
4j(4j 2)
=
2
4j
z
4j
((1
1
4z
2
)
2
4j
)
(4j 2)
j

2
4j
z
4j
(1
1
4z
2
)
(4j 2)
If [z[ >
1
2
, the series diverges (by a
j
th general term test).
Exercise 5.

j=1
(1 (2)
j
)z
j
.
Try limit comparison test .
The rst step is to test absolute convergence rst; its easier.

(1 (2)
j
)z
j

[(2z)
j
[
=

_
1
2
_
j
1

j
1
According to limit comparison test, for

(1 (2)
j
)z
j
to converge,

(2z)
j
must converge.
So if [z[ <
1
2
, then

j=1
(1 (2)
j
)z
j
absolutely converges.
If [z[ =
1
2
, z ,=
1
2
,

(1 (2)
j
)z
j
=

z
j

(e
2i
)
j
=
= 0

(e
2i+i
)
j
<
1
sin ( +)
If z =
1
2
,

1
2
_
j

1
j

Exercise 6.

j=1
j!z
j
j
j
A very big hint is to use Exercise 19 on pp. 399, in the section for Exercises 10.14.
203
Since

n1
j=1
f(j)
_
n
1
f(x)dx
n1

j=1
ln j
_
n
1
ln x = nln n n + 1
n

j=2
ln j
(n 1)! n
n
e
n
n n!
n!
n
n
ne
n
lim
n
n!
n
n
lim
n
ne
n
= 0
lim
n
n!
n
n
lim
n
n
2
e
n
= 0
= lim
n
n!
n
n
= 0
So then since
n!
n
n
is a monotonically decreasing convergent sequence of real terms; if

z is a bounded series, then by
Dirichlets test,

n!z
n
n
n
is convergent.
[z[ < 1; [z[ = 1 if z ,= 1
Try the ratio test, because its clear from the results of the ratio test where convergence and divergence begins and ends.
(n + 1)![z[
n+1
(n + 1)
n+1
n
n
n![z[
n
=
_
n
n + 1
_
n
[z[ =
=
_
1
1 + 1/n
_
n
[z[
n

1
e
[z[
Converges for [z[ < e .
Now try plugging in a complexied e:
e
j
_
j
j
j!
_ =
j!e
j
e
i2j
(j)
j

j
j
e
j
je
j
e
i2j
j
j
e
i2j

So the series diverges for [z[ = e.


Exercise 7.

j=0
(1)
j
(z+1)
j
j
2
+1
By the ratio test,
[z + 1[
j+1
j
2
+ 2j + 2
j
2
+ 1
[z + 1[
j
= [z + 1[
1 + 1/j
2
1 + 2/j + 2/j
2
j
[z + 1[
The series absolutely converges for [z + 1[ < 1.
For z = 0,

(1)
j
j
2
+1
converges since
1
j
2
+1
j
0
For z = 2,

(1)
j
(1)
j
j
2
+1
=

1
j
2
+1
and
1
j
2
+1
<
1
j
2
, but

1
j
2
is convergent (by integral test). So the series converges
for z = 2.
By Dirichlets test, the series converges as well, if we treat a
j
= (1)
j
(z + 1)
j
and b
j
=
1
j
2
+1
to be the monotonically
decreasing sequence.
= [z + 1[ 1 for convergence x
Exercise 8.

j=0
a
j
2
z
j
, 0 < a < 1
Use the root test .
(a
j
2
z
j
)
1/j
= a
j
z
j
0
So the series converges z C
Exercise 9.

j=1
(j!)
2
(2j)!
z
j
204
Use the ratio test
a
j+1
a
j
=
((j + 1)!)
2
z
j+1
(2(j + 1))!
(2j)!
(j!)
2
z
j
=
(j + 1)
2
z
(2j + 2)(2j + 1)
j

1(z)
4
for [z[ < 4,

a
j
absolutely converges
for [z[ > 4,

a
j
diverges
Lets test the boundary for convergence.
(j!)
2
(2j)!
4
j
=
(j!)
2
(2j)!
e
j ln 4
=
(j!)
2
(2j 2)!
e
j ln 4
(2j)(2j 1)

(j!)
2
(2j 2)!
e
j ln 4
(2j)
2

(j!)
2
(2j 2)!
j
2j
(j!)
2
=
j
2j
(2j 2)!

where we had used
(n 1)! n
n
e
n
n n! =ne
n

n!
n
n
n
e
n

n!
n
n
=
n
n
n!

e
n
n
Exercise 10.

j=1
3

j
z
j
j
=

j=1
e

j ln 3
z
j
j
e

j+1 ln 3
z
j+1
j + 1
j
e

j ln 3
z
j
=
_
1
j + 1
_
e
(

j+1

j) ln 3
z
_
j + 1
_
j =
_
1 +
1
j
1 1 +
1
2
_
1
j
_
1 =
1
2j
(for large j )
(for large j )
_
j
j + 1
_
e
1
2j
z 0
= Converges z C
Exercise 11.

j=1
_
135...(2j1)
246...(2j)
_
3
z
j
a
j+1
a
j
=
_
1 3 5 . . . (2j + 1)
2 4 6 . . . (2j + 2)
_
3
z
j+1
_
2 4 6 . . . (2j)
1 3 5 . . . (2j 1)
_
3
1
z
j
=
_
2j + 1
2j + 2
_
3
z =
_
1 +
1/2
j + 2
_
3
z
If [z[ < 1, it converges by ratio test, if [z[ = 1, then it converges by Gauss test
a
j+1
a
j
=
_
1 +
1/2
j + 2
_
3
z =
3

k=0
_
3
k
__
1/2
j + 2
_
k
[z[ =
= [z[
_
1 + 3
_
1/2
j + 2
_
+ 3
_
1/2
j + 2
_
2
+
_
1/2
j + 2
_
3
_
j
[z[
diverges for [z[ > 1 (by ratio test )
Exercise 12.

j=1
_
1 +
1
j
_
j
2
z
j
_
_
1 +
1
j
_
j
2
z
j
_
1/j
=
_
1 +
1
j
_
j
z
j
e
1
z
[z[ <
1
e
,

(1 +
1
j
)
j
2
z
j
converges by root test
[z[ >
1
e
,

(1 +
1
j
)
j
2
z
j
diverges by root test
1
e
= r
205
Exercise 13.

j=0
(sin aj)z
j
a > 0
[ sin (aj)z
j
[ [z[
j
By comparison test,

j=0
(sin aj)z
j
converges, since

j=0
[z[
j
converges absolutely, for [z[ < 1.
Note that if a = , the series is zero.

j=0
(sin aj) for a = 2 so r=1indeed.
Exercise 14.

j=0
(sinh aj)z
j
=

j=0
_
e
aj
e
aj
2
_
z
j
=
1
2
_

j=0
(e
a
z)
j

j=0
_
z
e
a
_
j
_
; a > 0
If [z[ <
1
e
a
, then

sinh ajz
j
converges. So then the radius of convergence is r =
1
e
a
Exercise 15.

j=1
z
j
a
j
+b
j
. Assume a < b
z
j
b
j
(1 +
_
a
b
_
j
)
(ratio test)
z
j+1
b
j+1
(1 +
_
a
b
_
j+1
)
_
_
b
j
_
1 +
_
a
b
_
j
_
z
j
_
_
=
z
b
_
1 +
_
a
b
_
j
1 +
_
a
b
_
j+1
_
j

_
z
b
_
So then [z[ b converges (diverges) by ratio test.
If a = b,

j=1
z
j
2a
j
=
1
2

j=1
_
z
a
_
j
By comparison test with x
j
, if [z[ [a[, the series converges (diverges).
Exercise 16.

j=1
_
a
j
j
+
b
j
j
2
_
z
j
Use ratio test on each of the sums, separately.
(a[z[)
j+1
j + 1
j
(a[z[)
j
=
a[z[
1 +
1
j
j
a[z[
=[z[ <
1
a
then the series converges
(b[z[)
j+1
(j + 1)
2
j
2
(b[z[)
j
= b[z[
_
1
1 +
1
j
_
2
j
b[z[
=[z[ <
1
b
So if a b, then r = a; (b)
Exercise 17.
_
1
0
f
n
(x) =
_
1
0
nxe
nx
2
=
e
nx
2
2

1
0
=
e
n
1
2
n

1
2
However,
lim
n
nxe
nx
2
= 0
This example shows that the operations of integration and limit cannot always be interchanged. We need uniform conver-
gence. Exercise 18. f
n
(a) =
sin nx
n
lim
n
sin nx
n
= 0
f(x) = lim
n
f
n
(x)
f

n
(x) =
ncos nx
n
=lim
n
f

n
(0) = 1
This example shows that differentiation and limit cannot always be interchanged.
Exercise 19.

j=1
sin jx
j
2
= f(x)
206
sin jx
j
2

1
j
2
x R
by comparison test,

j=1
[ sin jx[
j
2
converges, so

j=1
sin jx
j
2
converges

sin jx
j
2

1
N
2
x R; j N

N
1
N
2
converges, so

sin jx
j
2
uniformly converges.
Then by Thm., since
sin jx
j
2
are continuous,

sin jx
j
2
is continuous.
Since

sin jx
j
2
uniformly converges.
_

0

sin jx
j
2
=

_

0
sin jx
j
2
=

cos jx
j
3

0
=

(1)
j
1
j
3
=
= 2

j=1
1
(2j 1)
3
Exercise 20. It is known that

j=1
cos jx
j
2
=
x
2
4

x
2
+

2
6
if 0 x 2
(1) x = 2

j=1
1
j
2
=
(2)
2
4

2
2
2
+

2
6
=

2
6
(2) As shown in Ex. 19,

cos jx
j
2
is uniformly convergent on R

_
cos jx
j
2
=

_
sin jx
j
3
_

/2
0
=

(1)
j+1
(2j 1)
3
_
x
2
4

x
2
+

2
6
=
_
1
3
x
3
4

x
2
4
+

2
6
x
_

/2
0
=
= ()
3
_
1
12(8)

1
16
+
1
12
_
= ()
3
1
32
11.13 Exercises - Properties of functions represented by real power series, The Taylors series generated by a function,
A sufcient condition for convergence of a Taylors series, Power-series expansions for the exponential and trigono-
metric functions, Bernsteins Theorem. Sufcient Condition for convergence.
Theorem 37 (Bernsteins Theorem). Assume x [0, r], f(x), f
(j)
(x) 0 j N.
Then if 0 x < r
(25)

f
(k)
(0)
k!
x
k
converges
Proof. If x = 0, were done. Assume 0 < x < r.
f(x) =
n

k=0
f
(k)
(0)
k!
x
k
+E
n
(x)
E
n
(x) =
x
n+1
n!
_
1
0
u
n
f
(n+1)
(x xu)du
F
n
(x) =
E
n
(x)
x
n+1
=
1
n!
_
1
0
u
n
f
(n+1)
(x xu)du
Since f
(n+1)
> 0, f
(n+1)
(x(1 u)) f
(n+1)
(r(1 u))
=F
n
(x) F
n
(r) =
E
n
(x)
x
n+1

E
n
(r)
r
n+1
207
For f(x) =

n
j=0
f
(j)
(0)
j!
x
j
+E
n
(x) =E
n
(x)
_
x
r
_
n+1
E
n
(r)
f(r) =
n

j=0
f
(j)
(0)
j!
r
j
+E
n
(r) E
n
(r) since f
(j)
(0) 0 j
So then 0 E
n
(x)
_
x
r
_
n+1
f(r)
n and f(t) will be some non-innite value, so E
n
(x)
n
0.
Exercise 1.

j=0
(1)
j
x
2j
Consider absolute convergence. lim
j
(x
2
)
j
= 0 If x
2
< 1
If [x[ = 1, then consider
x
2(2j)
x
2(2j+1)
= x
4j
(1 x)

j=0
(1 x)x
4j
= (1 x)

j=0
(x
4
)
j
Indeed

j=0
(1)
j
x
2j
converges for [x[ 1
Exercise 2.

j=0
x
j
3
j+1
=
1
3

j=0
_
x
3
_
j
The series converges for [x[ < 3
Exercise 3.

j = 0

jx
j
_
x
0

j=0
jt
j1
=

j=0
x
j
So the series converges for [x[ < 1. Note that we had used the integrability of power series.
Exercise 4.

j=0
(1)
j
jx
j
.
jx
j
je
j ln x
lim
j
je
j ln x
=
_
if x > 1
0 if 0 < x < 1
If x = 1,
(2j)x
2j
(2j + 1)x
2j+1
= x
2j
(2j +(2j + 1)x) = 1

(1) =
So

(1)
j
jx
j
converges only for 0 x < 1, [x[ < 1.
Exercise 5.

j=0
(2)
j j+2
j+1
x
j
=

j=0
(1)
j
_
j+2
j+1
_
(2x)
j
lim
j
_
j + 2
j + 1
_
(2x)
j
= lim
j
(2x)
j
if [2x[ < 1
So when [x[ <
1
2
, the series converges.
_
2j + 2
2j + 1
_

_
2j + 3
2j + 2
_
=
(2j + 2)(2j + 2) (2j + 3)(2j + 1)
(2j + 1)(2j + 2)
4j
2
+ 8j + 4 (4j
2
+ 5j + 3)
4j
2
+ 6j + 2
=
3j + 1
4j
2
+ 6j + 2
=
(3j + 1)/2
2j
2
+ 3j + 1
=
(3j + 1)/2
(2j + 1)(j + 1)
3j + 1
4j
2
+ 6j + 2
<
3j + 1
4j
2
+
4
3
<
3(j + 1/3)
4(j
2
+ 1/3)
<
3
4
_
1
j
_
+
1
12j
2
Thus, it diverges, by comparison test with
1
j
for x =
1
2
.
Theorem 38. Let f be represented by f(x) =

j=0
a
j
(x a)
j
in the (a r, a +r) interval of convergence
(1)

j=1
ja
j
(x a)
j1
also has radius of convergence r.
208
(2) f

(x) exists x (a r, a +r) and


(26) f

(x) =

j=1
ja
j
(x a)
j1
Exercise 6.

j=1
(2x)
j
j
=

j=1
e
j ln 2x
j
= [x[ <
1
2
itll converge, since by comparison test,
e
j ln 2x
j
<
1
j
2
if
0 < x <
1
2
.
Exercise 7.

j=0
(1)
j
(2j+1)
_
x
2
_
2j
.

j=0
x
2j
2(j + 1/2)2
2j
<

j=0
x
2j
2
2j+1
j
=
1
2

j=0
_
x
2
_
2j
_
1
j
_
=
1
2

j=0
e
2j ln
x
2
j
since by comparison test,
e
2j ln
x
2
j
<
1
j
2
if 0 < x < 2
If x = 2 ,
1
2(2j) + 1

1
2(2j + 1) + 1
=
1
4j + 1

1
4j + 3
=
2
(4j + 1)(4j + 3)

1
8
1
j
2
(converges by comparison test to

1
j
2
)
For [x[ < 2 ,

j=0
(1)
j
(2j+1)
_
x
2
_
2j
converges.
Exercise 8.

j=0
(1)
j
x
3j
j!
=

j=0
(x
3
)
j
j!
= e
x
3
, which converges x R
Exercise 9.

j=0
x
j
(j+3)!
=
1
x
3

j=0
x
j+3
(j+3)!
=
1
x
3

j=+3
x
j
j!
=
1
x
3
(e
x
x
2
/2 x 1) Thus, it converges for x R.
Exercise 10.

j=0
(x1)
j
(j+2)!
=
1
(x1)
2

j=0
(x1)
j+2
(j+2)!
=
1
(x1)
2
_

j=2
(x1)
j
j!
_
=
1
(x1)
2
_
e
x1
(x 1) 1
_
=
e
x1
x
(x 1)
2
Exercise 11. a
x
= e
x log a
=

j=0
(log ax)
j
j!
(log ax)
j+1
(j+1)!
j!
(log ax)
j
=
(log ax)
j+1
j
0. By ratio test,

j=0
(log ax)
j
j!
converges for all x.
Exercise 12.
sinh x =
e
x
e
x
2
=
1
2
_
_

j=0
x
j
j!

j=0
(x)
j
j!
_
_
=

j=0
x
2j+1
(2j + 1)!
x
2j+3
(2j + 3)!
(2j + 1)!
x
2j+1
=
x
2
(2j + 3)(2j + 2)
j
0
Exercise 13.
sin
2
x =
1 cos 2x
2
=
1

j=0
(2x)
2j
(2j)!
(1)
j
2
=

j=1
2
2j1
x
2j
(1)
j+1
(2j)!
2
2j+1
x
2j+2
(2j + 2)!
(2j)!
2
2j1
x
2j
=
4x
2
(2j + 2)(2j + 1)
j
0
So the series converges x
Exercise 14.
1
1
x
2
=

j=0
_
x
2
_
j
1
2x
=

j=0
x
j
2
j+1
x
j+1
2
j+2
2
j+1
x
j
=
x
2
< 1 = (the series converges for [x[ < 2, by ratio test)
If x = 2, the series would diverge
If x = 2
1
2

j=0
(2)
j
2
j
=
1
2

j=0
(1)
j
= 0 but
1
2 (2)
=
1
4
x
209
Exercise 15. e
x
2
=

j=0
(1)
j
x
2j
j!
x
2j+2
(j+1)!
j!
x
2j
=
x
2
j+1
j
0
Exercise 16. sin 3x = sin 2xcos x + sin xcos 2x = 3 sin x 4 sin
3
x.
sin
3
x =
3 sin x sin 3x
4
=
3
4
_
_

j=0
x
2j+1
(1)
j
(2j + 1)!

j=0
(3x)
2j+1
(1)
j
(2j + 1)!
_
_
=
=
3
4
_
_

j=0
(1)
j
x
2j+1
(1 3
2j+1
)
(2j + 1)!
_
_
=
3
4
_
_

j=0
(1)
j+1
(3
2j+1
1)x
2j+1
(2j + 1)!
_
_
Exercise 17. log
_
1+x
1x
=
1
2
(log (1 +x) log (1 x)) =
1
2
_

j=1
(+x)
j
j
(1)
j1

j=1
(x
j
)(1)
j
j
_
ln (1 +x) =

j=0
(x)
j+1
j + 1
(1) =

j=1
(x)
j
j
(1) ln (1 x) =

j=0
x
j+1
j + 1
=

j=1
x
j
j
=

j=1
((1)
j
+ 1)x
j
j
=

x
2j+1
2j + 1
x
2j+3
2j + 3
2j + 1
x
2j+1
j
x
2
[x
2
[ < 1, converges, with radius of convergence of 1
Exercise 18.
3x
1+x2x
2
=
1
1x

1
1+2x
=

j=0
x
j
j

j=0
(2x)
j
j
=

j=0
x
j
j
(1 (2)
j
)
x
j+1
[(1 (2)
j+1
)[
j + 1
j
[x
j
(1 (2)
j
)[
= x

_
1
(2)
j
+ 2
_
_
1
(2)
j
1
_

j
2x < 1
[x[ <
1
2
For x =
1
2
x
2j
2j
(1 (2)
2j
) +
x
2j+1
2j + 1
(1 (2)
2j+1
) =
x
2j
((2j + 1)(1 2
2j
) +x(1 + 2
2j+1
)(2j))
(2j)(2j + 1)
_
1
2
_
2j
(2j + 1 (2j + 1)2
2j
+ 2jx + 2
2j+2
jx)
(2j)(2j + 1)
j

(2j + 1) + 4j
(2j)(2j + 1)
=
2j 1
(2j)(2j + 1)
0
So
3x
1+x2x
2
converges for [x[
1
2
Exercise 19.
125x
65xx
2
=

j=0
_
1 +
(1)
j
6
j
_
x
j
([x[ < 1).
12 5x
6 5x x
2
=
5x 12
(x + 6)(x 1)
=
1
1 x
+
6
6 +x
=

j=0
x
j
+

j=0
_
x
6
_
j
=

j=0
x
j
(1 +
_
1
6
_
j
)
[x[ < 1 since for x = 1, lim
j
(1 +
_
1
6
_
j
) = 1.
Exercise 20.
1
x
2
+x+1
=
2

j=0
sin
2(j+1)
3
x
j
([x[ < 1)
x
3
1 = (x 1)(x
2
+x + 1)
1
x
2
+x + 1
=
x 1
x
3
1
=
1 x
1 x
3
1
1 x
3
=

j=0
(x
3
)
j
1 x
1 x
3
=

j=0
_
(x
3
)
j
x
3j+1
_
By induction, it could be observed that x
3j
, x
3j+1
, 0 appear in sequences of 3 terms.
210
2

3
sin
2(j+1)
3
accounts for this.
=
2

j=0
sin
2(j+1)
3
x
j
[x[ < 1
sin
2(j+1)
3
x
j
< x
j
(so by comparison test to

x
j
, the radius of convergence is 1 )
Exercise 21.
1
1 x
=

j=0
x
j
;
_
1
1 x
_

=
1
(1 x)
2
=

j=1
jx
j1
=

j=0
(j + 1)x
j
1
(1 x)(1 x
2
)
=
1/4
1 x
+
1/2
(1 x)
2
+
1/4
1 +x
=
1
4
_
_

j=0
(x
j
+ (x)
j
) + 2

j=1
jx
j1
_
_
x
(1 x)(1 x
2
)
=
1
4
_
_

j=0
x
j+1
(1 + (1)
j
) + 2

j=1
jx
j
_
_
=
1
4

j=1
x
j
(1 + (1)
j1
+ 2j)
Exercise 22.
sin
_
2x +

4
_
=

j=0
(2x)
2j+1
(2j + 1)!
(1)
j
; cos 2x =

j=0
(2x)
2j
(2j)!
(1)
j
sin 2x =

j=0
(2x)
2j+1
(2j + 1)!
(1)
j
cos 2x =

j=0
(2x)
2j
(2j)!
(1)
j

j=0
a
j
x
j
=

2
2
(sin 2x + cos 2x)
For j = 98, a
j
=
2
98
(1)
49
98!
_

2
2
_
=
2
98
98!
_

2
2
_
Exercise 23.
f(x) = (2 +x
2
)
5/2
f

(x) =
5
2
(2 +x
2
)
3/2
(2x) = 5x(2 +x
2
)
3/2
f

(x) = 5(2 +x
2
)
3/2
+
15
2
x(2 +x
2
)
1/2
(2x) = 5(2 +x
2
)
3/2
+ 15x
2
(2 +x
2
)
1/2
f

(x) =
15
2
(2 +x
2
)
1/2
(2x) + 30x(2 +x
2
)
1/2
+
15x
2
2
(2 +x
2
)
1/2
(2x)
f

(x) = 15(2 +x
2
)
1/2
+
15
2
(2 +x
2
)
1/2
(2x)x + 30(2 +x
2
)
1/2
+
30x
2
(2 +x
2
)
1/2
(2x)+
+ 45x
2
(2 +x
2
)
1/2
+ 15x
3
_

1
2
_
(2 +x
2
)
3/2
(2x)
2
5/2
+ 0x +
5(2
3/2
)x
2
2!
+
0x
3
3!
+
45
4!

2x
4
Exercise 24. f(x) = e
1/x
2
if x ,= 0 and let f(0) = 0
(1)
f(x) =

j=0
_
1
x
2
_
j
j!
= 1 +
1
x
2
+

j=2
_
1
x
2
_
j
j!
=

j=0
(1)
j
x
2j
j!
f
(k)
=

j=0
(1)
j
(2j)(2j 1) . . . (2j (k 1))
j!
x
2j
=

j=0
(1)
j
(2j)!
(2j k)!j!
x
2j
211
Use ratio test :
(2(j + 1))!
(2j 2 k)!(j + 1)!
j!(2j k)!
(2j)!
x
2(j+1)
x
2j
=
(2j k)(2j k 1)
(j + 1)(2j)(2j 1)
x
2
j
0
Thus, by ratio test, every order of derivative exists for every x on the real line since the series representing the
derivative converges for every x.
(2) f(x) =

j=0
x
2j
j!
. There are no nonzero terms of positive power, i.e. no x
j
; j 1.
=f
(j)
(0) = 0 j 1
11.16 Exercises - Power series and differential equations, binomial series.
Exercise 1. For (1 x
2
)y

2xy

+ 6y = 0,
y =

j=0
a
j
x
j
y

j=1
ja
j
x
j1
y

j=2
j(j 1)a
j
x
j2
=

j=0
(j + 2)(j + 1)a
j+2
x
j
f(0) = 1 =a
0
= 1 f

(0) = 0 =a
1
= 0
2(1)a
2
+ 3(2)a
3
x +2(1)a
1
x + 6a
0
+ 6a
1
x +

j=2
((j + 2)(j + 1)a
j+2
j(j 1)a
j
2ja
j
+ 6a
j
)x
j
=
= 2a
2
+ 6 + 6a
3
x +

j=2
((j + 2)(j + 1)a
j+2
+a
j
(j + 3)(j 2))x
j
=
a
2
= 3
a
3
= 0
a
j+2
=
(j + 3)(j 2)
(j + 2)(j + 1)
a
j
For j = 2, a
4
= 0, so then a
j+2
= 0 for j = 2, 4, . . . . Likewise, since a
3
= 0, then a
j+2
= 0 for j = 3, 5, . . . .
=f(x) = 1 3x
2
Exercise 2. Using the work from above, then for (1 x
2
)y

2xy

+ 12y = 0
f(0) = 0 =a
0
= 0 f

(0) = 2 =a
1
= 2
2(1)a
2
+ 3(2)a
3
x +2(1)a
1
x + 12a
0
+ 12a
1
x +

j=2
((j + 2)(j + 1)a
j+2
j(j 1)a
j
2ja
j
+ 12a
j
)x
j
=
= 2a
2
+ 6a
3
x +4x + 0 + 24x +

j=2
((j + 2)(j + 1)a
j+2
a
j
(j + 4)(j 3))x
j
=
a
2
= 0
a
3
= 10/3
a
j+2
=
(j + 4)(j 3)
(j + 2)(j + 1)
a
j
For j = 3, a
5
= 0, so then a
j+2
= 0 for j = 3, 5, . . . . Likewise, since a
2
= 0, then a
j+2
= 0 for j = 2, 4, . . . .
=f(x) = 10/3x
3
+ 2
Exercise 3. f(x) =

j=0
x
4j
(4j)!
;
d
4
y
dx
4
= y
212
(x
4
)

= 4x
3
(x
4
)

= 12x
2
(x
4
)

= 24x
(x
4
)

= 24
_
x
4j
(4j)!
_

=
x
4j1
(4j 1)!
_
x
4j
(4j)!
_

=
x
4j2
(4j 2)!
_
x
4j
(4j)!
_

=
x
4j3
(4j 3)!
_
x
4j
(4j)!
_

=
x
4j4
(4j 4)!

j=1
x
4j4
(4j 4)!
= y

j=0
x
4j
(4j)!
= f(x)
To test convergence, use the ratio test
x
4j+4
(4j + 4)!
(4j)!
x
4j
=
x
4
(4j + 4)(4j + 3)(4j + 2)(4j + 1)
j
0 x R
So the series converges on R.
Exercise 4. f(x) =

j=0
x
j
(j!)
2
xy

+y

y = 0
y

j=1
jx
j1
(j!)
2
=

j=0
(j + 1)x
j
((j + 1)!)
2
y

j=1
(j + 1)jx
j1
((j + 1)!)
2

j=1
_
(j + 1)j
((j + 1)!)
2
+
(j + 1)
((j + 1)!)
2

1
(j!)
2
_
x
j
+
1
1!
1 =

j=1
_
1
(j!)
2

1
(j!)
2
_
= 0
Exercise 5. f(x) = 1 +

j=1
147...(3j2)
(3j)!
x
3j
; y

= x
a
y +b (Find a and b )
f

j=1
1 4 7 . . . (3j 2)
(3j 1)!
x
3j1
f

j=1
1 4 7 . . . (3j 2)
(3j 2)!
x
3j2
=

j=1
1 4 7 . . . (3j 5)
(3j 3)!
x
3j2
=
= x +

j=2
1 4 7 . . . (3j 5)
(3j 3)!
x
3j2
= x +

j=1
1 4 7 . . . (3j 2)
(3j)!
x
3j+1
x
a
f = x
a
+

j=1
1 4 7 . . . (3j 2)
(3j)!
x
3j+a
So then a = 1; b = 0 .
1 4 7 . . . (3j + 1)
(3j + 3)!
x
3j+3
_
(3j)!
1 4 7 . . . (3j 2)
_
1
x
3j
=
1(3j + 1)
(3j 2)(3j + 3)(3j + 2)(3j + 1)
x
3
j
0
So the series converges for all x.
Exercise 6. f(x) =

j=0
x
2j
j!
; y

= 2xy.
f

j=1
2jx
2j1
j!
= 2

j=1
x
2j1
(j 1)!
= 2

j=0
x
2j+1
j!
= 2xf
x
2j+2
(j + 1)!
j!
x
2j
=
x
2
j + 1
j
0 x
By ratio test, f converges x R.
Exercise 7. f(x) =

j=2
x
j
j!
y

= x +y
213
f

j=2
x
j1
(j 1)!
=

j=1
x
j
j!
= x +y
x
j+1
(j + 1)!
j!
x
j
=
x
j
j
0
So the series converges x R by ratio test.
Exercise 8.
f(x) =

j=0
(1)
j
(kx)
2j
(2j)!
f

j=1
(1)
j
(kx)
2j1
k
(2j 1)!
=

j=0
(1)
j+1
(kx)
2j+1
k
(2j + 1)!
f

j=1
(1)
j+1
(kx)
2j
(2j)!
k
2
f

k
2
f = 0
(kx)
2j+1
(2j + 1)!
(2j)!
(kx)
2j
=
kx
2k + 1
j
0 by ratio test, f converges x R.
Exercise 9.
f

j=1
(3x)
2j1
(2j 1)!
9 =

j=0
9(3x)
2j+1
(2j + 1)!
9(f x) = 9(x +

j=0
(3x)
2j+1
(2j + 1)!
x)
(3x)
2j+3
(2j + 3)!
(2j + 1)!
(3x)
2j+1
=
9x
2
(2j + 3)(2j + 2)
j
0
(by ratio test, f converges x R )
Exercise 10. J
0
(x) =

j=0
(1)
j x
2j
(j!)
2
2
2j
J
1
(x) =

j=0
(1)
j x
2j+1
j!(j+1)!2
2j+1
.
(1)
x
2j+2
((j + 1)!)
2
2
2j+2
(j!)
2
2
2j
x
2j
=
x
2
(j + 1)
2
4
j
0 by ratio test, f converges x R
x
2j+3
(j + 2)!2
2j+3
j!2
2j+1
x
2j+1
=
x
2
(j + 2)(j + 1)4
j
0 by ratio test, f converges x R
(2)
J

0
(x) =

j=1
(1)
j
x
2j1
(j 1)!(j!)2
2j1
=

j=0
(1)
j+1
x
2j+1
j!(j + 1)!2
2j+1
= J
1
(x)
(3)
j
0
(x) = xJ
0
(x) =

j=0
(1)
j
x
2j+1
(j!)
2
2
2j
j
1
(x) = xJ
1
(x) =

j=0
(1)
j
x
2j+2
j!(j + 1)!2
2j+1
j

1
=

j=0
(1)
j
x
2j+1
(j!)
2
2
2j
=j
0
= j

1
Exercise 11. x
2
y

+xy

+ (x
2
n
2
)y = 0.
214
n = 0 =x
2
y

+xy

+ (x
2
)y = 0
J
0
=

j=0
(1)
j
x
2j
(j!)
2
2
2j
=

j=1
(1)
j1
x
2j2
((j 1)!)
2
2
2j2
;
J

0
=

j=1
(1)
j
x
2j1
j!(j 1)!2
2j1
;
J

0
=

j=1
(1)
j
x
2j2
j!(j 1)!
(2j 1)
2
2j1

j=1
(1)
j
_
(2j 1)
j!(j 1)!2
2j1
+
1
j!(j 1)!2
2j1
+
2j
((j 1)!)j!2
2j1
_
= 0
n = 1 =x
2
y

+xy

+ (x
2
1)y = 0
J
1
(x) =

j=0
(1)
j
x
2j+1
j!(j + 1)!2
2j+1
=
x
2
+

j=1
(1)
j
x
2j+1
j!(j + 1)!2
2j+1
=
x
2
+

j=1
(1)
j1
x
2j1
(j 1)!(j)!2
2j1
J

1
=
1
2
+

j=1
(1)
j
(2j + 1)x
2j
j!(j + 1)!2
2j+1
J

1
=

j=1
(1)
j
(2j + 1)(2j)x
2j1
(j!)(j + 1)!2
2j+1
x
2
J

1
+xJ

1
+ (x
2
1)J
1
=
=

j=1
x
2j+1
__
(1)
j
(2j + 1)
(j!)(j + 1)!2
2j+1
_
((2j) + (1)) +
(1)
j
(1)
(j 1)!j!
_
j + 1
j + 1
__
j
j
__
1
2
2j1
__
2
2
2
2
_

(1)
j
(j + 1)!(j!)2
2j+1
_
+
+
x
2

x
2
=
=

j=1
_
(1)
j
x
2j1
(j!)(j + 1)!2
2j+1
_
((2j + 1)(2j + 1) + (1)(2j)(2j + 2) 1) = 0
Exercise 12. y

= x
2
+y
2
; y = 1 when x = 0.
215
y

(0) = 0 + 1
2
= 1
y = a
0
+a
1
x +a
2
x
2
+

j=3
a
j
x
j
y
2
= a
2
0
+a
2
1
x
2
+a
2
2
x
4
+
_
_

j=3
a
j
x
j
_
_
2
+
+ 2a
0
a
1
x + 2a
0
a
2
x
2
+ 2a
0

j=3
a
j
x
j
+
+ 2a
1
a
2
x
3
+ 2a
1

j=3
a
j
x
j+1
+ 2a
2

j=3
a
j
x
j+2
y

= a
1
+ 2a
2
x +

j=3
ja
j
x
j1
a
1
= 1 since y

(0) = 1
Consider the rst few terms of x
2
+y
2
a
2
0
+ 2a
0
a
1
x +a
2
1
x
2
+ 2a
0
a
2
x
2
+x
2
= a
1
+ 2a
2
x + 3a
3
x
2
=
a
1
= 1 = a
2
0
= a
0
= 1
2a
2
= 2a
0
a
1
= a
2
= 1
3a
3
= a
2
1
+ 2a
0
a
2
+ 1 = 4 = a
3
=
4
3
Exercise 13. y

= 1 +xy
2
with y = 0 when x = 0 =a
0
= 0
y =

j=1
a
j
x
j
y =

j=1
ja
j
x
j1
=

j=0
(j + 1)a
j+1
x6j
y = a
1
x +a
2
x
2
+a
3
x
3
+

j=4
a
j
x
j
y
2
= a
2
1
x
2
+a
2
2
x
4
+a
3
x
6
+
_
_

j=4
a
j
x
j
_
_
2
+
+ 2a
1
a
2
x
3
+ 2a
1
a
3
x
4
+ 2a
1

j=4
a
j
x
j+1
+
+ 2a
2
a
3
x
5
+ 2a
2

j=4
a
j
x
j+2
+ 2a
3

j=4
a
j
x
j+3
a
1
= 1
x : 2a
2
= 0 =a
2
= 0 x
2
: 3a
3
= 0 =a
3
= 0
x
3
: 4a
4
= 1
2
= a
4
=
1
4
x
4
: 5a
5
= 0 =a
5
= 0 x
5
: 6a
6
= 0 =a
6
= 0
x
6
: 7a
7
= 2a
1
a
4
+ 2a
2
a
3
= a
7
=
1
14
x
7
: 8a
8
= 0 + 2a
2
a
4
= 0 =a
8
= 0 x
8
: 9a
9
= 0 =a
9
= 0
x
9
: 10a
10
=
_
1
4
_
2
+ 2(1)
1
14
= a
10
=
23
1120
Exercise 14. y

= x +y
2
y = 0 when x = 0 =a
0
= 0
216
y

(0) = 0 + 0 = 0 =a
1
= 0
y =

j=2
a
j
x
j
y

j=2
ja
j
x
j1
=

j=1
(j + 1)a
j+1
x
j
y
2
=
_
_
a
2
x
2
+a
3
x
3
+a
4
x
4
+

j=5
a
j
x
j
_
_
2
=
= a
2
x
4
+a
2
3
x
6
+a
2
4
x
8
+
_
_

j=5
a
j
x
j
_
_
2
+
+ 2a
2
a
3
x
5
+ 2a
2
a
4
x
6
+ 2a
2
x
2

j=5
a
j
x
j
+ 2a
3
a
4
x
y
+ 2a
3
x
3

j=5
a
j
x
j
+ 2a
4
x
4

j=5
a
j
x
j
y

= x +y
2
x : 2a + 2 = 1 + 0 = a
2
=
1
2
x
2
: 3a
3
= 0 =a
3
= 0 x
3
: 4a
4
= 0 =a
4
= 0
x
4
: 5a
5
= a
2
2
= a
5
=
1
20
x
5
: 6a
6
= 0 =a
6
= 0x
6
: 7a
7
= 0 =a
7
= 0
x
7
: 8a
8
= 2(
1
2
)(
1
20
) = a
8
=
1
160
x
8
: 9a
9
= 0 =a
9
= 0 x
9
: 10a
10
= 0 =a
10
= 0
x
10
: 11a
11
= 2(
1
2
)(
1
160
) +
_
1
20
_
2
= a
11
=
7
8800
Exercise 15. y

= y
=

j=0
(j + 1)a
j+1
x
j
=

j=0
a
j
x
j
a
j+1
=
aj
(j+1)
a
j
=

j
j!
a
0
x (by induction)
Exercise 16. y

= xy
y

j=0
(j + 2)(j + 1)a
j+2
x
j
=
= 2a
2
+

j=0
(j + 3)(j + 2)a
j+3
x
j+1
=

j=0
a
j
x
j+1
=a
2
= 0 and a
j+3
=
a
j
(j + 3)(j + 2)
j = 0 a + 3 =
a
0
3 2
j = 3 a
6
=
a
3
6 5
j = 1 a
4
=
a
1
4 3
j = 4 a
7
=
a
1
7 6 4 3
a
3j
=
a
0
(3j)!
j1

k=0
(3k + 1) ; a
3j+1
=
a
1
(3j + 1)!
j1

k=0
(3k + 2)
Exercise 17. y

+xy

+y = 0
217
y =

j=0
a
j
x
j
y

j=1
ja
j
x
j1
y

j=2
j(j 1)a
j
x
j2
=

j=0
(j + 2)(j + 1)a
j+2
x
j
y

+xy

+y =

j=1
x
j
((j + 2)(j + 1)a
j+2
+ja
j
+a
j
) = 0 =a
j+2
=
a
j
(j + 2)
a
2
=
a
0
2
a
4
=
a
2
4
a
3
=
a
1
3
a
5
=
a
3
5
=
a
1
15
a
2j
=
(1)
j
a
0
(2j)!!
a
2j+1
=
a
1
(2j + 1)!!
(1)
j
(could be shown by induction)
Exercise 18. Recall that
y =

j=0
a
j
x
j
y

j=1
ja
j
x
j1
=

j=0
(j + 1)a
j+1
x
j
Knowing this, we could cleverly observe that e
2x
=

j=0
(2a
j
+ (j + 1)a
j+1
)x
j
is actually a differential equation!!!
=e
2x
= y

+ 2y
Solving this ODE using y(x) = e
A(x)
__
x
a
Q(t)e
A(t)
dt +y(a)
_
where A(x) =
_
x
a
P(t)dt,
y = e
2x
(x + 1)
We had obtained the necessary initial conditions to solve this ODE from the information given, that a
0
= 1, so that y(0) = 1.
By doing some simple computation and comparison of powers with e
2x
, then a
1
= 2, a
2
= 2, a
3
= 4/3
Exercise 19. cos x =

j=0
a
j
(j + 2)x
j
for f(x) =

j=0
a
j
x
j
.
Using cos x =

j=0
(x)
2j
(2j)!
(1)
j
representation, we can immediately conclude that for odd terms, a
2j+1
= 0 and by
matching powers of x,
a
2j
(2j + 2) = (1)
j
1
(2j)!
a
5
= 0
a
6
(6 + 2) =
(1)
3
6!
= a
6
=
7
8!
Now notice that for cos x =

j=0
a
j
(j + 2)x
j
=

j=1
ja
j
x
j
+ 2

j=0
a
j
x
j
is actually a differential equation, cos x =
xy

+ 2y. We can solve this rst-order ODE using


y(x) = e
A(x)
__
x
a
Q(t)e
A(t)
dt +y(a)
_
where A(x) =
_
x
a
P(t)dt. Then solving y

+
2y
x
=
cos x
x
,
y =
1
x
2
(xsin x + cos x (a sin a + cos a) +b)
218
Plugging 0 as a good guess back into the ODE, cos 0 = 1 = y(0)(2) =y(0) =
1
2
With this initial condition, we get
f(x) =
sin x
x
+
cos x 1
x
2
if x ,= 0
So f(0) =
1
2
and f() =
2

2
Exercise 20.
(1)
(1 x)
1/2
=

j=0
_
1/2
j
_
(x)
j
=
= 1 +
1
2
x +
_
1
2
_ _
3
2
_
2
x
2
+
_
1
2
_ _
3
2
_ _
5
2
_
3!
x
3
+
_
1
2
_ _
3
2
_ _
5
2
_ _
7
2
_
4!
x
4
+
+
_
1
2
_ _
3
2
_ _
5
2
_ _
7
2
_ _
9
2
_
5!
x
5
+ =
= 1 +
1
2
x +
3
8
x
2
+
5
8
x
3
+
35
128
x
5
+
63
256
x
5
+. . .
(2) To make the notation clear, (1 x)
1/2
=

j=0
_
1/2
j
_
(x)
j
=

j=0
b
j
x
j
=

j=0
a
j
Now
_

j+1
_
_

j
_ =
( 1) . . . ( (j + 1) + 1)
(j + 1)!
j!
( 1) . . . ( j + 1)
=
( j)
(j + 1)
So for =
1
2
,
a
j+1
a
j
=
_
1/2 +j
j + 1
_
(x) < x
Using this, we further nd that
b
j+1
< b
j
1
50
b
j+2
< b
j+1
1
50
< b
j
_
1
50
_
2
For x =
1
50
. So by induction, b
n+j
< b
n
_
1
50
_
j
r
n
=

j=1
a
n+j
<

j=1
a
n
_
1
50
_
j
= a
n
1/50
1 1/50
=
a
n
49
r
n
<
a
n
49
(3) Note that (1 x)
1/2
=
_
1
1
50
_
1/2
=
_
49
50
_
1/2
=
5

2
7
7
5
_
1
1
50
_
1/2
= 1 +
1
100
+
3
2
_
1
2(50)
_
2
+
5
2
_
1
2(50)
_
3
+
35
8
_
1
2(50)
_
4
+
63
8
_
1
2(50)
_
5

2 1.4142135624
Exercise 21.
(1)
1732
1000
_
1
176
3000000
_
1/2
=
1732
1000
_
3000000
2999824
_
1/2
Obviously, (3000000)
1/2
= 1000

3 so that we have 1732 (3/2999824)


1/2
.
With long multiplication, we could show easily that 1732 1732 = 2999824 (its harder to divide). So then
1732
1000
_
1
176
3000000
_
1/2
=

3
(2)
219
Exercise 22. arcsin x =
_
1

1x
2
(1 x
2
)
1/2
=

j=0
_

j
_
(x
2
)
j
= 1 +

j=1
_

j
_
(x
2
)
j
=arcsin x = x +

j=1
_
1/2
j
_
(1)
j
(2j + 1)
x
2j+1
_
1
2
_ _
3
2
_
. . .
_
1
2
j + 1
_
j(j 1) . . . (2)(1)
= (1)
j
(1)(3) . . . (1 + 2j 2)
(2j)!!
= (1)
j
(2j 1)!!
(2j)!!
= arcsin x = x +

j=1
(2j 1)!!
(2j)!!
x
2j+1
2j + 1
13.21 Exercises - The conic sections, Eccentricity of conic sections, Polar equations for conic sections.
Exercise 1. F is in the positive half-plane determined by N.
|X F| = ed(X, L)
|X F| = e[(X F) N +d[
Exercise 2.
(1)
|X F| = ed(X, L)
|X F| = e[(X F) N +d[
F = 0 =|X| = e[(X N) +d[; r = e[r cos +d[
=r = e(r cos +d) =r =
ed
1 e cos
(2) The right branch for the hyperbola is given by r =
ed
1e cos
because X N > 0. The left branch for e > 1,
|X F| = ed(X, L) = e[(X F) N +d[ =
= e[X N +d[ = e(d +r cos ) = r
r =
ed
(1 +e cos )
Exercise 3. For points below the horizontal directrix,
|X F| = ed(X, L)
F = 0 =|X| = ed(X, L) = e([(X F) N d[) = e[X N d[ = e[r sin d[
Now Thm. 13.18 says r =
ed
e cos + 1
if 0 < e 1
=r = e(d r sin ) =r =
ed
1 +e sin
For the right or upper-half branch of a hyperbola.
|X F| = e([(X F) N d[) = e(r sin d) = r
r =
ed
1 e sin
=
ed
e sin 1
Exercise 4. |X F| = ed(X, L); |X| = e[(X F) N d[ = e[r cos d[ = e(d r cos ) =r =
ed
1+e cos
e = 1, d = 2 .
Exercise 5. r =
3
1+
1
2
cos
=
6(
1
2
)
1+
1
2
cos
. e =
1
2
; d = 6
Exercise 6. r =
6
3+cos
=
2
1+
1
3
cos
. e =
1
3
; d = 6.
Exercise 7. r =
1
1
2
+cos
.
220
ed(X, L) = |X F| = e[(X F) N d[ = e[r cos d[ = er cos ed = r
ed
1 e cos
= r =
ed
e cos 1
So for r =
2
2 cos 1
, e = 2, d = 1.
Exercise 8. r =
4
1+2 cos
e = 2, d = 2.
Exercise 9. r =
4
1+cos
e = 1 d = 4.
Exercise 10. 3x + 4y = 25 =
3
5
x +
4
5
y = 5. N =
_
3
5
,
4
5
_
.
L = x = P +tA, N X = N P.
To nd the distance from the focus, at the origin, to the directrix,
dN = P +tA; dN N = d = N P
So for this problem, d = 5.
r = |X F| = ed(X, L) = e[(X F) N d[ = e[X N d[ =

3
5
r cos +
4
5
r sin 5

r =
1
2
_
5
3
5
r cos
4
5
r sin
_
r =
5/2
1 +
3
10
cos +
4
10
sin
Exercise 11. e = 1, 4x + 3y = 25
4
5
x +
3
5
y = 5; N =
_
4
5
,
3
5
_
.
d = 5.
|X F| = ed(X, L) = e[(X F) N d[ = e

4
5
r cos +
3
5
r sin 5

r = 5 r
_
4
5
cos +
3
5
sin
_
r =
5
1 +
4
5
cos +
3
5
sin
Exercise 12. e = 2, hyperbola, so theres 2 branches.
1

2
x +
1

2
y =
1

2
L = x = P +tA X N = N P
dN = P +tA; dN N = d = N P =
1

2
Note that the sign of d here tells you what side the focus, at the origin, lies on.
|X F| = ed(X, L) = |X| = e[(X F) N d[ = e(d
1

2
r cos
1

2
r sin )
r =
2/

2
1 +
2

2
cos +
2

2
sin
But for the right side branch,
|X F| = ed(X, L) = |X| = e[(X F) N d[ = e(d
1

2
r cos
1

2
r sin )
r =
2/

2
1
2

2
cos
2

2
sin
Exercise 13. e = 1 parabola.
221
(1)
|X F| = |X| = ed(X, L) = 1[(X F) N d[ = d X N = d r cos

3
d = r
_
3
2
_
=
3
2
10
8
mi
r =
3
2
10
8
mi
1 + cos
= 0, r =
3
4
10
6
mi
(2) Focus is in the positive half-plane determined by N.
|X F| = |X| = ed(X, L) = [(X F) N +d[ = r cos +d
d = r(1 cos ) = 10
8
mi(1 cos

3
) =
1
2
10
8
mi
r =
d
1 cos
=
1
2
10
8
mi
1 cos
r( = ) =
1
4
10
8
mi
13.24 Exercises - Conic sections symmetric about the origin, Cartesian equations for the conic sections.
Quick Review.
Consider symmetry about the origin.
|X F| = ed(X, L) = e[(X F) N d[ = e[X N F N d[ = [eX N e(F N +d)[
|X F|
2
= |X|
2
2X F +|F|
2
= e
2
(X N)
2
2aeX N +a
2
X X; X F = aeX N
X = (F aeN) = 0 =F = aeN; F N = ae; a =
ed
1 e
2
F =
e
2
d
1 e
2
N
=|X|
2
+ (ae)
2
= e
2
(X N)
2
+a
2
if X = aN; |X|
2
+ (ae)
2
= e
2
(X N)
2
+a
2
is satised
if X = bN

; b
2
+ (ae)
2
= e
2
(0) +a
2
b
2
= a
2
(1 e
2
)
Exercise 1. b
2
= a
2
(1 e
2
)
x
2
100
+
y
2
36
= 1
_
1
b
2
a
2
= e = e =
4
5
.
[F[ = [aeN[ = 10
_
4
5
_
= 8. f = (8, 0). (0, 0) center. Vertices (0, 6).
Exercise 2.
y
2
100
+
x
2
36
= 1.
4
5
= e; f = (0, 8).
(0, 0) center; vertices (6, 0), (0, 10).
Exercise 3.
(x2)
2
16
+
(y3)
2
9
= 1. Center (2, 3). [F[ = ae = 4

7
4
=

7.
_
1
b
2
a
2
=
_
1
9
16
=

7
4
= e; (2 +

7, 3), (2

7, 3) foci.
Vertices (6, 3), (2, 3), (2, 6), (2, 12).
Exercise 4.
x
2
(
25
9
)
+y
2
= 1. Center x = (0, 0). [F[ = ae =
_
5
3
_ _
4
5
_
=
4
3
.
e =
_
1
b
2
a
2
=
_
1
9
25
=
4
5
. Foci: (
4
3
, 0). Vertices (
5
3
, 0), (0, 1).
Exercise 5.
y
2
(1/4)
+
x
2
(1/3)
= 1
[F[ = ae =
1

3
_
1
2
_
=
1
2

3
.
_
1
1/4
1/3
=
1
2
= e. Foci:
_
1
2

3
, 0
_
. Center (0, 0).
Vertices (1/

3, 0), (0, 1/2).


Exercise 6. Center (1, 2).
222
_
1
b
2
a
2
=
_
1
16
25
=
3
5
= e; [F[ = ae = 5
3
5
= 3.
Foci: (1, 1), (1, 5).
Vertices: (1, 3), (1, 7), (3, 2), (5, 2).
Exercise 7. F = ae =
3
4
. a = 1, e =
3
4
. b
2
= a
2
(1 e
2
); b
2
= 1
_
1
4
_
.
x
2
+ 4y
2
= 1 .
Exercise 8. 2a = 4. a
2
= 4. 2b = 3. b
2
= 9/4. =
(x+3)
2
4
+
(y4)
2
4
= 1
Exercise 9.
(x+3)
2
9/4
+
(y4)
2
4
= 1.
Exercise 10. 2a = 6, a = 3.
(x+4)
2
9
+
(y2)
2
1
= 1.
Exercise 11. 2a = 10, a = 5. [F[ = ae = 5e = 4 e = 4/5. b
2
= a
2
(1 e
2
) = 25
_
1
16
25
_
= 9.
Exercise 12.
(x2)
2
a
2
+
(y1)
2
b
2
= 1;
a = 4 from (6, 1). b = 2 from (2, 3). =
(x2)
2
4
2
+
(y1)
2
4
= 1
Exercise 13. b
2
= a
2
(1 e
2
).
x
2
100

y
2
64
= 1; b
2
= 100(1 e
2
) = 64. 1 +
64
100
= e
2
.
Center (0, 0). e =
2

41
10
=

41
5
.
Vertices; (10, 0). F = ae = 2

41. Foci: (2

41, 0).
x
2
100
=
y
2
64
+ 1
x,y
; y =
4
5
x
Exercise 14.
y
2
100

x
2
64
= 1; Center (0, 0), a
2
= 100; b
2
= 64.
b
2
= a
2
(1 e
2
). e =

41
5
. Vertices (0, 10). F = ae = (0, 2

41).
x
2
64
+ 1 =
y
2
100
x,y

5
4
x = y.
Exercise 15.
(x+3)
2
4
(y 3)
2
= 1.
Center (3, 3). e =
_
1
b
2
a
2
=
_
1
1
4
=

5
2
.
Foci: ae = 2

5
2
=

5. (3 +

5, 3), (3

5, 3).
Vertices: (3, 4), (3, 2); (1, 3), (7, 3).
(x+3)
2
4
= 1 + (y 3)
2
x,y

(x + 3)
2
= y 3
Exercise 16.
x
2
144/9

y
2
144/16
= 1 =
x
2
16

y
2
9
.
e =
_
1
9
16
=
5
4
. Center (0, 0). [F[ = ae = 5. Foci: (5, 0), (5, 0). Vertices (4, 0).
Exercise 17. 20 = 5y
2
4x
2
. Center (0, 0). [F[ = ae = 2
_
3
2
_
= 3. Foci: (0, 3). 1 =
y
2
4

x
2
5
. e =
_
1
5
4
=
3
2
.
Vertices: (0, 2)
Exercise 18.
(x1)
2
4

(y+2)
2
9
= 1.
Center (1, 2). e =
_
1
9
4
=

13
2
; [F[ = 2

13
2
=

13. Foci: (1 +

13, 2), (1

13, 2).
Vertices: (5, 2), (3, 2).
Exercise 19. F = ae = 2(2) = 4.
x
2
4
+
y
2
12
= 1.
y
2
12
+ 1 =
x
2
4
x,y
y =

3x.
223
b
2
= a
2
(1 e
2
) = 4(1 4) = 12.
Exercise 20. F = ae =

2 = (1)e. b
2
= a
2
(1 e
2
) = 1(1 2) = 1. =y
2
x
2
= 1.
Exercise 21.
x
2
4

y
2
16
= 1
Exercise 22. (y 4)
2

(x+1)
2
3
= 1 where
F = ae = [ 2[ = ae. b
2
= a
2
(1 e
2
) = 1(1 4) = 3
Exercise 23.
(x2)
2
a
2

(y+3)
2
b
2
= 1
(3, 1) =
1
a
2

4
b
2
= 1
(1, 0) =
9
a
2

9
b
2
= 1
=
(y + 3)
2
27/8

(x 2)
2
(27/5)
= 1
Exercise 24.
x
2
1
3
= y
2
.
2x
3
= 2yy

. yy

=
x
3
.
3x 2y = C. m =
3
2
=y
0
9
2
= x
0
.
81
4
y
2
0
1 = 3y
2
0
=y
0
=
2

69
.
The asymptotes of y
2
=
x
2
1
3
are y =
x

3
.
3
_
9

69
_
2
_

69
_
=
23

69
= C
3x
_
23
3
= 2y
Exercise 25.
x
2
a
2
+
y
2
b
2
= 1
x
2
a
2

y
2
4a
2
= 1.
(3, 5) 9
25
4
= a
2
; a
2
=
11
4
.
=
x
2
11/4

y
2
11
= 1 .
Quick Review of Parabolas.
F on positive half plane to N.
|X F| = e[(X F) N +d[
Let N = e
x
; d = 2c; F = (c, 0); e = 1.
(x c)
2
+y
2
= e
2
((x c) + 2c)
2
= (x c)
2
+ 4c(x c) + 4c
2
y
2
= 4cx
Thus, for ellipses, the vertex is equidistant to the focus and directrix (conrming the other denition).
Let N = e
y
, d = 2c; F = (0, c), e = 1.
x
2
+ (y c)
2
= ((y c) + 2c)
2
= (y c)
2
+ 4c(y c) + 4c
2
x
2
= 4cy
Exercise 26. 4c = 8 (0, 0) vertex. y = 0 symmetry axis. x = 5 directrix.
Exercise 27. 4c = 3. Vertex: (0, 0). Symmetry axis: y = 0. Directrix: x = 3/4.
Exercise 28. (y 1)
2
= 12(x
1
2
). 4c = 12, c = 3. Symmetry axis: y = 1. Directrix:
_
5
2
, 1
_
.
Exercise 29. x
2
/6 = y. 4c =
1
6
c =
1
24
. Vertex: (0, 0). Directrix: y =
1
24
. Symmetry axis: x = 0.
Exercise 30. x
2
+ 8y = 0. 4c =
1
8
; c =
1
32
. y =
1
32
directrix; x = 0 axis.
Exercise 31. (x + 2)
2
= 4(y +
9
4
). 4c = 4; c = 1. Center (2, 9/4). Directrix: y = 13/4. Axis: x = 2.
Exercise 32. y = x
2
.
224
Exercise 33. x
2
= 8y.
Exercise 34. (y 3) = 8(x + 4)
2
.
Exercise 35. c =
5
4
5(x
7
4
) = (y + 1)
2
Exercise 36. y = ax
2
+bx +c
(0, 1) c = 1 (1, 0) 0 = a +b + 1(2, 0) 0 = 4a + 2b + 1 a =
1
2
=y =
1
2
x
2

3
2
x + 1
Exercise 37. 4c(x 1) = (y 3)
2
. 4c(2) = (4)
2
= 16. c = 2. 8(x 1) = (y 3)
2
.
Exercise 38. |X F| = ed(X, L) = [(X F) N d[
L = (x, y)[2x +y = 10;
2

5
x +
y

5
=
10

5
.
d = N = x
L
dN N = d = x
L
N =
10

5
.
F = 0 =|X|
2
= [X N d[
2
=
_
2

5
x +
y

5
+
10

5
_
2
= x
2
+y
2
5x
2
+ 5y
2
= (2x y + 10)
2
= 4x
2
+y
2
+ 100 + 4xy 40x 20y
=x
2
+ 4y
2
4xy + 40x + 20y 100 = 0
13.25 Miscellaneous exercises on conic sections.
Exercise 1.
y
2
b
2
= 1
x
2
a
2
y
2
= b
2

_
bx
a
_
2
= b
2
_
1
_
x
a
_
2
_
y = 2
_
a
a
b
_
1
_
x
a
_
2
dx = 2
_
1
1
ab
_
1 x
2
dx = (ab) area of a circle of radius 1
Exercise 2.
(1) Without loss of generality, let the major axis be 2a in the x-axis. y = b
_
1
_
x
a
_
2
V =
_
a
a
b
2
_
1
x
2
a
2
_
dx = b
2
a
_
1
1
(1 x
2
)dx =
4
3
(1)
3
b
2
a
(2) If rotated about the minor axis, suppose, without loss of generality, 2a is the minor axis (just note that
x
2
a
2
+
b
2
a
2
= 1
have x, y, a, b as dummy labels).
=V =
4
3
(1)
3
b
2
a, where 2a is the minor axis, 2b is the major axis.
Exercise 3.
x
2
(3/A)
+
y
2
(3/B)
= 1 By
2
= 3 Ax
2
=y
2
=
3
B

Ax
2
B
; y =
_
3
B

Ax
2
B
. So the area inside this ellipse is
2
_
1
B
_

3/A

3/A
_
3 Ax
2
dx = 2
_
3
B
_

3/A

3/A

1
x
2
_
3
A
_
For the other ellipse equation,
x
2
3/(A+B)
+
y
2
3/(AB)
= 1. y
2
=
_
3
AB
__
1
x
2
(3/(A+B))
_
; y =
_
3
AB
_
1
x
2
(3/(A+B))
.
Thus, the area inside this ellipse is
2
_
3
AB
_

3
A+B

3
A+B

_1
_
_
x
_
3
A+B
_
_
2
Equating the two areas after making an appropriate scale change,
2
_
3
B
_
3
A
_
1
1
_
1 x
2
dx = 2
_
3
AB
_
3
A+B
_
1
1
_
1 x
2
dx
225
Thus A
2
B
2
= AB = A
2
BA B
2
. Simply try treating B as a number and solve the quadratic equation in terms
of A.
A =
B
_
B
2
4(1)(B
2
)
2(1)
=
B B

5
2
=
B(1 +

5)
2
Exercise 4. y =
4h
b
2
x
2
.
_
b/2
b/2
_
4h
b
2
x
2
+h
_
=
4h
3b
2
x
3

b/2
b/2
+h
_
b
2
+
b
2
_
=
2hb
3
Exercise 5. y
2
= 8x.
_
2
0
8tdt = 4(2)
2
= 16pi
Exercise 6. y
2
= 2(x 1). y
2
= 4(x 2).
(1)
A = 2
_
2
1
_
2(x 1) + 2
_
3
2
_
2(x 1) 2

x 2 =
= 2

2
2
3
(x 1)
3/2

2
1
+ 2

2
2
3
(x 1)
3/2

3
2
4
2
3
(x 2)
3/2

3
2
= 2

2
2
3
+

2
4
3
(2)
3/2
2

2
2
3
4
2
3
= 8/3
(2)
_
2
1
2(x 1) = 2
_
1
2
x
2
x
_

2
1
= 2
_
1
2
(4 1) (2 1)
_
= 1
_
3
2
(2(x 1) 4(x 2)) dx =
_
3
2
(2x + 6)dx = x
2

3
2
+ 6x[
3
2
= (9 + 4 + 6(3 2)) = 1
=V =
_
2
1
2(x 1) +
_
3
2
(2(x 1) 4(x 2)) = 2
(3)
y
2
2
+ 1 = x,
y
2
4
+ 2 = x
2
_
2
0
_
_
y
2
4
+ 2
_
2

_
y
2
2
+ 1
_
2
_
= 2
_
2
0
_
3y
4
16
+ 3
_
dy = 2
_
3
80
y
5
+ 3y
_

2
0
=
= 2
_
3(32)
80
+ 6
_
= 2
_
96 + 480
80
_
= 2
_
384
80
_
=
48
5
Exercise 7. By Apostols denition of conic sections, we are basically given the conic section denition with e =
1
2
. So just
plug in the pt. (0, 4).
x
2
a
2
+
y
2
b
2
= 1
(0,4)
b = 4 b
2
= a
2
(1 e
2
) = 16 = a
2
_
1
_
1
2
_
2
_
x
2
64/3
+
y
2
16
= 1
Exercise 8. F = 0 |X F| = |X| = ed(X, F) = [X N +d[ =
x

2
+
y

2
+
1

2
because for the directrix
y +x = 1
N =
_
1

2
,
1

2
_
1

3
y +
x

2
=
1

2
X
L
= P +tA
X
L
NN P = 1/

2
dN = X
L
X
L
N = d = 1/

2
226
So by squaring both sides of the vector equation,
x
2
+y
2
=
x
2
2
+xy +
y
2
2
+
1
2
+x +y
x
2
2
+
y
2
2
xy x y =
1
2
x
2
+y
2
2xy 2x 2y = 1
Exercise 9. Center (1/2, 2) because we equate the asymptotes to see where they intersect: y = 2x + 1 = 2x + 3.
(y 2)
2
a
2

(x 1/2)
2
a
2
/4
= 1
(0,0)

4
a
2

1
a
2
=
3
a
2
= 1
(y 2)
2
3

(x 1/2)
2
3/4
= 1
Exercise 10. px
2
+ (p + 2)y
2
= p
2
+ 2p.
x
2
p+2
+
y
2
p
= 1.
(1) Since p +2 > p, the foci must lie on the x axis. a
2
= p +2; b
2
= a
2
(1 e
2
) = p = (p +2)(1 e
2
). e =
_
2
p+2
F = ae =

2. (

2, 0).
(2) F = ae =

2 = a(

3) =a =
_
2
3
; b
2
=
2
3
(1 3) =
4
3
.
x
2
2/3

y
2
4/3
= 1
Exercise 11. e = 1 for an ellipse.
|X F| = [X N a[ = a X N
|X F| = |X +F| = [ X N a[ = a +X N
|X F| +|X +F| = 2a
Exercise 12.
|X F| = e[(X F) N d[ = e(d (X F) N)
|X +F| = ed(X, L) = e[(X F) N +d[ = e(d (X F) N)
|X F| |X +F| = 2ed
X X so for the other branch, |X +F| |X F| = 2ed
Exercise 13.
(1)
(tx)
2
a
2
+
(by)
2
b
2
= 1
_
b
t
_
2
=
a
2
(1e
2
)
t
2
=
_
a
t
_
2
(1 e
2
)
(2) b
2
1
= a
2
1
(1 e
2
) b
2
2
= a
2
2
(1 e
2
).
1
b
2
1
a
2
1
= 1
b
2
2
a
2
2
;
b
2
1
a
2
1
=
b
2
2
a
2
2
x
2
1
a
2
1
+
y
2
b
2
1
= 1 =
__
b2
b1
_
x
_
2
a
2
2
+
_
b2
b1
y
_
2
(b
2
y)
2
(3)
(tx)
2
a
2

(ty)
2
b
2
= 1
_
b
t
_
2
=
a
2
(e
2
1)
t
2
=
_
a
t
_
2
(e
2
1).
b
2
1
= a
2
1
(e
2
1) b
2
2
= a
2
2
(e
2
1)
b
2
1
a
2
1
+ 1 = e
2
b
2
2
a
2
2
+ 1 = e
2
b
2
1
a
2
1
=
b
2
2
a
2
2

_
x
a
1
_
2

_
y
b
1
_
2
=
_
_
_
b2
b1
x
_
a
2
_
_
2

_
b2
b1
y
b
2
_
2
= 1
227
Exercise 14.
x
2
a
2
+
y
2
b
2
= 1. =
x
a
2
+
y
b
2
y

= 0
=y

=
b
2
x
ya
2
=
a
2
(1e
2
)x
a
2
y
=
(e
2
1)x
y
Exercise 15.
(1) y = ax
2
+bx +c ty = a(tx)
2
+btx +c y = atx
2
+bx +c/t = y = Ax
2
+b +C
(2) y = tx
2
, t ,= 0
Exercise 16. x y + 4 = 0 y = 4

x (y
2
= 16x); y

= 2x
1/2
.
y

(x = 4) = 1 (x, y) = (4, 8).


Exercise 17.
(1) If we treat the two given parabolas, y
2
= 4p(x a) and x
2
= 4qy, as two vector objects free from any specic
coordinate system then we observe that we can disregard the sign of q and p and simply state that they are both
positive. What matters is that we observe that p and q are the distance of the foci to the vertex for each of the
respective parabolas.
Second, observe that a is not given. By diagram, if p, q are given, a must be moved along the x-axis to t the
tangency condition. Thus, in terms of doing the algebra, just eliminate p and q from the relations.
If (h, k) is the point of contact,
x
2
= 4qy
x
2q
= y

(h) =
h
2q
y
2
= 4p(x a) y = 2

x a
y

p
1

x a
y

(h) =

p
1

h a
(Tangent condition)
_
h
2q
_
2
=
p
h a
=(h
2
)(h a) = (2q)
2
p
(one point of contact condition) with q =
h
2
4k
, p =
k
2
4(h a)
=h
2
(h a) =
_
h
2
2k
_
2
k
2
4(h a)
=(h a)
2
=
h
2
16
=h =
2a a/2
15/8
= 4a/3
(2)
h
2q
=

h a
2a
3q
=

p
_
a/3
=

3p

a
; 2a

a = 3
_
3pq
=4a
3
= 27pq
2
Exercise 18. First hint: Vector methods triumph over algebraic manipulations of Cartesian coordinates. Think of the locus
in terms of vector objects that are coordinate-free and the conic section will emerge. I mean, try evaluating |P A|
2
=
(x 2)
2
+ (x 3)
2
= (x +y)
2
A = (2, 3), N =
1

2
(1, 1), X = (x, y).
|X A| = x +y =

2(X N) =

2(X N (F N d))
where F N = d = A N =
5

2
d = distance from focus to the directrix .
y = x + 1 (axis of the hyperbola)
d =
5

2
=
_
(2 x)
2
+ (3 y)
2
=

2(2 x) x =
1
2
, y =
1
2
_
1
2
,
1
2
_
must also be the center. y
1
2
=
_
x +
1
2
_
is the directrix.
_
y
1
2
_
=
_
x +
1
2
_
is the general form of the asymptote.
228
Consider asymptotes in general. |X F| = ed(X, L).
|X F|
d(X, L)
= e =
|X F|
[X N (F N d)[
=
|X F|
(X F) N +d[
For |X F| , |X F| > d. To keep ratio of e, X F must be ultimately directed by N by a ratio of e.
=e =
|X F|
|X F| cos
=
1
cos
e.g. Consider N = e
x
.
x
2
a
2

y
2
b
2
= 1 =y =
b
a
x =

e
2
1x.
From the vector equation,
(X F) N = (x c, y) N =
_
(x c)
2
+y
2
cos = x c
_
(x c)
2
+y
2
x c
=
1
cos
= e;
(x c)
2
+y
2
(x c)
2
= e
2
;
y
2
(x c)
2
= e
2
1 =y =
_
e
2
1x
For our problem, consider the conic section approaching the asymptote. Then the conic section will look more like those
linear asymptotes.
_
(x 2)
2
+ (y 3)
2
= x +y
y
1
2
=(x+
1
2
)

__
x +
1
2
_

5
2
_
2
+
_

_
x +
1
2
_

5
2
_
2
= x +
_
x +
1
2
_
+
1
2
=

(1 +
2
)
_
x +
1
2
_
2
5(1 +)
_
x +
1
2
_
+
25
2
x

1 +
2
(1 +)
= = 0
The asymptotes are y =
1
2
and x =
1
2
.
In the second part, each quadrant must be checked. So far, I only have that quadrant II is lled: points in quadrant III and
quadrant IV cannot satisfy the given condition. To see this, consider quadrant II.
|x A| = x +y =

2(x, y)
_
1

2
,
1

2
_
For quadrant II, N =
_
1

2
,
1

2
_
. By diagram, (X F) N > 0 and X N > 0.
A N =
1

2
A N =
1

2
d =
1

2
[(X F) N +d[ = (X F) N +d
The equation for the axis of the conic section is y = (x 5).
By taking the asymptotic limit like above, we can show that = 0 again. We only sketch the part of the hyperbola in
quadrant II.
By similar procedure, I found that quadrant III, IV cannot satisfy the condition.
Exercise 19.
|X F| = d(X, L) = [(X F) N +d[
x
2
+y
2
= (X N +d
1
)
2
= y
2
+ 2yd
1
+d
2
1
F=0
x
2
= 2yd
1
+d
2
1
y

1
=
x
d
1
|X F| = [(X F) N d
2
[ = d
2
(X F) N
F=0
|X| = d
2
y
x
2
+y
2
= d
2
2
2d
2
y +y
2
=x
2
= d
2
2d
2
y y

2
=
x
d
2
229
Point of intersection x
2
0
= 2y
0
d
1
+d
2
1
= d
2
2
2d
2
y
0
2(d
1
+d
2
)y
0
= d
2
2
d
2
1
=y
0
=
d
2
d
1
2
x
2
0
= d
2
(d
2
2y
0
) = d
2
d
1
=
y

1
=

d
2
d
1
d
1
=
_
d
2
d
1
y

2
=

d
2
d
1
d
2
=
1
_
d2
d1
Exercise 20.
(1) Use X X symmetry.
|X F| = ed(X, L) = e[(X F) N +d[ = e[X N F N +d[ = [eX N a[
|X|
2
2X F +|F|
2
= e
2
(X N)
2
2ea(X N) +a
2
X X =|X|
2
+ 2X F +|F|
2
= e
2
(X N)
2
+ 2ea(X N) +a
2
=|X
2
| +|F|
2
= e
2
(X N)
2
+a
2
x
2
+y
2
+c
2
= e
2
x
2
+a
2
[F[ = c = ae
_
a
2
c
2
a
2
_
x
2
+y
2
= a
2
c
2
=
x
2
a
2
+
y
2
a
2
c
2
= 1
(2)
x
a
2
+
yy

a
2
c
2
= 0 =y

=
(a
2
c
2
)x
ya
2
xy(y

)
2
=
_
a
2
c
2
a
2
_
2
x
3
y
(x
2
y
2
c
2
)y

=
_
x
2
+
a
2
c
2
a
2
x
2
a
2
__
(a
2
c
2
)x
ya
2
_
=
= (a
4
+(a
2
c
2
)x
2
a
2
x
2
)
(a
2
c
2
)
a
4
x
y
xy =
xy
2
y
=
x(a
2
c
2

_
a
2
c
2
a
2
_
x
2
)
y
=
= (a
2
c
2
)(a
4
+a
2
x
2
)x/(a
4
y)
=xyy
2
+ (x
2
y
2
c
2
)y

xy = 0
(3) For y

, consider
1
y

at every (x, y).


xy
_
1
y

_
2
+ (x
2
y
2
c
2
)
1
y

xy = 0 =
xy
y

+ (x
2
y
2
c
2
)
1
y

+xy
if y

=0
xy + (x
2
y
2
c
2
)y

+xy(y

)
2
= 0
Thus S S since the dening differential equation is invariant under the transformation of the slope.
Exercise 21. For a circle centered at C, then |X C| = r
2
0
for all points X on that circle.
For the condition of being tangent to a given line, L = P + tA, then (X
C
C) A = 0 and the point lies on the circle so
|X
C
C| = r
2
0
.
Call the point that all the circles pass through F. Then |C F| = |C X
C
|. |C X
C
| is by denition d(X, L), the
distance from the circle center to the line. |C F| = |C X
0
| is by denition a parabola.
Exercise 22. Consider a circle thats part of the mentioned family that has its center directly below the given circle with radius
r
0
, and center Q.
Its given that the center is equidistant from the point of tangency and the line. This hints at a parabola because the parabolas
230
vertex is equidistant from the focus and the directrix. Thus, we need to show that d(X, L) is equal to the distance from the
circle center C to the bottom point of Q.
Let N be a unit normal vector pointing from the line towards the focus, placing the focus in the positive half-plane.
Let C be the center of an arbitrary circle in the family and r
1
its radius.
Let X
1
be the point of tangency between circle Q and circle C.
We want |(Q+r
0
N) C| = |X
2
C|.
The tangency condition between circle Q and C means that
(X
1
C) = (X
1
Q); > 0 =
r
1
r
0
Qr
0
N C = QX
1
r
0
N C +X
1
take the magnitude
|QX
1
|
2
+|X
1
C|
2
+r
2
0
+ 2(QX
1
)(X
1
C) + 2(X
1
Q)r
0
N + 2(C X)r
0
N
r
2
0
+r
2
1
+r
2
0
+ 2r
2
0
+ 2r
0
(1 +)(x
1
Q) N
2r
2
0
+r
2
1
+ 2r
1
r
0
+ 2(r
1
+r
0
)(X
1
Q) N
I had thought the key is to use the law of cosines to evaluate (X
1
Q) N =
1

(C X
1
) N.
Length l = d(X, L) = d(C, L).
But that just gets us back to the same place.
I had found the solution by a clever construction. But to come to that conclusion it required me to be unstuck - if
something doesnt work, move onto the next - dont try to make something work and go in circles. And persistence is key
because there can be many false eurekas.
Again, consider a particular circle with its center C
2
right below the given Q circle that just makes C
2
tangent with
the given line L
2
. The directrix is not going to be L
2
but L
1
, a line translated below L
2
, line of tangency, by r
0
, so that
|QC
2
| = r
2
+r
0
= d(C
2
, L
1
). It is a clever articial construction.
Lets show this for any circle C of radius r
1
in the family.
Tangent to the circle Q condition: X
1
C = (QX
1
).
So then |QC| = r
1
+r
0
Tangent to the line L
2
= B
2
+tA
2
: (X
2
C) A
2
= 0
|X
2
C| = r
1
Consider L
1
, a line translated by r
0
from L
2
away from Q.
If L
2
= B
2
+tA
2
, L
1
= B
2
r
0
N +tA
2
.
Since X
2
C = r
1
(N) then X
2
r
0
N C = (r
1
+ r
0
)(N) will point from C to L
1
, because (X
2
r
0
N) =
((B
2
+tA
2
) r
0
N) L
1
.
=|QC| = r
1
+r
0
= |X
2
r
0
N C| = d(C, L
1
).
Exercise 23. Without loss of generality, use y
2
= 4cx.
The latus rectum intersect the parabola at (c, +2c), (c, 2c).
Thus 4c = length of latus rectum = 2d = 2( distance from focus to directrix ).
y = 2

cx y

c/

x y

(c) = 1
Tangent lines: y = (x +c).
intersection
+(x +c) = (x +c) x = c (at the directrix)
Exercise 24. Center of circle is given to be 0.
Collinear with center and center not between them: P = Q; > 0
|P| |Q| = r
2
0
= |Q|
2
231
For the line dened in Cartesian coordinates as x + 2y 5 = 0, the vector form of this line is given by
X
L
= B +tA X
L
N = (x, y)
_
1

5
,
2

5
_
= N B + 0 =

5
A =
_
2

5
,
1

5
_
is a vector thats perpendicular to N ;
B = (1, 2) since we can simply plug it in to satisfy the equation
X
L
= (1, 2) +t
_
2

5
,
1

5
_
t R
Q = B +tA =|Q|
2
= B
2
+ 2tB A+t
2
A
2
= 5 +t(0) +t
2
= 5 +t
2
(5 +t
2
)() = r
2
0
= 4 =
4
5 +t
2
P = Q =
4
5 +t
2
_
(1, 2) +t
_
2

5
,
1

5
__
14.4 Exercises - Vector-valued functions of a real variable, Algebraic operations. Components; Limits, derivatives,
and integrals. Exercise 1. F

= (1, 2t, 3t
2
, 4t
3
).
Exercise 2. F

= (sin t, 2 sin t cos t, 2 cos 2t, sec


2
t)
Exercise 3. F

=
_
1

1t
2
,
1

1t
2
_
Exercise 4. F

= (2e
t
, 3e
t
).
Exercise 5. F

=
_
sinh t, 2 cosh 2t, 3e
3t
_
Exercise 6.
_
t
1+t
2
,
1
1+t
2
,
2t
(1+t
2
)
2
_
Exercise 7. F

=
_
2
1+t
2
+
4t
2
(1+t
2
)
2
,
4t
(1+t
2
)
2
, 0
_
.
F

F =
4t(1 +t
2
)
(1 +t
2
)
3
+
4t
2
(2t)
(1 +t
2
)
3
+
4t 4t
3
(1 +t
2
)
3
= 0
Exercise 8.
_
1
2
,
2
3
, e
1
1
_
Exercise 9.
(cos t, sin t, ln [ cos t[)[
/4
0
=
_

3
2
+ 1,

2
2
, ln

2
2
_
Exercise 10. (ln (1 +e
t
), t ln (1 +e
t
))[
1
0
=
_
ln
_
1+e
2
_
, 1 ln
_
1+e
2
__
Exercise 11.
_
te
t
e
t
, t
2
e
t
2te
t
+ 2e
t
, te
t
e
t
_

1
0
= (1, e 2, 2e
1
+ 1)
Exercise 12. (2, 4, 1) = A.
_
1
0
(te
2t
, t cosh 2t, 2te
2t
)dt =
_
1
2
te
2t
+
1
4
e
2t
,
t sinh 2t
2

cosh 2t
4
, 2
_
1
2
te
2t

e
2t
4
__

1
0
=
=
_
1
4
e
2
+
1
4
,
sinh 2
2

cosh 2
4
+
1
4
, 2
_
3
4
e
2
+
1
4
__
.
A B =
1
2
e
2
+2 sinh 2 + cosh 2 +
3
2
e
2
.
Exercise 13. F

(t) = B = 1 = |F

(t)| [B[ cos (t).


Given (t) =
0
constant, |F

(t)| must be a constant.


|F

(t)|
2
= F

(t) F

(t) = g g

= 2F

(t) F

(t) = 0 since |F

|
2
constant .
=F

(t) F

(t) = 0
Exercise 14.
232
F

= 2e
2t
A+2e
2t
B
F

= 4e
2t
A+ 4e
2t
B = 4(F)
Exercise 15. G

= F

+F F

= F F

Exercise 16.
G = F (F

)
G

= F

(F

) +F (F

+F

) = F (F

)
Exercise 17. If lim
tp
F(t) = A, jth component,

n
> 0,
j
> 0 such that [F
j
(t) A
j
[ <
_

n
if [t p[ <
j
Consider min
j=1,...n

j
=
0
n

j=1
[F
j
(t) A
j
[
2
<
n

j=1
__

n
_
2
= whenever [t p[ <
0
= lim
tp
|F(t) A| = 0
If lim
tp
|F(t) A| = 0, > 0, > 0 such that
_

n
j=1
(F
j
(t) A
j
)
2
< if [t p[ < .
=

n
j=1
(F
j
(t) A
j
)
2
<
>

n
j=1
(F
j
(t) A
j
)
2
> (F
k
(t) A
k
)
2
> 0
= > [F
k
(t) A
k
[ if [t p[ < .
Exercise 18. If F is differentiable on I, then
F

=
n

j=1
f

j
e
j
f

j
= lim
h0
1
h
(f
j
(t +h) f
j
(t))
F

=
n

j=1
lim
h0
1
h
(f
j
(t +h) f
j
(t)) = lim
h0
1
h
n

j=1
(f
j
(t +h) f
j
(t))e
j
= limh 0
1
h
(F(t +h) F(t))
If F

(t) = lim
hto0
1
h
(F(t +h) F(t)) = lim
h0
1
h

n
j=1
(f
j
(t +h) f
j
(t))e
j
=
=

n
j=1
lim
h0
1
h
(f
j
(t +h) f
j
(t))e
j
=

n
j=1
f

j
(t)e
j
So F

is differentiable.
Exercise 19. F

(t) = 0, j = 1 . . . n, f

j
(t) = 0. By one-dimensional zero-derivative theorem, f
j
(t) = c
j
constant. Thus
F(t) =

n
j=1
c
j
e
j
= C on an open interval I.
Exercise 20.
1
6
t
3
A+
1
2
t
2
B +Ct +D
Exercise 21. Y

(x) +p(x)Y (x) = Q(x). Then j = 1, . . . , n


y

j
(x) +p(x)y
j
(x) = Q
j
(x)
Since p, Q are continuous on I, and given this initial value condition y
k
(a) = b
k
,
y
j
(x) = e

x
a
p(t)dt
_
b
j
+
_
x
a
Q
j
(t)e

t
a
p(u)du
dt
_
=
n

j=1
j
j
(x) = Y (x) = e

x
a
p
_
B +
_
x
a
Qe

t
a
p
dt
_
Exercise 22.
233
tF

= F +tA =F

+tF

= F

+A
=tF

= A
F

(t) = A/t
=F

(t) = Aln t +B
=F(t) = A(t ln t t) +Bt +C
F(1) = A(1) +B +C = 2A
tF

= F +tA =At ln t +Bt = A(t ln t t) +Bt +C +tA


C = 0, B = 3A
F(t) = A(t ln t t) + 3At
F(3) = A(3 ln 3 3) + 9A = 3Aln 3 + 6A
Exercise 23.
F

(x) = e
x
A+xe
x
A+
1
x
2
_
x
1
F(t)dt +
1
x
F(x) =
= e
x
A+xe
x
A+e
x
A
F(x)
x
+
F(x)
x
= 2e
x
A+xe
x
A = (2 +x)e
x
A
F

(x) = (2 +x)e
x
A; F(x) = 2e
x
A+A(xe
x
e
x
) +C = Axe
x
+e
x
A+C
_
x
1
(Ate
t
+e
t
A+C)dt =
_
A(te
t
e
t
) +e
t
A+Ct
_

x
1
=
= A(xe
x
e
x
) +e
x
A+C(x 1) eA = Axe
x
+C(x 1) eA
xe
x
A+Ae
x
+
C(x 1)
x

eA
x
=C = eA
Exercise 24. F

(t) = (t)F(t)
=f

k
(t) = (t)f
k
(t); ln
_
f
k
(t)
f
k
(a)
_
=
_
t
a
(x)dx
f
k
= f
k
(a)e

t
a

F(t) =
n

j=1
f
j
(a)e6
_
t
a
e
j
= e

t
a

n

j=1
f
j
(a)e
j
= u(t)A
14.19 Exercises - Velocity and acceleration in polar coordinates, Plane motion with radial acceleration, Cylindrical
coordinates.
Exercise 1.
v =
dr
dt
=
dr
dt
e
r
+r
d
dt
e

= e
r
+re

( = t)
a =
_
d
2
r
dt
2
r
_
d
dt
_
2
_
e
r
+
1
r
d
dt
_
r
2
d
dt
_
e

= re
r
+ 2e

v = e
r
+te

= cos te
x
+ sin te
y
+t sin te
x
+t cos te
y
=
= (cos t t sin t)e
x
+ (sin t +t cos t)e
y
a = (t cos t 2 sin t)e
x
+ (t sin t + 2 cos t)e
y
Exercise 2.
v = e
r
+re

+e
z
= (cos t t sin t)e
x
+ (sin t +t cos t)e
y
+e
z
a = (t cos t 2 sin t)e
x
+ (t sin t + 2 cos t)e
y
Exercise 3. (a).
r = sin t; = t; z = log sec t; t <

2
(r cos )
2
+ (r sin
1
2
)
2
= r
2
r sin +
1
4
=
1
4
234
(b).
v =
dt
dt
e
r
+r
d
dt
e

+ log sec te
z
= cos te
r
+re

+
tan sec
sec
e
z
v
z
= tan ; v
2
= cos
2
t +r
2
+ tan
2
= sec
2

cos =
tan
sec
= sin = r = sin t
= arccos (sin )
Exercise 6.
A =
_
R
2
()d =
_
2
0
1
2
e
2c
d =
e
2c
4c

2
0
=
e
4c
1
4c
Exercise 7.
_

0
1
2
sin
4
d =
1
2
_

0
sin
2
(1 cos
2
)d =
1
2
_

0
__
1 cos 2
2
_

_
1 cos 4
2(4)
__
d = 3/16
Exercise 15. Place target at the center (without loss of generality). The strategy is to break up v into the polar coordinate unit
vectors.
r = re
r
v =
dr
dt
e
r
+r
d
dt
e

dr
dt
= v
r
= v cos ( ) = v cos
r
d
dt
= v sin
v sin
v cos
= tan =
r
d
dt
dr
dt
= r
d
dr
;
1
r
dr
d
= tan
r = e
tan
Exercise 17.
A rst order differential equation of the form y

= f(x, y) is homogeneous if f(tx, ty) = f(x, y). Then


f(r cos , r sin ) = f(cos , sin ) = f()
We nd that
dy
d
=
dr
d
sin +r cos
dx
d
=
dr
d
cos r sin
Thus
dy
dx
=
dr
d
sin +r cos
dr
d
cos r sin
= f()
Exercise 18.
v = k r
v =
dr
dt
e
r
+r
d
dt
e

v e
r
= 0, so
dt
dt
= 0; k r = r
d
dt
e

= re

= r
d
dt
[k r[
2
=
2
r
2
= r
2
_
d
dt
_
2
=

d
dt

, > 0
Exercise 19. (a)
235
v =
dr
dt
e
r
+r
d
dt
e

= r
d
dt
e

;
dr
dt
= 0; e

= e
z
e
r
v = r
d
dt
e
z
e
r
=
d
dt
e
z
re
r
= r
(b).
a = v

r + r

= r + r

= ( r) = ( r)
2
r
(c).
Now r = 0 =a =
2
r
Exercise 20.
The distance [r
p
(t) r
q
(t)[ is independent of t, so
d
dt
[r
p
(t) r
q
(t)[ = 0, which implies
d
dt
(r
2
p
2r
p
r
q
+r
2
q
) = 2r
p

dr
p
dt
2
dr
p
dt
r
q
2r
p

dr
q
dt
+ 2
dr
q
dt
r
q
= 0
dr
p
dt
(r
q
r
p
) =
dr
q
dt
(r
p
r
q
)
Suppose
v
p
=
p
r
p
v
q
=
q
r
q
Then
v
p
(r
q
r
p
) = v
q
(r
p
r
q
)

p
r
p
r
q
=
q
r
q
r
p
= =
q
r
p
r
q
((
p

q
) r
p
) r
q
= 0
Thus,
p
=
q
.
236

You might also like